You are on page 1of 224

The Math Less Traveled | Explorations in mathema...

http://mathlesstraveled.com/

The Math Less Traveled

Making a computer out of dominoes?


Posted on October 10, 2012

WhenImentionedcarryingoutcomputationalprocesseswitharoomfullofdominoes,Iwasntkidding.Matt ParkerisplanningtobuildadominocomputerattheManchesterScienceFestivalattheendofthemonth.The ManchesterScienceFestivalbloghasanicewriteupexplainingtheproject. HeresavideoofMattexplaininghowadominoANDgateworks(twochainsofdominoescomein,andone goesout;theoutgoingdominoeswillfallonlyifbothincomingchainsdo).UnfortunatelyitseemsIcantembed videosonthisblog,atleastnotwithoutgivingwordpresssomecash=(,soyoullhavetoactuallyclickthatlink towatchthevideo,butitstotallyworthit(trustme). IwishIcouldgoseeit,butitsabitfarforme.AnyMathLessTraveledreadersintheUKwhocanactuallygo watchthecomputerinactionandreportback?


Posted in computation, links, video | Tagged computer, domino, festival, Manchester, Matt Parker, science | 3 Comments

Factorization diagrams
Posted on October 5, 2012

InanidlemomentawhileagoIwroteaprogramtogenerate"factorizationdiagrams".Heres700:

Itseasytosee(Ihope),justbylookingatthearrangementofdots,thatthereare

intotal.

HereshowIdidit.First,afewimports:afunctiontodofactorizationofintegers,andalibrarytodrawpictures (yes,thisisthelibraryIwrotemyself;Ipromisetowritemoreaboutitsoon!).

1 of 224

FollowTheMathLess Traveled 10/16/12 16:55


Get everynewpost deliveredto your Inbox.

The Math Less Traveled | Explorations in mathema...

http://mathlesstraveled.com/

> module Factorization where > > import Math.NumberTheory.Primes.Factorisation (factorise) > > import Diagrams.Prelude > import Diagrams.Backend.Cairo.CmdLine > > type Picture = Diagram Cairo R2

TheprimeLayoutfunctiontakesanintegern(assumedtobeaprimenumber)andsomesortofpicture,and symmetricallyarrangesncopiesofthepicture.

> primeLayout :: Integer -> Picture -> Picture

Thereisaspecialcasefor2:ifthepictureiswiderthantall,thenweputthetwocopiesoneabovetheother; otherwise,weputthemnexttoeachother.Inbothcaseswealsoaddsomespaceinbetweenthecopies(equal tohalftheheightorwidth,respectively).

> primeLayout 2 d > > | width d > height d = d === strutY (height d / 2) === d | otherwise = d ||| strutX (width d / 2) ||| d

ThismeanswhentherearemultiplefactorsoftwoandwecallprimeLayoutrepeatedly,weendupwiththingslike

Ifwealwaysputthetwocopies(say)nexttoeachother,wewouldget

whichismuchclunkierandhardertounderstandataglance. Forotherprimes,wecreatearegularpolygonoftheappropriatesize(usingsometrigIworkedoutonanapkin, dontaskmetoexplainit)andpositioncopiesofthepictureatthepolygonsvertices.

Follow

2 of 224

FollowTheMathLess Traveled 10/16/12 16:55


Get everynewpost deliveredto your Inbox.

The Math Less Traveled | Explorations in mathema...

http://mathlesstraveled.com/

> primeLayout p d = decoratePath pts (repeat d) > > > > > > where pts = polygon with { polyType } w = max (width d) (height d) r = w * c / sin (tau / (2 * fromIntegral p)) c = 0.75 = PolyRegular (fromIntegral p) r , polyOrient = OrientH

Forexample,heresprimeLayout 5appliedtoagreensquare:

Now,givenalistofprimefactors,werecursivelygenerateanentirepictureasfollows.First,ifthelistofprime factorsisempty,thatrepresentsthenumber1,sowejustdrawablackdot.

> factorDiagram' :: [Integer] -> Diagram Cairo R2 > factorDiagram' [] = circle 1 # fc black

Otherwise,ifthefirstprimeiscalledpandtherestareps,werecursivelygenerateapicturefromtherestofthe primesps,andthenlayoutpcopiesofthatpictureusingtheprimeLayoutfunction.

> factorDiagram' (p:ps) = primeLayout p (factorDiagram' ps) # centerXY

Finally,toturnanumberintoitsfactorizationdiagram,wefactorizeit,normalizethereturnedfactorizationinto alistofprimes,reverseitsothebiggerprimescomefirst,andcallfactorDiagram'.

> factorDiagram :: Integer -> Diagram Cairo R2 > factorDiagram = factorDiagram' > > > . reverse . concatMap (uncurry $ flip replicate) . factorise

Andvoila!Ofcourse,thisreallyonlyworkswellfornumberswithprimefactorsdrawnfromtheset (andperhaps ).Forexample,heres121:


Follow

3 of 224

FollowTheMathLess Traveled 10/16/12 16:55


Get everynewpost deliveredto your Inbox.

The Math Less Traveled | Explorations in mathema...

http://mathlesstraveled.com/

Arethere11dotsinthosecircles?13?Icantreallytellataglance.Andheres611:

Uhhwell,atleastitspretty! Herearethefactorizationdiagramsforalltheintegersfrom1to36:

Follow

4 of 224

FollowTheMathLess Traveled 10/16/12 16:55


Get everynewpost deliveredto your Inbox.

The Math Less Traveled | Explorations in mathema...

http://mathlesstraveled.com/

Powersofthreeareespeciallyfun,sincetheirfactorizationdiagramsareSierpinskitriangles!Forexample,heres :

Powersoftwoarealsofun.Heres

Follow

5 of 224

FollowTheMathLess Traveled 10/16/12 16:55


Get everynewpost deliveredto your Inbox.

The Math Less Traveled | Explorations in mathema...

http://mathlesstraveled.com/

[ETA:asanonpointsout,thisfractalhasanametoo:Cantordust!] Onelastone:104.

IwishIknewhowtomakeawebsitewhereyoucouldenteranumberandhaveitshowyouthefactorization diagrammaybeeventually. (Incaseyouwerewondering, .)

Posted in arithmetic, pictures, primes, programming, recursion | Tagged diagrams, factorization, Haskell | 50 Comments

What I Do: Part 0


Posted on October 4, 2012

Follow

6 of 224

FollowTheMathLess Traveled 10/16/12 16:55


Get everynewpost deliveredto your Inbox.

The Math Less Traveled | Explorations in mathema...

http://mathlesstraveled.com/

ThisisthefirstinaplannedseriesofpostsexplainingwhatIdoinmy"dayjob"asacomputersciencePhDstudent. Theideaistowriteaseriesofpostsofincreasingspecificity,butallaimedatageneralaudience. HaveyoueverwonderedwhatIactuallydoallday,otherthanwritethisblog?(Well,probablytheansweris "no"since,asweallknow,peopleontheInternetdontactuallyhavereallives,inthesamewaythat kindergartenteachersliveintheclosetintheirclassroom.)ButwhatIdowillactuallybequiteinterestingto readersofthisblog,Ithink. So,tostartoff:Iamacomputerscientist.Whatdoesthatmean?

WhatIdontdo
Letmebeginbysayingthat"computerscience"isactuallyaterriblenameforwhatIdo.Itsakintoan astronomersayingtheystudytelescopescience,oramicrobiologistsayingtheystudymicroscopescience.Of course,astronomersdontstudytelescopes,theyusetelescopestostudystarsandsupernovas.Microbiologists dontstudymicroscopes,theyusemicroscopestostudycellsandDNA.AndIdontstudycomputers,Iuse computerstostudywell,what?

Computation
Inabroadsense,whatcomputerscientistsstudyiscomputation,bywhichwemeanprocessesofsomesortthat takesomeinformationandturnitintootherinformation.Questionsthatcanbeaskedaboutcomputation include: Whataredifferentwaysofdescribingacomputationalprocess? Howcaninformationbestructuredtomakecomputationalprocesseseasiertowrite,moreefficient,ormore beautiful? Howcantwodifferentcomputationalprocessesbecompared?Whenisoneprocess"better"thananother? Howcandifferentprocessesbecombinedintoalargerprocess? Howcanwebesurethatsomeprocessreallydoeswhatwewantitto? Whatsortsof"machines"canbeusedtoautomatecomputationalprocesses? What(ifany)arethelimitationsofcomputationalprocesses? Imsureotherquestionscouldbeaddedtothislist,butthesearesomeofthemostfundamentalones. Noticethatnoneofthesequestionsinherentlyhaveanythingtodowithcomputers.A"computationalprocess" couldbecarriedoutwithpilesofrocks,anabacus,paperandpencil(didyouknowthattheword"computer" usedtorefertoapersonwhosejobitwastodocarryoutcomputationalprocesses?),acarefullysetuproomfull ofdominoes,oracarefullysetuptesttubefullofDNA.Itsjustthatmoderncomputerscancarryout(most) computationalprocessesmanyordersofmagnitudefasterthananyothermethodweknowof,sotheymake exploringtheabovequestionspossibleinmuchdeeperwaysthantheywouldotherwisebe.Andindeed,the mathematicalrootsofcomputersciencegobackmanyhundreds,eventhousandsofyearsbeforetheadventof digitalcomputers.
Follow

7 of 224

FollowTheMathLess Traveled 10/16/12 16:55


Get everynewpost deliveredto your Inbox.

The Math Less Traveled | Explorations in mathema...

http://mathlesstraveled.com/

So,Istudycomputation.Butasyoucanseefromthelistofquestionsabove,thatsstillincrediblybroad.Infact, myresearchfocusesonthefirsttwoquestionsinthelistabove.InPart1Illdescribethosequestionsinabit moredetail.Also,Imhappytotrytoansweranyquestionsleftinthecomments!


Posted in computation, meta | Tagged computation, computer science, whatido | 5 Comments

CoM and Relatively Prime


Posted on September 17, 2012

Acouplethingstodrawyourattentionto: The90thCarnivalofMathematicsisupoveratWalkingRandomly.Theresquitealotofcoolstuffinthis edition,gocheckitout! ThefirstepisodeofRelativelyPrimeisup!Thisisaseriesofeightshowsallaboutthestoriesbehind mathematicsproducedbySamuelHansen.Ihaventactuallyhadachancetolistentothefirstepisodeyet, butIknowheflewallovertheworldinterviewingmathematiciansforthissoitoughttobeinteresting!I alsohelpedfunditonKickstartersoImquiteexcitedtoseethefruits.


Posted in links | Tagged Carnival of Mathematics, episode, podcast, show | 1 Comment

Three new books


Posted on August 23, 2012

Athree-for-onetoday!HerearethreebooksIwantedtomentiontoyou,dearreader,foronereasonoranother.

AWealthofNumbers BenjaminWardhaugh PrincetonPresskindlysentmeareviewcopyofthisbook.Asananthologyofpopularmathematicswritingfrom the1500 stothepresent,itsnotthesortofbookIusuallyreviewherebutitsnonethelessfascinating.The featuredexcerptsare,byturnsgripping,dull,lucid,incomprehensible,hilarious,irrelevant,andfun.Ididntlearn awholelotofnewmathematicsbyreadingthisanthology,butitgavemesomeeye-openingperspectiveonhow peoplehavethoughtandwrittenaboutmathematicsoverthelast500years. DeadReckoning:CalculatingWithoutInstruments RonaldW.Doerfler
Follow

8 of 224

FollowTheMathLess Traveled 10/16/12 16:55


Get everynewpost deliveredto your Inbox.

The Math Less Traveled | Explorations in mathema...

http://mathlesstraveled.com/

Theresalotofcontroversyovertheuseofcalculatorsandcomputersinmathclassrooms.Shouldtheybe welcomedaslabor-savingdevicesthatallowstudentstoexploremathematicsinnewways,oreschewedas crutchesthatturnstudentsintobutton-pushingautomatawithnorealunderstanding?Itsanimportantdebate, butitseemstomethatthosewhoargueagainstcalculatorssometimeshaveanonethelessimpoverishedviewof thealternative:justtheabilitytodothestandardWesternalgorithmsforthefourbasicarithmeticoperations, andunderstandwhythealgorithmswork. Thisbook(whichIreceivedasagiftaboutsixmonthsago)isntintendedtoenterthatdebatedirectlybutitis afar-rangingsurveyofmethodsforpencil-and-paper(ormental)calculation.Therearegeneralalgorithms,of course,(andnotjustforarithmetic,butforsquareroots,logarithms,trigfunctions)buttherearealsoallsorts oftricksandspecialcaseswhicharisefrom,andengender,intimatefamiliaritywithnumbers.Isit"practical"? Well,notreally.Butthatscertainlynotthepoint.Ihadsomuchfunreadingthroughthisbookandtryingoutall thedifferentalgorithms:multiplyinginmyhead,computingsquarerootstomanydecimalplacesusingjusta singlesheetofpaperHonestlyIdontrememberanyoftheactualalgorithmsanymore,butIcameawaywith betternumbersenseandImayjustpullitoutandtrysomeofthealgorithmsagain.Eventuallysomeofthem areboundtostick!

MathematicalLiteracyintheMiddleandHighSchoolGrades FaithWallaceandMaryAnnaEvans Imentionthisbookparticularlybecauseitcontains(inaboxonpage67)anessaybyyourstruly!Iwrote somethingabouttheexperienceofwritingthisblogandinspiringreaderswithmathematicalbeauty. Inolongerteachmiddleorhigh-schoolstudents,buttheideaofusingliteracytoinspireinterestinmathmakes alotofsensetome(andofcourse,itsperfectlyapplicableatthecollegelevelaswell).Thereareallsortsof interestingideasinheresomeofwhichIwillneveruse(discussingstatisticsandvotingviaAmericanIdol)but othersofwhichIdlovetotrysomeday(usingbiographyorfictiontoinspiremathematicalinterest).


Posted in books, review | Tagged anthology, calculation, literacy | 2 Comments

Introduction to Mathematical Thinking with Keith Devlin


Posted on August 22, 2012

IjustlearnedfromDeniseatLetsPlayMath!thatKeithDevlinisgoingtobeteachingacourseonCoursera calledIntroductiontoMathematicalThinking.Itsfreeandopentoanyonewithonlyabackgroundinhighschool math.Lookslikeitshouldbequiteinterestingandperhapsofinteresttosomeofmyreaders!


Posted in links, teaching | Tagged Coursera, free, Keith Devlin, mathematical thinking, online | 1 Comment

Follow

9 of 224

FollowTheMathLess Traveled 10/16/12 16:55


Get everynewpost deliveredto your Inbox.

The Math Less Traveled | Explorations in mathema...

http://mathlesstraveled.com/

Visualizing nim-like games


Posted on August 16, 2012

Inspiredbythecommentsonthispost,IvehadsomeideasbrewingforawhileImjustonlynowgetting aroundtowritingthemup. Thetopicisvisualizingwinningstrategiesfor"nim-like"games.WhatdoImeanbythat?Byanim-likegameImean agameinwhichtwoplayerstaketurnsremovingobjectsfromsomepiles(subjecttosomerules),andthelast playertoplayisthewinner(or,sometimes,theloser). Acutevariant,duetoPaulZeitz(andintroducedtomebySueVanHattum),istothinkofapetshopwith differenttypesofpets;playerstaketurnsvisitingthepetshopandbuyingsomepets,untilthestoreisalloutof pets.

Forgameswithonlytwopiles,orapetstorewithonlytwotypesofpets,playingthegamecanalsobethought ofasmakingmovesonasquaregrid.The -coordinaterepresents,say,thenumberofXoloitzcuintli,andthe y-coordinatethenumberofYaks;thesquarewithcoordinates meansthatthepetstorehas xolosand yaksleft.Buyingsomexoloscorrespondstomovingleft;buyingyaksmeansmovingdown;buyinganequal numberofeachmeansmovingdiagonallydownandleft;andsoon.

Hereareafewexamplesofnim-likegames:
Follow

10 of 224

FollowTheMathLess Traveled 10/16/12 16:55


Get everynewpost deliveredto your Inbox.

The Math Less Traveled | Explorations in mathema...

http://mathlesstraveled.com/

Inthegameofnim,youmayonlybuyonetypeofanimaloneachturn(butyoucanbuyasmanyasyou want).Onagrid,youareallowedtomoveanydistanceleftordown(butnotboth). Theinterestingthingisthatwecanvisualizethewinningstrategyforthisgameinthefollowingway. (ZacharyAbelhasamuchmoredetailedexplanationofthisidea.)Awinningpositionisasquarethat guaranteesawinthatis,ifitisyourturnandyouareonawinningsquare,then(assumingyoumakethe rightmoveandcontinuetoplayperfectly)youwillwinthegame.Illindicatewinningpositionsbylight greensquares,likethis: .Alosingpositionisapositionsuchthatyoucantwinnomatterwhatmoveyou make(assumingyouropponentplaysperfectly).Illindicatelosingpositionsbydarkbluesquares,likethis: .Infact,thewinningpositionsareexactlythosefromwhichthereexistsatleastonelegalmovetoalosing position;andthelosingpositionsarethosefromwhicheverylegalmoveistoawinningposition. Hereswhatitlookslikefornim:

Nottooexciting,butitmakessense.Ifthepetstorehasanequalnumberofeachtypeofpetremaining,the firstpersontomoveisgoingtolose:theirmovewillresultinanunequalnumberofpets(i.e.asquareoffthe mainbluediagonal),andalltheiropponenthastodoisbuyanequalnumberoftheothertypeofpetto restorebalance.Ultimatelythelosingplayerwillbeforcedtocleanthestoreoutofonetypeofpet,andthe otherplayerthenwinsbycleaningthestoreoutoftheothertype. InWythoffsgame,youmaybuyanynumberofasingletypeofanimal,or anequalnumberofboth.Onagrid, youareallowedtomoveanydistanceleft,down,orata45degreeangleleftanddown. Thevisualizationforthisgame,ofcourse,ismuchmoreinteresting:

Follow

11 of 224

FollowTheMathLess Traveled 10/16/12 16:55


Get everynewpost deliveredto your Inbox.

The Math Less Traveled | Explorations in mathema...

http://mathlesstraveled.com/

Youcanreadallaboutthefascinatinganalysisofthisgame(anditsvisualization)onZacharyAbelsblog. Inacomment,MaxdescribedagamefromtheInternationalOlympiadinInformatics: Youstartwitharectangle,andyoucancutiteitherverticallyorhorizontallyatintegersizes,each timekeepingthelargerpiece;thegoalistoobtainaunitsquare(sothatyouropponentcantmove). Itsnotasobvious,butthisisalsoanim-likegame.Thetwodimensionsoftherectanglecorrespondtothe numberofpetsoftwodifferenttypes.Forexample,a rectanglecorrespondsto10xolosand7yaks. Therectanglemustbecuteitherverticallyorhorizontally,meaningthatyoucanonlybuyonetypeofpeton agiventurn.Theinterestingtwististhatyoumustkeepthelarger pieceresultingfromacut,whichis equivalenttosayingthatyoumaybuyanynumberofpetsbutonlyuptohalfthenumberofpetsthestore currentlyhas.Forexample,ifthestorehas xolosand yaks,youmaybuyupto xolos,orupto yaks. Buyinganymorethanthatwouldcorrespondtocuttingtherectangleandkeepingthesmaller piece,whichis notallowed. Naturally,Iwonderedwhatthevisualizationofthisgamelookslike.Ifiguredithadtobesomething interesting,andIwasntdisappointed!Hereitis(notethatthebottom-leftsquarerepresents whereasinthevisualizationsfornimandWythoffsgameitrepresented ): here,

Follow

12 of 224

FollowTheMathLess Traveled 10/16/12 16:55


Get everynewpost deliveredto your Inbox.

The Math Less Traveled | Explorations in mathema...

http://mathlesstraveled.com/

Woah,neat!Itlooksasifthelosingsquaresfallalongdiagonallinesofslope forallinteger n(thatis, , , , , ,andsoon)thoughthelinesdontallpassthroughtheorigin.Itsnotsurprisingthatthemain diagonalconsistsofalllosingpositionsthestrategyisthesameasfornim;thefactthatonecanonlybuy uptohalfofacertaintypeofanimaldoesntmakeanydifference.Ifitisyouropponentsturntomoveand thestorehasanequalnumberofxolosandyaks,iftheybuyacertainnumberofyaksyoucanbuythesame numberofxolos,andviceversa.Eventuallythestorewillhaveoneofeach,atwhichpointyouropponent losessincetheycantbuyanymoreanimals(theywouldonlybeallowedtobuy,say,halfayak,butthepet storehasthesensiblepolicyofnotchoppingpetsinhalftoomessy). However,apparentlythereareadditionallosingpositionsotherthanthemaindiagonal.Forexample, accordingtothegraph,ifthestorehas4xolosand19yaks,thenthenextplayertomoveisgoingtolose! Youcanverifyforyourselfthatfromthispoint,theonlylegalmovesaretolightgreen(i.e.winning) positions,andthatfromanyofthesepositionstheotherplayercanmakealegalmovetoanotherdarkblue (i.e.losing)position,andsoon. Somedirectionswecouldgofromhere: Canyouthinkofanyvariantnim-likegamestoexplore?Ihaveaverygeneralprogramforcreatinggame visualizationsliketheabove,soifyoudescribeavariantgameinthecommentsIwillbehappytotryto generateavisualizationforit. Whataboutnim-likegameswiththree(ormore)piles?Tovisualizethestrategiesforthese,wehavetouse three(ormore!)dimensions.Ihopetoeventuallycomeupwithawaytodothis,atleastforthree dimensions.Ihappentoknowthatnimismuchmoreinterestinginthreedimensionsthanintwo! Canyouprovethatthevisualizationoftherectangle-cuttinggamereallylooksliketheabove?Canyoucome upwithanicewaytocharacterizethewinningstrategy,oratleastthelosingpositions?
Posted in games, pattern, pictures | Tagged nim, strategy, visualization, Wythoff | 4 Comments

Follow

13 of 224

FollowTheMathLess Traveled 10/16/12 16:55


Get everynewpost deliveredto your Inbox.

The Math Less Traveled | Explorations in mathema...

http://mathlesstraveled.com/

Searchable tiling database


Posted on August 2, 2012

Justalinktodaycheckoutthisawesometilingdatabase!Itsgottonsofbeautifulplanetilings(with informationandfurtherreadingabouteachone)andmanywaystosearchthroughthedatabase.Itsagreatway tofindexamplesofparticularsymmetriesortypesoftilingsbutitsalsofuntojustoohandahhoverrandom entriesfromthedatabase.

IwishIcouldrememberwhereIfirstcameacrossthis.

Posted in geometry, group theory, links, pattern, pictures | Tagged beauty, database, search, symmetry, tiling | 2 Comments

Book Review: The Enigma of the Spiral Waves


Posted on July 14, 2012

TheEnigmaoftheSpiralWaves(SecretsofCreationVolume2) wordsbyMatthewWatkins,picturesbyMattTweed MatthewWatkinsandMattTweedhavedoneitagain!Ipreviouslywrotea(verypositive) reviewofVolumeIthisbookisjustasengaging,ifnotmore.ItexplainstheRiemann Hypothesisoneofthebiggest,mostmysteriousopenquestionsinmathematicstodayin greatdetail.But,asonemightexpectafterreadingVolumeI,itremainsthoroughly accessible,eventothosewithnotmuchmathematicalbackground.Asamathematical writer,Ifinditincrediblyinspiring:itshowsthatwithenoughtimeandhardwork,itis possibletoexplainverytechnicalideasinawaythatisaccessiblebutstilldetailedand accurate. So,whatistheRiemannHypothesis?Simplyput,itstatesthatthesolutionstothefunction

(where isacomplexnumber)allliealongacertainline.Butthatmakesitsoundboring,likesayingthataroller coasterisamodeoftransportwithwheels.Beforereadingthisbook,though,Ididntknowmuchmorethan that.Ihadntthefaintestideawhyitissointerestinganddeep,orwhyanyonewouldthinkthatsolvingitwould beworthonemilliondollars.Thisbookexplainsallthatandmore.ItturnsoutthattheRiemannHypothesisis intimatelylinkedtothenatureoftheprimenumbers,whichare(still!)quitemysterious.Theyaredefinedby suchasimplerule,butseemtobehavesoerratically!Whatsgoingon? Ofcourse,therearewonderfulpicturestoo.Evenifyoureaditonlyfortheawesomepictures,itsstillworthit. Follow

14 of 224

FollowTheMathLess Traveled 10/16/12 16:55


Get everynewpost deliveredto your Inbox.

The Math Less Traveled | Explorations in mathema...

http://mathlesstraveled.com/

Highlyrecommendedforanyonewhowantsaglimpseintooneofthemostfascinatingandmysteriousopen questionsinmodernmathematics!
Posted in books, open problems, primes, review | Tagged Reimann Hypothesis, spiral, waves | 1 Comment

Blockly
Posted on June 28, 2012

ItseemsthatGoogleisdevelopingagraphicalprogramminglanguagecalledBlockly,inspiredbyScratchbut web-based,withtheabilitytocompiledowntoJavaScript,Dart,orPython(orrawXML,soyoucanprocessit further).IcantsayImallthatexcitedaboutthelanguageitselfnothingnewthere,justthesameoldtired imperativeprogrammingbutitsureisfun!Giveitatrycanyousolvethemaze?Howbigofaprogramdoyou need?


Posted in challenges, programming | Tagged Blockly, Google, graphical, maze, programming | 7 Comments

Picture this
Posted on June 13, 2012

Picturethis!isaverycoolinteractivethingy,madebyJasonDavies,intendedtogetstudents(oranyone,really) thinkingaboutsomeinterestingmath.Goplayaroundwithitandseeifyoucanansweranyofthelisted questions(oranyotherquestionsyoumightcomeupwithyourself).Itturnsouttobequiteintimatelyrelatedto somethingIvewrittenaboutbeforebutIwontspoilitbysayingwhat(atleast,notyet=).

Posted in challenges, links, pattern, pictures | Tagged interactive, picture, rectangles, this | 6 Comments

How to explain the principle of inclusion-exclusion?


Posted on June 11, 2012

Ivebeenremissinfinishingmyseriesofpostsonacombinatorialproof.Istillintendto,butImustconfessthat partofthereasonIhaventwrittenforawhileisthatImsortofstuck.Thenextpartofthestoryistoexplain thePrincipleofInclusion-Exclusion,butIhaventyetcomeupwithacompellingwaytopresentit.SoperhapsI shouldcrowd-sourceit.IfyouknowaboutPIE,howwouldyoumotivateandpresentit?Ordoyouknowofany linkstogoodpresentations?


Posted in combinatorics, meta | Tagged exclusion, inclusion, PIE, presentation | 7 Comments

Follow

15 of 224

FollowTheMathLess Traveled 10/16/12 16:55


Get everynewpost deliveredto your Inbox.

The Math Less Traveled | Explorations in mathema...

http://mathlesstraveled.com/

Wythoffs game at Three-Cornered Things


Posted on June 10, 2012

IvereallybeenenjoyingZacharyAbelsseriesofpostsonWythoffsgame[WythoffsGame:RedorBlue?;A GoldenObservation;TheFibonacciestString;WythoffsFormula],overonhisblogThree-CorneredThings.The Fibonaccinumbersshowupinthestrangestplaces!

Moregenerally,ifyouhaventseenZacharysblogbefore,gocheckitout.Ifyouenjoymyblog,Ithinkyoull enjoyhistoo.
Posted in fibonacci, games, links | Tagged Wythoff's game | 7 Comments

Fibonacci multiples, solution 1


Posted on June 9, 2012

Inapreviouspost,Ichallengedyoutoprove If evenlydivides ,then evenlydivides , ). andconsider .

where denotesthe thFibonaccinumber(

Heresonefairlyelementaryproof(thoughitcertainlyhasafewtwists!).Picksomearbitrary listingnotjusttheFibonaccinumbersthemselves,buttheirremainderswhendividedby choose ,sowewanttolisttheremaindersoftheFibonaccinumberswhendividedby

.Forexample,lets

Herearethefirst17Fibonaccinumbers:

Andherearetheirremainderswhendividedby 0):

,representedgraphically(red=1,orange=2,blankgap=

Remainders of the first 17 Fibonacci numbers mod 3

Follow

16 of 224

FollowTheMathLess Traveled 10/16/12 16:55


Get everynewpost deliveredto your Inbox.

The Math Less Traveled | Explorations in mathema...

http://mathlesstraveled.com/

Andindeed,aswewouldexpectifthetheoremistrue,everyfourthremainderiszero: random!Letstry

isevenlydivisibleby

.Butthereseemstobeabitmorethanthatgoingonthepatternofremainderswegotisdefinitelynot .Herearetheremainderswhenthefirst21Fibonaccinumbersaredividedby5:

The first 21 Fibonacci numbers mod 5

Hmm.Everyfifthremainderiszero,asweexpectedbuttheotherremaindersdontseemtofollowanice patternthistime. ordothey?Actually,ifyoustareatitlongenoughyoullprobablyfindsomepatternsthere!Nottomention thatIhaventreallyshownyouenoughofthesequence.Herearetheremaindersofthefirst41Fibonacci numberswhendividedby5:

First 41 Fibonacci numbers mod 5

Aha!Soitdoesrepeatafterall.Wejusthadntlookedfarenough. Andjustforfun,letsincludesomeFibonaccinumbersmod .Theserepeatmuchmorequicklythanfor .

Fibonacci numbers mod 8

OK,nowthatwevetriedsomespecificvaluesof ,letsthinkaboutthismoregenerally.Whenlistingthe remaindersoftheFibonaccinumbersdividedby ,theinitialpartofthelistwilllooklike

becausealltheFibonaccinumbersbefore

areofcourselessthan

,sowhenwedividethemby

theyare

simplytheirownremainder.Next,ofcourse, Well,

leavesaremainderofzerowhendividedbyitself.Thenwhat? is .OK,sofarwehave

bydefinition,soitsremaindermod

Infact,theruleforfindingthenextremainderinthesequencewillbethesameastheruledefiningtheFibonacci numbers,exceptthatwedoeverythingmod .Sothenextelementinthesequenceis again:


Follow

17 of 224

FollowTheMathLess Traveled 10/16/12 16:55


Get everynewpost deliveredto your Inbox.

The Math Less Traveled | Explorations in mathema...

http://mathlesstraveled.com/

andnow,itseems,wearestuck!Whatdowegetwhenweadd

?Whoknows? by ,likethis:

HereiswhereIwilldosomethingsneaky.Iamgoingtoreplacethesecondcopyof

Huh!?Whatdoesthatevenmean?Surelywecanneveractuallygetanegativenumberasaremainderwhen dividingby .Well,thatstrue,butfromnowon,insteadofstrictlywritingtheremainder when isdividedby .Thisisallthatreallymatters,sinceIjustwant . ,andthenwecan ,Illjustwritesomethingwhichisequivalentto modulo toseewhichpositionsareequivalenttozeromodulo So,theclaimisthat justsubtract Thenwhat? frombothsides. ;then ,andsoon:wegettheFibonacci

.Whyisthat?Well,

numbersinreverse,withalternatingsigns!

Forexample,herearethefirstfewFibonaccinumbersmod8again,butaccordingtotheabovepattern,with negativenumbersindicatedbydownwardspointingbars(andtheoriginalbarsshownmostlytransparent,for comparison):

Seehowthecolorsofthebarsrepeatnow,butrunningforwardsthenbackwards? If iseven,then willbepositive(asyoucanseeintheexampleabove);thatis,weget

Afterthispointweget

,andsoon,andthewholepatternrepeatsagain,asweveseen.

Whatif isodd?Then willbenegative,sowehavetogothroughonemorecyclewitheverythingnegated:

Thisexplainswhywehadtolookatalongerportionoftheremaindersmod5beforetheyrepeated.Heresmod 5again,withnegativesshowngraphically:
Follow

18 of 224

FollowTheMathLess Traveled 10/16/12 16:55


Get everynewpost deliveredto your Inbox.

The Math Less Traveled | Explorations in mathema...

http://mathlesstraveled.com/

Thecolorsofthebarsstillrepeat:forwards,thenbackwardsbutalternatingupanddown,thenforwardsbutall upsidedown,thenbackwardsandalternating(buttheotherway).Butattheendwerebackto ,sothewhole thingwillrepeatagain. Inanycase,whether isoddoreven,thesepatternsofremainderswillkeeprepeatingforever,witha always occurringevery positionsthatis,at ,so willalwaysbedivisibleby !

Thereareotherwaystoprovethisaswell;perhapsIllexplainsomeoftheminafuturepost.Itturnsoutthat theconverseisalsotrue:if evenlydivides ,then mustevenlydivide .Idontknowaproofoffthetopof myhead,butmaybeyoucanfindone?


Posted in fibonacci, modular arithmetic, number theory, pattern, pictures, proof, sequences | Tagged divisibility, fibonacci, proof, remainders | 5 Comments

Nature by Numbers
Posted on June 6, 2012

Thishasbeenmakingtheroundsofthemathblogosphere(blathosphere?),butincaseyouhaventseenityet, checkoutCristbalVilasawesomeshortvideo,NaturebyNumbers.EspeciallyappropriategiventhatIhave beenwritingaboutFibonaccinumberslately(Illpostasolutiontothechallengesoon).


Posted in fibonacci, golden ratio, links, video | Tagged fibonacci, nature, numbers, phi, video | 1 Comment

Fibonacci multiples
Posted on May 15, 2012

Ihaventwrittenanythinghereinawhile,buthopetowritemoreregularlynowthatthesemesterisoverI haveaseriesoncombinatorialproofstofinishup,somebookstoreview,andafewotherthingsplanned.Butto easebackintothings,heresalittlepuzzleforyou.RecallthattheFibonaccinumbersaredefinedby . Canyoufigureoutawaytoprovethefollowingcutetheorem? If evenlydivides ,then evenlydivides .

(Incidentally,theexistenceofthistheoremconstitutesgoodevidencethatthecorrectdefinitionof is ,not .) Forexample, evenlydivides ,andsureenough, enough, evenlydivides ). evenlydivides (inparticular,


Follow

. evenlydivides ,andsure

19 of 224

FollowTheMathLess Traveled 10/16/12 16:55


Get everynewpost deliveredto your Inbox.

The Math Less Traveled | Explorations in mathema...

http://mathlesstraveled.com/

Iknowoftwodifferentwaystoproveit;thereareprobablymore!NeitheroftheproofsIknowisparticularly obvious,buttheydonotrequireanydifficultconcepts.
Posted in arithmetic, challenges, fibonacci, number theory, pattern | Tagged divisibility, fibonacci | 12 Comments

Carnival of Mathematics 86
Posted on May 8, 2012

Welcometothe86thCarnivalofMathematics! nontotient,andnoncototient.Itisalsohappysince reversebut68isnotsemiprime). and

issemiprime, .Infact,itisthe

smallesthappy,nontotientsemiprime(theonlysmallerhappynontotientis68whichis,ofcourse,86in

However,themostinterestingmathematicalfactabout86(inmyopinion)isthatitisthelargestknowninteger forwhichthedecimalexpansionof containsnozeros!Inparticular, Althoughnoonehasproveditisthelargestsuch ,every upto . (whichisquitealot,althoughstill

slightlylessthanthetotalnumberofintegers)hasbeencheckedtocontainatleastonezero.Theprobability thatanylargerpowerof2containsnozerosisvanishinglysmall,givensomereasonableassumptionsaboutthe distributionofdigitsinbase-tenexpansionsofpowersoftwo. Eighty-sixisalsoapparentlysomesortofslangterminAmericanEnglish,butitreallyhasnothingtodowith math,sowhocares?Onwardtothecarnival!Ihadalotoffunreadingallthesubmissions,andhavedecidedto organizethemsomewhatthematicallythoughtheydontalwaysfitperfectly,sodontassumeyouwontbe interestedinapostjustbecauseofmycategorization!

Art

Follow

20 of 224

FollowTheMathLess Traveled 10/16/12 16:55


Get everynewpost deliveredto your Inbox.

The Math Less Traveled | Explorations in mathema...

http://mathlesstraveled.com/

ChristianPerfecthasstartedaseriesofpostsonthethemeofArtyMaths,withlinkstoartisticimagesand videoswithamathematicalbent.Aboveisacoolexample,somesortofstellatedpolyhedronmadeoutofmoney byKristiMalakoff(youcanfindmorehere). KatieStecklessubmittedalinktoRobbyIngebretsensblogpostFirstDigital3DRenderedFilm(from1972)and MyVisittoPixar.Katiesays, Thisispossiblytheearliestexampleofacomputeranimation,andoneofitstwocreators,Edwin Catmull,whowentontofoundPixar,iscreditedwithhavingwork[ed]out[the]mathtohandle thingsliketexturemapping,3Danti-aliasingandz-buffering.Fascinatingtothinkhehadtoinventall ofthatinordertodothis! Robbysblogpost(andtheextensivecommentsonit)givealotmorecontextandfascinatingdetails.And,of course,youcanwatchthevideoitself!

MikeCroucherofWalkingRandomlywritesaboutsomecoolmathematically-themedstainedglasswindows,and wonderswhetheranyoneknowsofanyothers.

Statistics/dataanalysis
ArthurCharpentierofFreakonometricswritesaboutNonconvexity,andplayingindoorpaintball:ifabunchof peopleinanonconvexplayingareaareallholdingwaterpistolsandshootattheclosestperson,whodoesntget wet? KatieStecklessubmittedalinktoData:itshowstoresknowyourepregnant,anarticlebyMatthewLaneof MathGoesPop!Everwonderhowcompaniescanpredictvariousthingsaboutyou(suchaswhetheryouare pregnant!)basedonyourbrowsinghabitsandotherpubliclyavailabledata?Thisarticleexplainssomeofthe basicmathunderlyingthissortofdatamining. JohnCookofTheEndeavouranswersthequestion:Whatisrandomness?inRandomisasrandomdoes.Itturns outthatthebestanswermightjustbetoavoidansweringatall!

Geometry
AugustusVanDusenofthinkingmachineblogsubmittedaposttitledSuperellipse,saying IreadanarticleaboutSergelstorg,aplazainStockholm,beinganexampleofasuperellipse.WhenI lookedupsuperellipseonWolframmathworld,Inoticedthattheareaformulainvolvedgamma
Follow

21 of 224

FollowTheMathLess Traveled 10/16/12 16:55


Get everynewpost deliveredto your Inbox.

The Math Less Traveled | Explorations in mathema...

http://mathlesstraveled.com/

functions.Ithendecidedtoderivetheresultmyselftoseeifitcouldbesimplifiedandhowitwould reducetothefamiliarformulafortheareaofanellipse. FrederickKohofWhiteGroupMathematicssharesageometricsolutiontoanoptimizationproblemthatdoesnt initiallyseemlikeithasanythingtodowithgeometry.

ZacharyAbelofThree-CorneredThingshaswrittenaseriesofthreeexcursionsintothemiraculousand interconnectedworkingsofthehumbletriangle:ManyMorleyTriangles,SeveralSneakyCircles,andThreeCorneredDeltoids.ThesearesomeofmypersonalfavoritesfromthismonthsCarnival:chock-fullofsurprising mathematicsandbeautifulillustrationsandanimations!

Teaching
ColinWrightwritesTheTrapeziumConundrum:howshouldatrapezium(akatrapezoidifyourefromtheUS) bedefinedwithexactlyonepairofparallelsidesoratleastonepairofparallelsides?Moregenerally,howare definitionsarrivedatandagreedupon?Theanswermaydependontheaudience. KarenG.ofschoolaramamusesupontherelationshipbetweenlanguageandlearningplacevalueinherpost LookingtoAsia. OnherblogMathMamaWrites,SueVanHattumwritesaboutLinearAlgebra:LeadingintotheEigenStuff.Sue says,Imteachinglinearalgebraforthefirsttimeinoveradecade.Thathasmeantrelearningitadelightful experience. PaulSalomonofLostInRecursionwritesExponentsandtheScaleoftheUniversea21stCenturymathlesson, afunstoryabouthowaninitiallydrylessononexponentsturnedintoaremarkablelearningexperience.

Fun
AlistairBirdsubmittedalinktoEnormousIntegers,saying, Itsstillacommonenoughmisconceptionthatpuremathematicsresearchmustbeaboutlargerand largernumbers,butitsstillnicetosometimesplayuptothisstereotype,asJohnBaezsblogposton AzimuthaboutEnormousIntegersdoes.Commentsareworthalooktoo. PatBallewwritesonPatsBlogaboutPandigitalPrimes:exploringpandigitalprimesandfindingouthowhandy
Follow

22 of 224

FollowTheMathLess Traveled 10/16/12 16:55


Get everynewpost deliveredto your Inbox.

The Math Less Traveled | Explorations in mathema...

http://mathlesstraveled.com/

Computerprogrammingskillsmightbe. Quick,whatcomesnextintheseries AlexandreBorovik.) ?Theanswer,asexplainedbySteven

Landsburgonhisblog,TheBigQuestions,maysurpriseyou!(ThankstoKatieStecklesforthesubmission,via

PaulSalomonofLostInRecursiondisplaysTheLostinRecursionRecursion.Canyoufigureoutwhatsgoing onwithoutgettinglostintheTheLostinRecursionRecursionrecursion?

StuffThatDidNotFitInAnyOtherCategoryButIsStillAwesome
ColinBeveridgeofFlyingColoursMathssubmittedSecretsoftheMathematicalNinja:Thesurprisingintegration ruleyoudontgettaughtinschool,andwrites, WhenIstumbledacrossthisrule,myreactionwaswhoa.Itsquick,itsextremelydirty,andits surprisinglyaccurate.Thekindofthingthemathematicalninjadreamsof. AndrewTaylorwritesaguestpost,ElectoralReformsandNon-TransitiveDice,onTheAperiodical,explaining WhyChoosingaVotingSystemisHardintermsofasetofnontransitivedice. PeterRowlett,ofTravelsinaMathematicalWorld,opinesinhispost,Whatanicejobyouhave,thatapopular rankinglistingmathematicianasoneofthetoptenbestjobsshouldntjustbeacceptedandrepeated uncritically. InherarticleHowcultureshapedamathematician,CarolClarkgivesaglimpseintothelifeandbackgroundof mathematicianSkipGaribaldi.Shewrites: Mathematiciansseetheworlddifferentlythanme.Iinterviewedamathematiciantogetaglimpseof thatview,andlearnedhoweverythingfromfinearttopopularfilmsandbooksplayedaroleinshaping thatview.

ThepreviousCarnivalofMathematicswashostedatTravelsinaMathematicalWorld;nextmonth,the87th CarnivalwillbehostedbyMr.ChaseatRandomWalks,sostartgettingyoursubmissionsreadynow!Forlistsof pastandfuturecarnivals,instructionsonsubmitting,andanswerstofrequentlyaskedquestions,seethemain CarnivalofMathematicssite.ThenextMathTeachersatPlaycarnivalisalsocomingupsoon,withasubmission


Follow

23 of 224

FollowTheMathLess Traveled 10/16/12 16:55


Get everynewpost deliveredto your Inbox.

The Math Less Traveled | Explorations in mathema...

http://mathlesstraveled.com/

deadlineofthisFriday.
Posted in links | Tagged Carnival of Mathematics | 3 Comments

IllbehostingtheCarnivalofMathematics,andthesubmissiondeadlineiscomingupsoonTuesday,May1.Pleasesubmitsomething!Itcouldbe somethingyouwrote,orsomethingsomeoneelsewrotethatyouenjoyed.Allmathematicsrangingfromelementarytoadvancediswelcome.
Posted on April 23, 2012

Book review: In Pursuit of the Traveling Salesman


Posted on April 7, 2012

Asmathematicalproblemsgo,thetravelingsalesmanproblem(TSP)is araregem:itissimultaneouslyofgreattheoretical,historical,and practicalinterest.Onthetheoreticalfront,itisawell-knownexampleof theclassofNP-completeproblems,whichlieattheheartofthemilliondollarPvsNPquestion(whichIstillintendtoexplainatsomepoint). Historically,ithasbeenstudiedforalmost200years(givenasufficiently inclusivedefinitionofstudy),andhasoccupiedaplaceinthepublic consciousnessforatleastthelast50.Andthisgreathistoricalinterestis partlyduetotheproblemspracticalsignificance. So,whatisit?Givenasetofpointsintheplane(or,moregenerally,aset ofpointswithdistancesspecifiedsomehowbetweeneachpairof points),theproblemistodeterminetheshortestpathwhichvisitsevery pointexactlyonceandthenreturnstothestartinglocation.Ofcourse,in onesensethisiseasy:justlistallpossiblepathsandcomputethe lengthofeach.Note,however,thatforasetof points,thereare (thatis, )possiblepathsthatvisiteverypointonceandthenreturntothestart.Evenwithonly possiblepathsifyoucouldcomputethelengthof years,slightlylongerthan points,thatsawhopping

onetrillionpathseverysecond,itwouldstilltake280millionmillenia(thats

Ivebeenalive)tocheckallofthem!Andthatsonly pointsinpractice,peoplewanttosolvetheTSPforsets ofpointsmuchlargerthan .Sothepointisnotjusttosolvetheproblem;therealquestionis,canitbe solvedefficiently? Amazingly,nooneknows!Butthathasntstoppedpeoplefromcomingupwithextremelycleveralgorithmsthat seemtoworkwellinpractice,onverylargesetsofpoints(i.e.thousands,oreventensofthousandsof points)eventhoughtherearepathologicalinputsforwhichthesealgorithmsdoessentiallynobetterthan justtryingeverypath.SothesealgorithmsconstituteagoodsolutiontotheTSPfromapracticalpointofview butnotatheoreticalone! WilliamCooksnewbook,InPursuitoftheTravelingSalesman:MathematicsattheLimitsofComputation,doesa wonderfuljobpresentingthehistoryandsignificanceoftheTSPandanoverviewofcutting-edgeresearch.Itsa beautiful,visuallyrichbook,fullofcolorphotographsanddiagramsthatenlivenboththenarrativeand mathematicalpresentation.Anditincludesawealthofinformation(perhapsevenabittoomuchattimes;Igot Follow

24 of 224

FollowTheMathLess Traveled 10/16/12 16:55


Get everynewpost deliveredto your Inbox.

The Math Less Traveled | Explorations in mathema...

http://mathlesstraveled.com/

lostinafewplaces).Butitactuallybillsitself,partly,asanintroductiontocutting-edgeideasinTSP researchandIthinkoverallitsucceedsadmirably,explainingideasinwaysthatareaccessiblebutnot patronizing.Readthisbookifyouwantafun,beautifullyillustratedintroductionto(thisonefascinatingpiece of)theedgeofhumanknowledge!


Posted in books, computation, geometry, open problems, review | Tagged book review, salesman, TSP. traveling | 6 Comments

New Carnival of Mathematics


Posted on April 5, 2012

TheCarnivalofMathematicshasbeenrevived!AbigthankstoMikeCroucherofWalkingRandomlyfor organizingitforthepastfewyears,andtoKatieSteckles,ChristianPerfect,andPeterRowlettfortakingover. Thelatestedition,carnival#85(wow,hasitreallybeengoingthatlong?)isnowupatPeterRowlettsblog, TravelsinaMathematicalWorld.Lotsofcoolstuffthere,sobesuretocheckitoutifyouhaventalready. IllbehostingCarnival#86here,sopleasesubmitsomething!ThedeadlineisMay1st,andIllpostthecarnival sometimetheweekafterthat.


Posted in links, meta | Tagged Carnival of Mathematics

Making our equation count


Posted on March 3, 2012

[Thisispost#4inaseries;previouspostscanbefoundhere:Differencesofpowersofconsecutiveintegers, Differencesofpowersofconsecutiveintegers,partII,Combinatorialproofs.] Werestilltryingtofindaproofoftheequation

whichexpressesthefactthatacertainarithmeticprocedurealwaysseemstoresult,strangelyenough,inthe factorialof .LasttimeIintroducedtheideaofusingacombinatorialproof,andgaveasimpleexampleinvolving abinomialcoefficientidentity. Inorderforthisideatoyieldanyfruit,weneedawaytointerpretthevariouspiecesoftheequationascounting something.Letsgooverthepiecesonebyoneanddiscusssomewaystointerpretthemcombinatorially.

Factorial,permutations,andmatchings
Letsstartwiththeright-handside, .Thisoneisnottoohard: countsthenumberofpermutationsof objects,thatis,thenumberofdifferentwaystotake distinctobjectsandarrangetheminanorderedlist.Why isthat?Well,thereare objectswecouldchoosetoputfirst;oncewevemadethatchoice,thereare remainingobjectswecouldchoosetogosecond;then choices.Forexample,herearethe choicesforthethirdobject,andsoon,foratotalof differentpermutationsofsize :

Follow

25 of 224

FollowTheMathLess Traveled 10/16/12 16:55


Get everynewpost deliveredto your Inbox.

The Math Less Traveled | Explorations in mathema...

http://mathlesstraveled.com/

However,theresanotherwaytothinkaboutpermutationswhichwillcomeinhandylater.Namely,wecanthink ofapermutationasamatchingbetweentwosetsofsize .Youknow,likethosepuzzlesthatgivetwo side-by-sidelistsandsaydrawalinematchingeachcartooncharacterwiththeirfavoritecheese!(or whatever).Likethis:

Herewehaveamatchingbetweentwosetsofsize .Eachdotontheleftismatchedwithexactlyonedotonthe right,andviceversa. Whyarematchingsanotherwayofthinkingaboutpermutations?First,itsnottoohardtoseethattherearealso matchingsbetweentwosetsofsize :wehave possiblechoicesofwhattomatchthefirstelementwith; thenthereare choicesleftoverforwhattomatchthesecondelementwith,andsoon.

Butwecanalsoseeacorrespondencebetweenpermutationsandmatchingsmoredirectly.Startbylabelingthe dotsontheleftofamatchingwithconsecutivenumbers:

Now,imagineeachnumbertravelingalongthecorrespondingrededgeuntilitreachesthedotontheother side.Likethis:

Follow

26 of 224

FollowTheMathLess Traveled 10/16/12 16:55


Get everynewpost deliveredto your Inbox.

The Math Less Traveled | Explorations in mathema...

http://mathlesstraveled.com/

Seehowthe traveleddownthesteepedgetoendupatthefourthdotfromthetop;the traveledacrossthe horizontaledgetostayinthesameposition;the traveleduptothetop;andsoon. Whatwegetoutisalistofthenumbersfrom16insomeorder;inthisexampleweget someparticularorder. Hereareallthepermutationsofsize again,thistimevisualizedasmatchings. .Inother

words,wecanviewamatchingasalittlephysicalmachinefortakingalistofobjectsandputtingtheminto

Now,atthispointIamverytemptedtogooffonatangentexploringgrouptheory,symmetrygroups,andall sortsofotherstuff,butIshallrestrainmyself(fornow!).

Binomialcoefficients
Anotherpieceoftheequationisthebinomialcoefficient coefficientscount asize- set.(Ialsotalkedaboutthislasttime.) .Butofcoursewealreadyknowwhatbinomial isthenumberofwaystochoose thingsoutof ,thatis,thenumberofsize- subsetsof

Exponentiationandfunctions
Whatabout ?Whatdoesthatcount?Itturnsoutthatexponentiationcorrespondstocountingfunctions:in particular, isthenumberoffunctionsfromasetofsize toasetofsize .Whyisthat?Well,foreachofthe elementsofthedomain,wehave choicesforwhereafunctioncouldsendit,andeachofthesechoicesis independentsothetotalnumberofchoicesis Forexample,hereareallofthe .

functionsfromasize- settoasize- set:

Hmmmthislooksfamiliar!Notethatsomeofthesefunctionsarematchings,andsomearent.Perhapsyoure startingtogetaninklingnowwhyIintroducedtheideaofpermutationsasmatchings Allthepiecesarealmostinplacenow.Theonepieceoftheequationwestillhaventyettalkedaboutisthat

Follow

27 of 224

FollowTheMathLess Traveled 10/16/12 16:55


Get everynewpost deliveredto your Inbox.

The Math Less Traveled | Explorations in mathema...

http://mathlesstraveled.com/

mysterious

.Itcertainlydoesntmakesensetointerpretthatasthenumberoffunctionsfromasetofsize

toasetofsizenegativeone,becauseofcoursethereisnosuchthingasasetwithanegativesize.Sohow canweinterpretitcombinatorially?TheanswerliesinsomethingcalledthePrincipleofInclusion-Exclusion(or PIEforshort),whichwillbethesubjectofmynextpost!


Posted in combinatorics, pictures | Tagged binomial coefficients, combinatorics, functions, matching, permutation | 3 Comments

Combinatorial proofs
Posted on February 17, 2012

Continuingfromapreviouspost,wefoundthatifwebeginwith thpowersof

consecutiveintegersand

thenrepeatedlytakesuccessivedifferences,itseemslikewealwaysendupwiththefactorialof ,thatis, .Wethenderivedanalgebraicexpressionfortheresultoftheiterativedifferenceprocedure.So thegoalnowistoprovethat

thatis,

Now,itspossible(probable,infact)thatthiscanbeprovedbypurelyalgebraicmeans.Ifyoucomeupwith suchaproofIwouldlovetoseeit!ButImustconfessthatIspentseveralhoursbangingmyheadagainstit (algebraicallyspeaking)withoutmakinganyprogress.EventuallyIturnedtotheideaofacombinatorialproof. WhatdoImeanbythat?Combinatoricsisthesubfieldofmathematicsconcernedwithcounting.Theessenceofa combinatorialproofistoshowthattwodifferentexpressionsarejusttwodifferentwaysofcountingthesame setofobjectsandmustthereforebeequal.Ivedescribedsomecombinatorialproofsbefore,incountingthe numberofwaystodistributecookies. Asanothersimpleexample,considerthebinomialcoefficientidentity

Itscertainlypossibletoprovethisalgebraically,byexpandingoutthebinomialcoefficients(using butwecangiveamoreelegantproof,basedonthefactthat thingsoutofasetof things.Forexample,herearethe isthenumberofwaystochooseasubsetof waystochoosethreethingsoutasetoffive:

),

Follow

28 of 224

FollowTheMathLess Traveled 10/16/12 16:55


Get everynewpost deliveredto your Inbox.

The Math Less Traveled | Explorations in mathema...

http://mathlesstraveled.com/

Considerthefirstelementofthesize- set.Everysubsetofsize eitherincludesthisfirstelement,oritdoesnt. Thenumberofsize- subsetswhichdonotincludethefirstelementis choose thingsfromtheremaining elementis . Heresanillustrationofhowthisworksintheparticularcasewhen and : ,becausetheycorrespondtochoosing ,sincethatsthenumberofwaysto things.Therefore elements.Thenumberofsize- subsetswhichdoincludethefirst oftheremaining

Noticehowthetensubsetsfromabovehavebeensplitintotwogroups:thefirstgroup,ontheleft,arethose thatdontincludethefirstelement;youcanseethateachofthemcorrespondstooneofthe correspondstooneofthe size- subsetsoftheremainingfourelements. size- subsets oftheremainingfourelements.Thesecondgroup,ontheright,arethosethatdoincludethefirstelement;each

Sothatstheideaofacombinatorialproof.Andwewanttodosomethingsimilarfortheidentitywearetrying toprovealthough,ofcourse,itsgoingtobeabitmoredifficult! Youmighthavefuntryingtothinkaboutwhatacombinatorialproofofourtargetequationmightlooklike; althoughifyoudonthavemuchexperiencewithcombinatoricsyoumayhavetroublecomingupwithwhatsorts ofthingsthetwosidesoftheequationmightbecounting!ThatswhatIlltalkaboutinmynextpost.


Posted in combinatorics, pictures, proof | Tagged binomial coefficients, combinatorial proof, identity | 12 Comments

Differences of powers of consecutive integers, part II


Posted on February 16, 2012

IfyouspentsometimeplayingaroundwiththeprocedurefromDifferencesofpowersofconsecutiveintegers (namely,raise consecutiveintegerstothe thpower,andrepeatedlytakepairwisedifferencesuntilreaching asinglenumber)youprobablynoticedthecuriousfactthatitalwaysseemstoresultinafactorialinthe factorialof ,tobeprecise. Forexample:


Follow

29 of 224

FollowTheMathLess Traveled 10/16/12 16:55


Get everynewpost deliveredto your Inbox.

The Math Less Traveled | Explorations in mathema...

http://mathlesstraveled.com/

Severalcommentersfiguredthisoutaswell.Doesthisalwayshappen?Ifso,canweproveit? Letsstartbythinkingaboutwhathappenswhenwedothesuccessive-differencingprocedure.Ifwestartwith thelist ,thenweget .(Iwanttokeepthelettersinorder,whichiswhyIwrote ,weget . (Thenegationof is ;addingthisto yields .)From weget insteadof . Insteadofsubtractingthefirstvaluefromthesecond,wecanthinkofitasaddingthenegationofthefirstvalue tothesecond.)Ifwestartwith

Doyouseeanypatternsyet?Letsdoonemore.Fromtheabovecalculationwecanalreadyseethatdoingfour iterationson givesus . Hmm.Letsmakeatable. Result 1 2 3 4 5 Iincludedonemorerow(whichyoucanverifyifyoulike).Nowdoyouseeapattern?Thecoefficientsseemto betakenfromPascalstriangle,butwithalternatingsigns! Infact,itsactuallynottoohardtoseewhythishappens.Ateachstepwetaketwooffsetcopiesoftheprevious row(by"offset"Imeanthatthelettersareshiftedbyone,like and )andaddthenegationof
Follow

willresultin

(doyouseewhy?).Doingonefinaliteration

30 of 224

FollowTheMathLess Traveled 10/16/12 16:55


Get everynewpost deliveredto your Inbox.

The Math Less Traveled | Explorations in mathema...

http://mathlesstraveled.com/

thefirsttothesecond.Sincethesignsarealternating,wereallyendupaddingabsolutevaluesofthe coefficients.Probablythebestwaytoreallyseethisisthroughanexample:

Noticehowweflipallthesignsinthefirstrow,sothattheymatchthesignsinthesecondrow.Butthisis exactlyhowPascalstriangleisgeneratedeachrowisthesumofthepreviousrowwithitself,offsetbyone. Now,intherealproblem,wedontstartwith ,butwiththe thpowersof consecutiveintegers. .Giventhis,wecan

Letscallthefirstinteger ,sothesequenceofconsecutiveintegersis

nowwritedownanexpressionforwhatweendupwithafterdoingtheiterateddifferenceprocedure:

Letsbreakthisdownabit.Weknowthatwegetasumof needtoinclude equalto when Finally,ofcourse,

terms;thatsthe

part(youcanreadmore ,whichis

aboutsigmanotationhere).Welluse toindextheterms.Wealsoknowthatthetermsalternatesign,sowe raisedtosomepowerinvolving ;thelasttermisalwayspositive,soweuse .Thebinomialcoefficient isthe thpowerofoneoftheintegerswestartedwith. givesusthe thentryonthe throwofPascalstriangle.

Theclaim,therefore,isthat

AndIwillproveittoyou,withprettypictures,aspromised!
Posted in arithmetic, iteration, pascal's triangle | Tagged binomial coefficients, consecutive, difference, integers, powers | 3 Comments

1717 4-coloring with no monochromatic rectangles


Posted on February 9, 2012

Quick,whatsspecialaboutthefollowingpicture?

Follow

31 of 224

FollowTheMathLess Traveled 10/16/12 16:55


Get everynewpost deliveredto your Inbox.

The Math Less Traveled | Explorations in mathema...

http://mathlesstraveled.com/

AsjustannouncedbyBillGasarch,thisisa

gridwhichhasbeenfour-colored(thatis,eachpointinthe

gridhasbeenassignedoneoffourcolors)insuchawaythattherearenomonochromaticrectangles,thatis,no fourgridpointsformingthecornersofanaxis-alignedrectangleareallofthesamecolor.Forexample,ifwe changethetop-leftgridpointtored,wecanseeseveralmonochromaticrectanglespopup:

OrheresanotherversionwhereIrandomlypickedagridpointinthemiddle,changeditscolor,andsureenough, moremonochromaticrectanglesresult:

Asyoucantryverifyingforyourself(andasIalsoverifiedusingacomputerprogram),therearenosuch monochromaticrectanglesinthefour-coloringatthetopofthispost!(Ifyouwanttoplaywiththefour-coloring yourself,hereitisinasimpledataformat.) Forseveralyearsnooneknewifthiswaspossible,andBillhadofferedaprizeof$289(thats ,ofcourse)to

anyonewhocouldfindsuchafour-coloring.TheprizewillbecollectedbyBerndSteinbachandChristian PosthoffyoucanfindmoredetailsinBillspost.Nooneyetknowsexactlyhowtheyfoundtheirfour-coloring,
Follow

32 of 224

FollowTheMathLess Traveled 10/16/12 16:55


Get everynewpost deliveredto your Inbox.

The Math Less Traveled | Explorations in mathema...

http://mathlesstraveled.com/

butapparentlytheywillbepresentingapaperaboutitinMay.Illtrytowritemoreaboutitthen(ifI understanditatall)! Ifyoureinterestedinreadingmoreaboutthehistoryandmathbehindthisproblem(andtogetsomeintuition forwhyitisdifficult),takealookatthesepostsbyBrianHayesonhisblog,bit-player:The1717challengeand 17x17:Anonprogressreport.IalsorememberseeingHeresafuninteractiveappletwhereyoucanplayaround withtheproblem,createdbyMartinSchweitzer.


Posted in open problems, pattern, people, pictures | Tagged 17x17, four-coloring, graph, grid, monochromatic, rectangles | 5 Comments

Book review: Nine Algorithms that Changed the Future


Posted on February 4, 2012

NineAlgorithmsthatChangedtheFuture:theIngeniousIdeasthatDriveTodays Computers,byJohnMacCormick.PrincetonUniversityPress,2012. Imoftenwaryofbookswrittenforgeneralaudiencesontechnicaltopics.Itsquite difficulttowriteinawaythatisbothaccessibletoawideaudienceandtechnically accurate.Manysuchbooksendupsacrificingaccuracyinthenameofaccessibility, tryingtoconveyjusttheintuitionorgeneralsenseofsometopic,butoftenend upgivingpeoplethewrongideainstead. Iwasquitehappytofind,therefore,thatJohnMacCormicknailsit:9Algorithms thatChangedtheFutureistechnicallyrightonthemoney,butmanagestoexplain thingsinwaysthatarebothunderstandableandfun.Wanttounderstandhow Googlerankssearchresults?OrhowAmazonmanagestoneverloseormessupyourorderinformation,even thoughtheygethundredsofthousandsoforderseachdayand(asweallknow)networksandharddrivesare unreliable?Everwonderhowyoucanordersomethingovertheinternetwithoutyourcreditcardnumberbeing stolen?Orhowzipisabletomakeyourfilessmaller,seeminglybymagic?Evenifyouhaveneverwondered aboutthesethings,perhapsIhavemadeyouwonderaboutthemjustnow.Andthatsexactlythepointofthis book:therearequiteafewingeniousalgorithmicideasthatmostofusrelyoneverydaythatwerarelyor neverevenstoptowonderabout. Forexample,Iactuallylearnedsomethingnew:Iknewaboutpublic-keycryptographybuthadneverreally knownmuchaboutDiffie-Hellmankeyexchange,whichiswhatallowsyourwebbrowsertotalkto,say, Amazonsserverssecurelyeventhoughtheyhavenevercommunicatedbefore.Itslikehavingasecret conversationincodewithapen-palwhomyouvenevermet,eventhoughlotsofpeoplearereadingyourmail. Howcanyoueveragreeonasecretcodeinthefirstplacewithoutthepeoplereadingyourmailfindingout(and hencebeingabletoreadallyoursubsequentcodedmessages)?Soundsimpossible,doesntit?Butitturnsout thatitispossible,withsomecleverideas,whichMacCormickskillfullyexplainsusingafunmetaphorabout mixingcolorsofpaint. Eachchapterstartsoutverysimply,graduallybuildingupmorecomplexexamplesuntilyoureachafull understandingofthealgorithmbeingexplained.AlongthewayMacCormickintroducesthetrickstheclever,
Follow

33 of 224

FollowTheMathLess Traveled 10/16/12 16:55


Get everynewpost deliveredto your Inbox.

The Math Less Traveled | Explorations in mathema...

http://mathlesstraveled.com/

centralideasthatmakeeachalgorithmwork.Thewritingisexcellent:clear,precise,andfun.Ihighly recommendthisbooktoanyonecuriousabouttheingeniousmathematicalandalgorithmicideasunderlying someoftodaysmostubiquitoustechnology.


Posted in books, computation, review | Tagged algorithms, history, John MacCormick

Computing with decadic numbers


Posted on January 30, 2012

[Thisistheninth,and,Ithink,finalinaseriesofpostsonthedecadicnumbers(previousposts:Acuriosity,An invitationtoafunnynumbersystem,Whatdoes"closeto"mean?,Thedecadicmetric,Infinitedecadicnumbers, Morefunwithinfinitedecadicnumbers,Aself-squarenumber,u-tube).] Inapreviouspost,wefoundadecadicnumber

withthecuriouspropertythatitisitsownsquare,eventhoughitisobviouslynotzeroorone.Wethenderiveda moreefficientalgorithmforgeneratingthedigitsof .HeressomeHaskellcode(explainedinthepreviouspost) whichimplementsthemoreefficientalgorithm,whichIincludeherejustsothatthispostwillbeavalidliterate Haskellfileinitsentirety.

Follow

34 of 224

FollowTheMathLess Traveled 10/16/12 16:55


Get everynewpost deliveredto your Inbox.

The Math Less Traveled | Explorations in mathema...

http://mathlesstraveled.com/

> {-# LANGUAGE TypeSynonymInstances > > > > module Decadic2 where > > import Control.Monad.State > > -- State for incrementally constructing u_n. > -- Invariant: curT = 10^n; un^2 = pn*curT + un > data UState = UState { pn > > > > > > -- u_1 = 5; > > uStep :: State UState Int > uStep = do > > > > > > > > > > > > > type Decadic = [Int] > > u :: Decadic > u = 5 : evalState (sequence $ repeat uStep) initUState return $ fromInteger d -- return the new digit put (UState p' u' (10*t)) -- record the new values let d u' p' = p `mod` 10 = d * t + u -- next digit -- prepend the next digit to u -- see above proof u <- gets un p <- gets pn t <- gets curT 5^2 = 25 = 2*10 + 5 > initUState = UState 2 5 10 deriving Show , un , curT } :: Integer :: Integer :: Integer #-} , FlexibleInstances

= (p + 2*d*u + d*d*t) `div` 10

Toroundthingsout,Idliketoshowoffsomeofthecoolthingswecandowiththis.First,asweknow,its possibletodoarithmeticwithdecadicnumbers.Soletsimplementit! Additionofdecadicnumbersisdonejustlikeadditionoftheusualdecimalnumbers:weaddcorresponding places(i.e.,lineupthenumbersoneundertheotherandthenaddincolumns).

> plus :: Decadic -> Decadic -> Decadic

First,wehavespecialcasesforzero,representedbytheemptylistofdigits:inthosecaseswejustreturnthe othernumberunchanged.

Follow

35 of 224

FollowTheMathLess Traveled 10/16/12 16:55


Get everynewpost deliveredto your Inbox.

The Math Less Traveled | Explorations in mathema...

http://mathlesstraveled.com/

> plus [] n2 = n2 > plus n1 [] = n1

Next,toaddadecadicnumberwhosefirstdigitisxtoadecadicnumberwhosefirstdigitisy,wejustaddxandy andthencontinueaddingrecursively.

> plus (x:xs) (y:ys) = (x+y) : plus xs ys

Ofcourse,werenotdone:thisdoesntdoanycarrying.Insteadofmodifyingourplusfunctiontodocarrying,we justwriteafunctionnormalizewhichmakessureeveryplaceinadecadicnumberisbetween and ;itwillcome inhandyformorethanjustaddition.

> normalize :: Decadic -> Decadic

Thenormalizefunctionsimplycallsarecursivehelperfunctionnormalize'whichkeepstrackofthecurrent"carry". Thestartingcarry,ofcourse,iszero.

> normalize = normalize' 0

Tonormalizezero(theemptylist)whenthecurrentcarryiszero,justreturntheemptylist.

>

where normalize' 0 [] = []

Withanonzerocarryandtheemptylist,wesimplyextendthelistwithaspecialzerodigitandcontinue normalizing.

>

normalize' carry [] = normalize' carry [0]

Inthegeneralcase,weaddthecurrentcarrytothenextdigitx,andcomputethequotientandremainderwhen dividingthissumbyten.Theremainderisthenextdigitd,andthequotientisthenewcarrywhichgetspassed alongrecursively.

> >

normalize' carry (x:xs) = d : normalize' carry' xs where (carry', d) = (carry + x) `divMod` 10

Andnowformultiplication,whichisbasedontheobservationthatzerotimesanythingiszero,andinthe generalcase
Follow

36 of 224

FollowTheMathLess Traveled 10/16/12 16:55


Get everynewpost deliveredto your Inbox.

The Math Less Traveled | Explorations in mathema...

http://mathlesstraveled.com/

> mul :: Decadic -> Decadic -> Decadic > mul [] _ = [] > mul _ [] = [] > mul (x:xs) (y:ys) = x*y : (map (x*) ys + (xs * (y:ys)))

Finally,wedeclareDecadictobeaninstanceoftheNumclass,whichallowsustousedecadicnumbersinthesame waysthatwecanuseothernumerictypes:

> instance Num Decadic where

Toaddormultiplydecadicnumbers,usetheplusandmulfunctionsandthennormalize.

> >

n1 + n2 = normalize (plus n1 n2) n1 * n2 = normalize (mul n1 n2)

Tonegateadecadicnumber,subtractthelastdigitfrom10andtherestofthedigitsfrom9.

> > > >

negate [] = [] negate (x:xs) = normalize $ (10-x) : negate' xs where negate' [] = repeat 9 negate' (x:xs) = (9-x) : negate' xs

Finally,toconvertanintegerintoadecadicnumber,puttheintegerintoalistofoneelementandnormalize.

>

fromInteger = normalize . (:[]) . fromInteger

So,letstryit!Wellwantawaytodisplaydecadicnumbers:

> showDecadic :: Decadic -> IO () > showDecadic d = putStrLn . dots $ digits > > > > where d' = take 31 d | otherwise digits = = ("..." ++) dots | length d' <= 30 = id concat . reverse . map show . take 30 $ d'

Normaldecimalintegerscanalsobeusedasdecadicnumbers:

Follow

37 of 224

FollowTheMathLess Traveled 10/16/12 16:55


Get everynewpost deliveredto your Inbox.

The Math Less Traveled | Explorations in mathema...

http://mathlesstraveled.com/

*Decadic2> showDecadic 7 7

Heres :

*Decadic2> showDecadic u ...106619977392256259918212890625

Andheres ;ithadbetterbethesameas !

*Decadic2> showDecadic (u^2) ...106619977392256259918212890625

Well,lookslikeitsthesameforthefirst30digitsatleast!Wecanalsocompute ,so thoughttheremightbeaself-squarenumberendingin ?Well,thisisit!

.Remember,if

then

shouldbeanotherself-squarenumber.Rememberhowwe

*Decadic2> showDecadic (1-u) ...893380022607743740081787109376 *Decadic2> showDecadic ((1-u)^2) ...893380022607743740081787109376

Finally,wecancheckthat

*Decadic2> showDecadic (u * (1-u)) ...000000000000000000000000000000

Ifyourecall,thisisinsomesensethefundamentalreasonwhythedecadicnumbersactsofunny,becauseithas zerodivisors:pairsofnumbers(like and ),neitherofwhichiszero,whoseproductisnonethelesszero.

Now,ifyouremember,fromevenfurtherback,whatgotusintothiswholedecadicmessinthefirstplace:

Follow

38 of 224

FollowTheMathLess Traveled 10/16/12 16:55


Get everynewpost deliveredto your Inbox.

The Math Less Traveled | Explorations in mathema...

http://mathlesstraveled.com/

Inthatfirstpost,Isaid ImanagedtoextendthispatternforafewmoredigitsbeforeIgotbored.Doesitcontinueforeveror doesiteventuallystop?Isthereanydeepermathematicalexplanationlurkingbehindthissupposed curiosity?Whatssospecialabout ?Dopatternslikethisexistforotherfunctions?

Well,bythispointIhopeitsclearthatthereisindeedadeepermathematicalexplanationlurking!Theequation

admitsthesolutions

and and

,butdoesitadmitanyotherdecadicsolutions?Noticethatgiven assolutions,then . (and )arealsosolutions:

,whichhas

Sointhiscaseweget

asasolution(theothersolutionisnotadecadicinteger). Toimplementit,weneedawaytohalvedecadicnumbers(Illletyouworkoutwhatsgoingonhere):

> halve :: Decadic -> Decadic > halve [] = [] > halve t@(s:_) > > > > > > > > | odd s where halve' [] = [] halve' [x] = [x `div` 2] halve' (x:x':xs) = (x `div` 2 + adj) : halve' (x':xs) where adj | odd x' = 5 | otherwise = 0 = error "foo" | otherwise = halve' t

Andnowwecandefine

> q = halve (3*u - 1)

*Decadic2> showDecadic q ...159929966088384389877319335937 *Decadic2> showDecadic (2*q^2 - 1) ...159929966088384389877319335937

Woohoo!Thisclearlyshowsthatthepatterndoes,infact,continueforever.Italsoshowsusthat

is
Follow

39 of 224

FollowTheMathLess Traveled 10/16/12 16:55


Get everynewpost deliveredto your Inbox.

The Math Less Traveled | Explorations in mathema...

http://mathlesstraveled.com/

notparticularlyspecial:anyquadraticfunctionthatfactorsas likethis,andprobablylotsofotherequationsdotoo.

,attheveryleast,willleadtoapattern

Ifyoureinterestedinreadingmore,hereswhereIgotsomeofmyinformation.Forexample,youcanread abouthowthereisanothernumber ,definedbystartingwith anditerativelyraising .Iteven tothefifthpower(justaswedefined bystartingwith andsuccessivelysquaring),suchthat promptedbymyblogposts!Isnttheinternetgreat?


Posted in arithmetic, programming | Tagged computing, decadic, numbers | 2 Comments

seemsthattheauthorofthatpage,GrardMichon,hasrecentlyaddedadiscussionofthisveryproblem,

Differences of powers of consecutive integers


Posted on January 29, 2012

PatrickVennebushofMathJokes4MathyFolksrecentlywroteaboutthefollowingprocedurethatyields surprisingresults.Choosesomepositiveinteger .Now,startingwith consecutiveintegers,raiseeach integertothe thpower.Thentakepairwisedifferencesbysubtractingthefirstnumberfromthesecond,the secondfromthethird,andsoon,resultinginalistofonly numbers.Dothesamethingagain,resultingin numbers,andrepeatuntilyouareleftwithasinglenumber. Forexample,supposewechoose alltothefourthpower,givingus ,andstartwiththefiveconsecutiveintegers .Weraisethem

Nowwetakepairwisedifferences: newlist

,then

,andsoon,andwegetthe

Repeatingthedifferenceproceduregives

OK,soweget .Sowhat? Well,ifyoutryenoughexamples,youmaynoticeasurprisingpattern.Illletyouplaywithitforawhile.Over thecourseofafewfuturepostsIllexplainthepatternandprovethatitalwaysholdsbuttheproofwillbea reallycoolcombinatorialone,withprettypictures!


Posted in arithmetic, pattern | Tagged consecutive, difference, integers, powers, surprising | 16 Comments

Follow

40 of 224

FollowTheMathLess Traveled 10/16/12 16:55


Get everynewpost deliveredto your Inbox.

The Math Less Traveled | Explorations in mathema...

http://mathlesstraveled.com/

u-tube
Posted on January 10, 2012

[Thisistheeighthinaseriesofpostsonthedecadicnumbers(previousposts:Acuriosity,Aninvitationtoa funnynumbersystem,Whatdoes"closeto"mean?,Thedecadicmetric,Infinitedecadicnumbers,Morefunwith infinitedecadicnumbers,Aself-squarenumber).] Inmypreviouspost,wefoundadecadicnumber

withthecuriouspropertythatitisitsownsquare. Wealsodiscoveredanalgorithmforcomputing :startingwith (thatis,wekeeponlythelast . (Asanaside,noticewhyweareallowedtodefine asalimit:becauseofthefunnydecadicmetric,thedistance betweensuccessive isactuallygettingsmallerbyafactorof andthatsomethingis .) So, eachtime;theyareconvergingonsomething, digits)togiveus ,wesquareeach andtaketheremainder .Then isdefinedasthelimitofthe as

Wecanprettyeasilyturnthisintoacomputerprogramtocompute toasmanydigitsaswelike.Asusual,Im usingmyfavoriteprogramminglanguage,Haskell.ItdoesntmatterifyoudontknowHaskell,Illtrytoexplain thingswellenoughthatyoucanstillfollowalonganyway. Firstwejustneedtoimportsomethingwellneedlater.

> module Decadic where > > import Control.Monad.State

Now,wedefineu1tobe5,andthefunctionnextUtakes and asinputandproduces

> u1 :: Integer > u1 = 5 > > nextU :: Integer -> Integer -> Integer > nextU n un = (un^2) `mod` (10^(n+1))

Nowwecandefinethe(infinite!)listof ,whichwellcallus.generateUstakesthecurrentvaluesof and and

Follow

41 of 224

FollowTheMathLess Traveled 10/16/12 16:55


Get everynewpost deliveredto your Inbox.

The Math Less Traveled | Explorations in mathema...

http://mathlesstraveled.com/

generatesalistwith asitsfirstelement,andtheremainderofthelistcomputedbyarecursivecallto
generateUs(with

and

asitsinputs).usjustgetsthingsstartedbycallinggenerateUswith

and

> us :: [Integer] > us = generateUs 1 u1 > where generateUs n un = un : generateUs (n+1) (nextU n un)

Letsseeifitworksbyaskingforthefirsttenelementsofthelistus:

*Decadic> take 10 us [5,25,625,625,90625,890625,2890625,12890625,212890625,8212890625]

Seemsgood! However,allisnotroses.Letsthinkaboutwhatourprogramhadtodoinordertocalculate ,itfirstsquaredittoget workneededtosquare butwaitaminute! .Given endsin

ofcourseitdoes,sincethatsthedefiningpropertyofthe .Butthatmeansthathalf(moreorless)ofthe wascompletelywasted!Theresnopointincomputingthelasthalfof ,since wealreadyknowitisgoingtobe .Weonlycareabouttherestofit. Realizingthatourprogramisdoingtoomuchworkisonething;turningthisintuitionintoactualimprovements isquiteanother!Initiallyitwasfarfromobvioustomehowtoavoidtherepeatedwork.ButIfinallyfiguredit out. Thekeyistorememberateachstepnotjust ,but .However,sinceweknowthat endswith ,wecan remember alongwiththeprefix(callit )of thatcomesbeforetheending .Forexample,for remember . Thekeyquestionnowis:given and ,howdowecompute thelastdigitof ;then Itscomputing out . ,theideaistoexpand : and ?Well,computing iseasy:itsjust tobe and (since ).Thatis,wewillalwaysinsistthat well

withanextradigittackedon,namely,thefinaldigitof .Inparticular,wecandefine

whichisthetrickypart.Sincewewanttohave

,manipulateitintothedesiredform,andseewhatwegetfor

Now,notethatwecanbreak intotwoparts:thelastdigit(whichwerecalling

)andtherest.Thatis,

Follow

42 of 224

FollowTheMathLess Traveled 10/16/12 16:55


Get everynewpost deliveredto your Inbox.

The Math Less Traveled | Explorations in mathema...

http://mathlesstraveled.com/

where

denotesroundingdowntothenearestinteger,so

meanstodivide by10andthrowawaythe

remainderthatis, withoutitsfinaldigit.Continuing,

andtherewehaveit!Wehaveexpressed

intheform

.Inparticular,

Noticethat

isactuallyguaranteedtobeaninteger,since isdivisibleby5thisisthesame

phenomenonweranacrossinprovingthatoursimplealgorithmforcomputing workscorrectly.Sowecan factoroutadivisionbyten(divisionisgenerallyslowsowedratheronlydoitonce):

Andnowforsomecode!Asonefinaloptimization,insteadofkeepingtrackof ,wekeeptrackof donthavetokeeprecomputing everyiteration. :

,sowe

First,thedatastructureweusetokeeptrackofthecurrentvaluesof , ,and

> -- State for incrementally constructing u_n. > -- Invariant: curT = 10^n; un^2 = pn*curT + un > data UState = UState { pn > > > > deriving Show , un , curT } :: Integer :: Integer :: Integer

Theinitialstatefor

has

,and

> -- u_1 = 5;

5^2 = 25 = 2*10 + 5

> initUState = UState 2 5 10

TheuStepfunctionistheheartofthealgorithm:ittakesthecurrentvaluesof , ,and , ,and .Italsoreturns

andupdatesthemto

asitsresult,whichwecanusetobuildup digit-by-digit.

Follow

43 of 224

FollowTheMathLess Traveled 10/16/12 16:55


Get everynewpost deliveredto your Inbox.

The Math Less Traveled | Explorations in mathema...

http://mathlesstraveled.com/

> uStep :: State UState Int > uStep = do > > > > > > > > > > > return $ fromInteger d -- return the new digit put (UState p' u' (10*t)) -- record the new values let d u' p' = p `mod` 10 = d * t + u -- next digit -- prepend the next digit to u -- see above proof u <- gets un p <- gets pn t <- gets curT

= (p + 2*d*u + d*d*t) `div` 10

So,didwegainanything?Asaconcretecomparison,letsseehowlongittakestocompute simplecode,ittakes7.2seconds:

.Usingourfirst,

*Decadic> :set +s *Decadic> length . show $ us !! 10000 10001 (7.23 secs, 208746872 bytes)

(Ijusthaditprintthenumberofdigitsof

toavoidwastingatonofspaceprintingouttheentirenumber.)

Andusingournewandhopefullyimprovedcode?

*Decadic> length . show . un . flip execState initUState $ replicateM_ 10000 uStep 10001 (1.55 secs, 225857080 bytes)

Only1.5seconds!Nice! TheothernicethingaboutuStepisthatitspitsoutonedigitof atatime,whichwecanusetodefine asan (infinite)listofdigitsasifthedigitswerecomingoneatatimedowna"tube",a -tube,onemightsaygetit, a tubehehnevermind.

> type TenAdic = [Int] > > u :: TenAdic > u = 5 : evalState (sequence $ repeat uStep) initUState

*Decadic> reverse $ take 20 u [9,2,2,5,6,2,5,9,9,1,8,2,1,2,8,9,0,6,2,5]

Follow

44 of 224

FollowTheMathLess Traveled 10/16/12 16:55


Get everynewpost deliveredto your Inbox.

The Math Less Traveled | Explorations in mathema...

http://mathlesstraveled.com/

Nifty!Nexttimetherealfunbegins,asIshowoffsomecoolthingswecandowithourshinynew implementationof .
Posted in computation, convergence, infinity, iteration, modular arithmetic, number theory, programming | Tagged decadic, Haskell, idempotent, streaming, u | 2 Comments

Herbert Wilf: 13 June 1931 7 January 2012


Posted on January 9, 2012

IwassadtolearnthatHerbertWilfdiedyesterday.Long-timereadersofthisblogmayrememberhimasoneof thediscoverersoftheCalkin-Wilftree,whichIwroteaboutinaten-partseriesofposts(1,2,3,4,5,6,7,8,9, 10).

The Calkin-Wilf Tree

Healsowrotegeneratingfunctionology,atextbookaboutgeneratingfunctions,atopicnearanddeartomyheart (whichIhopetowriteaboutheresomeday). AlthoughhewasanemeritusprofessorattheUniversityofPennsylvania(whereIamcurrentlydoingmyPhD)I sadlynevergotachancetomeethim.


Posted in people | Tagged Herbert Wilf | 1 Comment

A self-square number
Posted on January 4, 2012

[Thisistheseventhinaseriesofpostsonthedecadicnumbers(previousposts:Acuriosity,Aninvitationtoa funnynumbersystem,Whatdoes"closeto"mean?,Thedecadicmetric,Infinitedecadicnumbers,Morefunwith infinitedecadicnumbers).Iknowit'sbeenawhilesinceI'vewrittenonthistopic,soifyou'vebeenfollowing along,youmightwanttogobackandrefreshyourmemory.] Finally,aspromised,Icanshowyouthestrangenumberuwhichisitsownsquare(butwhichisntzeroorone!). Upuntilnowallthedecadicnumbersweveconsideredhavebeenequivalenttofamiliarrationalnumbers,but zeroandonearetheonlyrationalnumberswhicharetheirownsquare;clearlyumustbesomethingquite different! Assumingthatsuchaucouldexistandassumingitsadecadicinteger ,thatis,hasnodigitstotherightofthe decimalpointletsthinkforaminuteaboutwhatucouldpossiblybe.Forexample,whatcoulditslastdigitbe?
Follow

45 of 224

FollowTheMathLess Traveled 10/16/12 16:55


Get everynewpost deliveredto your Inbox.

The Math Less Traveled | Explorations in mathema...

http://mathlesstraveled.com/

Whenwemultiplytwointegers,thelastdigitoftheresultdependsonlyonthelastdigitsoftheintegersbeing multiplied,sincealltheotherdigitscontributesomepowerof10.Sowecannarrowdownthepossibilitiesfor thelastdigitofanyself-squaredecadicintegerbyseeingwhichdigitshavesquaresthatendinthesamedigit:

0 1 2 3 4 5 6 7 8 9

0 1 4 9 16 25 36 49 64 81 and ,butalso endsin and ends

Ivehighlightedthedigitswiththedesiredproperty:ofcourse,

in .Wealreadyknowwedontwanttoconsider and .Sofornow,letssupposethat endswith . Whataboutthelasttwodigitsof ?Again,thelasttwodigitsof dependonlyonthelasttwodigitsof .(If thisisntobvioustoyou,youshouldtryafewexamplestoconvinceyourself.Forexample,whatis ? Whatinformationdidyouneedtocomputethelasttwodigitsoftheanswer?)Sowhateverthelasttwodigits are,theirsquare,whenconsideredontheirownasatwo-digitnumber,mustbesomenumberthatendsinthe sametwodigits.Assumingthelastdigitis ,wecanturnthisrequirementintoamodularequationwhichwecan usetosolveforthesecond-to-lastdigit:

Sureenough,

whichendswith .

Canwetakethisfurther?Whataboutthelastthreedigits?

Check:

,whichindeedendswith

.Continuinginasimilarfashion(Illletyouworkoutthedetails
Follow

onyourown),wefindthatthefourthdigitmustbe ,andthefifthdigitmustbe .

46 of 224

FollowTheMathLess Traveled 10/16/12 16:55


Get everynewpost deliveredto your Inbox.

The Math Less Traveled | Explorations in mathema...

http://mathlesstraveled.com/

Areyouseeingapattern?Letsmakeatableoftheresultssofar:

1 2 3 4 5

5 25 625 0625 90625

25 625 390625 390625 8212890625

WhydidIputsomenumbersinbold?Well,hopefullyyouvenoticedbynowthateachnumberintheleft-hand columnalwaysseemstobeasuffixofthesquareofthepreviousnumber.Soperhapsthenextdigitwillbe8? Sureenough, endsin . ,

Willthisalwayswork?Yes,infact,itwill,andhereswhy.Letslet denotethelast digitsof (so generalizationofwhatwedidearlier):

,andsoon).Oncewehavefound ,wecansetupamodularequationtofindthenextdigit(thisisjusta

Now,

isclearlydivisibleby

sothattermgoesaway.Butwhatabout

?Itseemsthatweonly

knowitisdivisibleby with

,notnecessarilyby

.Ah,butwait!Weknowthat endswith ,andhenceis

divisibleby ;combiningthiswiththe givesusanotherfactorof !Sothistermgoesawaytoo,andweareleft

Now,

(bydefinition),sosubtracting frombothsidesleavesamultipleof .Dividingby

intheplaceof stdigitof

(namely, withtherightmost digitssettozero).Butwecanalsogetridofallthedigitstotheleftofthe stbecauseweareworkingmod . Soherestheprocedure:startingwith ,define wefindthat mustbeequaltothat

Thatis,squarethecurrentnumberoflength andtakethelast proofshowsthat

digitstogetthenextnumber.Theabove

Ateverystepwewillhaveanumber whosesquareendswiththedigitsof ; thisprocedurewillalwayswork;and thisproceduregivestheuniquesequenceof withthispropertywhenstartingwith !


Follow

47 of 224

FollowTheMathLess Traveled 10/16/12 16:55


Get everynewpost deliveredto your Inbox.

The Math Less Traveled | Explorations in mathema...

http://mathlesstraveled.com/

Sowehave

andsoon. Sowhatis ?Itissimplythelimitofcarryingoutthisproceduretoinfinity!

Weknowthatanysuffixof ,whensquared,yieldssomethingendingwithitself.Soitmakessense(althoughit takesabitofimagination!)thatsquaring itselfyields again.


Posted in arithmetic, infinity, iteration, modular arithmetic, proof | Tagged decadic, idempotent, self, square | 12 Comments

Four-figure offer
Posted on December 1, 2011

Thisjustarrivedinmyinbox: MynameisBeckyRaymond,ImaDomainBrokerageConsultantworkingonbehalfoftheownerof traveled.comtosellthispremiumasset. WhilesearchingonlineIcameacrossyourdomainmathlesstraveled.com;sincebothdomainshave listingsunderarelatedkeywordIthoughtperhapsyourcompanymaybeinterestedinacquiring traveled.com?Ifthisdomainisofinteresttoyou,pleasesubmitanofferinthefourfigurerangeand abovetoqualifyasapotentialbuyer. Surething,Becky!Hereismyfour-figureoffer:

Fig. 1. A graph of the first 1000 hyperbinary numbers.

Follow

48 of 224

FollowTheMathLess Traveled 10/16/12 16:55


Get everynewpost deliveredto your Inbox.

The Math Less Traveled | Explorations in mathema...

http://mathlesstraveled.com/

Fig 2. Hasse diagram for the subsets of a five-element set.

Fig. 3: Proof that

Fig. 4: Complete set of 15-bracelets.

Ihopeyouwillagreethatthisisaveryfinesetoffigures.Icouldprobablyaddacouplemorefigurestomyoffer ifthatbecomesnecessary. Ilookforwardtobeingtheproudowneroftraveled.com,theplacetogoforallyourmathematicaltravelneeds!


Posted in humor, meta | Tagged domain, four figure, offer | 7 Comments

Sigmas and sums of squares


Posted on November 29, 2011

CommenterRachelrecentlyasked,
Follow

49 of 224

FollowTheMathLess Traveled 10/16/12 16:55


Get everynewpost deliveredto your Inbox.

The Math Less Traveled | Explorations in mathema...

http://mathlesstraveled.com/

Howwouldyoufindthesumof

Seehereforanexplanationofsigmanotationinthiscaseitdenotesthesum

Ofcourse,foranyparticularvalueof wecanjustpluginvaluesanddoabunchofadding.ButIinterpret Rachelsquestiontomeancanwefindanalgebraicexpressionintermsof whichletsuscomputethesum morequicklythanactuallyaddingtheindividualterms? Letstry! First,weobservethat

Why?Ifyouthinkaboutitabit,youcanseethatthesametermsshowupontheleftandtheright,justina differentorder:theleftsideis Sinceadditionisassociative( Next,weobservethat , thatis, So,wehave .Thisisbecausemultiplicationdistributesoveraddition. . ;itisequalto .Sofar,we whereastherightsideis )andcommutative( )theseareequal. .

isjust copiesof ,soitisequalto .Ivewrittenbeforeabout have . Whataboutthatpesky ?Canitbesimplifiedatall?

Itcan,andIknowofafewdifferentwaystofigureitout.Illshowyouoneofthemtoday(andhopefullyothers inthefuture). Considerthesum

Forexample,if and

wehave

.Noticethatweendupwithboth and

infact,everythingcancelsexceptthe

atthebeginningandthe attheend.Ihopeyoucansee Follow

50 of 224

FollowTheMathLess Traveled 10/16/12 16:55


Get everynewpost deliveredto your Inbox.

The Math Less Traveled | Explorations in mathema...

http://mathlesstraveled.com/

thatingeneral,

Thissortofsumiscalledatelescopingsum,becausethewholethingcollapseslikeoneofthoseold-school telescopesthatpiratesuse.

However,wecanalsoexpandoutthe

andthensimplify:

Sonowweknowthat

sincebothsidesareequalto

.Fromherewejustneedsomealgebratoput

ononesideand

everythingelseontheother,andcleanthingsupabittoobtainwell,Illletyoufinishworkingitout.=)
Posted in algebra | Tagged notation, sigma, squares, sum, telescope | 6 Comments

Dimensions: go watch! now!


Posted on November 21, 2011

IfinallygotaroundtowatchingtheDimensionsvideos,whichImentionedoncebefore.Theyaresupercooland willbesuretoblowyourmind!Theystartbyexplainingsomesimpletools(stereographicprojection)and intuition(withreferencestoFlatland),thenjumpintofour-dimensionalpolytopes,complexnumbers,fractals, andfibrations.Thevideoscanbeatinybitcheesyandslow-movingattimesalthoughreally,iftheywentmuch fasteritwouldbedifficulttotakeitallin,andthereislotsofstunninglybeautiful3Danimation.Well,theres 4Danimationtoo,stereographicallyprojectedinto3-space,ofcourse.=)Watchingthemallisdefinitelyan investmentoftime(2hours),butdefinitelyworthit(andtheresnothingwrongwithskippingaroundabit).If youwanttoskiparound,formaximumbeautiful-mind-blowingnessmakesureyouatleastwatchthevideos about4Dpolytopesandfibrations.


Follow

51 of 224

FollowTheMathLess Traveled 10/16/12 16:55


Get everynewpost deliveredto your Inbox.

The Math Less Traveled | Explorations in mathema...

http://mathlesstraveled.com/

Posted in fractals, geometry, links, video | Tagged Dimensions, fibration, polytope, projection, video | 2 Comments

Old posts fixed


Posted on November 19, 2011

WhenImovedthisblogfrommyowncustomWordPressinstallationtowordpress.comlastMarch,the formatting, ,andimagesonanumberofoldpostsgotscrewedup.Ihavefinallyfinishedgoingback throughallmyoldpostsandfixingthingsup!Ifyoureboredandlookingforsomethinginterestingtoread,why nottakealookthroughmyarchives?=)


Posted in meta

Book review: Viewpoints: Mathematical Perspective and Fractal Geometry in Art


Posted on November 18, 2011

ThisbookiscertainlyquitedifferentfromthesortIusuallyreadand reviewbutIamalwaysinterestedinnewandcreativewaystoteach mathematics!Thisisquiteafunbook.Itsallaboutvisualartandsome ofthewaysitisinformedbymathematics,withfocusesonperspective andfractals.Itsfullofhands-onlearningactivities:reproducinga buildingusingmaskingtapeonaglasswindow,usingshishkebab skewerstohelpviewartworksatamuseum,drawingassignments,and, yes,somemathexercises.Interspersedbetweenthechaptersareartist vignettestellingthestory(andshowingoffthe(oftenamazing!) artwork)ofartistswhoseworkissomehowmathematicallyinspired.I didntlearnmuchmathfromthisbook,butIdidlearnsomefunthings aboutart,andIsuspectthatthereversewouldbetrueforartists withoutmuchmathbackground.Thisbookshouldworkwellfor studentswhohaventhadtoomuchfunwithmathinthepastbutare willingtotrysomethingnew.Itwouldespeciallyworkwellasthetextbookforahands-onclassorworkshop taughtbyanenthusiasticinstructoras,infact,itisdesignedtobe. Myonegripeiswiththequalityofmanyoftheincludedimages(andyes,IfinditstrangethatIshouldhaveto complainaboutthisinabookaboutart!).Firstofall,thebookisprintedinblackandwhite(withacollectionof colorplatesinthemiddle);yes,ofcourseIunderstandthattherearetradeoffsinvolvingthecostofprintingthe book,butstill,itsratherdisappointingtohaveabookaboutartprintedinblackandwhite.Theincluded diagramsandfigureslookgreat,butmanyoftheincludedimages(i.e.reproductionsofartworks)aremuchtoo darkandhardtosee,andIrepeatedlyfoundmyselfthinking,Ibetthispainting/photograph/artworkwouldbe reallybeautiful/interesting/amazingifonlyIcouldseeitincolor. Still,overall,thisisagreatbookwithfreshperspectiveontheintersectionofmathandartthatilluminatesboth subjects.
Posted in books, fractals, geometry, pattern, pictures, review | Tagged art, fractal, geometry, perspective, review, textbook

Follow

52 of 224

FollowTheMathLess Traveled 10/16/12 16:55


Get everynewpost deliveredto your Inbox.

The Math Less Traveled | Explorations in mathema...

http://mathlesstraveled.com/

Fun with repunit divisors: more solutions


Posted on November 17, 2011

InFunwithrepunitdivisorsIposedthefollowingchallenge: Provethateveryprimeotherthan2or5isadivisorofsomerepunit.Inotherwords,ifyoumakealist oftheprimefactorizationsofrepunits,everyprimeotherthan2or5willshowupeventually. Mypreviouspostexplainedtwodifferentproofs.AttheendofFunwithrepunitdivisorsIalsoposedaseries offollow-upchallenges;herearesolutionstothose. 1. Computearepunitwhichisdivisibleby2011(youllprobablywanttouseacomputer!). Aswenowknowfromthesecondproof(usingFermatsLittleTheorem),therepunitwith divisibleby thesmallestrepunitwhichisdivisibleby ?Inthatcasewecancompute , onesmustbe , .Soitsufficesto , ,butatthenext , .Forexample,Icomputedthisusing .SoIguessacomputerisnotreallynecessaryafterall!However,whatifwewanttocompute

,untilwegetzero.However,wedonthavetoactuallycomputeabiggerandbiggerrepuniteachtime! Eachrepunitisrelatedtothepreviousonebyanapplicationofthefunction keeponlytheremainder (reducing stepwecanreduce then secondwefindthat Haskellbydefining ateachstep,andapply toget .Thenwecompute whenneeded).Forexample,westartoutbycomputing , , toeachremaindertogetthenext

,andsoon.Iteratingthisprocessonacomputerisveryfast,andinafractionofa isthesmallestrepunitdivisibleby

fpx=(10*x+1)`mod`p smallestRepp=(+1).length.takeWhile(/=0).iterate(fp)$1

2. Provethateveryprimeotherthan2or5isactuallyadivisorofinfinitelymanyrepunits. Proof:Notethatmultiplying,say, by gives ;multiplyingby gives forall ,andsoon.In . general,wecanseethattherepunitwith digitsalwaysevenlydividestherepunitwith digitswhenever evenlydivides .Soif isadivisorof thenitisalsoadivisorof

3. Provethateveryintegerwhichisnotdivisibleby2or5isadivisorofsomerepunit. Proof:suppose hastheprimefactorization ; divides Hence mustbeadivisorof , divides ,since .Weknowthateach isadivisorofsomerepunit .Asasimpleexample, ,and isarepunititself. .Then,bytheargumentabove, mustbeadivisorof

.(Actually,wecanmakethisabitmoreprecise

inordertofindamuchsmallerrepunitthat

mustevenlydivide.Canyouseehow?)

4. Generalizealloftheabovetorepunitsinbasesotherthan10. Inbase ,wecandefinearepunitoflength tobe factorizationofthebase .


Follow

.Everythingaboveisstilltrue

aslongaswereplacethespecialexcludedprimes and withwhicheverprimesshowupintheprime

53 of 224

FollowTheMathLess Traveled 10/16/12 16:55


Get everynewpost deliveredto your Inbox.

The Math Less Traveled | Explorations in mathema...

http://mathlesstraveled.com/

5. Whatssospecialaboutrepunitshere?Canyougeneralizetoothersortsofnumbers? Illleavethisoneopenforthetimebeing!Thereareprobablylotsofinterestingwaysyoucouldanswerthis question.


Posted in arithmetic, iteration, modular arithmetic, number theory, primes, programming, proof, solutions | Tagged repunit

Fun with repunit divisors: proofs


Posted on November 16, 2011

Aspromised,herearesomesolutionstotherepunitpuzzleposedinmypreviouspost.(Stopreadingnowifyou dontwanttoseesolutionsyet!) Provethateveryprimeotherthan2or5isadivisorofsomerepunit.Inotherwords,ifyoumakealist oftheprimefactorizationsofrepunits,everyprimeotherthan2or5willshowupeventually. Proof1:Notethatwecanwritetherepunitcontaining onesas .Suppose isaprimeotherthan or

,andconsidertheremainderswhenrepunitsaredividedby .Sincethereareinfinitelymanyrepunitsandonly possibleremainders,bythePigeonholePrincipletheremustbesomeremainderwhichisrepeated,thatis, theremustbetwodistinctnumbers and (letssay )suchthat

Bringing

overtotheleft-handsideandsimplifyinggives

Since

wecanfactorout

,giving

Now,althoughitisperfectlyOKtoadd,subtract,ormultiplybythesamethingonbothsidesofamodular equation,wehavetobecarefulwithdivision:itisonlysafetodividebothsidesofamodularequationbysome numberwhichdoesnotshareanycommonfactorswiththemodulus.Well,inthiscasewebeganbyassumingthat isneither nor ,soitdoesnotshareanyfactorswith .Hence,dividingbothsidesby givesus

Look!Arepunitdivisibleby ! Whatsthebigideahere?Wehavesomesortofiterativeprocessandwanttoshowthatsomeparticularthing musteventuallyhappen.Ifthereareonlyafinitenumberofpossibilities,bythePigeonholePrincipleweknow theprocessmusteventuallyrepeat.Sowelookattheplaceswhereitrepeats,andusethosetocreatethething wewantedtoshow. Proof2:FermatsLittleTheoremstatesthatforanynumber whichisnotdivisiblebysomeprime ,


Follow

54 of 224

FollowTheMathLess Traveled 10/16/12 16:55


Get everynewpost deliveredto your Inbox.

The Math Less Traveled | Explorations in mathema...

http://mathlesstraveled.com/

Sowecanapplythisdirectlytoconcludethat simplynotethat isdivisibleby .

isdivisibleby foranyprimesotherthan , ,or ones.Tocompletetheproofwe

(note:whynot ?).Inotherwords,such alwaysdividetherepunitwith

ThisversionismuchshorterbutofcourserequiresyoutoknowFermatsLittleTheorem!Anditdoesntgiveas muchinsightunlessyoualreadyhavesomeinsightintowhyFermatsLittleTheoremistrue.PerhapsIcould explainaproofortwoofFermatsLittleTheoreminafuturepostifanyoneisinterested. AttheendofmylastpostIalsoposedsomefollow-upchallenges.Illpostsolutionstothosenexttime.


Posted in iteration, modular arithmetic, number theory, pattern, primes, proof | Tagged divisibility, Fermat, prime, proof, repunit | 1 Comment

Fun with repunit divisors


Posted on November 11, 2011

Inhonoroftodaysdate(11/11/11),heresafunlittleproblem(andsomefollow-upproblems)Iveseenposed inafewplaces(forexample,hereisaverysimilarproblem).IfIrecallcorrectly,itwasalsoaproblemona midtermformynumbertheorycourseincollege.Itsalovelylittleproblemwithanequallylovely solutiononceyouunderstandthesolutionyoullstarttoseemanyothersituationswhereasimilartechnique canbeapplied. Anumberoftheform ,withanynumberofrepeatedones,iscalleda(base-10)repunit.Prove

thateveryprimeotherthan2or5isadivisorofsomerepunit.Inotherwords,ifyoumakealistofthe primefactorizationsofrepunits,everyprimeotherthan2or5willshowupeventually. Forexample,herearetheprimefactorizationsoftherepunitsuptolength20:

Itsclearthatnorepunitisdivisibleby2or5(why?),butatfirstsightitmayseemunlikelythatalltheother primesaredivisorsofsomerepunit!Theredoesntseemtobealotofrhymeorreasonintheabovelistof factorizations.


Follow

55 of 224

FollowTheMathLess Traveled 10/16/12 16:55


Get everynewpost deliveredto your Inbox.

The Math Less Traveled | Explorations in mathema...

http://mathlesstraveled.com/

Theaboveproblemisreallytheheartofthematter,butonceyousolvethat,herearesomefunfollow-up problems: 1. Computearepunitwhichisdivisibleby2011(youllprobablywanttouseacomputer!). 2. Provethateveryprimeotherthan2or5isactuallyadivisorofinfinitelymanyrepunits. 3. Provethateveryinteger whichisnotdivisibleby2or5isadivisorofsomerepunit. 4. Generalizealloftheabovetorepunitsinbasesotherthan10. 5. Whatssospecialaboutrepunitshere?Canyougeneralizetoothersortsofnumbers? Illpostsomesolutionsinafewdays.Happy11/11/11!


Posted in arithmetic, challenges, modular arithmetic, number theory, primes | Tagged divisors, primes, repunit | 16 Comments

Post without words #3


Posted on October 29, 2011

Follow

56 of 224

FollowTheMathLess Traveled 10/16/12 16:55


Get everynewpost deliveredto your Inbox.

The Math Less Traveled | Explorations in mathema...

http://mathlesstraveled.com/

(Thisismy200thpost!=)
Posted in counting, pattern, pictures, posts without words, recursion | 11 Comments

More fun with infinite decadic numbers


Posted on October 28, 2011

Thisisthesixthinaseriesofpostsonthedecadicnumbers(previousposts:Acuriosity,Aninvitationtoafunny numbersystem,Whatdoesclosetomean?,Thedecadicmetric,Infinitedecadicnumbers). LasttimeIleftyouwithsomepartingexercises;herearetheanswers,alongwithsomeadditionalcommentary. 1. Whatnumberisrepresentedby Itturnsoutthat toitselfninetimesyields ?Howdoyouknow? ,andthereareseveralwaystoseethis.Thesimplestistonotethataddingit ,whichwealreadyknowis .


Follow

57 of 224

FollowTheMathLess Traveled 10/16/12 16:55


Get everynewpost deliveredto your Inbox.

The Math Less Traveled | Explorations in mathema...

http://mathlesstraveled.com/

Atthispointwemayalsoask,if yieldszero:

is

,thenwhatis

?Itmustbe

,sinceaddingthatto

Hmm,so :

is

,and

is

(sinceitis8times

).Thisalsomakessensebecause

Itseemstomakesensebutitsureisweird! (Idontactuallyknowtheanswer.) 2. Howabout ?

isnegativebut

ispositive.Canyoufigureouta

relativelysimplewaytotell,justbylookingataninfinitedecadicnumber,whetheritispositiveornegative?

MarkJamesnotedthat If is then . Indeed,andheresanotherwaytoseethesamething:if ,then ,so is lookslike since and

Hence

,so

.Asafurthercheck,

,so

whichoughttobe

:sureenough,adding to

yieldszero. willbeequal

Ingeneral,wecanseethattheinfiniterepeatingdecadicnumber to

wherethenumberof sinthedenominatoristhesameasthenumberofdigitsinthenumerator. ?Howabout ?Or ?

3. Canyoufindaninfinitedecadicnumberwhichrepresents is However, ,and is

;bythispointIhopeyoucanseewhy. whenmultipliedby

isdifferent:wewouldhavetofindsomenumberwhichyields

two.Obviouslynosuchnumberexists:twotimesanythingcannotpossiblyendina .Thesameistrueofany fractionwithadenominatorwhichisnotrelativelyprimeto(i.e.sharesacommondivisorwith) . However,wecanextendournotionofdecadicnumberstoaccommodatesuchfractions,byallowingdigits afteradecimalpoint.Soforexample willbe ,asusual,andthen willbe .Its


Follow

58 of 224

FollowTheMathLess Traveled 10/16/12 16:55


Get everynewpost deliveredto your Inbox.

The Math Less Traveled | Explorations in mathema...

http://mathlesstraveled.com/

importanttonotethatwecanonlyallowfinitestringsofdigitsafterthedecimalpoint:anumberlike ismeaninglessbecausethesequenceofnumbers pointareOK. Ingeneral,wecalldecadicnumberswithnodigitsafterthedecimalpoint(includinginfiniteones)decadic integers.Obviouslyallthenormalintegersarealsodecadicintegers;butsoarefractionssuchas denominatorsarerelativelyprimeto .Otherfractionssuchas Exercises: A. Showthatanyfractionwhosedenominatorhasonly and asfactorsisrepresentedbyafinitedecimal number. B. Showthatbyallowingafinitenumberofdigitsafterthedecimalpoint,wecanrepresentanyfractionasa decadicnumber.(Hint:factorthedenominatorintoonepartconsistingonlyoftwosandfivesandanother partwitheverythingelse.) YoumayrecallthatinapreviouspostIpromisedtoshowyouastrangedecadicnumber ,whichisnotzero,but isequaltoitsownsquare.WevenowfinallyseenenoughformetotellyouwhatitiswhichIwilldoinmy nextpost.Inthemeantimeyoumaywanttotrydiscoveringit!
Posted in arithmetic, infinity, number theory | Tagged decadic, decimal, fractions, integers, representation | 4 Comments

doesnotconvergeto

anything;infact,theyaregettingfurtherandfurtherapart!Butanyfinitenumberofdigitsafterthedecimal

whose

arenot.

Book review: Magical Mathematics


Posted on October 20, 2011

OccasionallysomeonefromPrincetonUniversityPresssendsmealistof upcomingtitlesandasksifthereareanyIdliketoreview.Ijumpedwhen Isawthisone:abookaboutmagictricksandmath!ByPersiDiaconisand RonGraham!RonGrahaminparticularisoneofthecoolest mathematiciansIknowof. SoIsaidIwasinterestedandthenpromptlyforgotaboutitthatwas severalmonthsago.Imaginemydelightwhenthebookshowedupinthe mail!AssoonasIopeneditIknewIwasinforatreat:itissimplya beautifulbook.Thedesign,layout,typography,eventhepaperis beautiful. SoIsettledintoreaditoverthepastfewdaysandwasnotdisappointed:thisisoneofthemostfun,engaging newpopularmathematicsbooksIveseeninalongtime.Ilovehowitreadsmorelikeaconversationthana traditionalbook:theauthorswendtheirwayfromexplainingmagictricks,toexplainingthemathbehindthem, tospeculatingonopenmathematicalquestions(andwhethertheiranswerscanbeturnedintogoodmagic tricks),topersonalreminiscencesandstoriesaboutgreatmathematicalmagicians.Theresachapteronjuggling (completewithinstructionswithpictures!),achapterontheIChing,achapteronshufflingcards.Thereare
Follow

59 of 224

FollowTheMathLess Traveled 10/16/12 16:55


Get everynewpost deliveredto your Inbox.

The Math Less Traveled | Explorations in mathema...

http://mathlesstraveled.com/

lotsofgreatmagictricks,manyexplainedsothatyoucanperformthemyourself(andafewthatarent!).In short,whatitboilsdowntoisacoupleofbrilliantandfascinatingpeoplehavingaheckofalotoffunand lettingyouinonsomeofit. Themathisatalevelthatshouldbeaccessibletomosthighschoolorparticularlymotivatedmiddleschool students.Indeed,givingthisbooktoahighschoolstudentisprobablydangerousifyouwantthemtopay attentiontoanythingelseforthenextfewweeks;theyarelikelytobehooked!


Posted in books, people, review | Tagged book, magic, Persi Diaconis, review, Ron Graham | 3 Comments

Infinite decadic numbers


Posted on October 19, 2011

Torecap:wevenowdefinedthedecadicmetric onintegersby

where isnotdivisibleby10,andalso closer,withadistanceof . and

.Accordingtothismetric,twonumbersareclosewhentheir and areatadistanceof ,but and are arecloserstill,withadifferenceof and are .Putsimply:themorefinal

differenceisdecadicallysmall.So,forexample,

digitstwointegersshare,theclosertheyare.Thisisareversaloftheusualsituationinwhichintegersareclose whentheysharealotofinitialdigits(like withoutsharingdigitsforexample, and ).MarkJamesnotesthatitisalmostasifwewere apart.Butinthiscasethenegativenumberendswith reversingthenumbers.Jonathannotesthatthisisnotquiteright,negativeandpositiveintegerscanbeclose digitsthataredualtothedigitsthepositivenumberendswith. Letskeepthinkingaboutnegativenumbersabit. apart. to and arecloserstill, and :specifically, arealways ,so and are apart. and are isveryclose . , ,

.Infact,Ihopeyoucanseethatingeneral, apart.Sothemore9sweadd,thecloserwegetto

Whatifweletthisprocessgoonforever?Afterall,wedothiswiththenormaldecimalnumbers: ,etc.getcloserandcloserto ,sowedefinetheinfinitesequence here:wedefinetheinfinitedecadicnumber

.Wecandothesamething

tobeequalto

.Weird,huh?Butitgetsweirder(andcooler).Thereallyneatthingisthatarithmeticstill wehadbetter

worksinthesamewayasitdoesfornormalintegers.WhatdoImean?Well,ifweadd and get ,right?Letstryit:

Adding9and1givesus10,sowewrite0andcarrythe1givingus10again,andsoonforever. Andifweadd toitselfweoughttogetsomethingthatbehaveslike ,right?


Follow

60 of 224

FollowTheMathLess Traveled 10/16/12 16:55


Get everynewpost deliveredto your Inbox.

The Math Less Traveled | Explorations in mathema...

http://mathlesstraveled.com/

Well,addingthelasttwo9 sgivesus18,sothelastdigitoftheansweris8wecarrythe1,sothenext additiongivesus19;wewritedown9andcarrythe1andthispatternisnowobviouslygoingtorepeat forever,so

whichIclaimrepresents

:youshouldhavenotroubleverifyingthat

Herearesomepartingexercisesforyoutoworkonbeforemynextpost: 1. Whatnumberisrepresentedby 2. Howabout ? ?Howabout ?Or ? ?Howdoyouknow?

3. Canyoufindaninfinitedecadicnumberwhichrepresents

Posted in arithmetic, convergence, infinity, number theory | Tagged decadic, negative, numbers | 7 Comments

The decadic metric


Posted on September 28, 2011

Continuingmyseriesofpostsexploringthedecadicnumbersinmypreviouspost,Iexplainedthatwewill defineanewsizefunction,ormetric ,differentfromtheusualabsolutevalue,andwritten numberswillbeclosetoeachotherwhenthesizeoftheirdifferenceissmall. Fornowwewilldefine ; ontheintegers.First,everynonzerointeger canbefactoredas ; .Now,for notdivisibleby ,wesimplydefine . So, ; ; .Thisdefinitiondoesntcoverzero,however,because ,sowedefine for ,since ,where is .Two

notdivisibleby .Inotherwords, isthelargestpowerof whichevenlydivides .Forexample,

any .Butnoticethatthedistancefromanynumbertoitselfwillbe thedistancefromanythingtoitselfoughttobezero.

So,whatpairsofintegersareclosetoeachother,underthisnewmetric?Howabout compute

and

?We

.Atfirstsightthatmightseemprettyclose.Afterall,undertheusualabsolute forsomenonnegative ,anditseasytoseethat isactuallythelargest and areactuallyratherfarapart.Weird,right?(Beforereadingon,

valuemetric,thatstheclosesttwonaturalnumberscanpossiblybetoeachotherwithoutbeingequal.Butthe decadicmetricisalwaysoftheform possiblevalueforthismetric!So

canyoucomeupwithapairofintegersthatarecloser?) Asanaside,notethatthenumberlineisnolongerhelpful:itgivesusavisualsenseofhowfarapartanytwo integersare,butonlyfortheusualabsolutevaluemetric!


Follow

61 of 224

FollowTheMathLess Traveled 10/16/12 16:55


Get everynewpost deliveredto your Inbox.

The Math Less Traveled | Explorations in mathema...

http://mathlesstraveled.com/

Forourcurrentpurposesitsmoreusefultothinkoftheintegersaslivinginabigsoup:eachintegerstillexists andmeansthesamethingitusuallymeans,butthephysicalrelationshipoftheintegerstooneanotherisquite differentthanwhatweareusedto.(Theirarithmetic relationshipsareunchanged:forexample, .) and thesoup!Partofourjobwillbetofigureoutmoreaboutwhatthatsoupactuallylookslike. stillequals arerightnexttoeachotheronthenumberline,buttheyareoncompletelyoppositesidesof

So,whataboutapairofintegersthatareclosertogetherthan absolutevaluemetric!Letscomputetheirdecadicdistance:

and

?Howabout

and

Thosedontseemverycloseatall!Youmightprotest.Aha,butyouarestillthinkingintermsoftheold

Aha,sotheyreallyareabitclosertogether.Whatabout

and

Thosearecloserstill! Doyouseewhatsgoingon?Canyoucomeupwithaconcisedescriptionofwhentwointegerswillbeclose? Howclosecantwointegersbe?


Posted in arithmetic, number theory, pattern | Tagged decadic, distance, integers, metric, number line, numbers, p-adic, soup | 5 Comments

What does close to mean?


Posted on September 22, 2011

Continuingfromlasttime,considerthe(normal,decimal)number

withaninfinitenumberof3 safterthedecimalpoint.Now,youprobablyknowthatthisrepresents why?Howdowedefinewhatsuchaninfinitesequenceofdigitsmeans?

.But
Follow

62 of 224

FollowTheMathLess Traveled 10/16/12 16:55


Get everynewpost deliveredto your Inbox.

The Math Less Traveled | Explorations in mathema...

http://mathlesstraveled.com/

Thestandardansweristhatwethinkoftheinfinitedecimalnumber thesequence

asashorthandforthelimitof

Thatis,thesequenceofrationalnumbers

,andsoon,getinfinitelyclosetosomenumber,namely,

whichistakenasthemeaningofthesequence.(Iamwavingmyhandsabithere;thisisusuallymademore precisethroughthenotionofaCauchysequence.Buttheintuitionisthesame.) Now,inthepreviousparagraphIsaidthatthenumbers ifwegiveadifferentanswerthanusual. First,letsthinkaboutwhatclosetomeansinthecontextoftheusualrealnumbers.Thedistancebetweentwo numbers and isdefinedtobe ,where denotestheusualabsolutevalueofanumber.Wecanthinkof theabsolutevaluefunctionasassigningasizetoeachnumber:42and-42bothhavethesamesize,namely,42. Sothedistancebetweentwonumbersisthesizeoftheirdifference. Thenameofthegamenowwillbetodefineadifferentsizefunction,whichwewillwrite issmall.
Posted in convergence, number theory | Tagged absolute value, Cauchy, distance, limit, sequence | 3 Comments

,getinfinitelyclosetosomenumber.Whatdowe

meanbycloseto?Youmaythinkthisasilly,obviousquestion.Butitturnsoutthatinterestingthingshappen

.Usingthissize

functionwillgiveusadifferentmeaningofcloseto:twonumbers and willbeclosetoeachotherwhen

An invitation to a funny number system


Posted on September 17, 2011

Considertheequation

Solvingthisequationisnosweat,right?Letsdoit.First,wesubtract frombothsides:

Nowwecanfactoran outoftheleftside:

Now,iftheproductoftwothingsiszero,oneofthemmustbezero.Soeither

,or

,thatis,

Easy,right?Sure.ButwhatIdliketotalkaboutoverthecourseofthenextfewposts,inspiredbythiscuriosity, isafunnyalternatenumbersystemcalledthe10-adic ,ordecadicnumbers.Inthedecadicnumbers,theequation hastheusualsolutions0and1,butitalsohastwoother funnysolutions,whichdontcorrespondtoany familiarrealnumbers.


Follow

63 of 224

FollowTheMathLess Traveled 10/16/12 16:55


Get everynewpost deliveredto your Inbox.

The Math Less Traveled | Explorations in mathema...

http://mathlesstraveled.com/

Thefirstsolution,whichIwillwriteaboutinanupcomingpost,wellcall .Thatis, issomefunnysortof numberwiththepropertythat alsoasolution,since .(Afancymathwordforthisisidempotent.)Noticethatthismeans . .Thisisquite is

Noticealsowhathappenswhenwemultiplythesetwosolutions:

strange:twonumbers,neitherofwhichiszero,whichmultiplytogivezero!Suchnumbersarecalledzero divisors.Thisisthestepinoursolutionabovewhichfailsforthedecadicnumbers:oncewefactoredtoobtain ,wereasonedthatiftwothingsmultiplytogivezero,thenoneofthemmustbezero.ButIvejust claimedthatthisisnottrueforthedecadicnumbers. Oftenmathematiciansstayawayfromsystemswithzerodivisors,sincetheyruinsomeniceproperties.For example,noteverynonzeronumberhasamultiplicativeinverse.(Recallthatthemultiplicativeinverseofa number isanothernumber suchthat ;forexample,themultiplicativeinverseof is .)For homework,provethatthe wediscussedabovedoesnothaveamultiplicativeinverse!(Hint:trysupposingthat itdoes,andderiveabogusequation)Butwerenottryingtoaccomplishanythinginparticular,justtohavefun andlearnsomething,sobringonthezerodivisors! OK,sowhatexactlyarethesedecadicnumbers?Whatis ?Andwhatelsecanbesaidaboutthecuriosityinmy previouspost?Allthisandmore,comingsoontoaMathLessTravelednearyou (Ifyoureimpatient,youcanreadsomeaboutp-adicnumbershere,andaboutthedecadicnumbershere.Butas usual,Iplantoexplainthingsatamoreleisurelypace.)
Posted in arithmetic, number theory | Tagged idempotent, numbers, ten-adic, zero divisor | 8 Comments

A curiosity
Posted on September 14, 2011

FromFutilityCloset,afunblogofrandomtidbitsIenjoyreading,comesthefollowingcurioussequenceof equations,attributedtoJ.A.H.Hunter:

ImanagedtoextendthispatternforafewmoredigitsbeforeIgotbored.Doesitcontinueforeverordoesit eventuallystop?Isthereanydeepermathematicalexplanationlurkingbehindthissupposedcuriosity?Whats sospecialabout ?Dopatternslikethisexistforotherfunctions?


Follow

Posted in arithmetic, modular arithmetic, number theory, pattern | 20 Comments

64 of 224

FollowTheMathLess Traveled 10/16/12 16:55


Get everynewpost deliveredto your Inbox.

The Math Less Traveled | Explorations in mathema...

http://mathlesstraveled.com/

Some words about Post without words #2


Posted on September 11, 2011

Mypreviouspostdisplayedthispicture:

AsYuriyKashnikovguessed,Imadethispictureusingdiagrams,aHaskelllibraryIamdevelopingforcreating imageslikethis.(Youcanseethesourcecodeforthispicturehere.) Ifyouhaventhadachancetopondertheimageandlookforpatterns,maybeyouwanttodothatnow!Iwant tosayafewthingsaboutit.First,asJMobserved,onewaytolookatthisisatreewitheachbranchasking, `WhichcolorshouldIeliminate? Heresanotherwaytothinkaboutit(whichreallyamountstothesamething).Eachgraphisapictureofallthe subsetsofacertainset,arrangedsothattheemptysubsetisatthebottom,thecompletesetisatthetop,andall thesetsinbetweenarearrangedsothatthereisanupwards-pointingedge(orpathofseveralupwards-pointing edges)connectingeachsettotheothersetsofwhichitisasubset.Soifwestartatthebottomandtravel upwardsfollowingtheedges,moreandmorecolorswillbeadded,butoncewehaveseenacertaincolor,itwill remaininallthesetsweencounterfromthenon(aslongaswekeeptravelingonlyupwards).Thesesortsof
Follow

65 of 224

FollowTheMathLess Traveled 10/16/12 16:55


Get everynewpost deliveredto your Inbox.

The Math Less Traveled | Explorations in mathema...

http://mathlesstraveled.com/

diagramsareactuallycalledHassediagrams.(Hassediagramsareabitmoregeneral;theycanactuallybeusedto visualizeanypartiallyorderedset.) Now,xandercountedthenumberofsetsineachrow.HecorrectlypointedoutthatIshouldhaveincluded anotherdiagramatthetop,namely,anemptydottedbox.Ifweincludethat,countingthenumberofsetsinthe rowsofeachdiagramgivesaninterestingpattern: 1 11 121 1331 14641 Doyourecognizethispattern?Thatsright,itsPascalstriangle!Wewouldexpectthenextrowtobe1510105 1.Letssee:

Sureenough!Thisisnottoosurprising,actually:eachrowofoneofthesediagramsshowsallthesize-ksubsets ofasize-nset,thatis,thenumberofwaystochoosekcolorsoutofn.Thisisalsocalledabinomialcoefficient, andthebinomialcoefficientsareexactlythenumbersthatmakeupPascalstriangle. Now,someoneelsealsomentionedhypercubes.CertainlythefirstdiagramIshowedlookslikealine(a one-dimensionalcube),thesecondlookslikeasquare(atwo-dimensionalcube),anditsnottoohardto imaginethenextoneasadrawingofathree-dimensionalcube.Buthowdoweknowwhetherthenextonelooks likeafour-dimensionalcube?Andwhattheheckisafour-dimensionalcube,anyway?Illtalkaboutthatinmy nextpost!


Posted in counting, pascal's triangle, pattern, pictures | Tagged binomial coefficients, diagram, Hasse, Pascal, subset | 4 Comments

Post without words #2


Posted on September 6, 2011

Follow

66 of 224

FollowTheMathLess Traveled 10/16/12 16:55


Get everynewpost deliveredto your Inbox.

The Math Less Traveled | Explorations in mathema...

http://mathlesstraveled.com/

Posted in counting, pascal's triangle, pattern, pictures, posts without words | 10 Comments

Relatively Prime: Stories from the Mathematical Domain


Posted on July 28, 2011

SamuelHansenneedsyourhelp!Hescurrentlyraisingmoneytocreateaseriesofpodcastsexplainingmathtoa wideaudience.Fromthelinkedvideo,itlooksliketheywillbealotoffun,andIthinkitwouldbeashameifhe wasntabletomakethem.Hestillneedstoraiseanother$4,000orsointhenextsixdayseverybithelps. Hedidntaskmetopostthis;Ijustthinkitsaworthycause.Ifyoureunsureofhiscredentials,youcancheck outtheothermathematicalpodcastshesmadeinthepast.


Posted in links, people | Tagged podcast, Samuel Hansen, stories

Book review: Roads to Infinity


Posted on July 5, 2011

Whatisinfinity?Whatisproof? Thesearetwoofthebiggestquestionsmathematicianshavegrappledwithoverthe years.Inthiswell-writtenandfascinatingbook,JohnStillwelltakesusonatour


Follow

67 of 224

FollowTheMathLess Traveled 10/16/12 16:55


Get everynewpost deliveredto your Inbox.

The Math Less Traveled | Explorations in mathema...

http://mathlesstraveled.com/

throughsomeoftheanswerstothesequestionsandthesurprisingwaysthatthe answersarerelateddiscoveredoverthepast100years.Diagonalarguments, transfiniteordinals,thecontinuumhypothesis,formalcomputationalsystemsandthe haltingproblem,incompletenessandunprovability,formalarithmeticandproof strength ThisbookissomewhatabovetheusuallevelofbooksIreviewhere.Readerswillneedawell-developedsenseof logictofollowallthethreads.Butthisisbyfarthemostengagingandaccessibleexpositionofthesetopicsthat Iveeverseen,andIcouldntresistreviewingsuchawell-writtenbookaboutsuchbeautifulandsurprising mathematics.


Posted in arithmetic, books, computation, induction, infinity, logic, proof, review | Tagged infinity, John Stillwell, proof, roads

Happy Tau Day!


Posted on June 28, 2011

Happy day!

,ofcourse,isthefundamentalcircleconstantwhichrepresentstheratioofany

circlescircumferencetoitsradius.(Inthepastpeoplehavealsousedthesymbol torepresenthalfof ; perhapsyouveheardofit.) TheinimitableViHarthasmadeavideoabout :

Andhereswhat soundslike:

Follow

68 of 224

FollowTheMathLess Traveled 10/16/12 16:55


Get everynewpost deliveredto your Inbox.

The Math Less Traveled | Explorations in mathema...

http://mathlesstraveled.com/

Formore linkgoodness,checkoutDenisespost.
Posted in famous numbers, links, video | Tagged day, pi, tau | 3 Comments

The wonderful world of Mike Reilly


Posted on June 14, 2011

TheotherdayIreceivedanemailfromMikeReilly,whointroducedhimselfasaprofessionalgameandpuzzle inventor,andsuggestedthatImightbeinterestedintakingalookatafewofhiswebsites.Iwasntquitesure whattoexpect:gameandpuzzleinventorsoundsawesome,butontheotherhand,Iroutinelygetemailsof theformhey,Ijustsawyourblog,youmightbeinterestedinlookingatmywebsiteandtheyoftenturnout tobepeoplejusttryingtomakemoneywithoutanythingveryinterestingtooffer. So(asyoucanprobablyguessfromthefactthatIamactuallywritingablogpostaboutit)Iwasthrilledto discoverthatMikeisdefinitelynotoneofthosesortsofpeople!Hehascreatedsomereallyfascinatingand beautifulgamesandpuzzles.

IfirstgotsuckedintohisHourMazepuzzles(thefirstoneispicturedabove)whichshouldticklethefancyof manyMathLessTraveledreaders.Therulesaresimplefillinthegridsothateverynumberfrom1-12isused anequalnumberoftimes,andthenumbersinadjacent(i.e.notseparatedbyawall)cellsdifferbyone,modulo 12(so1and12differby1).Butcompletingthemcanbequitetricky!Theresdefinitelylotsofinterestingmath lurkingtherebutIhaventyetbeenabletocomeupwithanygeneralstrategy.Maybethatwillbethesubjectof afuturepost.


Follow

69 of 224

FollowTheMathLess Traveled 10/16/12 16:55


Get everynewpost deliveredto your Inbox.

The Math Less Traveled | Explorations in mathema...

http://mathlesstraveled.com/

MikealsolinkedmetohisKickstarterproject,Reillyscube,wherehesraisingmoneytoproduceabeautiful woodencubepuzzlehedesignedandyoucangetyourownifyousupporttheproject!Youcanalsoread thereaboutothergamesandpuzzleshesdesigned(someofwhichyoucanalsogetasrewardsforsupporting theproject).


Posted in games, links, people, puzzles | Tagged cube, hour, Kickstarter, maze, puzzle, Reilly | 2 Comments

Collatz conjecture: an apology


Posted on June 7, 2011

InthecommentsonmypreviouspostaboutGerhardOpfersproposedproofoftheCollatzconjecture,several differentpeopleexpressedtheopinionthatmytonewasratherarrogant,andIthinktheyhaveapoint.So:I apologizeformytone.Ihavefallenpreytotheall-too-easypleasureofpointingouthowotherpeopleare wrong. Idonot,however,apologizeforthecontentofmypost:IamstillfirmlyoftheopinionthatOpfersproof containsanunfixablehole.


Posted in meta, open problems, proof | 9 Comments

The Collatz conjecture is safe (for now)


Posted on June 4, 2011

AfewdaysagoJohnCookreportedadraftpaperclaimingtosolvetheCollatzconjecture.Ofcourse,sincethe Collatzconjectureissosimpletostate,itconstantlyattractstonsofwould-besolvers,andmostofthepurported proofstheygeneratearenotevenworthmathematicianstimetolookat.Sowhyshouldthisonebedifferent? Well,onthesurface,itseemedtobeamuchmoreseriousattemptthanthevastmajority.OfScottAaronsons TenSignsaClaimedMathematicalBreakthroughisWrong,thispaperexhibitedonly#6(jumpingstraightinto technicalmaterialwithoutpresentinganewidea)and#10(usingtechniquesthatseemtoowimpy).But#6 couldjustbeduetopoororganizationofthepaper,and#10isnotobviousuntilyougettotheend. Butandyouknewthiswascomingitiswrong.Ivespentseveralhoursoverthepastfewdaysreadingover thepaperandcametothisconclusionindependenlyandthenfoundseveralotherpeoplewhohadcometo thesameconclusion,forthesamereason. ConsiderthefollowingproofoftheCollatzconjecture.WeconsiderrunningtheCollatzfunction(callit ) backwards,tofindoutwhichnumber(s)couldhaveprecededagivennumberinaCollatzsequence.Wealways have ,sofrom wecangobackwardsto .Also, when isan oddinteger,thatis,when isonemorethananoddmultipleof .Thesearetheonlytwopossibilities.So, startingfrom ,wecangobackwardsto ;from wecangobackwardsto ;from wecangoto or ;from we get ;from weget or ;andsoon.Inthiswaywecanbuildupaninfinitetreeofnumbers.Butthistree containseverynaturalnumbersincewecanalwaysworkupthetreefrom untilwefindanynumberwewant. Henceeverynumbermustreach wheniteratingtheCollatzfunction. Doyouseetheproblemwiththisproof?Well,thisisessentiallywhatGerhardOpfersproofboilsdowntoas
Follow

70 of 224

FollowTheMathLess Traveled 10/16/12 16:55


Get everynewpost deliveredto your Inbox.

The Math Less Traveled | Explorations in mathema...

http://mathlesstraveled.com/

well(thedetailsarenotexactlythesame,buttheformisprettymuchidentical).Thereisalotofstufffirst aboutlinearoperatorsovercomplexfunctions,butforthispartheisjustrelyingonworkthatsomeoneelse alreadydidanyway,andintheendhejustendsuplookingatcoefficientsofpowerseries,whichjustboilsdown tosomenumbertheory.Hebuildsatreebyinvertingacertainfunction(nottheCollatzfunctionbutaclosely relatedone),andthewholeargumentrestsonthefactthatthistreecontainseverynaturalnumberwhichhe statesbutdoesnotprove!AnditseemscleartomethatprovingthiswouldbenoeasierthanprovingtheCollatz conjectureitself.Intheend,forallthedetailedargumentandcontortion,hehascomebackpreciselytowhere hestarted. (Bytheway,Iamnotinterestedinreadingyour supposedproofoftheCollatzconjecture,sopleasedonotpost alinktoitinthecomments!)
Posted in open problems, proof | 43 Comments

Post without words #1


Posted on May 24, 2011

Posted in fibonacci, pattern, pictures, posts without words | Tagged fibonacci, partition | 13 Comments

Follow

71 of 224

FollowTheMathLess Traveled 10/16/12 16:55


Get everynewpost deliveredto your Inbox.

The Math Less Traveled | Explorations in mathema...

http://mathlesstraveled.com/

Cassinis identity
Posted on May 15, 2011

MypreviouspostaskedyoutotakeanyFibonaccinumber,squareit,andalsomultiplythetwoadjacent Fibonaccinumbers,andseeifapatternemerged.HeresatableImadeforthefirst6Fibonaccinumbers:

(Hmm,thenumbersinthatlastrowsurelookfamiliar)ItseemsthatthesquareofaFibonaccinumberandthe productofitstwoadjacentFibonaccinumbersalwaysdifferbyexactlyone.Moreover,whichoneisbigger alternates:thesquareisbiggerforoddvaluesof andtheadjacentproductisbiggerforevenvaluesof . Algebraically, . Thisisactuallytrue,andisknownasCassinisidentity,sinceitwasfirstpublishedbytheItalianastronomerGian DomenicoCassiniin1680.Letsproveit! First,wecancheckthatitholdswhen : . Nowwecanassumeitholdsforsome ,andshowthatitalsoholdsfor :

Sobyinduction,Cassinisidentityholdsforall anotherpost!)

.(Actually,thereisasensiblewaytodefinenegative

FibonaccinumberswhichmakesCassinisidentitytrueforallintegers ,butperhapsthatcanbethesubjectof

Posted in algebra, fibonacci, induction, pattern, proof, solutions | Tagged Cassini, fibonacci, identity | 8 Comments

A Fibonacci pattern
Posted on May 13, 2011

RecalltheFibonaccinumbers, , thesequenceofnumbersbeginningwith whereeachsubsequentnumberisthesumoftheprevioustwo:


Follow

72 of 224

FollowTheMathLess Traveled 10/16/12 16:55


Get everynewpost deliveredto your Inbox.

The Math Less Traveled | Explorations in mathema...

http://mathlesstraveled.com/

Trythis:pickanyFibonaccinumber.Squareit.Now,lookatthetwoFibonaccinumbersoneithersideofyour chosenone,andmultiplythem.Forexample,supposewepick . sideof are and ;theirproductis . .ThetwoFibonaccinumbersoneither

TrythiswithafewdifferentFibonaccinumbers.Betteryet,makeatableofyourresultsforthefirst Fibonacci numbers.Whatdoyounotice?Canyouexpressyourobservationalgebraically?Canyouproveitistrueforevery Fibonaccinumber?Whatdoesthishavetodowiththeareaparadox?


Posted in algebra, arithmetic, challenges, fibonacci, pattern, sequences | Tagged fibonacci, number, pattern | 4 Comments

Area paradox unmasked


Posted on May 8, 2011

InmylastpostIpresentedaparadox,whereasetoffourpiecesformingan88squarecouldapparentlybe rearrangedtoforma513rectangle,summoninganextraunitofareaoutofthinair. Quiteafewcommentersrealizedthatthepiecesdontactuallyquitefittogetherinthesecondfigure,leavinga smallgapwhichiscoveredupbythethickblacklines(that,orelsethesecondfigureusesslightlydifferentlyshapedpieces!).Herearethefiguresagain,thistimewithoutthethicklines:

Thatthinwhiteareainthemiddlehasanareaofyouguesseditonesquareunit. InasubsequentpostIllexplainwhythisworkssowell.AsnotedbyJM,itsnotacoincidencethatallthe dimensionsinvolvedareFibonaccinumbers


Posted in geometry, pictures, puzzles, solutions | Tagged area, Lewis Carroll, paradox, solution | 1 Comment

An area paradox

Follow

73 of 224

FollowTheMathLess Traveled 10/16/12 16:55


Get everynewpost deliveredto your Inbox.

The Math Less Traveled | Explorations in mathema...

http://mathlesstraveled.com/

Posted on May 2, 2011

HeresafunparadoxwhichhasbeenaroundforquiteawhileandwasapparentlyafavoriteofLewisCarroll.

Asyoucanverifyforyourself,thetwofiguresabovearecomposedoftwodifferentrearrangementsofthesame fourpieces.However,ifwecomputetheareasofthetwofigureswegetasurprise:theareaofthesquareis ,whereastheareaoftherectangleis !Wheredidtheextraunitofareacomefrom?

Feelfreetopostquestions,comments,thoughts,guesses,etc.below.(However,pleaserefrainfromcommenting ifyouhaveseenthisparadoxanditsexplanationbefore!)
Posted in challenges, geometry, paradox, pictures | Tagged area, Lewis Carroll, paradox | 18 Comments

Learn You a Haskell!


Posted on April 27, 2011

IhavewrittenseveraltimesbeforeaboutusingtheHaskellprogramminglanguageasa toolforexploringmathematics,butuntilnowIhadnogoodrecommendationforpeople whowereinterestedinlearningitforthatpurpose(RealWorldHaskellisagreatbookif youarealreadyaprogrammerandwanttoGetThingsDonewithHaskell,butnotso greatifyoujustwanttohavefunwithit). IamthereforehappytoreportthatNoStarchPresshasjustpublishedLearnYoua HaskellforGreatGood!ABeginnersGuidebyMiranLipovaa.Itsafantastic,fun, thoroughintroductiontoHaskell,spicedupbyMiransgreatsenseofhumorandzanyillustrations.Youcanbuy apapercopyorreaditonthewebforfree!


Follow

74 of 224

FollowTheMathLess Traveled 10/16/12 16:55


Get everynewpost deliveredto your Inbox.

The Math Less Traveled | Explorations in mathema...

http://mathlesstraveled.com/

Justforfun,inhonoroftheoccasion:

>--@powmodbaseexpm@efficientlycomputes(base^exp(modm)). >powmod::Integer->Integer->Integer->Integer >powmod_0_=1 >powmodbaseexpm|evenexp=(x*x)`mod`m >|otherwise=(x*x*base)`mod`m >wherex=powmodbase(exp`div`2)m > >fn=powmod2nn > >--http://oeis.org/A015910 >a015910=mapf[1..] > >--http://oeis.org/A050259 >test=all(==3) >(mapf[4700063497 >,3468371109448915 >,8365386194032363 >,10991007971508067 >,63130707451134435989380140059866138830623361447484274774099906755 >] >)

*Main>take50a015910 [0,0,2,0,2,4,2,0,8,4,2,4,2,4,8,0,2,10,2,16,8,4,2,16,7,4,26,16,2,4,2,0,8,4,18,28,2,4,8,16,2,22,2,16,17,4,2,16,30,2 *Main>:set+s *Main>test True (0.01secs,1110184bytes)

Posted in books, programming | Tagged Haskell, review, textbook, tutorial

Triangular number equations via pictures: solutions


Posted on April 14, 2011

Herearesomesolutionstomypreviouspost.However,theyarealmostcertainlynottheonlysolutions!Ifyou haveothercoolwaystovisualizeanyofthese(oranyothertriangularnumberequations)feelfreetopostinthe commentseitheralinktoanimageifyouhaveone,orjustadescriptionifyoudonthaveagoodwayto makeanimage.

Follow

75 of 224

FollowTheMathLess Traveled 10/16/12 16:55


Get everynewpost deliveredto your Inbox.

The Math Less Traveled | Explorations in mathema...

http://mathlesstraveled.com/

Posted in arithmetic, pattern, pictures, proof, solutions | Tagged number, pictures, proof without words, triangular | 10 Comments

Triangular number equations via pictures


Posted on March 24, 2011

TheotherdayIwasfiddlingaroundabitwithtriangularnumbers.Byonlydrawing picturesIwasabletocomeupwiththefollowingtriangularnumberequations,where denotesthe thtriangularnumber(thatis,thenumberofdotsinatriangularfigurewith dotsinthebottomrow, top). 1. 2. 3. 4. Now,noneofthesearehardtoprovealgebraically,butthatsnotthepoint.Canyoucomeupwithpicturesto illustratethevalidityofeachequation?Canyouuseapicturetofigureouthowtogeneralize#2inthesameway that#3generalizes#1? (thisoneisageneralizationofthefirstone)
The fourth triangular number

dotsinthenextrow,andsoon,andonedotonthe

Follow

76 of 224

FollowTheMathLess Traveled 10/16/12 16:55


Get everynewpost deliveredto your Inbox.

The Math Less Traveled | Explorations in mathema...

http://mathlesstraveled.com/

Posted in challenges, pictures, proof | Tagged equation, proof, triangular, visual | 8 Comments

Triangunit divisors and quadratic reciprocity


Posted on March 20, 2011

Recallthatthetriangunitnumbersaredefinedasthenumbersyougetbyappendingthedigit1totheendof triangularnumbers.Putanotherway, where denotesthe thtriangularnumber,and the th triangunitnumber.Thechallenge,posedbyPatrickVennebush,istodiscoverandprovesomethinginteresting aboutthedivisorsoftriangunitnumbers.(Therearespoilersahead,soifyoustillwanttothinkaboutthis problemyourselffirst,stopreadingnowandgoreadmypreviouspostinstead.) Asyoumayhavediscovered,allthedivisorsoftrianguntnumbersseemtoendineither1or9.Thisiscertainlya strikingpattern,andifitstrue,canweexplainwhy? First,wecanstartwiththeformulafortriangularnumbers(whichIhaveshownhowtoderivebefore), ,andcomputeasfollows:

Soeverytriangunitnumberisoftheform

forsomenaturalnumber .

Thenextthingtonoticeisthatifwetakeanytwonumberswhichendwith1or9andmultiplythemtogether, theirproductalsoendswith1or9.Thelastdigitofaproductdependsonlyonthelastdigitsofthefactors; multiplyingby1doesntchangeanything;and9times9is81whichendswith1.Anotherwaytolookatitisto notethat9isthesameas(-1)whenworkingmod10,sowerereallyjustsayingthat .

Soitsenoughtoshowthatalltheprimedivisorsoftriangunitnumbersendwith1or9;ifthisistruethenall theotherdivisors,beingproductsoftheprimedivisors,willalsoendwith1or9. AsInotedbefore,theOEISpageforthetriangunitnumbersliststhefollowingproof,duetoNickHobson: If isanoddprimedifferentfrom5then or . implies ,whence

Butthisisextremelytelegraphic,andonlymakessensebyitselfifyouhaveagoodbackgroundinnumbertheory soitsagreatprooftoincludeontheOEISbutnotsogoodforalltherestofus!Thankfully,AdamGlesser
Follow

77 of 224

FollowTheMathLess Traveled 10/16/12 16:55


Get everynewpost deliveredto your Inbox.

The Math Less Traveled | Explorations in mathema...

http://mathlesstraveled.com/

flesheditoutabitinacommentonmypreviouspost.Imre-formulatinghisexplanationinmyownwords, partlytotrytomakeitasaccessibleaspossible,andpartlytomakesureIunderstandit! Suppose isaprimenumberwhichdividesthetriangunitnumber of5alwaysendwith0or5). Thefactthat isafactorof canbewrittenusingamodularequationas . (Incaseyouhaveneverseenthissortofequationbefore, remainderwhendividedby .) ThenextstepisthecrucialonewhichIneverfiguredoutbutIdontfeeltoobadaboutit(andneithershould you,ifyoudidntfigureitouteither)sinceitrequiressomeexperienceandbackgroundinnumbertheoryto thinkofit.Theideaistocompletethesquaresincethisletsususequadraticreciprocity.Whatsthat,yousay? Readon!Wemultiplybothsidesoftheequationby20,thendoabitofrearranging: meansthat and havethesame .Obviously, cannotbe2since

triangunitnumbersalwaysendwith1(andarethereforeodd). alsocannotbe5forthesamereason(multiples

OK,sowhyisthishelpful?Thisiswherethelawofquadraticreciprocitycomesin.Illexplainexactlywhatitis inmynextpost,butinthisparticularinstance,fromthefactthat5iscongruenttoasquare toconcludethat mustalsobecongruenttoasquare ,itallowsus (althoughnotnecessarilythesamesquare!). :

Well,therearenttoomanypossibilitiesforsquareswhenworking

Wealreadyknow isnot5.Soitmustbecongruentto1or4 Butitcantendwith4or6becauseitisprime.Tada!

.Thatmeansthatitendswith1,4,6,or9.

Butwhataboutthatmagicalquadrilateralrecipricosi-thingummyorwhateveritwasinthemiddle!?Ihearyou cry.Neverfear,Ishallexplainitsoon!Itisratherfamousandinterestinganddeservesitsownpost
Posted in arithmetic, modular arithmetic, number theory, primes, proof | Tagged OEIS, quadratic, reciprocity, triangular, triangunit | 5 Comments

Moved to wordpress.com
Posted on March 20, 2011

IspoketoosoonbeforeoverthepastseveralweeksIvecontinuedtohaveproblemswithmyhosting,soI finallybitthebulletandtransferredeverythingtowordpress.com(theURLofmyblogisthesameasbefore thoughmathlesstraveled.com).UnfortunatelythisprobablymeansthatthereareabunchofbrokenimageFollow

78 of 224

FollowTheMathLess Traveled 10/16/12 16:55


Get everynewpost deliveredto your Inbox.

The Math Less Traveled | Explorations in mathema...

http://mathlesstraveled.com/

links,linkstooldpostsmaynotworkanymore(alltheoldpostsarestilltherethough,justmaybeunder differentURLs),Icanthavemybookshelfwidgetinthesidebaranymore,anditwilltakemeawhiletoget everythingsetupandformattedthewayIdlikebutIcanbreathealoteasieraboutuptime!IthinkIcanreally meanitthistimewhenIsayexpectsomenewgreatpostssoon!


Posted in meta | Tagged hosting, wordpress

Triangunit divisors
Posted on March 1, 2011

HeresaneatproblemfromPatrickVennebushofMathJokes4MathyFolks: Appendthedigit1totheendofeverytriangularnumber.Forinstance,from3youdget31,andfrom 666youdget6,661.Nowtakealookatallofthedivisorsofthenumbersyouvecreated.Whatarethe unitsdigitsofthedivisorsforeverynumbercreatedinthisway?Canyouprovethatthisresultalways holds? JustforfunIhavedubbedthesenumberstheonesyougetbyappendingaunittotriangularnumbersthe triangunitnumbers.Ivefoundapatternandevencheckeditforthefirst10,000triangularnumbersusinga computer,butsofaraproofhaseludedme! Bytheway,thetriangunitnumbersaresequenceA062786intheOnlineEncyclopediaofIntegerSequences,but thatpagecontainsaspoiler(Ithink,Ididntreadit)sodontpeekifyouwanttofigureitoutyourself!
Posted in number theory, pattern, puzzles | Tagged divisors, numbers, triangular, triangunit, unit | 9 Comments

My blog is back (hooray!) and a quick puzzle


Posted on February 22, 2011

Myblogisfinallybackupafteralonghiatusduetohostingproblems(notthefirsttime).Imseriously contemplatingmovingsomewheremorereliable,butitshardtocomeupwithasolutionthatwouldnot(a) invalidateanylinkstomyblogorparticularposts,(b)costalotofmoney(mycurrenthostingisfree),or(c) requirealotofworktoconvert/transfereverything.Ideaswelcome. Ihopetogetbacktobloggingsoon.Inthemeantime,heresanoldchestnutforyoutopuzzleover,ifyou haventalreadyseenit: Acertainmathclassmeetseveryweekday.OneFridayattheendofclass,theprofessortellstheclass thatsometimeduringthenextweek,theyaregoingtohaveasurprisequiz.Iamnotgoingtotellyou onwhatdaythequizwillbe,shesays.TheonlythingIwillsayforsureisthatitwillbea surpriseyouwillnotknowwhatdaythequizwillbeuntilthemomentIhanditoutatthebeginning ofclass. Whenwillthequizbe? Feelfreetopostquestions,ideas,andsolutionsinthecomments(sodontpeekatthecommentsifyouwantto

Follow

79 of 224

FollowTheMathLess Traveled 10/16/12 16:55


Get everynewpost deliveredto your Inbox.

The Math Less Traveled | Explorations in mathema...

http://mathlesstraveled.com/

thinkaboutityourselffirst).
Posted in logic, meta, puzzles | Tagged logic, puzzle, quiz, surprise | 8 Comments

Prime Time in Haskell


Posted on January 6, 2011

Inarecentblogpost,PatrickVennebushofMathJokes4MathyFolksnotedthat2011canbeexpressedasa sumofconsecutiveprimenumbers,andchallengedhisreaderstoworkouthow.Healsoposedacouplefurther challenges: Asitturnsout,2011isextracoolbecauseitcanbewrittenasasumofaprimenumberofconsecutive primenumbers. Whenwillthathappenagain? Andfinally Whatisthenextyearthatwillbeaprimenumberandalsobeasumofaprimenumberofconsecutive primenumbers?(Wow,thatsamouthful,aintit?) JustforfunIdliketosharehowIcomputedsolutionstoPatrickschallengesinmyfavoriteprogramming language,Haskell.Itsgreatfordoingthissortofrecreationalmathematics(amongmanyotherthings),asIhope youwillsee.Dontworryifyoudontknowit,Ivetriedtowritethispostinsuchawaythatanyonecanfollowit tosomeextent.Also,thispostisLiterateHaskell;youcancopyandpastetheentiretextofthepostdirectlyinto afilewithan.lhsextensionandloaditinaHaskellinterpreterlikeghci. WARNING:spoilersahead!!Ifyoudliketoworkoutthechallengesyourself,stopreadingnow. Okay,firstthingsfirst.Somesmartpersonhasalreadyputtogetheranicelibraryforefficientlylistingprime numbers,soletsuseit:

> import Data.Numbers.Primes (primes, isPrime)

Afewotheroddsandendswewillneed:

> import Control.Arrow ((>>>), (&&&)) > import Data.List (tails, find, intercalate) > > infixl 0 >$> > x >$> f = f x -- backwards function application

First,letswriteafunction,primeSums,whichtakesatargetnumberandfiguresoutifitcanbeexpressedasasum ofconsecutiveprimes.
Follow

80 of 224

FollowTheMathLess Traveled 10/16/12 16:55


Get everynewpost deliveredto your Inbox.

The Math Less Traveled | Explorations in mathema...

http://mathlesstraveled.com/

> -- Find runs of consecutive primes that sum to n. > primeSums :: Int -> [(Int, Int)]

ThissaysthatprimeSumstakesanIntasinputandreturnsalistoforderedpairs.Thefirstnumberineachpairis theindexofthefirstprimenumberintherun(2isthe0thprime,3isthe1stprime,andsoon),andthesecond numberisthelengthoftherun.Forexample,primeSums


120 = [(8,4)],since120isequaltothesumoffour

consecutiveprimesstartingwiththeninthprimethatis,120=23+29+31+37. WeregoingtowriteprimeSumsbystartingwiththelistofallprimenumbersandtransformingitincrementallyby applyingaseriesoffunctionstoit.Notethatitreallyistheinfinitelistofallprimenumbersoneofthevery nicefeaturesofHaskellisthatitislazy,whichmeansthatwecanprogramwithlistsandotherdatastructures thatareinfiniteintheory,butonlythepartsweactuallyendupusingwilleverbecomputed. Sincewearelookingforrunsofatleasttwoconsecutiveprimesthataddupton,primeslargerthanhalfofnare obviouslyofnouse.Sowestartbykeepingonlytheprimesuptohalfofn[ThankstoSteveforthecorrection;see commentbelow!]:

> primeSums n = > primes >$> takeWhile (< n)

Nextweapplytailswhichgivesusthelistofallsuffixes,or"tails",ofalist.Forexample,tails

[1,2,3] =

[[1,2,3], [2,3], [3], []].Thisissowecanconsiderallthepossiblestartingplacesforrunsofconsecutive

primes.Wedontwanttheemptylistattheendsoweuseinitwhichthrowsawaythelastelementofalist.

> >

>>> tails >>> init

Wenowhavealistoflistsofprimenumbers,eachliststartingatadifferentplace.Wewanttoapplythesame processingtoeachlist,whichwecandowithmap:

>

>>> map (

Witheachlistofprimeswefirstcomputeallthepartialsumsusingscanl
[1,2,3,4] = [0,1,3,6,10].

(+) 0.Forexample,scanl (+) 0

>

scanl (+) 0

The0atthebeginningisclearlyuselesssowethrowitawaywithtail.
Follow

81 of 224

FollowTheMathLess Traveled 10/16/12 16:55


Get everynewpost deliveredto your Inbox.

The Math Less Traveled | Explorations in mathema...

http://mathlesstraveled.com/

>

>>> tail

Weonlycareaboutsumswhicharelessthanorequaltoourtargetnumbern,sowekeeponlythatpartofthe list.(NotethatduetoHaskellslaziness,thebiggersumsthatwouldhavecomelaterinthelistwontevenget computed.)

>

>>> takeWhile (<= n)

Soatthispoint,ifwehadstartedwiththelist[2,3,5,7,11,13,17,19,23]andthetargetnumber28,wewouldhave
[2,5,10,17,28](2=2;5=2+3;10=2+3+5;;28=2+3+5+7+11;andthesumsincluding13and

beyondaretoobig,sotheygetthrownaway). Now,allwereallycareaboutisthelastnumberinthelistwehavesofar,sinceitiseitherequaltothetarget number,meaningwehavefoundasequenceofconsecutiveprimessummingtoit,oritislessthanthetarget number.Wealsowanttorememberthelengthofthelistsincethattellsushowlongtherunofconsecutive primesis.

> >

>>> length &&& last )

&&&allowsustoapplytwofunctionstoasinglevalueandgetoutapairofvalues.Forexample,inthiscase (length &&& last) [2,5,10,17,28] = (5,28).

Okay,wearenowdoneprocessingeachlistofprimes,andintheplaceofeachlistweareleftwithapairof numbers;thosepairswhosesecondnumberisequaltothetargetnumbermarkplaceswhereaconsecutiverunof primesaddeduptothetargetnumber.Wewanttorememberwhichpairiswhich(soweknowwhereeachrun started),sowefirstusezip


[0..]tocombineeachpairwithitsindex.

>

>>> zip [0..]

Nowtheelementsofthelistlooklike,forexample,(0,

(5,28))whichmeansthatstartingfromindex0thereisa

sequenceoffiveconsecutiveprimeswhichaddupto28. Weonlywanttokeeprunsthatactuallyadduptothetargetnumber;filterkeepsonlythoseelementsofalist thatsatisfysomecondition:

>

>>> filter ((== n) . snd . snd)

Finally,wecanthrowawaythesumfromeachelementsinceweknowwhatitis;wekeeponlythestartingindex

Follow

82 of 224

FollowTheMathLess Traveled 10/16/12 16:55


Get everynewpost deliveredto your Inbox.

The Math Less Traveled | Explorations in mathema...

http://mathlesstraveled.com/

andlengthofeachrun.

>

>>> map (fst &&& fst . snd)

Andthatsit!WecantryitoutbyloadingthisfileinghciandevaluatingprimeSumsappliedtovariousarguments attheprompt:

*Main> primeSums 28 [(0,5)] *Main> primeSums 311 [(4,11),(10,7),(15,5),(25,3)] *Main> primeSums 2011 [(36,11),(120,3)]

Asexpected,28isthesumoffiveconsecutiveprimesstartingwiththe0th(thatis,2).311canbewrittenasa sumofconsecutiveprimesinfourdifferentways,startingwitharunof11consecutiveprimesstartingwiththe primeatindex4(thatis,11).Andsureenough,2011canbewrittenasthesumof11consecutiveprimes startingatindex36,orthreeconsecutiveprimesstartingatindex120.Uhwhichprimesarethose?Letswrite alittlefunctionto"expand"runssowecanactuallyseewhattheyare:

> expandRun :: (Int, Int) -> [Int] > expandRun (i,n) = primes >$> drop i >>> take n

Giventhepair(i,n)representingastartingindexiandnumberofprimesn,wesimplytakethelistofprimes, dropthefirstiofthem,andthentakethenextn.

*Main> map expandRun (primeSums 2011) [[157,163,167,173,179,181,191,193,197,199,211],[661,673,677]]

Thatsbetter!Apparently2011=157+163+167+173+179+181+191+193+197+199+211= 661+673+677.Andwhynotaddthemupjusttomakesure?

*Main> map (sum . expandRun) (primeSums 2011) [2011,2011]

Ohgood! NowontothenextbitofPatrickschallenge.Whenisthenexttimetheyearwillbeexpressibleasthesumofa primenumberofconsecutiveprimenumbers?First,weneedafunctiontotestwhetherthisistrue:

Follow

83 of 224

FollowTheMathLess Traveled 10/16/12 16:55


Get everynewpost deliveredto your Inbox.

The Math Less Traveled | Explorations in mathema...

http://mathlesstraveled.com/

> isSumOfPrimeConsecPrimes = any (isPrime . snd) . primeSums

isSumOfPrimeConsecPrimessimplycomputestheprimeSumsofanumberandthencheckswhetheranyofthesecond

(snd)elementsofthepairsreturnedareprime(remember,thesecondelementsgivethelengthsoftheruns). Letsmakesureitstruefor2011:

*Main> isSumOfPrimeConsecPrimes 2011 True

Sureenough.Andwhenisthenextone?

*Main> find isSumOfPrimeConsecPrimes [2012..] Just 2015 *Main> primeSums 2015 [(76,5)]

Lookslikewewonthavetowaitverylong;2015isasumoffiveconsecutiveprimes.Butwaitaminute,2011is asumofaprimenumberofconsecutiveprimesintwodifferentways.Whenisthenexttimethatwillhappen? (Patrickdidntaskthat,butImjustcurious!)


iSoPCPiMW(isSumofPrimeConsecutivePrimesinMultipleWays)checksjustwhatyouwouldthinkitdoes;we

gettospecifyhowmanywayskwerequire.Itkeepsonlyrunswhoselengthisprime,countshowmanysuch runsthereare,andtellsuswhetherthatnumberisnolessthank.

> iSoPCPiMW k = (>=k) . length . filter (isPrime . snd) . primeSums

*Main> find (iSoPCPiMW 2) [2012..] Just 2155 *Main> find (iSoPCPiMW 3) [2012..] Just 2303 *Main> find (iSoPCPiMW 4) [2012..] Just 2867

Well,wehavetowaitabitlongerforthis.Theyearwontbeexpressibleasasumofaprimenumberof consecutiveprimesintwowaysagainuntil2155!Andasyoucansee,justforfunIalsofoundthenexttimes whentherewillbethreeorfoursuchways.(Interestingly,Ialsotriedsearchingforayearwithfiveways;butmy computerisstillsearchingaftertenminutes,eventhoughalloftheabovesearchestooklessthanasecond!So eitherthereisntone,oritwonthappenforaverylongtime,butIdontknowwhich.Maybeyoucanfigureit out?) WhataboutPatrickslastquestion?2015clearlyisntprime.Well,wecanjustsearchthroughthelistofprimes

Follow

84 of 224

FollowTheMathLess Traveled 10/16/12 16:55


Get everynewpost deliveredto your Inbox.

The Math Less Traveled | Explorations in mathema...

http://mathlesstraveled.com/

insteadofsearchingthelistofallnumbersbiggerthan2011:

*Main> find isSumOfPrimeConsecPrimes $ dropWhile (<=2011) primes Just 2081

2081,eh?Thatsstillalongtimefromnow,butunlike2155theresasmallchanceIllstillbealivethen(Iwould be99)! Well,Ihopethishasbeenfun.Feelfreetoleaveacommentwithanyquestions.Ifyoureinterestedinlearning moreaboutHaskell,youcandownloadithere,andIrecommendstartingbyreadingLearnYouaHaskellfor GreatGood!.


Posted in arithmetic, number theory, primes, programming | Tagged consecutive, Haskell, primes, sum | 8 Comments

Vi Hart's math doodles


Posted on December 14, 2010

ViHartsmathdoodlevideoshavebeenmakingtheroundsofthemathblogosphere(Iheardofthemfrom DeniseatLetsPlayMath!).Therearefourvideossofarandtheyarewellworthwatchinginspirational, creative,fullofgreatmath,andjustplainfun! OneofthesedaysIamgoingtofigureouthowtomakesomefunvideosofmyown


Posted in games, humor, links, people, video | Tagged doodles, hart, vi, video | 2 Comments

The mathematics of human knots


Posted on November 19, 2010

Haveyoueverplayedhumanknot?Itseemstobeacommonicebreaker/team-buildingsortofexercise(justdo aGooglesearchforhumanknotandyoullfindquiteafewpagesdiscussingit).Everyonestandsinacircle, thenreachesacrosswithbothhandstorandomlygrabthehandsoftwoothers.Thegoalisthentountanglethe knotintoacircle,withoutlettinggoofanyhands.

IplayedthisgameseveraltimesgrowingupandIneverthoughttwiceaboutit.Manyofthematerialsthatcame upfirstwhenIsearchedonlineseemtoindicatethatitwillalwaysbepossiblefortheparticipantstountangle themselvesintoacircle.Ifoundafewplacesthatacknowledgeditisnotalwayspossible,sinceinsteadyou


Follow

85 of 224

FollowTheMathLess Traveled 10/16/12 16:55


Get everynewpost deliveredto your Inbox.

The Math Less Traveled | Explorations in mathema...

http://mathlesstraveled.com/

mightgettwoorthreeseparatecircles.Onesitesuggestedthatiftheparticipantsseemedreallystuck,the facilitatormightallowtwoofthemtoletgooftheirhandsandthenrejointhemontheothersideofsome obstruction;butitwasphrasedasmoreofaconcessiontopreventtheparticipantsfromgettingtoo frustratedthanasanecessity. Butmathematically,thisisallhogwash:itispossibletogetarbitrarilycomplicatedknots(or,moregenerally, links,whichareknotsformedfromseveralseparateloops)bythisprocess,wherenomatterhowhardthe participantstrytountanglethemselves,theywillneverreachacircle.Forexample,hereisafigureeightknot:

Itmaynotbeimmediatelyobviousyoucantturnthisintoacirclejustbyfiddlingaroundwithit,butyoudont havetotakemywordforit:itcanbemathematicallyproventhatyoucant.(Perhapsthatsasubjectforanother post.)Ileaveitasanexercisetofigureouthowpeoplestandingaroundacirclecouldgraboneanothershands inordertoformafigureeightknot.ThegamereallyoughtbecalledHumanUnknot,sinceifyouactuallyform aknotthegameisnofunatall! Knotscanalsohavearbitrarilyhighunknottingnumber ,whichistheminimumnumberoftimestheknotwould needtopassthroughitself(correspondingtobreakingandrejoiningonelinkinthechain)inordertobecome unknotted;soallowingtheparticipantsonebreak/relinkmightnotbeenough. Peopleplaythisgameallthetime,andyetmostofthedescriptionsofthegamegrosslymisunderstandthe possibleoutcomes.Howcanthisbe?Forme,itraisesthefollowingquestion: Ifnpeoplestandaroundacircleandrandomlytakeoneanothershands,whatistheprobabilitythattheyforma knot? Therearealotmorequestionsonecouldask,andIveleftthequestionsortofvagueonpurpose;toreally answeritwouldrequirepinningdownwhatwemeanmuchmoreprecisely.ButfornowIlljustopenitupto discussion!
Posted in games, people, probability | Tagged games, human, icebreaker, knot, team, teambuilding | 9 Comments

Fractals without a computer


Posted on November 15, 2010

Follow

86 of 224

FollowTheMathLess Traveled 10/16/12 16:55


Get everynewpost deliveredto your Inbox.

The Math Less Traveled | Explorations in mathema...

http://mathlesstraveled.com/

Whathappenswhenyouhookupthreeprojectorstoasinglevideocamera,andthenpointthecameraatthe projectedimages? RIDICULOUSLYAWESOMETHINGS,thatswhat!

Posted in fractals, video | Tagged camera, fractals, projectors, video | 6 Comments

Book Review: The Mystery of the Prime Numbers


Posted on November 7, 2010

Severalmonthsago,MatthewWatkinssentmeareviewcopyofhisnewbook,Secretsof CreationVolumeOne:TheMysteryofthePrimeNumbers.Itstakenmeawhiletoget aroundtoreviewingit,butnotforlackofenthusiasm.Letmecutrighttothechase: Thisisafantasticbook.Afabulousbook.Asplendiferousbook! Itseasyenoughtosaywhatthebookisabout:itexplainssomeextremelyfascinating propertiesofthedistributionofprimenumbers.Butthatdoesnttellyouwhyitsso great;booksaboutprimenumbersareadimeadozen. Firstofall,Watkins(withthehelpoftheillustrations,whichIllgettoshortly)managestoexplaineverything extremelyclearly,beginningfromfirstprinciplesandIreallymeanfirstprinciples;hebeginswiththedefinition ofthenaturalnumbersandgoesfromthere.Theonlythingthatcouldpossiblystandbetweenelementaryschool studentsandanappreciationofthisbookistheirreadingabilityandattentionspan,NOTtheirmathematical abilities.ButandhereisthetrulyastoundingthingI,aPhDstudentwhohasstudiedmathmywholelife, couldnotputthisbookdown.NotonlywasInotbored,Ilearnednewthings!Howisthispossible?Partly,itwas duetothefresh,creative,insightfulwayeverythingisexplained,coupledwithexcellentwriting;partly,itwas duetothesubject,whichstartsconventionallyenoughbutsoonwandersintofascinatingterritoryunfamiliarto manymathematicians(includingthisone).Abooklikethis,accessibletoyoungchildrenandengagingtoadults, isarareandwonderfulaccomplishmentindeed!
Follow

87 of 224

FollowTheMathLess Traveled 10/16/12 16:55


Get everynewpost deliveredto your Inbox.

The Math Less Traveled | Explorations in mathema...

http://mathlesstraveled.com/

Ohyes,andtheillustrations.Theillustrations!TheillustrationsbyMattTweedmakemesohappy.Thereisthis cutelittlewizard(youcanseehimonthecoverofthebookabove)andthesetwocutelittlechildrenandthey areconstantlyengagedinvarioussillyhijinksthatbothadmirablyillustratethecontentofthetextandsimply bringasmiletoyourface.Andtherearerobotsandpilesofbeansandaliensandladybugsandmoleculesand Myonlycriticismlieswiththefirstchapter,whichWatkinsspendsonasortofintroductoryrantaboutthe unhealthyobsessionofourculturewithallthingsquantitative.WhileItendtoagreewithhimaboutmuchofit, itcomesoffasheavy-handedandmuchtoolong.Butdontletitstopyou:asWatkinshimselfsuggests,youare freetoskipthefirstchapter. IeagerlyawaitVolumeII(tobepublishednextyear)!


Posted in books, number theory, primes, review | Tagged book review, mystery, numbers, prime | 3 Comments

Penn Alexander: subset counting and Gray codes


Posted on November 2, 2010

ImvolunteeringagainthisyearwiththemiddleschoolmathclubatPennAlexander.Imgoingtotrytobe betterthisyearaboutpostingwhatwedoeachweek,forposterityssakeandincaseitinspiresanyoneelse!

Lastweek,wegotoutahugebucketofred,blue,yellowandgreensquaretiles,andIstartedbyaskingthemto countthenumberofsubsetstheycouldformfromred,blue,andyellowtilesIdidntusethewordsubset, though,IjustdemonstratedwhatImeantwithafewexamples.Thispromptedimmediatewildguessing(six! nine!)butalsosomegoodquestions,suchaswhetherduplicatetileswereallowed(no)andwhethertheorderof thetilesmattered(no).OnesurprisewasthatIreallyhadtoencouragethemrepeatedlytogetoutenoughtiles thattheycouldactuallyformallthesubsetsrightthereinfrontofthem;initiallymostweresittingtherewith threetilesandjusttryingtomovethemaroundandcountintheirhead.IfIdiditagainIwouldspendmore timeencouragingthemtothinkaboutsystematicwaystoensurethattheywerentmissinganypossibilities.

Anyway,eventuallytheyallagreedonseven,andthenwetalkedaboutincludingtheemptysubset.Nextwe addedthegreentileandtheyeventuallyallagreedonsixteen.Theywereabletocountthesubsetswithoneand twotilesintheirheadsbythispoint,andthentheyallprettyquicklycaughtontothepattern(thatthenumber ofsubsetsdoubleseachtimeyouaddanewtile)althoughonlyafewunderstoodwhythiswasso.

FinallyIaskedthemwhethertheycouldtakeallthethree-tilesubsetsandputtheminanordersothatanytwo adjacentsubsetsdifferedonlybytheaddition(ordeletion)ofasingletile.Thiswasthehardestparttoexplain, andIhadtodemonstratewhatImeantseveraltimes.Buteventuallymostofthemunderstoodwhatwewere tryingtodo,andafewdidfigureitoutonestudentevenfiguredoutanorderingforallsixteenfour-tile


Follow

88 of 224

FollowTheMathLess Traveled 10/16/12 16:55


Get everynewpost deliveredto your Inbox.

The Math Less Traveled | Explorations in mathema...

http://mathlesstraveled.com/

subsets,andwithabitofpromptingfrommefiguredoutageneralalgorithmforfindingorderingsforevenmore tiles!

Theinspirationforthisactivitycamefromthiswebsite,whereyoucanfindabunchmoreinformationand resources.
Posted in counting, pattern | Tagged code, counting, Gray, Penn Alexander, subsets | 1 Comment

Back in business
Posted on October 24, 2010

Ihadsomehostingissuesforawhiletherebutshouldbebackinbusinessnow.Thedatabasehadtoberestored fromabackupsoIlostmymostrecentpost,buthadthetextofitsavedelsewhereandjustre-postedit.Ihope togetbacktopostingsoon!


Posted in meta | 1 Comment

Book reviews: Math Jokes 4 Mathy Folks and Easy as Pi


Posted on October 24, 2010

MathJokes4MathyFolks

Afewmonthsago,PatrickVennebushwaskindenoughtosendmeareviewcopyofhisnewbook, MathJokes4MathyFolks.Itsatreasure-troveofmath-relatedjokeswithahugerangeof material,fromwordplaytolightbulbjokestovisualgagstolongerstories,fromelementaryto graduatelevelmathematics,fromcornytosubtle.Trulytheressomethinghereforeveryone!Here areacoupleofmyfavorites: Astatisticsstudentacceleratedbeforecrossingeveryintersection.Hispassengerfinallyasked,Whydo yougosofastthroughintersections?Thestudentreplied,Statisticallyspeaking,yourefarmorelikely tohaveanaccidentatanintersection,soItrytospendlesstimethere. Whenthelogicianssonrefusedtoeathisvegetables,thefatherthreatenedhim,Ifyoudonteatyour veggies,youwontgetanyicecream!Theson,frightenedattheprospectofnothavinghisfavorite dessert,quicklyfinishedhisvegetables.Afterdinner,impressedthathissonhadeatenallhisvegetables, thefathersenthissontobedwithoutanyicecream. Patrickhasalsostartedablogasanextensionofthebook,whichcontainsjokessubmittedbyreaders,new jokesdiscoveredbytheauthor,detailsaboutspeakingappearancesandworkshops,andotherrandombitsof informationthatmightbeinterestingtothestrangefolkswholikemathjokes.Thatwouldbeme!
Follow

89 of 224

FollowTheMathLess Traveled 10/16/12 16:55


Get everynewpost deliveredto your Inbox.

The Math Less Traveled | Explorations in mathema...

http://mathlesstraveled.com/

EasyasPi:TheCountlessWaysWeUseNumbersEveryDay
IalsorecentlyreceivedareviewcopyofEasyasPi:TheCountlessWaysWeUseNumbersEveryDay,byJamie Buchan.Itsabitlightontechnicaldetailsformytastes;butthenagainIhaveprettyweirdtastes.Puttingaside mypersonalloveofrigor,itsfulloffunandinterestingtidbitsaboutthehistory,sociology,andmathbehindthe numbersthatshowupallovertheplaceinourculture.Thisisagreatbooktokeepnexttoyourtoilet(andI meanthatinthebestpossibleway!)tointrigueyournon-mathematicalguests.
Posted in books, humor, links, review | Tagged humor, jokes, numbers, pi

P vs NP: Whats the problem?


Posted on September 1, 2010

Aspromised(betterlatethannever),Imgoingtobeginexplainingthe(in)famousPvsNPquestion(seethe previouspostforabitmorecontext).Asastart,heresasuper-concise,30,000-footversionofthequestion: Arethereproblemswhosesolutionscanbeefficientlyverified,butwhichcannotbeefficientlysolved? Ofcourse,thisissovagueastobealmostmeaningless!Myjoboverthenextfewpostswillbetoun-vagueifyit foryou.Lookingatthestatement,threequestionsmayimmediatelycometomind,andwellconsidereachin turn: Whatcountsasaproblem? Whatdowemeanbyefficient? Whatdowemeanbyverify? Todaywellstartbyconsideringthefirstquestion:whatcountsasaproblem? ThePvsNPquestionisusuallystatedintermsofdecisionproblems,whichareproblemswithayes/noanswer. Forexample,IsthereawaytodrivefromheretoKalamazoowithoutdrivingonamajorhighway?Eitherthereisa waytodoit,orthereisnt,andwemighthopethatacomputer(givenanappropriatemapasinput)wouldbe abletotelluswhich.

Ofcourse,noteveryproblemisadecisionproblem.Forexample,HowlongdoesittaketogettoKalamazooifyou gobythefastestpossibleroute?Wemighthopethatacomputercouldanswerthisquestiontoo,buttheanswer wearelookingforisanamountoftime,ratherthanyesorno.Thisisnotanisolatedexample;therearelotsof


Follow

90 of 224

FollowTheMathLess Traveled 10/16/12 16:55


Get everynewpost deliveredto your Inbox.

The Math Less Traveled | Explorations in mathema...

http://mathlesstraveled.com/

interestingandimportantproblemslikethis.Sowhyrestrictourselvestojustdecisionproblems? Itseasytoseethatdecisionproblemsarethesimplestpossiblekindofproblem.Theanswercarriesjustasingle bitofinformation.(Theonlythingthatcouldpossiblybesimplerthanaproblemwithayes/noanswerisa problemwithano/noanswer,whichcarrieszeroinformation.Forexample,Doesthisdressmakemelook fat?Butthesearentthesortsofproblemsweareusuallyinterestedinsolvingwithacomputer.)Thereisalong scientifictraditionofstudyingthesimplestpossiblekindofsomething,inordertogaininsightintomore complicatedthingsaswell.Thatswhatweredoinghere. Butcanstudyingdecisionproblemsreallygiveusinsightintoproblemswithmorecomplicatedanswersaswell? Actually,yes!ToseeanexampleofwhatImean,consideragainourquestionHowlongdoesittaketogetto Kalamazooifyougobythefastestpossibleroute?Thisisaproblemwithacomplicatedanswer(youmightthink thatasinglenumberisnotverycomplicated,butyouhavetoadmitthatitiswaymorecomplicatedthana simpleyes/no!).Butnowconsiderthisrelatedquestion:IsitpossibletogettoKalamazooin minutesorfewer? Actually,thisisawholefamilyofquestionsparameterizedbythetime .Foranyparticular ,thisisclearlya decisionproblemwithayesornoanswer.Butifwehadacomputerprogramwhichcouldefficientlysolvethis problemforany ,wecoulduseittosolvetheoriginalproblemtoanydesiredaccuracy.Forexample,wemight askthefollowingseriesquestions,withtheanswertoeachquestionshowninbold: IsitpossibletogettoKalamazooin1000minutesorfewer?YES IsitpossibletogettoKalamazooin500minutesorfewer?NO IsitpossibletogettoKalamazooin750minutesorfewer?YES IsitpossibletogettoKalamazooin625minutesorfewer?NO andsoon Afteronlyafewmorequestions,wewouldlearnthatifwegobythefastestpossiblerouteandgoexactlythe speedlimitthewholetimeandneverstopforfoodorgasortousearestroom,itwouldtakeexactly665 minutestogettoKalamazoo. Thatsallverywell,youmightsay,Icanseehowthatworksforproblemswithnumericalanswers.Butwhat aboutanswersthataremorecomplicatedyet?Forexample,whatifwewantedtoknownotonlythetimeit takestogettoKalamazoobythefastestroute,butalsotherouteitself?Thisiscertainlynotjustanumber,itis alistofturns,oralistofstreetnames,orsomethingcomplicatedlikethat.Surelywecantreducethistojust somedecisionproblem? Well,nomatterhowcomplicatedtheanswertoaproblem,itisalwayspossibletoencodetheanswerinbinary, asasequenceofonesandzeros.Forexample,toencodeaseriesofturnswemightagreetoencodealeftturnas 01,arightturnas10,goingstraightas11,anduse00toindicatetheendofthedirections.So011110101100 wouldmeanTurnleft,thengostraight,thenturnrighttwice,gostraightagain,andyourethere.Inorderto encodestreetnameswewouldneedamuchmorecomplexencodingbutitcanbedoneeasilyenough.For examplewemightencodetheletterAas00000,Bas00001,Cas00010,andsoon. OK,sowhat?Heresthepunchline:givenaproblemPwithsomecomplicatedanswerandasuitablewayto encodetheanswertoPinbinary,wecanmakethefollowingdecisionproblem:
Follow

91 of 224

FollowTheMathLess Traveled 10/16/12 16:55


Get everynewpost deliveredto your Inbox.

The Math Less Traveled | Explorations in mathema...

http://mathlesstraveled.com/

Isthe thbinarydigitinthebinaryencodingoftheanswertoPaone? Ihopeyoucanseehowwecoulduseacomputerprogramcapableofsolvingthisproblemforany tosolvethe originalproblem:justaskaboutthebinarydigit(orbit)ateachposition,andbuilduptheanswertoPonebitat atime. Perhapsyouthinkthisisbitcontrived.Butitshowsthatinsomesense,theresnothingfundamentallyinteresting goingonwithproblemsthathavecomplicatedanswers;wecanalwaysreducethemtodecisionproblems.Sofor thesereasons(andperhapsafewothers),thePvsNPquestionisphrasedintermsofdecisionproblemsonly. WecannowrestatethePvsNPquestionabitmoreprecisely: Aretheredecisionproblemswhosesolutionscanbeefficientlyverified,butwhichcannotbeefficientlysolved? Inmynextpost,Illexplainwhatwemeanbythathand-wavywordefficiently.
Posted in computation, open problems | Tagged binary, decision, Kalamazoo, P vs NP, problem | 9 Comments

P NP?
Posted on August 10, 2010

Afewdaysago,VinayDeolalikar,aPrincipalResearchScientistatHPLabs,begancirculatingadraftofapaper entitledPNP.Themathematicsandcomputersciencecommunitiesimmediatelyeruptedinafrenzyof excitementandactivity. Themilliondollarquestion:whytheexcitement?Well,thatsexactlyit:itisamillion-dollarquestion.ThePvs NPproblemisoneofthesevenMilleniumPrizeProblemsestablishedin2000bytheClayMathematicsInstitute; eachproblemcarrieswithitaprizeof$1,000,000.OneoftheseventhePoincarConjecturehasalready beensolved,bytheRussianmathematicianGrigoriPerelman(whofamouslyrefusedtoacceptbothaFields Medalandthe$1million).Anditsnotevenreallyaboutthe$1millionprize;thePvsNPproblemiswidely agreedtobethemostsignificant(anddifficult)openquestionintheoreticalcomputerscience,soforsomeone tosolveitwouldbeaReallyBigDeal. So,didDeolalikarreallysolveit?Willhewinthe$1million?Wellitswaytooearlytotell!Hereareafew thingstokeepinmind: SincePvsNPissuchafamousproblem,therearetonsofattemptsatsolvingitpublishedallthetime.Most arebypeoplewhoeitherhaveanoverinflatedestimateoftheirmathematicalunderstanding,orbelievethe solutionwastoldtotheminadreambybenevolentaliens,orboth.Hencetheyarentevenreallyworth seriousmathematicianstimetoread.Sadbuttrue.Butthisisclearlynotthecasehere:Deolalikarisa respectedresearcherwhohasdonerelatedworkinthepast,andhis100-pagepaperiswell-writtenand demonstratesagoodunderstandingoftherelevantissues(youdonthavetotakemywordforit).Evenifthe proofendsuphavingsomesortoffatalflaw,itsclearthathehassomeinterestingnewideasthatmaylead tootherdiscoveries.Hencetheexcitement. Unfortunately,therehavealreadybeensomepossibleflawspointedout.Butkeepinmindthatanypaperof thismagnitudeisboundtocontaintonsoferrors,omissions,andtypos(ifyoudontbelieveme,trywriting
Follow

92 of 224

FollowTheMathLess Traveled 10/16/12 16:55


Get everynewpost deliveredto your Inbox.

The Math Less Traveled | Explorations in mathema...

http://mathlesstraveled.com/

onesometime!).Onlytimewilltellwhethertheseflawsturnouttobefatalproblemsthatinvalidate Deolalikarsentireapproach,mistakesthatcanbefixedrelativelyeasily,orjustmisunderstandingsonthe partofthepeoplewhopointedthemout. SupposingtheflawsarefixableandDeolalikarsproofendsupbeingacceptedbythemathematical community,itwillstillbequiteafewyearsbeforehecouldpossiblywinthe$1million.First,theproof needstobepublishedinamajormathematicaljournal(whichwillprobablytakeatleastayear),thenthere isamandatorytwo-yearwaitingperiod,andthenaspecialcommitteehastoexaminetheproofanddecide whethertoawardtheprize!Sodontholdyourbreath. Atthispoint,youmayalsobewonderingwhattheheckthePvsNPproblemactuallyis,andwhyitisso important.Fortunately,itstheoneMilleniumPrizeproblemthatIamactuallyqualifiedtoexplain,andinthe followingpostortwoIintendtodojustthat!(UnfortunatelyIamemphaticallynotqualifiedtoexplainanything aboutDeolalikarsattemptedproof.)Ivebeenintendingforquiteawhiletowriteaboutsomeinterestingtopics intheintersectionofmathematicsandcomputerscience(mydayjob,afterall,isasacomputerscientist!)and thiswillprovidetheperfectexcuse.
Posted in computation, links, open problems, people, proof | Tagged Deolalikar, Millenium, P vs NP, prize | 7 Comments

More cookies
Posted on July 27, 2010

IrecentlyreceivedthefollowinginterestingproblemfromShadowcat,whichisageneralizationofthecookie problemIvewrittenaboutpreviously.Weagainwanttocountthewaystodistributeidenticalcookiesto non-identicalstudents,withthetwistthatweimposeanupperboundonthenumberofcookiesreceivedbyeach student(quitereasonableifwewanttobemindfulofthestudentsnutrition): Imaginethatinsteadoftencookiesandfivestudents,youhavefiftycookiesandtenstudents.(Itseasier toquantifythissituationusinglargernumbers.)Howmanywayscanyoudistributethesecookies amongthestudentssothatnostudenthasanymorethantencookies? Studentsmaybegivenanynumberofcookieslessthanorequaltoten,includingzero.Thecookiesare identical,justasintheoriginalproblem,so,aswiththeoriginalproblem,itdoesntmatterwhich cookiethestudentgets,justhowmany.Butthestudents,again,are*not*identical,sowhichstudent getsaspecificnumberofcookiesisimportant. Iunfortunatelyhaventhadmuchtimetothinkaboutityet.Feelfreetoleavecomments,thoughts,partial solutions,andsolutionsinthecomments!
Posted in arithmetic, challenges, counting | Tagged cookies, counting, students | 9 Comments

Right answers for the wrong reasons


Posted on June 30, 2010

Follow

93 of 224

FollowTheMathLess Traveled 10/16/12 16:55


Get everynewpost deliveredto your Inbox.

The Math Less Traveled | Explorations in mathema...

http://mathlesstraveled.com/

HeresarecentxkcdwhichasamatheducatorIfoundparticularlyfunny.Somequestionsformyreaders: 1. Whatnumbersbesides3and9wouldworkhere? 2. Doyouhaveanyparticularlyfunnyorinterestingstoriesofstudentsgettingsomethingrightforthewrong reason?Posttheminacomment(or,betteryet,onyourownblogifyouhaveone!). Also,manypartsofmyoptimalchange-carryingchallengeremainunanswered(althoughthisislikelydueinpart totheunfortunatedowntimeIexperiencedshortlyafterpublishingit).Ifyouhaventalready,giveitatry,youre suretofindsomeaspectoftheproblemthatinterestsyou!


Posted in arithmetic, humor, links | Tagged arithmetic, errors, right for the wrong reason | 8 Comments

Optimal change-carrying
Posted on June 24, 2010

RecentlyMichaelleftthefollowingchallengeinacomment: Ivebeentryingtooptimizemychange-carryinghabits.Whatisthesmallestamountofquarters,dimes, nickelsandpenniesonecancarrywhilestillbeingabletogiveperfectchange(twodecimals)? Itsnot100%clearwhatMichaelmeantbygiveperfectchange,butletsassumethegoalistobeabletomake anyamountbetween1and99cents.Fornon-USreaders,UScoinsareworth1,5,10,and25cents. Somequestionsforexploration: 1. Whatsthesmallestnumberofcoinsyoucancomeupwiththatworks?Whatarethey? 2. Aretheremultiplesolutions,oristhesolutionunique? 3. Howcanyouprovethataproposedsolutionisoptimal?Unique? 4. Canyouanswerthequestionforadifferentsystemofcoins?Forexample,Iamcurrentlyspendingthe summerinCambridge,England,wherecoinsareworth1,2,5,10,20,and50pence.Whatifyoualsoinclude the1-and2-pound(100and200pence)coins,andwanttobeabletomakechangeforeveryamountupto5 pounds(thesmallestnote)? 5. TheUSandUKcoinsystemsbothhavethepropertythatagreedystrategyworksforgivingthesmallest amountofchange.Thatis,tomakeacertainamountofchangewiththesmallestpossiblenumberofcoins, youcanjustkeeppickingthebiggestcoinlessthanorequaltotheremainingamount.Whataboutcoin systemswithoutthisproperty?Dotheymakethisproblemharder?Easier? 6. Ifyougottodesignyourownsystemofcoinswithwhateverdenominationsyouwanted,howwouldyou
Follow

94 of 224

FollowTheMathLess Traveled 10/16/12 16:55


Get everynewpost deliveredto your Inbox.

The Math Less Traveled | Explorations in mathema...

http://mathlesstraveled.com/

designitsothattheminimumnumberofcoinsneededtomakeallamountsbetween1and99centsisas smallaspossible?AsLARGEaspossible? 7. Whataresomemethodologiesforattackingthissortofquestioningeneral? Feelfreetocomeupwithyourowngeneralizationsaswell.Postquestions,discussion,andsolutionsinthe commentsbutdontpeekuntilyouvetriedsolvingit!Ivepostedanotherchange-makingpuzzlebeforeaswell; thediscussiontheremightalsogiveyouideas.


Posted in arithmetic, challenges, counting | Tagged change, minimum | 11 Comments

Manufactoria
Posted on May 25, 2010

Afriendofminejustpointedmetoamostexcellentpuzzlegame,Manufactoria,whereinyoubuildlittle machinestotestrobots.FornowIwontgiveawaythesecretofwhatrealmath/computersciencetopicthe gameteachesyou,butIllwritemoreaboutitlater.Fornow,justhavefunplayingthegame!=)

Posted in games, links, logic, programming, puzzles | Tagged game, robot, test | 17 Comments

The broken weight problem: solutions and further exploration


Posted on May 11, 2010

Firstofall,letmesaytoallmyreadershowfantasticitfelttopostapuzzle,afternotpostinganythingfortwo months,andgeteighteenthoughtful,insightfulcommentsinjustthreedays;itseverybloggersdream.Youall arefantasticandmakethisalotoffunthanksforreading! IthoughtIdtakeapostjusttosummarizesomeoftheresponsesandsolutionstothebrokenweightproblem. Asmanycommentersrealized,thesolutionisthattheweightsareofsizes1,3,9,and27.Hmm,powersof threecoincidence?Ofcoursenot! Asseveralcommentersnoted,somethinginvolvingbasethreereadilypresentsitselfifwerealizethatthereare threepossibilitiesforeachweight:itcanbeontheleftofthebalancescale,ontheright,ornotonthescaleat all.Sinceonesideofthescalecorrespondstoaddingtheweightandtheothersidetosubtracting,weare essentiallywritingnumbersinbasethree,butusingthedigits-1,0,and1insteadoftheusual0,1,2.For example,25canbewrittenas10(-1)1,thatis, (ifwewerereallygoingtousethissystem wedwanttocomeupwithabettersymbolfor-1).Infact,thisisknownasbalancedternary,anditisafact(as provedbyafewcommenters)thatndigitsofbalancedternaryallowustouniquelyrepresenteveryinteger
Follow

95 of 224

FollowTheMathLess Traveled 10/16/12 16:55


Get everynewpost deliveredto your Inbox.

The Math Less Traveled | Explorations in mathema...

http://mathlesstraveled.com/

between

.Withfourdigits(asintheproblem)wecanuniquelyrepresenteveryintegerbetween-40and40.

Therewasabitofconfusioninthecommentsaboutbeingabletorepresentsomeintegersinmorethanone way,butIthinkifyoutryitoutyouwillfindthatthisisnotthecase. Fromthisproblem(generalizingittoarbitrarynumbersofweights),wecanseethat

JMnotedthatthisgeneralizesto

andwonderedwhetherthishasanythingtodowithsolvingtheproblem,orwithproblemslikeit.Indeedit does;heresoneforyou:supposeyouaretaskedwithdesigningasetofweights.Theweightsshouldmakeit possibletoweighasmanydifferentintegerweightsaspossible,withoutleavinganyout,justliketheweights1, 3,9,27makeitpossibletoweigheveryintegerweightupto40withoutleavinganyout.Theonedifferenceis thatyouwanttohavetwoidenticalcopiesofeachweight.Whatisthebestyoucando? Ivelefttheproblemslightlyvagueonpurpose,butIhopeyouwillhavefunsolvingitandfiguringoutwhatit hastodowithJMsobservation!Canyoucomeupwithotherinterestinggeneralizationsoftheproblem? Finally,SamShahpostedalinktoadescriptionofhisexperienceusingtheprobleminareal-lifeproblem-solving session.


Posted in arithmetic, challenges, number theory, solutions | Tagged balanced, broken, puzzle, ternary, weight | 6 Comments

The broken weight problem


Posted on May 1, 2010

HeresafantasticproblemIrecentlyheard.ApparentlyitwasfirstposedbyClaudeGaspardBachetdeMziriac inabookofarithmeticproblemspublishedin1612,andcanalsobefoundinHeinrichDorries100Great ProblemsofElementaryMathematics. Amerchanthadafortypoundmeasuringweightthatbrokeintofourpiecesastheresultofafall.When thepiecesweresubsequentlyweighed,itwasfoundthattheweightofeachpiecewasawholenumberof poundsandthatthefourpiecescouldbeusedtoweigheveryintegralweightbetween1and40pounds. Whatweretheweightsofthepieces? Notethatsincethiswasa17th-centurymerchant,heofcourseusedabalancescaletoweighthings.So,for example,hecouldusea1-poundweightanda4-poundweighttoweigha3-poundobject,byplacingthe 3-poundobjectand1-poundweightononesideofthescale,andthe4-poundweightontheotherside. Thesolutiontothispuzzleisreallyfascinatingandleadsintoallsortsoffungeneralizationsandothertopics;Ill writemorelater.Fornow,asalways,feelfreetoleavequestions,observations,andsolutionsinthecomments (sodontlookatthecommentsbeforeyouvesolveditifyoudontwanttoseetheanswer!).
Follow

96 of 224

FollowTheMathLess Traveled 10/16/12 16:55


Get everynewpost deliveredto your Inbox.

The Math Less Traveled | Explorations in mathema...

http://mathlesstraveled.com/

Posted in arithmetic, challenges, number theory, puzzles | Tagged puzzle, weights | 22 Comments

A gentle introduction to the 5th Polymath project


Posted on April 21, 2010

IhighlyrecommendreadingJasonDyersdescriptionoftheErdsdiscrepancyproblem,thesubjectofthemost recentPolymathproject(thePolymathprojectsareanexperimentinmassivelycollaborativemathematics,where anyoneatallcancontributesomethingtowardsasolution).TheErdsdiscrepancyprobleminparticularis fascinatingbecauseitisquiteeasytounderstandthestatementoftheproblemwithonlyaknowledgeofbasic arithmeticbutithasbeenunsolvedforalmost80years!!Jasonsexplanationoftheproblemisclearand engaging,andhascutepicturesgogiveitaread!


Posted in arithmetic, counting, links, number theory, open problems | Tagged discrepancy, Erds, polymath

Penn Alexander math club: map coloring


Posted on February 23, 2010

TodayinmathclubIhadthestudentsexploremapcoloring.Itriedtoleaveitasopen-endedaspossibleto startIjustsaidthatweweregoingtodrawmapswithcountries,andtrytogiveeachcountryacolor,sothat notwoadjacentcountrieshavethesamecolor.Iwascarefulnottospecifywhatacountryis,orwhatitmeans fortwocountriestobenexttoeachother! Prettymuchontheirown,theyfiguredouthowtodrawamapwithfourcountriesalltouchingeachother, whichthereforerequiredfourcolors.WhenIchallengedthemtodrawamapwithfivecountriesalltouching eachother,theycameupwithmapsinvolvingcountriestouchingatacorner,andwithsatellitedisconnected regionsthathadtobegiventhesamecolorasthemothercountry.Theyalsofiguredoutthatifweallowthese things,wecandrawmapsrequiringanarbitrarynumberofcolors,andconjecturedthatwithoutthesethingswe canthavefivecountriesalltouchingeachother.Ithentoldthemaboutthefour-colortheoremandwehadfun tryingtofour-coloramapofNorthAmerica(includingtheUSstates). ThenIshowedthemhowtointerpretmapsasgraphs,whymapscorrespondtoplanargraphs,andhowtoturn themap-coloringquestionintoagraph-coloringquestion.Ishowedthemthecompletegraphon5verticesand howitwouldcorrespondtohavingfivecountriesalltouchingeachother.ThenforfunIposedthethreeutilities problem,whichafterplayingwithforawhile,theycorrectlyguessedcouldnotbesolvedwithinthegiven constraints.Onestudentdidcomeupwithaningenioussolutioninvolvingapairofteleporters,which (althoughIdidntpointthisouttothematthetime)correspondstothefactthat of canbeembeddedona torus!Ithenshowedthemhowtointerpretthisalsoasastatementaboutgraphs(specifically,thenon-planarity ),andthentoldthemKuratowskisTheorem(whichIstillfindratheramazingandmagical).

Tofinishup,Ihadthemexploretheideaofdividingupacontinentintocountriesonlybydrawingstraightlines thatwentcompletelyfromonesideofthecontinenttotheother.Theycorrectlyfiguredoutthatsuchmaps wouldalwaysbetwo-colorable.Welookedatsomeexamplesandtheircorrespondinggraphs(which,theynoted, alwaysconsistofabunchofquadrilaterals).WhenIshowedthemtheinductiveproofoftwo-colorability,oneof thestudentsnotedthattheproofgeneralizedtonon-straightboundaries,aslongastheboundariesaredrawnas continuouslineswithbothendpointsonthecoastlineofthecontinent(whichIhadntrealized)!


Follow

97 of 224

FollowTheMathLess Traveled 10/16/12 16:55


Get everynewpost deliveredto your Inbox.

The Math Less Traveled | Explorations in mathema...

http://mathlesstraveled.com/

Allinall,thiswasprobablyourmostfunandengagingmeetingyet!NowthattheMathcountscompetitionis over,Ithinkwellhavealotoffundoingsomemoreopen-ended,exploratorythingslikethisratherthan practicingwithproblemsets.


Posted in geometry, pattern, puzzles, teaching | Tagged four-color theorem, graphs, map coloring, math club | 4 Comments

Irrationality of pi: the integral that wasn't


Posted on February 11, 2010

Andnowforthepunchline!Todaywellshowthat,forlargeenoughvaluesof ,

completingtheproofoftheirrationalityof . First,letsshowthat that ispositivewhen .Weknowthat ispositivefor .ButIclaim

istoo.Rememberthat

and areclearlypositivewhen ispositive;and positiveaswell. Forthesecondpart,notefirstthatfor ,

isalsopositivewhen

.Fromherewesimply

notethatifafunctionispositiveoveranentireinterval,theintegralofthefunctionoverthatintervalwillbe

Whyisthis?Well,clearly that

(since

),andalso

)andhence

,soweconclude

Thisdoesntyetincludethe

,butnoticethatmultiplyingby

canonlymakethingssmaller,since

is

atmost .Now,herestheslightlysneakypart:Iclaimthatwecanmake enough.Whyisthis?Noticethatwecanrewriteitas

assmallaswewantbymaking big

Now, thedenominatorof mightbeverylarge.Itmighthavefourteenmillionzilliondigits.Butnomatter howbig is,therewillofcoursebeaninteger whichisbiggerthan ,so reallylongtimetocancelouttheenormousproduct .Andthen ,and , andsoonofcourse,multiplyingbysomethinglessthanonemakesthingssmaller.Anditmighttakeareally ,butifwejustwaitpatientlyitwillgetsmaller andsmallerandeventuallytherewillcomesome forwhich
Follow

98 of 224

FollowTheMathLess Traveled 10/16/12 16:55


Get everynewpost deliveredto your Inbox.

The Math Less Traveled | Explorations in mathema...

http://mathlesstraveled.com/

Actually,eventhisisntquitesmallenough:wewanttheintegralfrom to of thatsnotaproblem;toensurethatwecanjustpick bigenoughsothat insidea by

tobelessthan1.But (ifthegraphof fits

box,thenitsintegralonthisintervalmustbelessthantheareaofthebox).

Voila!Anintegralwhichisanintegerabsurdlybetween0and1,allbecauseweassumed wasrational. Theinescapableconclusion,whichprobablywouldhavedriventheancientGreekscrazy,isthat isirrational!


Posted in algebra, calculus, convergence, famous numbers, proof, trig | Tagged inequality, integral, irrational, Niven, pi | 8 Comments

Irrationality of pi: the impossible integral


Posted on February 6, 2010

Weregettingclose!Lasttime,wedefinedanewfunction andthat .So,considerthefollowing:

andshowedthat

and

arebothintegers,

Thefirststepusestheproductrulefordifferentiation(recallingthat stepiswhatweshowedlasttime.Nowweseethepointofdefining waytotalkabouttheantiderivativeof ofderivativesof butitsmuchclearerthisway,dontyouagree?

and

);thelast

:itsjustsothatwehaveaconvenient

.Wecouldjustdoeverythingdirectlyintermsofalternatingsums

Nowthatweknowtheantiderivativeof thefollowingintegral:

,wecanusetheFundamentalTheoremofCalculustocompute

Notethatthevalueofthisintegralisaninteger,sinceboth

and

areintegers.Butnexttimewellshow

thatitisalsostrictlybetween0and1(forasuitablechoiceof ),whichisclearlynonsense!
Posted in famous numbers, proof | Tagged Fundamental Theorem of Calculus, integral, irrational, Ivan Niven, pi, proof | 4 Comments

Dimensions
Posted on February 3, 2010

Iveonlywatchedthetrailersofar,butthislooksextremelycool!Somebeautiful,fascinatingvideosaboutmath, withlotsofextraaccompanyingmaterialandexplanationsonthewebsite.

Follow

99 of 224

FollowTheMathLess Traveled 10/16/12 16:55


Get everynewpost deliveredto your Inbox.

The Math Less Traveled | Explorations in mathema...

http://mathlesstraveled.com/

HattiptoPhilWadler.
Posted in links, video

Divisor nim
Posted on February 3, 2010

YesterdayinmathclubIhadthestudentsplayagamewhichIdimlyrememberseeingsomewherebutforget where.SinceIdontknowwhatitisreallycalled,Imcallingitdivisornim.Hereshowitworks:
Follow

100 of 224

FollowTheMathLess Traveled 10/16/12 16:55


Get everynewpost deliveredto your Inbox.

The Math Less Traveled | Explorations in mathema...

http://mathlesstraveled.com/

1. Theplayerspickapositiveinteger. 2. Thetwoplayersworktogethertowritedownallthedivisorsofthechoseninteger(beingsuretoinclude1 andtheintegeritself). 3. Theplayersnowalternatemovesasfollows:onaplayersturn,shemustchooseoneofthedivisors ,and thencrossoutthatdivisoraswellasalloftheotherlistednumberswhicharedivisibleby . 4. Onsubsequentturns,playersmayonlychoosenumberswhicharenotyetcrossedout. 5. Whoeverisforcedtochoose1(becauseitistheonlynumberleft)istheloser! Forexample,supposethechosennumberis12.Wewritedownthedivisorsof12: 1,2,3,4,6,12. Nowsupposethefirstplayerchooses4(actually,thisisabadmove;Illletyoufigureoutwhy);theythencross out4and12,since12isdivisibleby4.Thegamenowlookslike 1,2,3,4,6,12. Nowitstheotherplayersturn;supposetheypick3(thisisactuallyabadmovetoo!),sotheycrossout3and 6.Nowthegamelookslike 1,2,3,4,6,12. Thefirstplayernowcrossesout2,andthesecondplayerisforcedtochoose1,sothefirstplayerwins. Thekidsthoughtthiswasalotoffunanditleadstoallkindsofinterestingdiscussions.First,ofcourse,you havetofigureouthowtowritedownallthedivisorsofthestartingnumber(howdoyouknowwhenyouve listedthemall?whataresomesystematicstrategiesforlistingthedivisors?).Thenyoucantalkaboutstrategies forplayingthegame.Imighttalkaboutsomeofthesethingsinsomefutureposts.FornowIwilljustnotethat thisactuallyhassomedeepconnectionstothetheoryofposets(wearebasicallyjustusingeachintegerasan abbreviationforitsposetofdivisors).AlthoughIveplayedaroundwithitabitIdontyetknowofageneral strategyalthoughanyparticularstartingintegernecessarilygivesawinningstrategyforONEofthetwo players,anditsnottoohardtofigureitoutbyworkingbackwards.Moreonthislater,Isuppose. Inthemeantime,havefunplaying!
Posted in games, number theory, pattern | Tagged divisor, game, lattice | 2 Comments

Battlestations!
Posted on February 1, 2010

TheworldsLARGESTFRACTALDORITO!
Posted in fractals, humor, links, video | Tagged Blownapart Studios, dorito, fractal | 2 Comments

Irrationality of pi: curiouser and curiouser


Posted on January 30, 2010

Follow

101 of 224

FollowTheMathLess Traveled 10/16/12 16:55


Get everynewpost deliveredto your Inbox.

The Math Less Traveled | Explorations in mathema...

http://mathlesstraveled.com/

Ivebeenremissinpostingherelately,whichIwillattributetoChristmasandNewYeartravellingandgeneral craziness,andthenstartinganewsemestercrazinessbutthingshavesettleddownabit,soherewegoagain! Sinceitsbeenawhilesincemylastpostinthisseries,heresaquickrecap:ImpresentingaproofbyIvanNiven that isirrational,thatis,thatitcannotberepresentedastheratiooftwointegers(andhenceitsdecimal expansiongoesonforeverwithoutrepeating).Myfirstpostjustgavesomebackgroundandanoutlineofthe generalargument.Inmysecondpost,webeganbyassumingthat isrational,anddefinedthefunction

(really,afamilyoffunctions,oneforeachvalueof )where and arethenumeratoranddenominatorof . Wethenshowedthat showedthatallthederivativesof ,andinfactthat issymmetric,with intermsof .Inmythirdpost,we ,andshowsomeofits takeoninteger valueswhenevaluatedatboth0and .Wereabout

halfwaythere!Todaywellcontinuebydefininganewfunction

properties.Recalltooouroverallplan:weregoingtowindupwithanintegralwhichisstrictlygreaterthan0, strictlylessthan1,andalsoaninteger!Sincethisisclearlynonsense(therearenointegersbetween0and1)we willconcludethatourinitialassumptionthat isrationalwasbogus,andthat mustbeirrationalafterall. Sowithoutfurtherado,heresournewfunction .Actually,thistooistechnicallyafamilyoffunctions ,

oneforeach ;butagain,everythingweproveaboutitwillbetruenomatterwhat is.

Inwords, as

isthealternatingsumofalltheevenderivativesof

.(Isayallbecause,asnotedinmylast

post,anyderivativeof

higherthan iszero.)UsingSigmanotation,wecanalsowritethismoreconcisely

Thereareafewthingstonote.First,thinkwhathappenswhenweevaluate takeonintegervaluesat0,and Ofcourse,thesamethinggoesfor Next,consider isjustasumofabunchofderivativesof .

:sinceallthederivativesof , mustbeanintegertoo.

Sincethederivativeofasumisthesumofthederivatives,wecancompute

as

Thatis,

turnsinto attheendof

turnsinto

,andsoon.Butwaitaminute,yousay.Shouldntthe in ?Infact,itdoesbutasnotedbefore, hasacorrespondingtermin is ofthe


Follow

turninto

zero,sothattermjustgoesaway.Nowwenotethateverytermof

102 of 224

FollowTheMathLess Traveled 10/16/12 16:55


Get everynewpost deliveredto your Inbox.

The Math Less Traveled | Explorations in mathema...

http://mathlesstraveled.com/

oppositesign,except except :

,whichhasnocorrespondingterm.Sowhenweadd

and

,everythingcancels

Astutereaderswillnoteafunnyresemblancebetweenthedefinitionof

andtheTaylorseriesfor and .

and

indeed,nexttimewellstartmakingsomeconnectionswithouroldtrigonometricfriends,
Posted in famous numbers, proof | Tagged derivatives, irrationality, Ivan Niven, pi, proof | 10 Comments

Perfect age
Posted on January 10, 2010

Todayismybirthday!Thisisthesecondand(barringanymiraculousadvancesinmedicalscience)finaltimethat myagewillbeaperfectnumber.Unfortunately,thefirsttimemyagewasaperfectnumber,Ididntknowwhat aperfectnumberwas.


Posted in meta | Tagged age, birthday, perfect | 15 Comments

Book review: Riot at the Calc Exam and Other Mathematically Bent Stories
Posted on January 8, 2010

YouveheardthestoryofRumpledStiltsken,right?Youknow,theonewherethe topologistsdaughterislockedinthegradstudentloungeandforcedtoturncoffee intotheoremsbymorning?whatsthat,yousayyouhaventheardthatone?Funny, Ithoughteveryoneknewthatstory.Well,itsafascinatingandsoberingtalefullof insightintolifeandthenatureofoops,wait,thoseweremynotesforTheKite Runner .RumpedStiltskeniswell,justreadit,OK? ColinAdamsentertainsuswiththisandmanyotherhumorouslymathematical (mathematicallyhumorous?)storiesinhisnewbook,RiotattheCalcExamandOther MathematicallyBentStories.TipsonhowtoavoidRERI(RepetitiveEyeRollInjury), advicefromamathematicalethicist,storiesabouteveryonesfavoritePrincipalInvestigator,DirkMangum,P.I.,a transcriptfromthehitradioshowMathTalkwithPlugandChugthelistgoeson.Somearefunnierthan others,ofcourse(bytheend,theconceitofanthromorphizing/metaphorizingmathematicaltheoremsandthe processorprovingthemhadgottenparticularlyold),butonbalancemyFunny-o-Meterwasdefinitelypointing somwherebetweenamusingandhilarious.Thisbookwouldmakeagreatgiftforthatspecialpersoninyour lifewholikestoreadfunnystoriesaboutmathwhiletheyareinthebathroom,orforanyonewholikesreading funnystoriesaboutmathingeneral,oranyonewholikesfunnystories,orwholikesmath.Thisbookwouldnot makeagoodgiftforgrumpypeoplewhohatemath.DontsayIdidntwarnyou. Fulldisclosure:theAMSkindlysentmeafreereviewcopyofthisbook.Also,ColinAdamswasactuallyoneof myprofessorsincollege,whichyoumightthinkwouldmakemesomewhatbiased,whichisprobablytrue,butit alsomeansthatIhappentoknowthathereallyisquitefunny,andalsothatheistheFastestDraw(erof3D surfaceswithcoloredchalk)intheWest(ernMassachusetts).Alsoalso,thismorningforbreakfastIateabowlof Follow

103 of 224

FollowTheMathLess Traveled 10/16/12 16:55


Get everynewpost deliveredto your Inbox.

The Math Less Traveled | Explorations in mathema...

http://mathlesstraveled.com/

shreddedwheatcereal.
Posted in books, humor, review | Tagged humor, short stories

MangaHigh.com
Posted on December 28, 2009

IrecentlyreceivedanemailsuggestingthatIcheckoutthewebsiteMangaHigh.com,whichhasinteractive math-basedgamesforelementarythroughhighschoolstudents.Now,Iamgenerallyprettyskepticalofsuch things.Forone,theyareusuallyofrelativelypoorquality.Ifyoureallywantstudentstobeinterestedina computergame,youhavetocompetewithgamecompanieswhichpourmillionsofdollarsintodetail,graphics, andgameplayandkidscantellthedifference!Foranotherthing,tryingtomakemathinterestingand relevantbyspicingitupwithinteractivegamescanbackfire:whywouldyouneedtodothatunlessitis actuallyboringandirrelevant?Itisliketryingtogetyourchildrentoeatasparagusbyhidingitinsidetheir hamburgers.Kidsarenotfooledbythis.(Infact,asparagusisoneofthemostdeliciousvegetablesIknow,but onlyifitisfreshandcookedright;ifnotfreshorovercooked,itisdisgusting.Iwillletyoumaketheappropriate metaphoricalinferences.) Nevertheless,Iwasintrigued,especiallysincemycorrespondentclaimedthatthiswebsitewasendorsedbythe eminentmathematicianandeducatorMarcusduSautoy.SoIvisitedthesiteandtriedplayingafewgames andwaspleasantlysurprised!Thegamesarefairlyhigh-qualityandhumorous(Iactuallyspenttwentyminutes orsoplayingthefirstgameItried,eventhoughitwasrathereasyforme),andthesitepromisestotrackpoints andaccomplishmentsforstudentswhoregister(adefiniterequirementifyouwanttogetstudentshookedon thegames). Ontheflipside,thecommercialstatusofthesiteisntcompletelyclearyoucanplayallthegamesforfreebut itclaimsthisisforalimitedtime,soImnotsurewhathappensafterthelimitedtimeisup.Thesitealso appearstohaveverylittletodowithManga,sothetitleisabitodd.Buttheseareminorconsiderationsatthe moment. Imstillnotsoldontheideaofinteractivegamesforteachingmathbutifyourelookingforsuchthings, MangaHigh.comseemslikeoneofthebestsitescurrentlyoutthere.
Posted in games, links, teaching | Tagged education, games, interactive | 1 Comment

Irrationality of pi: derivatives of f


Posted on December 20, 2009

Inmypreviouspostinthisseries,wedefinedthefunction

andshowedthat and

.Todaywellshowthesurprisingfactthat,foreverypositiveinteger ,although meansthe thderivativeof ;


Follow

arenotnecessarilyzero,theyarealwaysintegers.(Thenotation

thatis,takethederivativeof ,thenthederivativeofthat,thenthederivativeofthat, times.)Putmore

104 of 224

FollowTheMathLess Traveled 10/16/12 16:55


Get everynewpost deliveredto your Inbox.

The Math Less Traveled | Explorations in mathema...

http://mathlesstraveled.com/

succinctly:everyderivativeof takesonintegervaluesat

and

Whymightthisbesurprising?Itssurprisingbecauseofthe inthedenominatorof .Forexample,considerthe function(whichIjustmadeup):

Itseasytoseethat goingon!

.Butletstakethederivative:

so

,whichisclearlynotaninteger. )theremustbesomefancycanceling

Forthederivativesof toalwaysgiveanintegerat

(letaloneat

Fornowwewillconsideronly termcontaining todisappear,so constanttermof

(wellcomebackto

later).Ofcourse,substituting for causesevery .Hence,wemustshowthatthe

isjusttheconstanttermof

isalwaysaninteger. ,thatis,

Considerthenumeratorof

Notethat

,whenexpandedout,isapolynomialoftheform

,wheretheellipsiscontains .(Infact,wecouldusethe

abunchoftermswithintegercoefficientsandpowersof between1and theyareintegers.)Multiplyingby ,weseethat

BinomialTheoremtocomputetheprecisecoefficientsbutitreallydoesntmatter;allwewillcareaboutisthat

so

isapolynomialwithtermsofdegree through ,andhencesois hasnoconstantterm,so

,sincedividingby changesthe butwealreadyknewthat.)

coefficientsbutnottheexponents.(Notethat Recallthatthederivativeof is one.Sothefirstderivativeof Wecanseethatnoneofthefirst certainlyaninteger)for in! Aswenotedabove,whenexpandedout

,sotakingthederivativeofapolynomialreduceseachoftheexponentsby isapolynomialwithtermsofdegree through derivativesof through (andhenceaconstant (whichis (stillnoconstantterm);andsoon.

termofzero);thesecondderivativehastermsofdegree

willhaveaconstantterm,so

.Whataboutthe thderivativeandhigher?Thisiswherethefancycancelingcomes

isasumofabunchoftermsoftheform

where

and issomeinteger.Whenwetakethederivative,thistermwillturninto ;anotherderivativegivesus

;ifwe

takethederivativeagain,itwillbecome times,wewillendupwiththeconstantterm

,andsoon.Doyouseewhatishappening?Aftertakingthederivativeexactly

Follow

105 of 224

FollowTheMathLess Traveled 10/16/12 16:55


Get everynewpost deliveredto your Inbox.

The Math Less Traveled | Explorations in mathema...

http://mathlesstraveled.com/

andheresourfancycanceling:

isclearlydivisibleby ,sothisissomeintegertimes ,whichisalsoan hasdegreeatleast ,bythetime

integer.Voila!Saidadifferentway,andmoresuccinctly:sinceeachtermof

wehavetakenthederivativeenoughtimesforittoyieldaconstantterm,the willbecanceledfromthe denominator,sincewewillhavetakenthederivativeatleastateachpowerof from downto . Finally,ifwetakethederivativeof morethan times,weget ,sonoproblemsthere. Great,so isalwaysaninteger.Butwhatabout ?Well,remember,lasttimeweshowedthat ),weget

.Ifwetakethederivativeofbothsideswithrespectto (beingcarefultousethechainruleonthe leftside,notingthatthederivativeof withrespectto is

Wecanrepeatthisprocesstofindthat case, ,whichisaninteger.

(thetwonegativescancelontheleftside),

,andsoon.Buttheextranegativesignforoddderivativesdoesntreallymatter:ineither

Gettingcloser!Nexttime,wewilldefineanotherspecialfunction function willhelpuscompute integerstrictlybetween and (whichisimpossible).

intermsof

anditsderivatives;this

which(ifyourecallthepunchline)willturnouttobean

Posted in calculus, famous numbers, proof | Tagged derivatives, irrationality, Ivan Niven, pi, proof | 10 Comments

Math Teachers at Play #21


Posted on December 19, 2009

MathTeachersatPlay#21isupatMathMamaWrites,anditincludesthiscutepuzzle,whichSueapparently madeupherself: TheNumberlandNewsrunspersonalads.21waslookingforanewfriendandputanadin. Two-digit,semi-prime,triangular,Fibonaccinumberseekssame.Imabinarypalindrome,what aboutyou? Will21findafriend? Asemi-primeisanumberwithexactlytwoprimefactors,like6.Seethispostforadefinitionoftriangular number ,thispostforsomehintsonhowtofigureoutageneralformulaforcomputingtriangularnumbers,and thisoneforthesolution.Fibonaccinumbersarediscussedhere.Finally,apalindromeisanumber(orword,or phrase)whichisthesameforwardsandbackwards;abinarypalindromeisanumberwhichisapalindromewhen expressedinbasetwo.
Posted in fibonacci, links, number theory, puzzles | Tagged binary, fibonacci, MTaP, palindrome, puzzle, semi-prime, triangular | 13 Comments

Follow

106 of 224

FollowTheMathLess Traveled 10/16/12 16:55


Get everynewpost deliveredto your Inbox.

The Math Less Traveled | Explorations in mathema...

http://mathlesstraveled.com/

The haybaler
Posted on December 16, 2009

AtPennAlexandersmathclubyesterday,thestudentsworkedonafunpuzzlethatIdneverseenbefore.Itgoes likethis: Youhavefivebalesofhay. Forsomereason,insteadofbeingweighedindividually,theywereweighedinallpossiblecombinations oftwo.Theweightsofeachofthesecombinationswerewrittendownandarrangedinnumericalorder, withoutkeepingtrackofwhichweightmatchedwhichpairofbales.Theweights,inkilograms,were80, 82,83,84,85,86,87,88,90,and91. Howmuchdoeseachbaleweigh?Isthereasolution?Aretheremultiplepossiblesolutions? Unfortunately,theproblemseemedalittlebeyondthem(oratleast,theythoughtitwasbeyondthem,sothey quicklylostinterest)butthisseemslikeagreatproblemtouseinmiddleschoolorhighschoolmathclasses.In middleschool,keepthemtalkingandfocusonthemethodstheyemploytotrytosolveit.Inhighschool, perhapsoncetheysolveityoucouldgetthemtotrygeneralizingtheproblem(toothersetsofweights,more thanfivebales,etc.).
Posted in arithmetic, challenges, logic, puzzles, teaching | Tagged bales, hay, weights | 11 Comments

The Christmas Price Index


Posted on December 14, 2009

IvewrittenbeforeaboutthemathematicsofthetraditionalsongTheTwelveDaysofChristmas.Fora differentangle,checkoutPNCBanksChristmasPriceIndex:apparently,everyyeartheycomputethetotalcost ofallthegiftsinthesongasawhimsicalmeasurementoftheeconomy.Thisyeartheyhaveareallycutevideo alongwithgamesandactivitiesthatcanbeusedbyeducatorsgiveitalook!Thisyear,aTrueLovewouldhave toshellout$87,402.81forall364gifts(uponly1.8%fromlastyear)! (Note,whenIvisitthesiteIgetawarningaboutabadsecuritycertificatebutafterpokingaroundabitnothing fishyappearstobegoingon,itshouldbeOKtotellyourbrowsertomakeanexceptionifyougetasimilar warning.)


Posted in games, links, video | Tagged Christmas, gifts, index, PNC Bank, price | 1 Comment

Irrationality of pi: the unpossible function


Posted on December 12, 2009

Recallfrommylastpostwhatwearetryingtoaccomplish:byassumingthat isarationalnumber,wearegoing todefineanunpossiblefunction!So,withoutfurtherado: Suppose ,where and arepositiveintegers.Definethefunction likethis:

Follow

107 of 224

FollowTheMathLess Traveled 10/16/12 16:55


Get everynewpost deliveredto your Inbox.

The Math Less Traveled | Explorations in mathema...

http://mathlesstraveled.com/

(Incaseyouveforgotten, ,pronouncednfactorial,istheproductofallthenumbersfrom1to .)OK butwhatis ?Ihearyouask.Goodquestion.Theshortansweris,itdoesntmatter: canbeanypositive integer.Wewillshowabunchofthingsthataretrueabout nomatterwhat is.Later,wewillseethatwegeta contradictiononlyforvaluesof whicharebigenough.ButthatsOK;sinceeverythingweproveuptothat pointwillbetruenomatterwhat is,wecanpickavalueof whichisasbigaswelike. Letsexploresomepropertiesof aswell(rememberingthat .First,itseasytoseethat ,ofcourse,whichmeansthat .Itsnottoohardtoseethat ):

So

haszerosat

and .Thatis,

.Butmoreistrue:infact,

issymmetric (amirrorreflectionofitself)

aroundtheline

Letsprovethis:

Idontseewhatssounpossibleabout sofar,yousay?Patience!(Ofcourse,itisntreally itselfwhichisthe problem;theproblemisourinsistencethat isactuallydefinedintermsofthenumeratoranddenominator of ) Nexttime,wellseethatthederivativesof alsohavesomespecialproperties.


Posted in calculus, famous numbers, proof | Tagged irrational, Ivan Niven, pi, proof, symmetric | 7 Comments

Irrationality of pi
Posted on December 7, 2009

Everyoneknowsthat theratioofanycirclesdiametertoitscircumferenceisirrational,thatis,cannotbe writtenasafraction .Thisalsomeansthat sdecimalexpansiongoesonforeverandneverrepeatsbut haveyoueverseenaproofofthisfact,ordidyoujusttakeitonfaith? Theirrationalityof wasfirstproved(accordingtomodernstandardsofrigor)in1768byLambert,buthisproof wasrathercomplicated.Amoreelementaryproof,usingonlybasiccalculus,wasgivenin1947byIvanNiven. Youcanreadhisoriginalpaperhere,butitsratherterse!JustasIdidforCalkinandWilfspaper,Recountingthe Rationals,IplantowriteaseriesofpostsexplainingNivensproofinabitmoredetail,withsomeaccompanying intuition.Illassumeabasicknowledgeofcalculus;ifyoudontknowcalculus,justhangtightforafewposts! Heresthebasicoutlineoftheproof.Webeginbysupposingthat isrational:inparticular,suppose someintegers and .Wellthenusethesevaluesof and todefineaspecialfunction for
Follow

,aboutwhichwewill

108 of 224

FollowTheMathLess Traveled 10/16/12 16:55


Get everynewpost deliveredto your Inbox.

The Math Less Traveled | Explorations in mathema...

http://mathlesstraveled.com/

showthefollowing: isaninteger,AND . Butthisisabsurd!Therearenointegersgreaterthanzeroandlessthanone.Theinescapableconclusionwillbe thatourinitialassumptionthat wasfalse. andbeginexploringsomeofitsproperties.

Inmynextpost,welldefinethespecialfunction

Posted in calculus, famous numbers, proof | Tagged irrational, Ivan Niven, pi, proof | 14 Comments

Carnival of Mathematics this Friday!


Posted on December 1, 2009

ThenextCarnivalofMathematicsgoesupthisFridayat idiotsbloggetyoursubmissionsin!
Posted in links, people | Tagged Carnival of Mathematics

Who Am I?
Posted on November 29, 2009

AnexcellentpuzzlefromJD2718: Therearefivetrueandfivefalsestatementsaboutthesecretnumber.Eachpairofstatementscontains onetrueandonefalsestatement.Findthetrues,findthefalses,andfindthenumber. 1a.Ihave2digits 1b.Iameven 2a.Icontaina7 2b.Iamprime 3a.Iamtheproductoftwoconsecutiveoddintegers 3b.Iamonemorethanaperfectsquare 4a.Iamdivisibleby11 4b.Iamonemorethanaperfectcube 5a.Iamaperfectsquare 5b.Ihave3digits Pleasedontpostthesolutioninacomment,soasnottospoilitforothers.Butfeelfreetoleaveacommentif youneedahint,ortoemailmeifyouthinkyouhavesolveditandwanttocheckifyouarecorrect.(Actually,its easytocheckyourself:justmakesurethateachofeachpairofstatements,exactlyoneistrueandoneisfalse!)
Follow

109 of 224

FollowTheMathLess Traveled 10/16/12 16:55


Get everynewpost deliveredto your Inbox.

The Math Less Traveled | Explorations in mathema...

http://mathlesstraveled.com/

Posted in challenges, logic, number theory, puzzles, teaching | Tagged number, puzzle, secret | 13 Comments

m-bracelets code
Posted on November 27, 2009

Bypopulardemand,hereistheHaskellcodeIusedtogeneratetheimagesinmypreviouspost.Thispostis literateHaskell;youcansimplycopyandpastethisentirepostintoafilecalledBraceletGraph.lhs(oranything youlike,aslongasitendswith.lhs)andrunityourself.Also,IusedRobertGreayerslovelyBlogLiteratelytool towriteandformatthispost. First,afewimports:weuseMartinErwigsfglfunctionalgraphlibraryforconstructinggraphcs,andIvan MiljenovicsbindingstotheGraphvizlibraryinordertooutputgraphdescriptions.

> import Data.Graph.Inductive > import Data.GraphViz > > import System.Environment

A"link"inanumberbraceletisapairofdigits;knowingapairofdigitscompletelydeterminestheremainderof thebracelet.

> type Link = (Int,Int)

Thenextlinkinabraceletisobtainedbyaddingandtakingtheresultmodm.

> nextLink :: Int -> Link -> Link > nextLink m (a, b) = (b, ((a + b) `mod` m))

Weconstructthelistofallpossiblelinks.

> braceletLinks m = [ (a,b) | a <- [0..m-1], b <- [0..m-1] ]

Wenowalsoconstructthelistofallthe"edges"fromonelinktothenext.

> braceletEdges m = [ (l, nextLink m l) | l <- braceletLinks m ]

Wewillalsoneedafunctiontoconvertalinkintoauniquenumericrepresentation,sincethegraphlibrary assumesthatverticesarenamedbyintegers.

Follow

110 of 224

FollowTheMathLess Traveled 10/16/12 16:55


Get everynewpost deliveredto your Inbox.

The Math Less Traveled | Explorations in mathema...

http://mathlesstraveled.com/

> linkToLabel m (a, b) = a*m + b

Therestofthecodesimplyinterfaceswiththefglandgraphvizlibrariestocreateagraphandoutputitasa.dot file.Thiswasthehardestpartofthecodetowritebutitwashardonlyinthesensethatittookmeawhileto lookthroughthedocumentationforthefglandgraphvizlibrariestofigureouthowtodowhatIwanted.

> braceletGraph :: Int -> Gr Link () > braceletGraph m = mkGraph (map mkNode (braceletLinks m)) > > > > > dotGraph m = graphToDot True (braceletGraph m) > > > > > linkAttrs (_, (a,b)) = > > > > > > > > colors = cycle $ map ColorName [ "lightblue" > > > > > > > > > > > > main = do > > (m:_) <- fmap (fmap read) getArgs putStr . printDotGraph $ dotGraph m , "red" , "orange" , "yellow" , "green" , "blue" , "purple" , "brown" , "pink" , "grey" ] [ Label . StrLabel . show $ a , Shape Circle , Color [colors !! a] , FillColor $ colors !! a , Style [SItem Filled []] ] [] linkAttrs (const []) where mkNode l (map mkEdge (braceletEdges m)) = (linkToLabel m l, l) mkEdge (l1,l2) = (linkToLabel m l1, linkToLabel m l2, ())

Wecanrunthisprogrambypassingitthevalueofmwewanttouse,anditoutputsafilein.dotformat,which wecanturnintoanimageusingoneofgraphizsgraphlayouttools,likeneatoorcirco.Forexample:

$ ghc --make BraceletGraph.lhs $ ./BraceletGraph 9 > bracelets9.dot $ neato -Tpng bracelets9.dot > bracelets9.png

Posted in arithmetic, computation, links, pictures, programming | Tagged bracelets, graph, graphviz, Haskell, literate | 3 Comments

Follow

111 of 224

FollowTheMathLess Traveled 10/16/12 16:55


Get everynewpost deliveredto your Inbox.

The Math Less Traveled | Explorations in mathema...

http://mathlesstraveled.com/

m-bracelets
Posted on November 22, 2009

Itiseasytogeneralizenumberbraceletstomoduliotherthan10ateachstep,addthetwopreviousnumbers andtaketheremainderoftheresultwhendividedbym.HerearesomeprettypicturesImadeoftheresulting braceletsformrangingfrom1through12.Clickonanyofthemtogetalargerversion. m=1

m=2

m=3

m=4

Follow

112 of 224

FollowTheMathLess Traveled 10/16/12 16:55


Get everynewpost deliveredto your Inbox.

The Math Less Traveled | Explorations in mathema...

http://mathlesstraveled.com/

m=5

m=6

m=7

Follow

113 of 224

FollowTheMathLess Traveled 10/16/12 16:55


Get everynewpost deliveredto your Inbox.

The Math Less Traveled | Explorations in mathema...

http://mathlesstraveled.com/

m=8

m=9

m=10

m=11

Follow

114 of 224

FollowTheMathLess Traveled 10/16/12 16:55


Get everynewpost deliveredto your Inbox.

The Math Less Traveled | Explorations in mathema...

http://mathlesstraveled.com/

m=12

IusedalittleHaskellprogramtooutputdescriptionsofthegraphs,andGraphviztogeneratetheimages.Idbe happytopostthecodeifanyoneisinterested.
Posted in arithmetic, fibonacci, iteration, pattern, pictures, sequences | 6 Comments

One of these things is not like the others


Posted on November 21, 2009

HattiptoTanyaKhovanova.
Posted in pattern | Tagged odd one out, puzzle | 9 Comments

Number bracelets
Posted on November 17, 2009

RecentlyIvebeenvolunteeringwiththemiddleschoolmathclubatPennAlexander,aPreK-8schoolinmy neighborhood.Todaywedid(amongotherthings)afunactivityIdneverseenbefore,callednumber bracelets.Thestudentsseemedtoenjoyit;itworkedespeciallywellwithabunchofstudentsallworkingonit atthesametimesincetheywereabletocomparenotes. Herestheidea:startwithanytwoone-digitnumbersyoulike.Forexample,letschoose4and6.Next,addthe twonumbers:4+6=10.Throwawaythetensdigitofyouranswer(ifany);thisisthenextnumberinthe sequence.Inourcaseweget0.Nowwehave4,6,0andwedothesamethingwiththelasttwonumbers,6and 0:6+0=6.Sonowwehave4,6,0,6.Continuing,weget 4,6,0,6,6,2,8,0,


Follow

115 of 224

FollowTheMathLess Traveled 10/16/12 16:55


Get everynewpost deliveredto your Inbox.

The Math Less Traveled | Explorations in mathema...

http://mathlesstraveled.com/

Trysomedifferentstartingnumbers.Dothesequenceseverrepeat?Howmanydifferentsequencescanyou make?Howlongcantheybe?Canyougeneralizethistoothersortsofrulesforgeneratingsequences? Amuchbetterdescription(withprettypictures,morequestionsforexploration,andspoilers)canbefoundhere.


Posted in arithmetic, iteration, pattern, sequences, teaching | Tagged activity, bracelets, number, Penn Alexander | 12 Comments

minim
Posted on November 4, 2009

AfungameIdiscoveredrecently,minim.Ineachlevelyoustartoutwithanetworkofnumberednodes,andthe objectistosuccessivelycombinethenodesaccordingtocertainmathematicalrulesinordertoendupwithonly asinglenode.Thelevelsstartouteasybutgetfiendishlydifficultbytheend.Myonlycomplaintisthatitisnt possibletomakeupyourowncustomlevels.


Posted in games, links | Tagged arithmetic, game | 1 Comment

Logicomix: An Epic Search for Truth


Posted on November 2, 2009

IrecentlyacquiredacopyofLogicomix:AnEpicSearchforTruth,byApostolosDoxiadisandChristos Papadimitriou,withartbyAlecosPapadatosandAnnieDiDonna.Itdefiescategorization:isitacomicbook?A biography?Abookofphilosophy?Ofhistory?Ofmathematics?Well,itsallofthesethings,andmanagestopull itoffwithgraceandstyle. LogicomixisagraphicnovelfocusingonthestoryofBertrandRussell,theEnglishmathematicianand philosopher,andthequestatthebeginningofthetwentiethcenturytodiscoveralogicallyrigorousfoundation formathematics.Thisquesthadasurprisingoutcome:inonesense,itwasafailure;inanothersense,itled directlytothedevelopmentofcomputers!IfyouwanttoknowwhatImeanyoullhavetoreadthebook. Itsbeatifullyandsmartlyillustrated,andtellsarivetingstory,interspersedwithmeta-narrativeaboutthe authorsofLogicomixandtheirprocessofplanningandwritingit.IfyoureadLogicomixexpectinga comprehensivemathematicalhistoryofthistimeperiodincomicbookform,youllbedisappointed.Itreallyis tellingthestoriesofthepeopleinvolved,withbigmathematicalideasexplainedasnecessary(althoughthe mathematicalexplanationsitdoesincludearecreativeandclear).Fortunately,thepeopleareratherfascinating! LikeE.T.BellsMenofMathematics,Logicomixcertainlydispelsthemyththatmathematiciansaredull.Logicomix wouldmakeagreatadditiontoanylibrary,butIparticularlyrecommendittomiddleandhighschoolteachers forlendingtostudents!
Posted in books, logic, review | Tagged Bertrand Russell, book review, foundations, graphic novel, logic, philosophy, set theory | 2 Comments

The hyperbinary sequence and the Calkin-Wilf tree


Posted on October 18, 2009

Andnow,theamazingconclusiontothisseriesofpostsonNeilCalkinandHerbertWilfspaper,Recountingthe
Follow

116 of 224

FollowTheMathLess Traveled 10/16/12 16:55


Get everynewpost deliveredto your Inbox.

The Math Less Traveled | Explorations in mathema...

http://mathlesstraveled.com/

Rationals,andtheanswerstoallthequestionsaboutthehyperbinarysequence.Holdontoyourhats!

TheCalkin-WilfTree
First,recalltheCalkin-Wilftree,definedasfollows:therootofthetreeisthefraction1/1,andeverynode labeled inthetreehastwochildren,withtheleftchildlabeled ,andtherightlabeled .Heres apictureofthefirstfourlevelsofthetree(ofcourse,thetreeisinfinite):

Nowwaitjustoneminutethosenumberslookfamiliar!Letslistthosefractionsinorder,onelevelatatime, fromlefttoright:

Thatsright,itsthehyperbinarysequence!Inparticular,thenumeratorsarethehyperbinarysequencestartingat index0,andthedenominatorsfollowthesamesequence,offsetbyone. butwhy?!TheCalkin-Wilftreeisdefinedbyarecursiveruleformanipulatingfractions,andthehyperbinary sequenceisdefinedintermsofwaystowritenumbersassumsofpowersoftwoitcertainlyisntapriori obviouswhytheseshouldhaveanythingtodowitheachother!

LabellingtheCW-tree
LetslabelthenodesintheCalkin-Wilftreewithpositiveintegers,startingwith1attherootandproceedingin orderbylevels,fromlefttoright.Thismeansthatthefractionatnode is weretryingtoprove!Heresanillustrationofournumberingscheme: atleast,thatswhat

Now,whatdoyounoticeabouttherelationshipbetweenthelabelonanodeandthelabelsonitschildren?Its nottoohardtoguessthepattern:theleftchildofthenodelabelled islabelled ,andtherightchildis labelled .Howcanweprovethis?


Follow

117 of 224

FollowTheMathLess Traveled 10/16/12 16:55


Get everynewpost deliveredto your Inbox.

The Math Less Traveled | Explorations in mathema...

http://mathlesstraveled.com/

Well,itscertainlytruefortherootnote:itslabelled1,anditschildrenarelabelled2and3.Nowwenotethatif thepatternholdsforthenodeimmediatelyprecedingnode thatis,thechildrenofnode thenthechildrenofnode comeimmediatelyafterthechildrenof .Ifyouthinkaboutit,thisistrueregardlessofwhether ofoneleveland isthebeginningofthenext. WecanthinkoflabellingeachedgeoftheCalkin-Wilftreewitheithera0ora1:0forleftedges,and1forright edges.Thentakingallthezerosandonesalongthepathfromtheroottoanynodeandstickinganextra1atthe beginninggivesusthelabelofthatnodeinbinary!Forexample,thepaththatgoesleft,left,rightcorresponds to0,0,1,andaddinganextra1tothefrontgivesus1001whichisindeedthebinaryrepresentationof9! are and istheend ,sotheyarenumbered and

and areonthesamelevel,or

So,isitreallytruethatthefractionatnode isalways .Next,supposethatthefractionatnode is child(whichisnode )is bythewaytheCalkin-Wilftreeisdefined,theleftchildis

?Itisindeed,andwenowhavethetoolswe ,whichisindeed .Butthisisnoweasy: ;weneedtoshowthatthefractionattheleft

needtoseewhy.Firstofall,therootnodeofthetreethatis,node1is(bydefinition) ,andthefractionattherightchildis

Theequality,ofcourse,followsdirectlyfromtherecurrencerelationforthehyperbinarysequence!Likewise,the rightchildis

Solvingthe

mystery

AspointedoutbyJM,thisconnectionwiththeCalkin-WilftreeisexactlywhatweneedtoproveFergalDalys conjectureaboutthenumberofprimaryoccurrencesofanygivennumber. Wedefinedaprimaryoccurrencesimplyasanumberatanevenpositioninthehyperbinarysequence.Ofcourse, even-labellednodesintheCalkin-Wilftreeareexactlytheoneswhichareleftchildren,andsotheprimary occurrencesarethedenominatorsofleftchildren(equivalently,thenumeratorsofrightchildren).Also,sinceleft childrenarealwaysoftheform ,leftchildrenalwayshavedenominatorsbiggerthantheirnumerators. Finally,asyoumayrecall,weshowedearlierthateverypositiverationalnumberoccurssomewhereinthe Calkin-Wilftree,inlowestterms,exactlyonce.


Follow

118 of 224

FollowTheMathLess Traveled 10/16/12 16:55


Get everynewpost deliveredto your Inbox.

The Math Less Traveled | Explorations in mathema...

http://mathlesstraveled.com/

Puttingthisalltogether:eachprimaryoccurrenceof correspondstoafraction where foreachpositiveinteger

intheCalkin-Wilftree, such

.Andsinceeveryfractionoccursexactlyonceinlowestterms,thereisexactlyonesuchfraction whichisrelativelyprimeto .Ofcourse,bydefinition,thereare primaryoccurrencesof ,sincethats

numbers,sothisistheproofofFergalsconjecture!Thereareexactly

howmanyreducedfractionstherearebetween0and1whichhaveadenominatorof .

Computingprimaryoccurrences
TheconnectionwiththeCalkin-Wilftreealsoallowsustocompute (in time)computealltheprimaryoccurrences suchthat thatis,givenany ,wecanefficiently !

Howdoesthiswork?Well,ifyouremember,theEuclidanAlgorithmgivesusawaytofindourwayupthe Calkin-Wilftreefromanystartingfraction.Combiningthiswiththerelationshipwediscoveredearlierbetween thelabelonanodeandthelabelsonitschildrenallowsustocomputethelabelonthenodewherewestarted. Asanexample,letscomputeoneoftheprimaryoccurrencesof5.Inparticular,letsdotheonethatcorresponds tothefraction .Since3issmallerthan5, .Finally, ; .Therefore mustbetheleftchildof istheleftchildof .3isbiggerthan2,so isnode ; ;and istotheleft istherightchildof istotheleft,soitsnode again,soitsnode .Nowwecanworkbackwards:

istotherightofthat,soitsnode

isaprimaryoccurrenceof5!Wecanalsothinkofthismore

succinctlybyrecallingthatpathsintheCalkin-Wilftreecorrespondtobinaryrepresentationsofthelabels.Sowe canruntheEuclideanAlgorithm,gettingazerooroneateachstep,andthenreversethemandputanextra1at thebeginningtogetthebinaryrepresentationoftheprimaryoccurrence.Tobemoreprecise: 1. Startwiththepair 2. Given ,if where . isrelativelyprimeto .Callthecurrentpairofnumbers .Otherwisewrite . .

thenwrite

3. Repeatstep2untilreaching

4. Readofftheprimaryoccurrenceof byprefixing1tothereverseofthebinarydigitswrittenoverthearrows. Allofthisleadstothefollowing(somewhatmind-blowing)observation:thepositionofeachprimaryoccurrence of inthehyperbinarysequenceisaproof (encodedinbinary)that isrelativelyprimetothenumberthatcomes immediatelybeforeit. Totiethisalltogether,heresalittleHaskellprogramIwrotetocomputeprimaryoccurrences:

Follow

119 of 224

FollowTheMathLess Traveled 10/16/12 16:55


Get everynewpost deliveredto your Inbox.

The Math Less Traveled | Explorations in mathema...

http://mathlesstraveled.com/

importSystem.Environment(getArgs) importControl.Monad(forM_) importData.Maybe(catMaybes) importData.List(sort) --FindafractionintheCalkin-Wilftree.findCWabcomputesthe --locationofa/bintheCalkin-Wilftree,orreturnsNothingifa --andbarenotrelativelyprime. findCW::Integer->Integer->MaybeInteger findCW11=Just1 findCWab|a<b=left`fmap`findCWa(b-a) |a>b=right`fmap`findCW(a-b)b |a==b=Nothing leftx=2*x rightx=2*x+1 --Findtheprimaryoccurrencesofn. primaryOccs::Integer->[Integer] primaryOccsn=sort.catMaybes$[findCWmn|m<-[1..n]] main::IO() main=do(from:to:_)<-getArgs forM_[readfrom..readto]$\i->do putStr$showi++":" putStrLn.unwords.mapshow.primaryOccs$i

Iftheresinterest,Icouldwriteanotherpostexplaininghowthisprogramworksinabitmoredetail,but hopefullyyoucanmostlyunderstandwhatitsdoingfromtheprecedingdiscussion.Letscheckthatitworks:

[brent@archimedes:~/teaching/mlt/hyperbin]$./prim-occs216 2:2 3:46 4:814 5:10121630 6:3262 7:1822242864126 8:2026128254 9:34464860256510 10:38565121022 11:3640545866949612410242046 12:445020484094 13:4252707811212013019019225240968190 14:6880110122819216382 15:721182583823845081638432766 16:92981341582242483276865534

Yup,looksgood!Andtoshowthatitreallyisefficient,letsuseittocomputetheprimaryoccurrencesof1000 (yes,all400ofthem!):

Follow

120 of 224

FollowTheMathLess Traveled 10/16/12 16:55


Get everynewpost deliveredto your Inbox.

The Math Less Traveled | Explorations in mathema...

http://mathlesstraveled.com/

[brent@archimedes:~/teaching/mlt/hyperbin]$./prim-occs10001000 1000:397704227856024585326839670568714447567476688850468648092576104030 ...386moreprimaryoccurrences... 1071508607186267320948425049060001810561404811705533607443750388370351051124936 1224931983788156958581275946729175531468251871452856923140435984577574698574803 9345677748242309854210746050623711418779541821530464749835819412673987675591655 43946077062914571196477686542167660429831652624386837205668069374

Onmycomputerthisrunsinabout0.03seconds.Sweet!Notethatthepositionofthefinalprimaryoccurrence of1000has303digitsthatsapproximatelyonecentillion.Icertainlywouldntwanttogothroughthefirstone centillionnumbersinthehyperbinarysequencelookingfor1000byhand,wouldyou?

FurtherReading
Ihopeyouveenjoyedthislittle(1.5-year-long!)excursionintothefascinatingworldopenedupbyCalkinand Wilfspaper.(Ivesureenjoyedwritingit!)Ifyoureinterestedinexploringmore,IsuggestlookingatRoland BackhouseandJooFerreiraspaper,RecountingtheRationals,Twice!whichalsoexplainstheconnectionto anotherfamoustreeoffractions,theStern-Brocottree.Ifyoureinterestedinthecomputationalaspectofthe hyperbinarynumbers,havealookatEnumeratingtheRationals,byJeremyGibbons,DavidLester,andRichard Bird.
Posted in arithmetic, computation, induction, iteration, number theory, pattern, proof, recursion, sequences, solutions | Tagged algorithm, binary, Calkin-Wilf, Euclidean, Haskell, hyperbinary, tree | 6 Comments

Hyperbinary conjecture seeking proof for a good time, long walks on the beach
Posted on October 12, 2009

Heresthelatestprogressonthehyperbinarysequence.Weretryingtofigureouttheinverserelationofthe function :givenaparticularnumber ,wheredoesitoccurinthehyperbinarysequence?Thatis,whatarethe valuesof forwhich ?

Thereareinfinitelymany,butinapreviouspostIarguedwhyweonlyneedtofindoccurrencesatevenpositions ofthesequence,whichwecallprimaryoccurrences.Ihavenoideahoweasyorharditistogiveageneral methodforfindingallprimaryoccurrences.Butsomeprogresshasbeenmade: Brendanprovedbyinductionthat Brendanalsoprovedthat least,itwasnttome!). FergalDalyconjecturedthatthenumberofprimaryoccurrencesof is totientfunction; . denotestheso-calledEuler isdefinedtobethenumberofpositiveintegerssmallerthan whicharerelativelyprime ! and .Thesecorrespondtothenumbersthatoccur ).

rightnextto1inthesequence(wesawearlierthat

.Thisisimpressive,sincethispatterncertainlyisntobvious(at

to .Anexplanationofthisfact,ifitistrue(anditreallylookslikeitmightbe!)wouldprobablygoalong waytowardsfindingageneralmethodforcomputing CananyonefindaproofofFergalsconjecture?


Posted in challenges, pattern, people, proof, sequences | Tagged Euler, hyperbinary, inverse relation, totient | 4 Comments

Follow

121 of 224

FollowTheMathLess Traveled 10/16/12 16:55


Get everynewpost deliveredto your Inbox.

The Math Less Traveled | Explorations in mathema...

http://mathlesstraveled.com/

New and improved: bookshelf and Carnival of Mathematics


Posted on October 10, 2009

Twothingstoday:first,Iveupdatedmybookshelf(gotohttp://www.mathlesstraveled.com/,scrolldowna bitandlookintherightmargin).ThebookscomefrommyLibraryThinglibrary,andIvenowfiguredouthowto writereviewsofthebooksandhavethemshowupdirectlyinmybookshelf!Iveaddedoneortworeviewsbut plantoaddmoreoverthecomingweeks.IalsoplantowriteblogpostsreviewingafewparticularbooksIve acquiredrecently.Ifyourelookingforafunmathbook,Ihopeyoullfindthisagoodplacetostart! ImalsoexcitedbytherebirthoftheCarnivalofMathematics,whichhadfallenonhardtimesrecently.Mike CroucherofWalkingRandomlyhasgraciouslyvolunteeredtotakeoverorganizationalduties.JasonDyerwillbe hostingthenextoneonNovember6.


Posted in books, meta, people | Tagged Bookshelf, Carnival of Mathematics, reviews

Minus times minus is plus


Posted on September 29, 2009

Iveseenseveralrecentpostswonderingaboutanintuitiveexplanationforthefactthataminustimesaminus equaltoaplus.Iunderstandwhypeoplewouldwonderaboutthisthewayitsoftentaught,itseemstobe justanarbitraryrulewithnowhy!Butinfact,itsquitelogical. Paulatcrossedstreams.comgivesanexplanationinvolvingtwoquantitieswithreal-worldinterpretations associatedwithnegativevalues:networth(negativemeansadecreaseinnetworth),andtime(thepastis negative).Itsfairlyintuitive,butinsomesenseitonlyshowswhyitisnicethataminustimesaminusisaplus, sinceitallowsustomodelthisreal-worldsituation;itdoesntreallyshowwhyitistrueinadeepsense.Mikeat WalkingRandomlygivesaproofessentiallyfromthefieldaxioms,but(ashehimselfadmits)itisnot particularlyintuitive. HereshowIwouldexplainit.Itisntrigorousatall,andImnotevencompletelysatisfiedwithit,butIhopeit ishelpfulforbuildingsomeintuition. Thinkaboutthefamiliarnumberline:positivenumbersgoofftotheright,andnegativenumbersaretothe left.Addingapositivenumbercorrespondstomovingrightalongthenumberline.Addinganegativenumber (thatis,subtractingapositiveone)correspondstomovingleftalongthenumberline.Sowithadditionwe alreadyseethisideaofnegativecorrespondingtodoingsomethingintheoppositedirection. So,whatdoesmultiplicationcorrespondtoonthenumberline?Ofcourse,multiplicationcorrespondstoscaling, orstretching:forexample,ifwestartatapointonthenumberlineandmultiplyby3,wewillendupatapoint threetimesasfarfromzeroaswestarted.Andwhataboutmultiplyingbyanegativenumber?Itcorrespondsto ascaleintheotherdirection:forexample,ifwestartatapointonthenumberlineandmultiplyby-3,weendup atapointontheothersideofzero,andthreetimesasfar.Thatis,multiplyingbyanegativenumbermeansthat weflipfromonesideofzerototheother.So,ofcourseifwestartfromtheleftofzero(anegativenumber)and multiplybyanegative,weendupontherightofzero(apositivenumber)! [Asanafterthought,Ithinktryingtoexplainitanymoredeeplythanthisreallydoesrequirebringinginthe

Follow

122 of 224

FollowTheMathLess Traveled 10/16/12 16:55


Get everynewpost deliveredto your Inbox.

The Math Less Traveled | Explorations in mathema...

http://mathlesstraveled.com/

distributivepropertyinsomeway---itistheonlythingthatformallyconnectsaddition(remember,negative numbersaredefinedasadditiveinverses)andmultiplication.IreallyliketheideapostedbyEricasacomment onMike'spostregardinganillustrationofthedistributiveproperty. Also,Imeanttoputnicepicturesinthispost,butfornowifIwaittomakenicepicturesitwillneverhappen.So I'lljustpostitfornow,andmaybeI'llcomebackandaddsomepictureslater.]


Posted in arithmetic | Tagged intuition, multiplication, negative | 14 Comments

More hyperbinary fun


Posted on September 28, 2009

WhenIoriginallyposedChallenge#12,acertainDavepostedaseriesofcommentswithsomeexplorationsand partialsolutionstopartII(thehyperbinarysequence).AlthoughIgavethesolutioninmylastpost,no solutiontoanyproblemiseverthelastwordtheresalwaysmoretoexplore,moreconnectionstosee!Its worthtakinganotherposttoexploresomeofDavesdiscoveriesandconjectures,andhowtheyconnecttomy lastpost.Iveuseddifferentnotations,madeupsometerms,triedtosimplifythingswherepossible,andsoon, butalltheideasinthispostarefundamentallyDaves.

Firstthingsfirst
Davefirstnoticedthat

forall

.Inotedthisinmypreviouspostaswell,butsaidonlythatthisisapparentfromthinkingabout

therecursiveconstructionIdescribed.However,itsworthgoingthroughaproofofthismoreformally;itsa goodexampleofaproofbyinduction.First,letsrecalltherecurrencerelationthatdefinesthehyperbinary sequence:

Nowwecanshowthat Now,letssupposethat

for

.First,thebasecase:when

;check!

forsomeparticular

;thisiscalledtheinductivehypothesis,whichIwill isobviouslyodd,sowewanttoapplyrule

abbreviateasIH.wewanttoshowthat

aswell.

(O).Theonlytrickistogetitintotherightformfirst:

Presto!Sincethethingwewantedtoshowistruewhen inductionitmustbetrueforall .

andistruefor

wheneveritistruefor ,by

Follow

123 of 224

FollowTheMathLess Traveled 10/16/12 16:55


Get everynewpost deliveredto your Inbox.

The Math Less Traveled | Explorations in mathema...

http://mathlesstraveled.com/

Secondthings,thirdthings
ButDavethenwentontonoticethatthissortofpatternisntuniqueto1.Forexample,

for

,and

alsofor

.Daveconjecturedthatthisgeneralizes,andindeeditdoes:mostgenerally,wecansaythat

for

.Thatis,onceacertainnumberoccursatposition .Notethat

,itwillkeepoccurringinaregularpatternafter isaspecialcasewith .

that,ateverypositionoftheform

Whyisthis?Intuitively,itsbecauseofthecopyingthebluenumbersinmypreviouspost.Onceanumber occurs,itwillkeepgettingcopied,butwiththedistancebetweenthecopiesdoublingeachtime.Butagain,this isworthprovingformally.Illletyoudothisonetheproofbyinductionissimilartotheproofabove! Butwhydooccurrencesof1correspondto ,andoccurrencesof2to ,andoccurrencesof3to and

,andsoon?Arethereanypatternstobefoundhere?ThisiswhatDaveexplorednext;letsfollow.

Primaryoccurrencesandtheinverseofh
Letsthinkalittlemoreaboutthisequation when form (inwhichcase . Thismeansthatif isanyoddnumber,wecanwriteitintheform ,andso :thatis, .Letssupposethat isodd(ifitisnt,wecan isalwaysodd,except

alwaystakeanotherfactoroftwooutofittocombinewiththe ).Noticethat

,whichiseven).Moreover,wecanalwayswriteanyoddnumberinthe

,where isodd:justaddone,andfactoroutasmanycopiesof2aspossible.Forexample,

everyoddpositioninthehyperbinarysequenceisjustcopiedfromanearlierevenposition.Ofcourse,thiscan alsobeseenfromtherecursiveconstructioninmypreviouspost;butnowwehavereallyprovedit!Andwhat abouttheevenpositions?Thevaluesofthehyperbinarysequenceatevenpositionsareobtainedbyadding, ratherthancopying.If occursatanevenposition,wewillcallitaprimaryoccurrenceof . Thenicethingaboutprimaryoccurrencesisthat(a)thereareonlyfinitelymanyprimaryoccurrencesofany particular (challenge:provethis!),and(b)onceweknowthepositionsoftheprimaryoccurrencesof ,we knowthepositionsofalltheoccurrencesof :theyareatpositionsoftheform occurrence. So,given ,canwefindallitsprimaryoccurrences?Essentiallyweareaskingifwecancompute (bywhichI ,where isaprimary

meantheinverserelationof ;since isnotinjectiveitsinverseisobviouslynotafunction),butleavingout non-primaryoccurrencessincewecaneasilyfindthosefromprimaryones.Letsstartbymakingatablewith Follow

124 of 224

FollowTheMathLess Traveled 10/16/12 16:55


Get everynewpost deliveredto your Inbox.

The Math Less Traveled | Explorations in mathema...

http://mathlesstraveled.com/

somesmallvaluesof :
n Primary occurrences

1 2 3 4 5 6 7 8 9 10 11 12 13 14 15 16

0 2 4,6 8,14 10,12,16,30 32,62 18,22,24,28,64,126 20,26,128,254 34,46,48,60,256,510 38,56,512,1022 36,40,54,58,66,94,96,124,1024,2046 44,50,2048,4094 42,52,70,78,112,120,130,190,192,252,4096,8190 68,80,110,122,8192,16382 72,118,258,382,384,508,16384,32766 92,98,134,158,224,248,32768,65534

Doyouseeanypatterns?Ifso,canyouprovethembyinduction?Illtalkaboutafewinmynextpost.ButIstill dontknowwhethertheresaneffectivewaytocompute !

Posted in challenges, induction, pattern, proof, recursion, sequences, solutions | Tagged hyperbinary, induction | 12 Comments

Challenge #12 solution, part II


Posted on September 19, 2009

Yes,thatsright,thatChallenge#12,postedoneyear,fivemonths,andadayago.Yousee,Ihavethisnasty habitofstartingthingsandnotfinishingthemwell,betterlatethannever! Questiontwooftheaforementionedchallengeaskedthis: Givenapositiveintegern,inhowmanywayscannbewrittenasasumofpowersoftwo,wheneach powerisallowedtooccuratmosttwice?Say functionh?


Follow

denotesthenumberofdifferentwaystowritenasa

sumofpowersoftwowithatmosttwocopiesofeachpowerallowed.Whatcanyousayaboutthe

125 of 224

FollowTheMathLess Traveled 10/16/12 16:55


Get everynewpost deliveredto your Inbox.

The Math Less Traveled | Explorations in mathema...

http://mathlesstraveled.com/

Theserepresentationsusingatmosttwocopiesofeachpoweroftwoaresometimescalledhyperbinary representations.Forexample,11canbewrittenas hyperbinaryrepresentationsof11,so . oras .Itturnsoutthesearetheonly

Howtoanalyzethis?WecanuseasimilarapproachtothatoutlinedinthesolutiontopartIII(whichaskeda similarquestion,butallowinganynumberofcopiesofeachpoweroftwo).Firstletsthinkaboutoddnumbers, thatis,numbersoftheform .Anyrepresentationofanoddnumbermustincludeanoddnumberofcopies of (otherwisethesumwontbeodd!),butsinceatmosttwocopiesareallowed,theremustbeexactlyone copyof .Ifwetakeawaythe anddivideeverythingelsebytwo,wegetavalidrepresentationof .The reverseworksaswell:ifwestartwithanyvalidrepresentationof ,multiplyeverythingbytwo,andaddacopy of ,wegetavalidrepresentationof . Whataboutevennumbers?Avalidrepresentationof haseithertwocopiesof ,ornone.Ifithasnone, (andviceversa);iftwo,wecantakeawaythe .Thismeansthat

wecansimplydividebytwotogetavalidrepresentationof

copiesof andthendividebytwotogetarepresentationof ,andviceversa.Therefore, . Thesetwoequations, completelycharacterize and .Wecaneasilycompute ,alongwiththefactthat forthefirstfewvaluesof : ,areenoughto

1,1,2,1,3,2,3,1,4,3,5,2,5,3,4,1,5,4,7,3,8,5,7,2,7,5,8,3,7,4,5,1 Itsimmediatelystriking,ofcourse,that .Infact,evenasthevaluesof hidinginthere.Infact, every . isntstrictlyincreasing:thatis,justbecause ,andtheobviousconjectureisthat doesntmean for

continuetogetbiggeronaverage,therearestillsomevaluesof1

Thenextthingwenotice,afterstaringabit,isthatthe

sandwichedinbetweentwo1 sseemstoreadthesameforwardsandbackwards.Letsmakesomegraphssowe canseethesepatternsmoreeasily:belowIvemadegraphsshowing 10000,respectively. as goesfrom1to100,1000,and

sequencecontainsalotofpalindromes!Everysection

Follow

126 of 224

FollowTheMathLess Traveled 10/16/12 16:55


Get everynewpost deliveredto your Inbox.

The Math Less Traveled | Explorations in mathema...

http://mathlesstraveled.com/

Pretty!Wecanclearlyseethesymmetricstructurehere,andthewaythatitoccasionally(every downto1. Heresanotherwaytothinkaboutbuildingthesequence.Westartwiththefirsttwoelements:

)jumpsback

Nowwecopytheredpartontotheendofthesequence(thecopywillbeblue),leavingablankspaceinfrontof eachbluenumber: , , , Nowwefillineachblank(ingreen)byaddingthetwonumbersoneithersideofit: , , , Nowtheblueandgreenpartbecomesthenewredpart: , , Nowwerepeat!Copytheredparttotheend,leavingablankinfrontofeachcopiednumber: , , , , , , Fillineachblankbyaddingthetwonumbersoneitherside:


Follow

127 of 224

FollowTheMathLess Traveled 10/16/12 16:55


Get everynewpost deliveredto your Inbox.

The Math Less Traveled | Explorations in mathema...

http://mathlesstraveled.com/

, , , , , , Andagain:blueandgreenbecomethenewreds,copytherednumberstotheendwithablankinfrontofeach: , , , , , , , , , , , , Andfillintheblanksbyadding: , , , , , , , , , , , , Andsoon.Ittakesalittlebitofthought,butitcanbeseenthatthemethodIvedescribedhereisreallyjusta differentwayofexpressingtheequations waymakesitapparentwhya1alwaysshowsupat and !Butthinkingaboutitthis ,andalsowhyweendupwithpalindromes.

Illstoptherefornow;inanupcomingpostIllbringthingsfullcirclebyshowinghowthesenumbersrelateto theCalkin-Wilftreeof rationals.


Posted in challenges, counting, induction, pattern, sequences, solutions | Tagged binary, hyperbinary | 15 Comments

Book review: The Mathematical Mechanic


Posted on August 1, 2009

Iamnotaphysicist. Myhighschoolphysicsclasswastaughtbyamanwhowasprobablybrilliant.Isayprobablybecausehewas sobadatexplainingthingsthatitwasimpossibletotell.Hislectureswentlikethis:hewouldstateaproblem anddrawsomesortofdiagramontheboard.Hewouldthenproceedtomumbleincomprehensiblethingswhile drawing(apparentlyrandom)arrowsandequationsonthediagram.Thenhewouldwritedownthecorrect answer.Ourfinalexamconsistedofthemultiple-choiceportionofanoldAPphysicsexam;Igotascoreof17. Outof200. Itwasthehighestscoreintheclass. Incollege,Ioccasionallythoughtabouttakingaphysicscourse,butitneverquitefitmyschedule.Besides,Ihad thisvaguesense,leftoverfromhighschool,thatphysicswasmessy,inelegant,andfullofequationsthatdidnt yieldanyintuitiveinsightintoanything. Iwaspleasantlysurprised,then,toenjoyMarkLevisnewbook,TheMathematicalMechanic:UsingPhysical ReasoningtoSolveProblems(PrincetonUniversityPress,2009).[Fulldisclosure:PUPkindlysentmeafree reviewcopy.Doesthatmakemebiased?Well,probably.] Herestheidea:youhaveathornymathematicalproblemtosolve.Forexample,letssayyouwanttoprovethe PythagoreanTheorem(whichis,infact,thesubjectofChapter2).Youthen(thisisthehardpart)comeupwith somesortofappropriateidealizedphysicalsystem,suchasafreelyrotatingtriangulartankfullofwater,ora
Follow

128 of 224

FollowTheMathLess Traveled 10/16/12 16:55


Get everynewpost deliveredto your Inbox.

The Math Less Traveled | Explorations in mathema...

http://mathlesstraveled.com/

frictionlessringslidingonasemicircleandattachedtotwosprings,andsoon.Youthenmakesomesortof observationthatqualifiesyouasbeingatleastthreeyearsold,suchas,Thetankfullofwaterdoesntmove. Becauseatankfullofwaterdoesntjustmovebyitself.Andthenyouwritedownsomeequationswhich representthefactthatthewaterdoesntmove,andheypresto!ThePythagoreanTheorem! Thebookischock-fulloftheseseeminglymagicalphysicalthoughtexperimentsinvolvingbicyclewheels,pistons, springs,soapfilms,pendulums,andelectriccircuits,withapplicationstogeometry,maximizationand minimizationproblems,inequalities,optics,integrals,andcomplexfunctions,obviouslycollectedoveran (intellectuallyadventurous)lifetime.Idoubtitwillactuallyhelpmesolveproblemsanytimesoon,butinsome sensethatisntreallythepoint;itgavemenewintuitionandinsightintobothphysicsandmathematics,anda newappreciationmissedinhighschoolfortheeleganceandbeautyfoundinthestudyofaphysicalworld thatcanbedescribed(perhapsunreasonablyso)bymathematicalequations. Theproseispithy,andIhadnotroublefollowingthegistofthearguments.Myonecomplaintisthatthedetails areoftenhardtofollowforanon-physicistlikeme:althoughthebookclaimstobeself-contained,withan appendixcoveringallthenecessarybackground,theappendixissoconciseastobenotallthatusefulunlessyou alreadyknowsomephysicsinwhichcaseyouprobablydontneedtheappendixinthefirstplace!Butinsome sense,thisismoreacomplaintaboutmylackofeducationthanaboutthebook.Itisntmeanttobe(norshould itbe)aphysicstutorial,soitsappropriateforittoassumesomebackground;Imprettysurethatanyonewitha (decent!)highschoolcourseinphysicscouldfollowmostofitwellenough. Overall,Ihighlyrecommendittoanyonewhois(evenslightly)interestedinphysics,andappreciates mathematicaleleganceandcleverness.Itwouldmakeagreatgiftforalmostanyone,whetherahighschool studentoruniversityprofessor,armchairphysicistorprofessionalmathematician.
Posted in books | Tagged physics, problem solving, review | 1 Comment

Curta Calculating Machines


Posted on June 20, 2009

[Tryingtoclearoutthequeueofdraftpostshere...IstartedwritingthispostinSeptemberof2008butnever posteditforsomereason!] ThankstoRodatReasonableDeviations,IrecentlyreadaboutCurtacalculatorsamazingmechanical calculatorsthatweremanufacturedfromthe1940sto1970s.

Follow

129 of 224

FollowTheMathLess Traveled 10/16/12 16:55


Get everynewpost deliveredto your Inbox.

The Math Less Traveled | Explorations in mathema...

http://mathlesstraveled.com/

Ifyouwanttoplaywithone,heresasimulatorthatsomeonemade.Ifwouldbeevenmorefuntoplaywithone inreallife,ofcourse,butIheartheyareratherexpensivenow!Isawoneinreallifeinadisplaycaseinthe PrincetoncomputersciencedepartmentbutofcourseIdidntgettoplaywithit. IalsoencourageyoutogoreadRodspost,whichincludesanumberofotherlinkstoinformationaboutthese fascinatingmachines!


Posted in computation, links, video | Tagged calculator, Curta, mechanical | 4 Comments

Ways to multiply
Posted on June 12, 2009

at360hasacoupleofposts(withmoretocome!)listingabunchofdifferentalgorithmsforperforming
aboutsomeofthem.
Posted in computation | Tagged algorithms, methods, multiplication

multiplication.Didyouevenknowtherewasmorethanone?Wouldyoubesurprisedtolearnthatoverthe

years,peoplehavecomeupwith(atleast)twentyfivedifferentmethods?Itstrue!Headoverto360toread

Square roots with pencil and paper: method 2


Posted on June 11, 2009

AlittlewhileagoIwroteabouttheBabylonianmethodforapproximatingsquarerootswithpencilandpaper.In thatpostInotedthattheBabylonianmethodisquiteefficient,butannoyinginsomewayssinceitmakesyou dealwithbigfractionsand/orlongdivision.TodayImgoingtodescribeanothermethodforcomputingsquare rootswithpencilandpaperwhichislessefficient,butexplicitlydealswiththedecimalrepresentationandhence moreconvenientinsomeways. Hereshowitworks.Letssayweretryingtofindthesquarerootof7.First,wewrite7as7.00000andgroup thedigitsbytwos,likeso:

Follow

130 of 224

FollowTheMathLess Traveled 10/16/12 16:55


Get everynewpost deliveredto your Inbox.

The Math Less Traveled | Explorations in mathema...

http://mathlesstraveled.com/

Thefirststepistowritedownthefirstdigitofthesolution.Thisiseasy: example,00).

(toosmall),but

(toobig),

sowewritedown2,thensubtract4from7andwritetheremainder,andbringdownthenexttwodigits(inthis

Now,heresthepartthatseemslikevoodoo,butdontworry,Illexplainwhyitworkslater!FornowIlljust sticktothehow.Ouranswersofaris2.Wedoubleit,getting4,andnowwelookforthelargestdigitdforwhich .I'mabusingnotationabithere;by Idon'tmean4timesd,Imeanthenumberyougetbypasting thedigitdafterthedigit4 .Someexperimentingshowsthat bringdownthenexttwodigits: anditsthenextdigitintheanswer.Wewrite6alongthetop,andsubtract276(whichis46*6)from300,then

.So6isthedigitwearelookingfor,

Nowwerepeattheprocess:wefirstdoubletheanswersofar(26)toget52,thenwelookforthelargestdigitd forwhich .Again,someexperimentingshowsthat and ,sothedigitwe arelookingforis4.Wewrite4alongthetop,subtract ,andbringdownthenexttwodigits:

Illcarryitonemorestep:twice264is528,and

(whereas

istoobig),soweget

Follow

131 of 224

FollowTheMathLess Traveled 10/16/12 16:55


Get everynewpost deliveredto your Inbox.

The Math Less Traveled | Explorations in mathema...

http://mathlesstraveled.com/

Sofar,wehavecomputedthat

.Andindeed,wecancheckthat

,but

So,whydoesthiswork?Well,supposewehaveworkedoutsofarthat

andwewanttofindanotherdigitinthisapproximation.First,wecanmultiplybothsidesoftheequalityby100:

Nowwewanttofindthevalueofbsothat

whichwillgiveusonemoredigitofprecision.Expandingoutthebinomialontheleft:

Factoringandrearrangingabit:

istheremainder;itswhatslistedatthebottomunderthemostrecentsubtraction.Thefactorof100 correspondstobringingdownthenexttwodigits.Andwhatdowewantontheleft?Wewantavalueofbfor which islessthanthisremainder.Thislooksfamiliar!The iswherewemultiplythecurrent correspondstoappendingthedigitbto . answerbytwo,andthen

So,whichisbetter,thismethodortheBabylonianmethod?Idontknow,theyareverydifferent.Icanthinkof situationswhereIwouldwantoneortheother.IguessIllleavethedeterminationuptoyou!
Posted in algebra, computation | Tagged pencil and paper, square root | 8 Comments

Carnival of Mathematics #53


Posted on June 5, 2009

Welcometothe53rdCarnivalofMathematics!Asomewhatshortercarnivalthistime,butIgreatlyenjoyed readingthesubmissionsIdidget,andIhopeyoudotoo. MeaghanMontroseatTutorFipresentstentipsfordailypracticingyourmathskills. ErinatNotefromtheTeachersharessometipsonhowtogettutoring,practicematerials,andothermath helpforyourchildwithoutcoughingupatonofmoney. MathMama,akaSueVanHattum,linkstotheInstituteforFiguring,anamazingorganizationwhichis,to quotetheaboutpage, dedicatedtoenhancingthepublicunderstandingoffiguresandfiguringtechniques.Fromthe physicsofsnowflakesandthehyperbolicgeometryofseaslugs,tothemathematicsofpaperfolding andgraphicalmodelsofthehumanmind,theInstitutetakesasitspurviewacomplexecologyof figuring.
Follow

132 of 224

FollowTheMathLess Traveled 10/16/12 16:55


Get everynewpost deliveredto your Inbox.

The Math Less Traveled | Explorations in mathema...

http://mathlesstraveled.com/

Thesiteiswellworthexploring!Mathmamaparticularlylikestheirphotogalleries,whichshowoffcrocheted explorationsofhyperbolicgeometry! BarryLeibaofStaringatEmptyPagesexploresthetrigonometryofcardoors.Ifyouretryingtogetoutof yourcarinatightparkingspot,whichwouldyouratherhave,awidedoororanarrowone? PatBallewtalksaboutthesubfactorialoperation!n,whichcountsthenumberofwaysofreorderingnthings sothatnoneofthemisinitsoriginalposition(knownasderangements),andwonderswherethenotation camefrom. Whatcouldbreastfeedingpossiblyhavetodowiththegoldenratio?JohnCookdiscussesrational approximationsoverathisblog,TheEndeavour. BadalJoshi,amathgradstudentatOhioStateUniversity,hasaneatnewblog,TheSquaredCircle.I especiallylikehisrecentseriesofpostsongamesandprobability,butthereslotsofothergreatstuffaswell. TheNumberWarrior,JasonDyer,writesaboutanapparentbuginWolframAlpha.HowcanwetrustWolfram togetcomplexintegralsrightiftheycantevenmanageBabyloniannumbernotation? Lastbutnotleast,DaveRichesonatDivisionbyZerohassomebeautifulGeoGebraappletsforplayingwith theJapaneseTheorem,whichrelatestheradiiofcirclesinscribedintriangulationsofcyclicpolygons.
Posted in links, meta, people | Tagged Alpha, approximation, Babylonian, breastfeeding, car, Carnival of Mathematics, coral, crocheting, derangements, doors, games, GeoGebra, golden ratio, hyperbolic, Japanese Theorem, notation, probability, rational, subfactorial, tips, trigonometry, tutoring, Wolfram | 9 Comments

Carnival of Mathematics: here, this Friday!


Posted on May 31, 2009

IwillbehostingtheCarnivalofMathematicsherethisFriday(June5).Pleasesendinyoursubmissions! IMPORTANT:itsalongstorybutIamnotyetlistedasthehostontheblogcarnivalsite,soDONOTsubmit postsusingtheblogcarnivalsubmissionform!Pleaseleaveyoursubmissionsasacommentonthispost,orsend meanemail(byorgeyatgmail).Ifyoualreadysubmittedsomethingusingtheblogcarnivalsubmissionform, pleasere-submitittome. Thesubmissiondeadlineis8amonFriday,June5.Submitbeforethedeadline,orelsebepreparedtofacemy ummilddispleasure!


Posted in meta | Tagged blog, carnival, Carnival of Mathematics | 6 Comments

MMM #33: Super divisible


Posted on May 25, 2009

ThisweeksMondayMathMadnessisupatWildAboutMath!.Lookslikeafairlyaccessibleproblemthisweek: Whatstheprimefactorizationofthesmallestwholenumberthatisdivisiblebyallintegersfrom1 uptoandincluding50? Submitananswer(seethepostforinstructions),andmaybewinaprize!


Posted in challenges, links, number theory, primes | Tagged divisible, factorization, smallest

Follow

133 of 224

FollowTheMathLess Traveled 10/16/12 16:55


Get everynewpost deliveredto your Inbox.

The Math Less Traveled | Explorations in mathema...

http://mathlesstraveled.com/

Square roots with pencil and paper: the Babylonian method


Posted on May 18, 2009

Everyoneknowshowtoadd,subtract,multiplyanddividewithpencilandpaper;butdoyouknowhowtofind squarerootswithoutacalculator?(Incidentally,IhighlyrecommendreadingTheFeelingofPowerbyIsaac Asimov,ashortstoryaboutafutureinwhichhumansaresoreliantoncomputersthattheyhaveforgottenhow todoarithmetic.) Anobviousmethodistoguessandcheckwhilekeepingtrackoflowerandupperbounds.Forexample,ifwe wantedtofindthesquarerootif7,wemightstartbyguessingthatthesquarerootis2.Computing seethat2istoosmall.Sowetry3: inbetween2and3.Letstry2.5: ,we ,so3istoobig!Soweknowthesquarerootof7mustbesomewhere .So2.5istoosmall,andthesquarerootof7issomewherebetween

2.5and3.Wemighttry2.7next(toobig),andsoon. Thisworks,butitisextremelytediousandinefficient!Wecancutthesearchrangeinhalfateachstep,butthis meansthatonaverageweonlyaddasinglenewdecimalplaceevery3.3stepsorso( bettermethods;Illshareoneofthemtodayandoneinafuturepost. ThefirstmethodisoftencalledtheBabylonianmethodsinceitwasknowntotheancientBabylonians.Heres howitworks.SaywearetryingtofindthesquarerootofN .Justlikewiththeguessandcheckmethod,westart outwithsomeguessR.ThenwecomputeanewvalueforRasfollows: ).Notto mentionthatateachstepwehavetocomputethesquareofincreasinglylongnumbers.Thereareatleasttwo

Repeatingthisprocesswillresultincloserandcloserapproximationsto Letstryanexample,againusing and

asourinitialguess.Wecancomputeafewiterationsofthe

processaccordingtotheaboveformula:

Howclosedidweget?Thetruevalueof

,to15decimalplaces,is

(Incidentally,IcomputedthisusingWolfram|Alphabytypingsqrt7to15digits.)Hereareour approximations,withthecorrectdecimalplacesinbold:

Follow

134 of 224

FollowTheMathLess Traveled 10/16/12 16:55


Get everynewpost deliveredto your Inbox.

The Math Less Traveled | Explorations in mathema...

http://mathlesstraveled.com/

Wow!Thatconvergesprettyfast.Infact,thismethodconvergesquadraticallythenumberofcorrectdecimal placesapproximatelydoubleswitheverystep! So,whydoesthiswork?Well,firstofall,notethatif operation),then (thatis,ifRisafixedpointofthis

Also,itisnottoohardtoseethat eachstep.

mustlieinbetween and

,since

;sotakingtheiraverage at

(whichisessentiallywhattheBabylonianmethoddoes)willnecessarilygiveusabetterapproximationto

TheBabylonianmethodisoneofthefastest-convergingmethodsforcomputingsquareroots,butitcanbe somewhatinconvenient.Youhavetochoosewhethertodoallthecalculationswithfractionsandthenconvertto adecimalrepresentationattheend(asIdidabove),whichmeansyouhavetodealwithmultiplyingratherlarge numbers;oruseadecimalrepresentationthroughout,whichmeansyouhavetodosomeannoyinglongdivision. Theresanothermethodwhichdoesntconvergeasquicklybutcanbemuchmoreconvenient,sinceitexplicitly usesdecimalnotationandinvolvessomewhatmoremanageableoperations;Illdescribethisothermethodinan upcomingpost. FormorereadingontheBabylonianmethodandanumberofrelatedgeneralizations,checkoutthisMathPages article.


Posted in algebra, computation, convergence, iteration, number theory | Tagged Babylonian, method, pencil and paper, square root | 10 Comments

Wolfram|Alpha
Posted on May 16, 2009

TheInternetisabuzzwiththereleaseofWolfram|Alpha,Wolframsnewcomputationalknowledgeengine. (WolframisthecompanythatmakesMathematica.)SoIwenttotryitout.Itssortoflikeasearchenginein thatyoutypeinqueries;butitdoesntactuallysearchtheweb,itsearchesan(apparentlyhuge)setofdatabases thatithasandcomputesvariousanswersandtypesofinformationbasedonyourquery.Forexample,Itypedin mybirthday(January10,1982)andittoldmewhatdayoftheweekitwas,theamountoftimethathaspassed sincethen(inyears+months+days,intotalweeks,intotaldays),anyholidaysthatwereobserved,famous peoplewhodied/werebornonthatday,sunrise,sunset,andthephaseofthemoon.Inoticedthatthetotal numberofdayssincemybirthdaywas9988almost10,000!SoItriedtypinginJanuary10,1982+10000 daysandsureenough,itworked:ittoldmethatmy10,000thdayofbeingalivewilloccuronThursday,May 28,12daysfromnow,whenthemoonwillbewaxingcrescentandsunrisewillbeat5:37AM.Neat!
Follow

135 of 224

FollowTheMathLess Traveled 10/16/12 16:55


Get everynewpost deliveredto your Inbox.

The Math Less Traveled | Explorations in mathema...

http://mathlesstraveled.com/

ItypedinanotherqueryanditinformedmethatmyquerycouldntbeprocessedbecauseWolfram|Alphawas undertoomuchload.Instead,itofferedtoshowmealivevideofeedoftheWolfram|Alphacontrolroom,which wasamusing(justenvisionlotsofguyssittingaroundinfrontofcomputersshowingspikymulticoloredgraphs =).


Posted in computation, links | Tagged Wolfram|Alpha | 5 Comments

Full Speed Ahead!


Posted on May 15, 2009

Hoorayforsummer!Nowthatfinalsarefinallyover,youcanexpectalotmorefrommeoverthenextfew months.Ivegotalotofexcitingthingsplanned,including adescriptionandexplanationofanalgorithmforfindingsquarerootsbyhand(whichmostpeopledont knowanymore); finallyFINALLYforrealfinishingmyseriesofpostsonRecountingtheRationals(yes,Jason,thisoneisfor you!); goingthroughaneatlittlepaperbyIvanNivengivingaproofthatpiisirrational; alittlenumbertheory,focusingontheFundamentalTheoremofArithmetic; funlinkstointerestingmathematicsaroundtheweb; andwhateverelseticklesmymathematicalfancy! Togetthingsstarted,heresalinkforyou:NormanWildbergersMathFoundations,aseriesofvideosexplaining thefoundationsofmathematics.Idontagreewithallhisopinionsparticularlyregardinginfinitesetsbutthis isasubjectonwhichImwillingtoagreetodisagree;asalegitimate,publishedresearchmathematicianwhos beendoingmathematicslongerthanIvebeenalive,hescertainlyentitledtohisopinions!AndIcertainlydo agreewithmanyofhisopinions,especiallyregardingmathematicseducation.Inanyevent,thevideosarevery well-done,andthereslotsofinterestingstuffthere!
Posted in links, meta, people, video | Tagged foundations, Normal Wildberger, plans, summer | 3 Comments

Monday Math Madness #31


Posted on April 27, 2009

ThisweeksMondayMathMadnessisanicelittleprobleminvolvingcomplexexponentiation.Gocheckitout, andmaybewinaprize!
Posted in challenges, complex numbers, links | Tagged complex, exponentiation, madness, math, monday

Idempotent endofunctions
Posted on April 17, 2009

ViaTopologicalMusingscomesanotherneatlittlecountingproblem. Afunctionisidempotentifapplyingittwicegivesthesameresultasapplyingitonce:thatis, for


Follow

anyinputx.Endofunctionisjustafancywayoftalkingaboutafunctionwhosedomainandcodomainarethe

136 of 224

FollowTheMathLess Traveled 10/16/12 16:55


Get everynewpost deliveredto your Inbox.

The Math Less Traveled | Explorations in mathema...

http://mathlesstraveled.com/

same. Thechallenge:considertheset expressionintermsofn. Feelfreetopostvariousapproachesandsolutionmethodsinthecomments,butIwontbepostingasolution later;forthatIlljustdirectyoutoVishalspostwhereIgotthisproblem.Hegoesontodiscusstherelationto exponentialgeneratingfunctionsandcombinatorialspecies,whichisanamazingtopic(althoughquiteaways abovethenormallevelofTMLT;youhavebeenwarned!).


Posted in challenges, counting | Tagged counting, function, idempotent | 8 Comments

;howmanydifferentidempotentfunctionsaretherefrom to ? , ,and .Youranswershouldbean

Forexample,whenn=3onesuchfunctionisgivenby

Distributing cookies: solutions


Posted on April 14, 2009

Andnowforsomesolutionstothecookiedistributionproblem.Imactuallygoingtodescribefourdifferent methodsofsolution,andthereby(re)discoversomenicecombinatorialidentitiesalongtheway.ThisiswhatI loveaboutcombinatoricsyoudiscoverallthisrichstructurejustbycomingupwithdifferentwaystocount things!

Solution1:CookiesandFences
Thefirstsolution,asdescribedinacommentbyMike,consistsinconsideringpermutationsofcookiesand fences.

Ifwesetupthecookiesinarow,wecanusefourfencestoseparatethemintofivesections;thefirststudent willgetthefirstsectionofcookies,thesecondstudentwillgetthesecondsection,andsoon.Intheexample above,studentAgetsonecookie,studentBgetstwo,studentCdoesntgetany,andstudentsDandEgetthree andfour,respectively.Itseasytoseethateverydistributionofcookiescorrespondstoanarrangementoffences, andviceversa.So,howmanywaysaretheretoarrangethefences?Thinkofitthisway:wehavefourteen slots,andintoeachslotwecanputeitheracookieorafence.Choosingadistributionisjustchoosinginwhich ofthefourteenslotstoputthefourfences,andthereare waystochoosefouroutoffourteenslots.(If youdontknowaboutbinomialcoefficients,youcanreadaboutthemhere.)Incidentally,thisisalsothenumber ofwaystodistributefourfencestoelevenstudents;justusethecookiestodividethefencesintoelevensections (mostofwhichwill,ofcourse,beempty).AlthoughIhavenoideawhystudentswouldwantfences. Thiscanbeeasilygeneralized:theaboveargumentshowsthatthereare students. So,theproblemissolved,right?Whatmorecantherebetosayaboutit? plenty!Thisisaniceelegantsolution,butittakessomecleverness(and/orluck,and/orexperiencewiththis sortofproblem)toseeit,andinfactitisntthefirstsolutionIcameupwith.Lookingatsomedifferentmethods Follow waystodistributec cookiestos

137 of 224

FollowTheMathLess Traveled 10/16/12 16:55


Get everynewpost deliveredto your Inbox.

The Math Less Traveled | Explorations in mathema...

http://mathlesstraveled.com/

ofsolutionwillalsoilluminatesomeoftherichstructureofbinomialcoefficients.

Solution2:DontBeSoMean
Itseemsmeanthatsometimessomestudentsdontgetanycookiesatall.Wouldntitbemuchnicerifevery studentgotatleastonecookie(eveniftheystillmightbedistributedunevenly)?Thekeyinsight,aspointedout byJonathan,isthis:thenumberofwaystodistributetencookiestofivestudentsisthesameasthenumberof waystodistributefifteencookiestofivestudentsinsuchawaythateachstudentgetsatleastonecookie. Whyisthis?Well,supposeyouhavedistributedfifteencookiestofivestudentssothateachstudenthasatleast one.Nowtakeonecookieawayfromeachstudentandyouhavenowdistributedtencookiestofivestudents. Conversely,ifyouhavedistributedtencookiestofivestudentssomehow,ifyounowgiveeachstudentanextra cookie,youhavedistributedfifteencookiesandeachstudenthasatleastone.Eachdistributionoftencookies correspondstopreciselyonedistributionoffifteencookieswhereeachstudentgetsatleastone,andviceversa, socountingoneisthesameascountingtheother.Now,howmanywaysaretheretodistributefifteencookies likethis?Wecanlinethecookiesupandimaginechoosingpointsatwhichtodividethem;todividetheminto fivegroups,wehavetochooseexactlyfouroutofthefourteenpossibleplacestodividethecookies.(Thereisat leastonecookiebetweenanytwodividingpoints,whichmeansthateachstudenthastogetatleastonecookie.)

TheaboveillustrationshowsadistributioninwhichstudentsA,B,C,D,andEgettwo,three,one,four,andfive cookiesrespectively.Notethatthisisthedistributionoffifteencookiesthatcorrespondstothedistributionof tencookiesillustratedwiththefencespreviously. So,thenumberofwaystodistributefifteencookiestofivestudentssothateachgetsatleastoneis generally,thenumberofwaystodistributec cookiestosstudentssoeachgetsatleastoneis tosstudentssothateachgetsatleastone. ;more

.Also,thetotal

numberofwaystodistributec cookiestosstudentsisthesameasthenumberofwaystodistributec+scookies

Solution3:Oneatatime,please!
Anotherwayofapproachingtheproblemcomescourtesyofmeichenl.Todistributecookiestosstudents,we firsthavetogivesomenumberofcookiestothefirststudent;wecangiveheranywherefromzerototen cookies.Ifwegiveherkcookies,thereareckcookieslefttodistributetotheothers1students.Ifwelet standforthenumberofwaysofDistributingc cookiestosstudents,wecanwritethisobservationas

Thatis,wesplitthedistributionupintocasesbasedonhowmanycookieswegivetothefirststudent;ineach casewevereducedtheproblemtoasimplerdistributionproblem.Addingeverythingup,wegettheabove recurrencewevedefined recursively,thatis,intermsofitself.Thisdefinitionactuallygetsofftheground foranys(theresonlyonewaytodistributezerocookies


Follow

becausewecanidentifysomebasecases,thatis,simplesituationsinwhichweknowtheanswerwithout breakingthingsdownfurther.Inparticular,

138 of 224

FollowTheMathLess Traveled 10/16/12 16:55


Get everynewpost deliveredto your Inbox.

The Math Less Traveled | Explorations in mathema...

http://mathlesstraveled.com/

namely,togivezerocookiestoeachstudent),and cookiestoasingleluckystudent!).

foranyc (theresonlyonewaytodistribute

So,oncewehavethisrecurrence,whatdowedowithit?Wecanuseitfordirectcalculation,asMarkJames apparentlydid;healsousedtherecurrenceinacomputerprogramtoprintallthedistributions.Asobservedby DanielKlein,thissortofrecurrenceisalsoperfectlysuitedforaprogrammingtechniquecalleddynamic programming;itseasytoturnthisrecurrenceintoacomputerprogramtocompute foranyvaluesofc and ). Soifwe s(althoughinthisparticularcase,asweveseen,therearemoreefficientwaystocompute Butheresanotherinterestingthing:wealreadyknowwhat substitutethisintotheaboverecurrence,weget is,fromsolutions1and2:its

Ifwesubstituted+1inplaceofs1everywhere,wecanmakethisalittlenicerwithoutchanginganything(d isjustanewvariableImadeup,itdoesntstandforanythinginparticular):

Wevediscoveredanidentityinvolvingsumsofbinomialcoefficients!Neat!Now,youmightthink,OK,lets provethisbyinduction.Butweneeddonosuchthing.Wehavealreadyarguedthattheexpressionontheleft andtheexpressionontherightcountthesamethingssotheymustbeequal!Thisistheepitomeofa combinatorialproof:toshowthattwothingsareequal,justshowthattheyrepresenttwodifferentwaysof countingthesamething.

Solution4:Howmeandoyouwanttobe?
Thelastsolution(atleast,thelastsolutionIwilldiscussinthispost!)comescourtesyofDave.Insteadof breakingthedistributionintocasesaccordingtohowmanycookieswegivetothefirststudent(asinthe previoussolution),wecanbreakitintocasesaccordingtohowmanystudentsgetanycookies:wecangiveall thecookiestoonestudent,orgivethemalltotwostudents,orthree,andsoon. Thefirstcaseiseasy:ifwegiveallthecookiestoonestudent,thereare5waystochoosewhichstudenttogive thecookiesto,andonlyonewaytogivethemthecookies.Whatifwegiveallthecookiestotwostudents?Well, thereare waystochoosewhichtwostudentsgetcookies.Nowwewanttodistributethetencookiestothese twostudentsinsuchawaythateachstudentgetsatleastonecookie(otherwisewearebackinthefirstcase). Aspreviouslynotedinsolution2,thereare waystodothis.Ifthreestudentsgetcookies,thereare waysto choosethethreestudents,and waystodistributethecookies,andsoon.Generalizing,weseethat

Egads!Wevediscoveredanothercombinatorialidentity,thisoneinvolvingasumofproductsofbinomial coefficients!
Follow

139 of 224

FollowTheMathLess Traveled 10/16/12 16:55


Get everynewpost deliveredto your Inbox.

The Math Less Traveled | Explorations in mathema...

http://mathlesstraveled.com/

TheresyetanothermethodofsolutionImawareofthetheoryofcombinatorialspeciesandgenerating functionsbutthatcanwaitforanotherpost,Ithinkwevehadquiteenoughcombinatorialgoodnessforone day!


Posted in counting, proof, solutions | Tagged binomial coefficients, combinatorial identities, combinatorics, cookies, counting, distribution | 4 Comments

Distributing cookies
Posted on April 1, 2009

HeresaneatproblemIsawinarecentpostbyStevenMillerontheWilliamsCollegemathdepartmentblog.The problemcomesfromanoldPutnamcompetition,oneofthemostprestigiouscollegemathematicscompetitions. (Itsalsooneofthemostdifficultoutofapossible120points,themedianscoreis1!)

Therearetenidenticalcookiesandfivestudents.Howmanywayscanthecookiesbedistributedamongthe students? Notethatthecookiesareidentical,soitdoesntmatterwhichcookiesastudentgets,justhowmany.The students,ofcourse,arenotidentical,sostudentAgettingfourcookiesandstudentBgettingtwoisdifferent thanAgettingtwoandBgettingfour.Assumethatthecookiescantbesplitintopieces.Notethatgivingcookies tosomestudentsbutnotothersisavalidwayofdistributingthem,aslongasallthecookiesaredistributed.For example,givingsixcookiestostudentA,fourtostudentC,andnonetoanyoneelseisavaliddistribution. MywifeandIhadafuntimesolvingthisproblem,whichleadstoallkindsofinterestingcombinatorialinsights. Illdescribeour analysisinanupcomingpost.
Posted in challenges, counting | Tagged combinatorics, cookies, Putnam | 23 Comments

Chessboard counting: solutions and further challenges


Posted on March 30, 2009

Andnowforsomesolutionstothechessboardcountingchallenges. 1. Thefirstchallengewastocountthenumberofsquaresofanysizeonan88chessboard.Thekeyhere(as
Follow

140 of 224

FollowTheMathLess Traveled 10/16/12 16:55


Get everynewpost deliveredto your Inbox.

The Math Less Traveled | Explorations in mathema...

http://mathlesstraveled.com/

withmanycountingproblems)istobreaktheproblemdowninanappropriateway.Inthiscase,wecan thinkaboutcountingeachsizesquareseparately.Thereisonlyonesquarewithsidesoflength8.Thereare4 squaresofsize7theycanbeinoneoftwopositionshorizontally,andoneoftwopositionsvertically.Its nothardtoseethatthereare squaresis . totalsquares.Theres .(Showinghowtoderivethisformulaisa ,asexpected.) 1by1rectangles, jbyk squaresofsize6, ofsize5,andsoon.Sothetotalnumberof

2. Thispatterncontinues;onannbynchessboard,thereare actuallyaniceclosedformulaforthissumforgeneraln: goodsubjectforanotherpost.Wecancheckitforn=8: 1by2rectangles, 2by1rectangles,andsoon.Ingeneral,thereare

3. Countingrectanglesinsteadofsquaresturnsoutnottobetoomuchharder.Thereare rectangles.Sothetotalnumberofrectanglesis

Thatlooksannoyingtoaddup,buttheresaclevertrickwecanpull.Noticethatwecanfactoran8outofthe firstparenthesizedexpressionabove,a7outofthesecond,andsoon,yielding

Butnowwecanfactor

outofthis,yielding

Ah,thatsmuchnicer! 4. Forageneralmbynchessboard,thesamemethodyields

totalrectangles.Usingtheclosedformulaforthesumofthenumbersfrom1throughn,wecanrewritethis as .

Seethecommentstothepreviouspostforsomefurtheranalysis.Alsointhecomments,Vekyposedthe followingfollow-upchallenge:howmanysquares(orrectangles)arethereifwealsoallowsquares(rectangles) whicharenotparalleltothesidesofthechessboardtheonlyrestrictionisthattheirverticesmustoccurat crosshairpointsinbetweenthesquaresofthechessboard?


Posted in challenges, counting, solutions | Tagged chess, chess board, counting, rectangles, squares | 2 Comments

Chessboard counting
Posted on March 13, 2009

Iamcurrentlydoingaunitoncombinatorics(themathematicalstudyofcounting)withmyprecalculusstudents, andIwasinspiredtopostafewcounting-themedchallengeproblemsforyourenjoyment.(Also,itsmyspring break!) Asyouprobablyknow,achessboardconsistsof64squaresarrangedineightrowsandeightcolumns.


Follow

141 of 224

FollowTheMathLess Traveled 10/16/12 16:55


Get everynewpost deliveredto your Inbox.

The Math Less Traveled | Explorations in mathema...

http://mathlesstraveled.com/

1. Howmanysquaresofanysizeareonachessboard?Eachofthe64smallestsquarescount,ofcourse,but therearealsolargerones;forexample,thefoursquaresintheupperleftcornerforma22square. 2. Howmanysquaresofanysizewouldtherebeona9by9chessboard?10by10?nbyn? 3. Howmanyrectanglesofanysizeandshapearethereonachessboard?Isthiseasierorharderthancounting squares? 4. Howaboutrectangleson9by9,10by10,nbyn,ormbynchessboard?


Posted in challenges, counting | Tagged chess, chess board, counting, rectangles, squares | 10 Comments

Pentaflakes
Posted on February 7, 2009

Justalinktoday:MikeCroucheroveratWalkingRandomlyhassomegorgeouspicturesoffractalconstructions calledpentaflakes,madebyrecursivelygluingpentagonstogetherinvariousways.Hesalsomadea Mathematicademonstrationforplayingaroundwithvarioussortsofn-flakesinteractively(youdontneed MathematicatotryitoutjustthefreeMathematicaPlayer).


Posted in fractals, geometry, links | Tagged demonstration, interactive, Mathematica, pentaflake | 1 Comment

Carnival of Math #48, and Monday Math Madness #25


Posted on January 31, 2009

The48thCarnivalofMathematicsispostedatConcreteNonsense.MyfavoritepostsincludeFoxmathspost aboutastrangeiteratedsequenceinvolvingpiandthisamazingpictureofafractalcabbage.Alsonearanddear tomyheartisMarkDominusspostonmonadsandclosureoperators. Also,checkoutMondayMathMadness#25overatWildAboutMath!.Thisweeksproblemisaveryinteresting countingproblem.


Posted in challenges, counting, famous numbers, fractals, links | Tagged carnival, counting, fractal cabbage, monads, pi | 1 Comment

More on repetend lengths


Posted on January 27, 2009

Inapreviouspost,Inotedthatthelengthoftherepetend(repeatingportionofthedecimalexpansion)ofa fractionwithprimedenominatorpisatmostp-1,andinfactdividesp-1.Ialsosaid: Infact,theresevenmorethatcanbesaidaboutnon-primedenominators,aswell. Thiswassomethingofacop-out,andtodayImgoingtocorrectthat!Ingeneral,supposethedenominatordcan befactoredas

thatis,dcanbefactoredastheproductoftheprime tothe power,theprime tothe power,andsoon. (Forexample, .)Thenitturnsoutthatthelengthoftherepetendofanyfractionwithdenominatord


Follow

142 of 224

FollowTheMathLess Traveled 10/16/12 16:55


Get everynewpost deliveredto your Inbox.

The Math Less Traveled | Explorations in mathema...

http://mathlesstraveled.com/

willbeadivisorof

(The denotesaproduct;youcanreadaboutthenotationhereifyouhaventseenitbefore.)Whendisprime, wecanseethatthisjustreducestod-1aswewouldexpect.Foreachadditionalprimepinthefactorization,we multiplybyp-1;foreachadditionalpowerofaprimepbeyondthefirst,wemultiplybyp. So,forexample,therepetendlengthforfractionswithdenominator221=13*17shouldevenlydivide ;andindeed,therepetendlengthof1/221is48.Asanotherexample,therepetendlengthof is726,whichindeeddivides .

YoumaywonderhowIknowthis.Well,askingfortherepetendlengthofafractionwithdenominatord amountstoaskingforthesmallestpowerof10whoseremainder,whendividedbyd,is1.Anotherwaytosay thisisthatwewanttoknowtheorderoftheelement10inthegroup (thegroupofpositiveintegersless ?Theanswer,asexplainede.g. thanandrelativelyprimetodundermultiplication).ByLagrangesTheorem,theorderofanyelementofagroup dividestheorderofthegroup;sothequestionbecomes,whatistheorderof onp.155ofContemporaryAbstractAlgebrabyJosephGallian(HoughtonMifflin,2002),istheaboveformula. Now,maybethatwentwayoveryourhead.Itcertainlydidifyouveneverstudiedanygrouptheory.ButIdont knowasimplerwaytoexplainit!Perhapsthereisabetterway,andifyouknowofone,Idlovetohearaboutit inthecomments.Nonetheless,thisisagreatexampleofasimplequestionthatquicklyleadsintosomevery deepstructure. YoumayalsowonderhowonearthIknowthattherepetendlengthof1/17303is726!No,Ididntsitthereand dolongdivision;ofcourse,Iwroteacomputerprogram.PerhapsIwillpostitsoon.
Posted in group theory, number theory, pattern, primes | Tagged decimal, expansion, fractions, length, repetend | 6 Comments

New bookshelf entry: The Annotated Turing


Posted on January 26, 2009

FormybirthdayIgot,amongotherthings,acopyofTheAnnotatedTuring:AGuidedTour ThroughAlanTuringsHistoricPaperonComputabilityandtheTuringMachine,byCharles Petzold.Ihaventfinishedreadingityet,butsofar,Ihighlyrecommendit;notonlythat, butImpleasedtoreportthatIrecommenditevenforpeoplewhodonthavemuch backgroundincomputerscience.Petzolddoesagoodjobofsettingthestage,soto speak,describingthenecessarymathematicalbackgroundandcontext.Andsofarhis explanationofTuringspaperitselfisquitereadableandengaging. Whymightyoubeinterestedinreadingsuchathing?AlanTuringisoftenregardedasthe fatherofcomputerscience,anddidimportantfoundationalworkincomputerscience,mathematicsandwhat wewouldnowcallartificialintelligence.HisinventionoftheTuringmachineasimple,imaginarymachine whichservesasaformalizationofcomputationwasanachievementofcreativegeniusthatlaidthe foundationsformuchofmoderncomputerscience.Atthesametime,italsoputtorestoneofthebiggestopen Follow

143 of 224

FollowTheMathLess Traveled 10/16/12 16:55


Get everynewpost deliveredto your Inbox.

The Math Less Traveled | Explorations in mathema...

http://mathlesstraveled.com/

questionsinmathematicsofhisday,theEntscheidungsproblem,whetheritwaspossibletocomeupwithapurely mechanicalprocedurefordeterminingthetruthorfalsityofanarbitrarymathematicalstatement(writtendown inasuitablyformalway).Turingssurprisinganswer(alsoarrivedatindependentlybyAlonzoChurch,who inventedasecondimportantformalizationofcomputation,thelambda-calculus)wasno. Inshort,ifyoureatallcuriousaboutsomeofthebrilliantmathematicalcreativitythatwentintomaking possiblethemachinethatyouareusingatthisverymomenttoreadthisblogpostyoushouldreadThe AnnotatedTuring!


Posted in books, computation, people | Tagged review, Turing, Turing machine | 6 Comments

More on decimal expansions


Posted on January 21, 2009

Today,IdliketoanswersomeofthequestionsIraisedintheDecimalExpansionZoo: 1. Whichdecimalexpansionsterminate,andwhicharerepeatingandhowdoesitrelatetothedenominator? Asweknow,thedecimalexpansionofeveryrationalnumbereitherterminatesorrepeatsbutinasense, theyallrepeat;theonesthatterminatejusthappentorepeatthedigitzero.Thatis,0.25isreallyjust 0.25000000.Thisshouldgiveusacluethattheterminatingisnotreallyafundamentaldifference,butan artifactoftheparticularwaywevechosentorepresentnumbers,inbase10.Andindeed,asyoucancheck, thefractionswithterminatingrepresentationsarethosewhosedenominatorsaredivisibleonlyby2and5 (since2and5arethedivisorsof10).Ifweused,say,base21insteadofbase10,thefractionswith terminatingrepresentationswouldbetheoneswhosedenominatorsaredivisiblebyonly3and7,andsoon. 2. Howarethedifferentcyclesforagivendenominatorrelatedtoeachother,andwhy? Ifsomefractionhasadecimalexpansionwitharepeatingportionlike[abcde],theneverycyclic rearrangementof[abcde](thatis,[bcdea],[cdeab],[deabc],andsoon)alsooccursastheexpansionof someotherfractionwiththesamedenominator.Toseewhythisissoisnothardifyouthinkaboutthe processoflongdivision;theremainderateachstepuniquelydeterminesthenextremainder,andsoon,so giventhesamedivisor,wearealwaysgoingtoseetheexactsamesequenceofdigitsinthedecimalexpansion followingagivenremainder. 3. Howarethelengthsofthecyclesforagivendenominatorrelatedtothedenominatoritself? AsnotedbyJonathan, Thelengthoftherepeatingunitislessthanorequaltoonelessthanthedenominator.Thatscool. Indeed!Andunderstandingwhythismustbethecaseisnothard,again,ifwethinkabouttheprocessoflong divisiontoproducethedecimalexpansionforsomefraction.Supposethefractionhasdenominatord.At eachstepofthelongdivision,wemustgetaremainderlessthand.Ifweevergetaremainderofzero,the expansionterminates.Ifweevergetaremainderthatweveseenbefore,theexpansionwillbegintorepeat. So,thelongestanexpansioncanpossiblygobeforerepeatingis(d-1). However,asnotedbysilverpie,theresmore:
Follow

144 of 224

FollowTheMathLess Traveled 10/16/12 16:55


Get everynewpost deliveredto your Inbox.

The Math Less Traveled | Explorations in mathema...

http://mathlesstraveled.com/

Notonlyisitalwayslessthanorequaltod-1;forprimedenominators,itsadivisorofd-1.(The quotientisequaltothenumberofdifferentpatterns13,forexamplehas12/6ortwo6-digit patterns.) Thisistrue!Infact,theresevenmorethatcanbesaidaboutnon-primedenominators,aswell.However, unlikethepreviousobservations,thisoneisextremelynon-obvious.TheonlywayIknowhowtoproveit takesadetourthroughgrouptheory.PerhapsIllwriteaboutitsomeday,butfornowIllleaveyoutobe amazed.=)


Posted in number theory, pattern | Tagged decimal, expansion | 15 Comments

Carnival of Mathematics and other links


Posted on January 18, 2009

The47thCarnivalofMathematicshasbeenputtogetherbyjd2718;gogiveitaread!Someofmyfavoriteposts includefactoringbypuzzles,teachingbinarynumbers,the2009game,cliquesofdichotomies,butthereislots ofotherinterestingstuffaswell. KalidAzadofBetterExplainedhasagreatarticleaboutdevelopingyourintuitionformathematicsthatIalso wantedtoshare. Finally,hereisxkcdsusefulguidetoconvertingtometric.


Posted in links | Tagged Carnival of Mathematics, intuition, metric, xkcd | 1 Comment

Predicting pi: pretty graphs and convergents


Posted on January 16, 2009

RecallthechallengeIposedinapreviouspost:giventhesequenceofintegers

,whatcanyou

learnabout (assumingyoudidntknowanythingaboutitbefore)?Theanswer,asexplainedinanotherpost,is thatyoucanlearn towhateverprecisionyoulike,ifyouwaitlongenough.Notonlythat,butyoucandobetter thanisinitiallyobvious.Youcanlearnndigitsofthedecimalexpansionof ifyouwaitfor throwingawaytoomuchinformation:instead,notingthat on fromeachterminthesequence: ,butthisis ,wegetupperandlowerbounds

Letsgraphtheseupperandlowerboundsforvariousvaluesofn:

Follow

145 of 224

FollowTheMathLess Traveled 10/16/12 16:55


Get everynewpost deliveredto your Inbox.

The Math Less Traveled | Explorations in mathema...

http://mathlesstraveled.com/

Neato!Wecanviewthisinaslightlydifferentwaybygraphingtheerror (theamountunderorover )ofthe approximationsinsteadoftheapproximationsthemselves:

So,thatlooksprettycoolbutthebestapproximationsquicklygetsocloseto thatitshardtoseewhats reallygoingon.Sonowweregoingto(a)zoomwayoutthegraphwillgoton=50000insteadofjust200, and(b)usealog-logscalethatis,wellgraphthelogarithmofboththeerrorandn.

Follow

146 of 224

FollowTheMathLess Traveled 10/16/12 16:55


Get everynewpost deliveredto your Inbox.

The Math Less Traveled | Explorations in mathema...

http://mathlesstraveled.com/

Cool!Nowwecanseesomemoreinterestingstructureatthebottomofthegraph(inthepreviousgraph,the approximationsquicklygotsoclosetotheaxisthatitwasimpossibletotellwhatwasgoingon;thisiswhy plottingthelogarithmoftheerrorhelps).Wecanseethatmostoftheapproximationsareinthatbigmass slopinggentlydownandtotheright.Thisrepresentsabouthowwellwewoulddoatapproximatingpiifwejust lookedatthe thelementsofthesequence.Buttheinterestingthingistheapproximationswhicharenotpart ofthatbigmass.Atseveralplacesmostnotablyataroundn=5,n=100,andn=30000therearedownward spikes,representingapproximationswhichjumpoutasbeingwaybetterthanmostoftheotheronesatthat point.Thesearepreciselytheconvergentsof ,whichare(inaspecifictechnicalsense)thebestrational approximationsto .Thisiswhy,bylookingatalltheapproximationsandchoosingthebestones,wecando muchbetterthanjustlookingatthe thelementsofthesequence(imaginealinedrawnthroughthe bottommostpointsinthegraph;itsmuchsteeperthantheaverageslopeofthebigmassofapproximations aboveit).InafuturepostIhopetowriteabitmoreabouttheseso-calledconvergentsandwheretheycome from;Iveactuallywrittenabitaboutthembefore,butdidntpointitoutatthetime!

Posted in convergence, famous numbers, pattern, sequences | Tagged approximation, convergents, graphs, pi | 5 Comments

Post #100 102!


Posted on January 3, 2009

AfterjustovertwoandahalfyearsofwritingTheMathLessTraveled,Ivereached100posts!(Well,102 actually,Iseemtohavemissed100)ItsalittlehardtobelieveIveactuallywrittenthatmuch,andkeptthis littleprojectgoingforsolong.Thanksforreading!AndIknowIhaventwrittenanythinginawhilegradschool andsuchbutlookforsomemoreniftymathcomingsoon!


Posted in meta | 6 Comments

Sine of an inscribed angle

Follow

147 of 224

FollowTheMathLess Traveled 10/16/12 16:55


Get everynewpost deliveredto your Inbox.

The Math Less Traveled | Explorations in mathema...

http://mathlesstraveled.com/

Posted on December 6, 2008

Didyouknowthattheratiobetweenthesideofanytriangleandthesineoftheoppositeangleisequaltothe diameterofthetrianglescircumcircle?Ididnt!Ijustlearnedittodaywhenresearchingthelawofsines.Allthat timespentonthelawofsinesinhighschool,andnooneeverbotheredtotellmethatinanytriangle,notonly arealltheratiosbetweensidelengthsandsinesofoppositeanglesequaltoeachother ,theyarealsoequalto somethingelseinterestingnamely,thediameterofthecircumcircle! Amongotherthings,thismeansinparticularthatifyouinscribeanyangleinacirclewithdiameterone,the lengthofthechorditsubtendsisequaltothesineoftheangle:

Nifty,eh?Wanttoseeaproof?Hereitis:

Ratherthanexplainitindetail,Illjustletyoustareatitforawhile,andleaveacommentifyouhavequestions. =)Theonethingyouneedtorememberfromgeometrythatyoumightnotrememberisthatanangleinscribed inacircle(like intheabovepicture)subtendsanangletwiceaslarge.


Posted in geometry, proof | Tagged circumcircle, inscribed angle, law of sine, proof without words, sine | 12 Comments

Predicting Pi: solution


Posted on October 23, 2008

Nowforthesolutiontothequestioninmypreviouspost,whichaskedwhatyoucanlearnabout ,giventhe sequenceofintegers NickJohnsoncommented:


Follow

148 of 224

FollowTheMathLess Traveled 10/16/12 16:55


Get everynewpost deliveredto your Inbox.

The Math Less Traveled | Explorations in mathema...

http://mathlesstraveled.com/

Well,theobviousthingonecanlearngivenjust|(10^n)r|isthefirstndigitsofthedecimalexpansion ofr. Forexample,whenwearegiven justwaitforthe thitemofthelist. ,wecansaywithconfidencethat .Ifwewaitlongenough,

wecanlearnasmanydecimaldigitsof aswewant;inparticular,tolearnthefirstndecimaldigitsof ,wecan

Thisseemslikeanawfullylongtimetowait,though.Notonlythat,butifweonlylookatthefirst,tenth, hundredth,thousandth,elementsofthesequence,wewouldbeignoringalmostalltheinformationinthe sequence!Intuitively,itseemsthatweshouldbeabletodobetterbytakingmoreofthesequenceintoaccount. Andindeed,wecan. Thekeyistorealizethat

Doyouseewhy?Takingthefloorofsomethingnevermakesitbigger,so

(infact,itcanneverbeequal

since isirrational,butthatsnotimportantforourpurposes).Also,takingthefloorofsomethingreducesitby someamountlessthanone,soaddingonetothefloorgivesussomethingbiggerthantheoriginal;hence .Dividingthroughbyn,wefindthat

Thatis,ifthenthelementofthesequenceisk,thenweknowthat mustbebetweenk/nand(k+1)/n.Of course,thisworksforanynumberr,notjustfor . Letsseehowthisworks.Thefirstfewnumbersinthesequence weknowthat .Afterseeingthe6,weknowthat now(3.5)thanwedidbefore(4).Afterseeingthe9,weknow boundhasn'tchangedyet.Thatwon'thappenuntilwegetto are .Afterseeingthe3,

sowehaveabetterupperbound ,andsoon.Noticethatourlower ,whenwelearnthat .Soour

newlowerboundis3.125.Buttheupperboundatthisstep,26/8=3.25,isactuallyworsethantheupper boundthatwewouldhavefoundonthepreviousstep,namely22/7=3.142857Soingeneral,theupperand lowerboundsthatwefindineachstepmightnotbebetterthanallthepreviousones;wecanjustkeepthe greatestlowerboundandtheleastupperboundthatwe'veseensofar. So,howgoodarethesebounds?Iwrotealittleprogramtocomputethebestupperandlowerboundsfor variouspointsinthesequence.Here'satableshowingthebestlowerandupperboundsatvariouspointsinthe sequence,andthedecimaldigitstheygiveusconfidenceabout.


Term Best bounds so far

10 100

3.1 3.14

Follow

149 of 224

FollowTheMathLess Traveled 10/16/12 16:55


Get everynewpost deliveredto your Inbox.

The Math Less Traveled | Explorations in mathema...

http://mathlesstraveled.com/

1000 10000 100000 1000000

3.1415 3.141592 3.141592653 3.1415926535

Asyoucansee,thismethoddoesmuchbetterthanthemethodofjustlookingatpowersoften!Atn=one million,wealreadyknowtendecimaldigitsofpi;bylookingjustattheonemillionthelementofthesequence, wewouldonlyknowsixdigits.Itturnsoutthatingeneral,itisthecase(whichIwillnotprove)thatonaverage, wecangetapproximatelytwiceasmanydecimaldigitsbyfindingbestupperandlowerboundsthiswayinstead ofjustlookingatpowersoften. InafuturepostIhopetomakeagraphoftheseupperandlowerboundsandtalkalittlebitmoreaboutwhats goingonitturnsouttobeprettyinterestingstuff!


Posted in convergence, pattern, sequences, solutions | Tagged approximants, approximation, floor, pi, sequence | 5 Comments

Predicting Pi
Posted on October 1, 2008

InspiredbyarecentpostoveratFoxmaths!,heresaninterestingchallengequestionforyoutothinkabout: SupposeIgiveyouthesequenceofintegers ,thenextnumberwouldbe ,andsoon,where denotesthegreatestintegerless

thanorequaltoxinotherwords,itmeanstorounddown.Sothefirstnumberinthesequencewouldbe ,andsoon.Giventhissequence,whatcanyoulearnabout (assumingthatyoudidntalreadyknowanythingaboutit)? Amoregeneralquestion:giventhesequenceofintegers fork=1,2,3,whatcanyoulearnaboutr ?

Theanswerhasmanyinterestingconnectionstothetheoryofirrationalnumbersandcontinuedfractions.
Posted in challenges, pattern, sequences | Tagged approximation, floor, pi, sequence | 3 Comments

Decimal expansion zoo


Posted on September 23, 2008

Inacommentonapreviouspostaboutrationalnumbersanddecimalexpansions,SteveGilbergnoted: Ivebeenfascinatedathowanymultipleof1/7thatsnotanintegerrepeatsthesamedigitsindecimal expression,onlystartingatdifferentpointsinthesequence: 1/7=.142857 3/7=.428571 2/7=.285714 6/7=.857142


Follow

150 of 224

FollowTheMathLess Traveled 10/16/12 16:55


Get everynewpost deliveredto your Inbox.

The Math Less Traveled | Explorations in mathema...

http://mathlesstraveled.com/

4/7=.571428 5/7=.714285 Iverearrangedtheorderofthefractionstomakethepatternobvious. Thisisprettycoolindeed,andprobablywell-knowntomany.Buttheresnothingparticularlyspecialabout7, otherthanthefactthatitissmall.Inordertotalkaboutthesortsofpatternswefindintheexpansionsof rationalnumbersandwhyImgoingtostartbyhavingaspecialexhibit:theDecimalExpansionZoo!Havea lookaroundandseeifyounoticeanypatterns.Illfollowtheconventionofenclosingrepeatingportionsin [squarebrackets].So,forexample,0.23[48]means0.234848484848andsoon. Somequestionsforyoutothinkaboutwhileyouwanderaboutthezoo:whatpatternsdoyounotice?Which decimalexpansionsterminate,andwhicharerepeatingandhowdoesitrelatetothedenominator?Howare thedifferentcyclesforagivendenominatorrelatedtoeachother,andwhy?Howarethelengthsofthecyclesfor agivendenominatorrelatedtothedenominatoritself?Illanswerallthesequestions,andmore,inanupcoming post!

Follow

151 of 224

FollowTheMathLess Traveled 10/16/12 16:55


Get everynewpost deliveredto your Inbox.

The Math Less Traveled | Explorations in mathema...

http://mathlesstraveled.com/

1/2=.51/8=.125 3/8=.375 1/3=.[3]5/8=.625 2/3=.[6]7/8=.875 1/4=.251/9=.[1] 3/4=.752/9=.[2] 3/9=.[3] 1/5=.24/9=.[4] 2/5=.45/9=.[5] 3/5=.66/9=.[6] 4/5=.87/9=.[7] 8/9=.[8] 1/6=.1[6] 5/6=.8[3]1/10=.1 3/10=.3 1/7=.[142857]7/10=.7 3/7=.[428571]9/10=.9 2/7=.[285714] 6/7=.[857142]1/12=.08[3] 4/7=.[571428]5/12=.41[6] 5/7=.[714285]7/12=.58[3] 11/12=.91[6] 1/11=.[09] 2/11=.[18]1/13=.[076923] 3/11=.[27]10/13=.[769230] 4/11=.[36]9/13=.[692307] 5/11=.[45]12/13=.[923076] 6/11=.[54]3/13=.[230769] 7/11=.[63]4/13=.[307692] 8/11=.[72]2/13=.[153846] 9/11=.[81]7/13=.[538461] 10/11=.[90]5/13=.[384615] 11/13=.[846153] 1/17=.[0588235294117647]6/13=.[461538] 10/17=.[5882352941176470]8/13=.[615384] 15/17=.[8823529411764705] 14/17=.[8235294117647058]1/14=.0[714285] 4/17=.[2352941176470588]3/14=.2[142857] 6/17=.[3529411764705882]5/14=.3[571428] 9/17=.[5294117647058823]9/14=.6[428571] 5/17=.[2941176470588235]11/14=.7[857142] 16/17=.[9411764705882352]13/14=.9[285714] 7/17=.[4117647058823529] 2/17=.[1176470588235294]1/15=.0[6] 3/17=.[1764705882352941]2/15=.1[3] 13/17=.[7647058823529411]4/15=.2[6] 11/17=.[6470588235294117]7/15=.4[6] 8/17=.[4705882352941176]8/15=.5[3] 12/17=.[7058823529411764]11/15=.7[3] 13/15=.8[6] 1/18=.0[5]14/15=.9[3] 5/18=.2[7] 7/18=.3[8]1/16=.0625 11/18=.6[1]3/16=.1875 13/18=.7[2]5/16=.3125 17/18=.9[4]7/16=.4375 9/16=.5625 1/19=.[052631578947368421]11/16=.6875 10/19=.[526315789473684210]13/16=.8125 5/19=.[263157894736842105]15/16=.9375 12/19=.[631578947368421052] 6/19=.[315789473684210526]1/31=.[032258064516129] 3/19=.[157894736842105263]10/31=.[322580645161290] 11/19=.[578947368421052631]7/31=.[225806451612903] 15/19=.[789473684210526315]8/31=.[258064516129032] 17/19=.[894736842105263157]18/31=.[580645161290322] 18/19=.[947368421052631578]25/31=.[806451612903225]

Follow

152 of 224

FollowTheMathLess Traveled 10/16/12 16:55


Get everynewpost deliveredto your Inbox.

The Math Less Traveled | Explorations in mathema...

http://mathlesstraveled.com/

Posted in number theory, pattern | Tagged decimal, expansion, zoo | 14 Comments

Two Very Large primes


Posted on September 17, 2008

Aspromised,IcannowrevealtheidentityofthetwonewlydiscoveredMersenneprimes.Thesmallerofthe two,discoveredonSeptember6byHans-MichaelElvenichinLangenfeld,Germany,is

an11,185,272-digitnumberwhichyoucandownloadhere.Thelargeronewasactuallydiscoveredfirst,on August23,byEdsonSmith,whohadinstalledtheprime-checkingsoftwareoncomputersatUCLA.Itisnowthe largestknownprime,weighinginatawhopping12,978,189digits,andisequalto

Youcandownloadithere.Ofcourse,asIsuspected,thesearebothlongerthantenmilliondigits,whichmeans thatthefirstonetobediscoverediseligiblefora$100,000prize! Theseareridiculouslyhugenumbers.Foralittleperspective,thetotalnumberofatomsintheuniverseis estimatedatsomewherearound ,anumberwithonlyeighty-onedigits.Nowgobackandreadagainhow manydigitsthesenewlydiscoveredprimenumbershave.


Posted in computation, famous numbers, links, primes | Tagged EFF, GIMPS, Mersenne, prime | 3 Comments

New Mersenne primes!


Posted on September 15, 2008

TheGreatInternetMersennePrimeSearchjustannouncednotone,buttwonewMersenneprimes!Youmight alsorecallthelasttimetheyannouncedanewprime,inSeptember2006,sothesenewprimeswerefound almostexactlytwoyearsafterthepreviousone.Theyhaventactuallyannouncedwhattheprimesareyet,but bothofthemarealmostsuretobelongerthantenmilliondigits,longenoughtoclaimthe$100,000prize offeredbytheElectronicFrontierFoundation!Iftheybothendupbeinglongerthantenmilliondigits,Iguessit suckstobetheguywhosecomputerdiscoveredthesecondone(justtwoweeksafterthefirst).Youdontgetany moneyfordiscoveringthesecondofanything. Anyway,assoonastheyannouncetheactualnumbers,Illbesuretoletyouknow!


Posted in computation, famous numbers, links, primes | Tagged announcement, discovery, GIMPS, Mersenne, prime | 1 Comment

Teaching precalculus in 2008-2009


Posted on September 14, 2008

ThisyearIwillbeteachingprecalculusviacorrespondencetotwohomeschoolstudents.Dontaskhowthis happened,itsalongstory,butImexcited!Itwillbefunformetogainmoreexperienceteachingandwriting, andtoexperimentabitwiththecurriculuminwaysthatIthinkwillmakeitmorecurrent.Ivetaughtprecalculus


Follow

153 of 224

FollowTheMathLess Traveled 10/16/12 16:55


Get everynewpost deliveredto your Inbox.

The Math Less Traveled | Explorations in mathema...

http://mathlesstraveled.com/

oncebefore,atapublichighschoolinWashington,DC,andeversincethatexperienceIvehadsomestrong opinionsaboutwaysthatthecurriculumshouldbedifferent.PerhapsIllwritemoreonthatrantlater. Atanyrate,thepointisthatIllbemakinganumberofmymaterialsavailableonlineunderaCreativeCommons Attribution-Noncommerciallicense(thesamelicensethatappliestoallthecontentonthisblog).Thereare alreadyacoupleassignmentsposted,alongwithabasicLaTeXtutorialandasyllabus.Feelfreetouseitfor learning,teaching,orwhatever.Iimaginethattheweeklyassignmentscouldbeprofitablyusedforenrichment, extracredit,orevenstraight-upassignmentsaspartofotherclassesatvariouslevels.ImalsomakingtheLaTeX sourceavailablesoyoucanevenmakeyourownmodifications,orjustuseindividualexercises,paragraphs,or whatever,aslongasyoucitemeasthesource.Iwontbemakingsolutionsavailable,forhopefullyobvious reasons,butifthereisenoughdemandandIhavetimeImightbeabletowriteupsomesolutionsandhave themavailablefordistributiontoteachersbyrequest,soletmeknowifyoudbeinterested. IllprobablyoccasionallywritesomethingherewhenIvepostedsomenewmaterials,butifyoureparticularly interestedtoknoweachtimeIvepostedsomethingnew,letmeknowandIcouldperhapssetupsomesortof automatednotificationsystem.
Posted in links, meta, teaching | Tagged correspondence course, creative commons, latex, precalculus, teaching | 6 Comments

Rational numbers and decimal expansions


Posted on September 7, 2008

Asyoumayrememberfromschool,rationalnumbershaveaterminatingoreventuallyrepeating(periodic ) decimalexpansion,whereasirrationalnumbersdont.So,forexample,0.123123123123,with123repeating forever,isrational(infact,itisequalto41/333),whereassomethinglike0.123456789101112131415,which willneverrepeat,isirrational. Butdoyouknowwhythisistrue?(Despitewhatyourteachersmayhavetoldyou,themostimportantquestion inmathematicsisnothow,itiswhy!)TodayIwillshowwhyeveryrationalnumberhasaterminatingor eventuallyrepeatingdecimalexpansion,andinafuturepostIwillshowwhyeveryrepeatingorterminating decimalexpansionrepresentsarationalnumber.Fromthesetwopiecesofinformation,ofcourse,wecanalso deducethateverydecimalexpansionwhichdoesntterminateorrepeatmustrepresentanirrationalnumber,and viceversa. Continuereading

Posted in infinity, iteration, number theory, pattern | Tagged decimal, periodic, rational | 9 Comments

Weird black bars in LaTeX images


Posted on September 2, 2008

RecentlyIvebeenseeingstrangeblackbarstotherightofalltheimagesgeneratedfromLaTeXexpressionson thisblog.ButImstartingtosuspectthatitssomesortofbuginFirefox,becausewhenIdownloadtheimages andviewthemwithsomeotherprogram,theylookfine.


Follow

154 of 224

FollowTheMathLess Traveled 10/16/12 16:55


Get everynewpost deliveredto your Inbox.

The Math Less Traveled | Explorations in mathema...

http://mathlesstraveled.com/

Doyouseeablackbartotherightoftheaboveformula?Pleaseleaveacommentifyoudo,ideallystatingwhat operatingsystemandbrowseryouuse,alongwiththeirversionnumbers.(Ifyoudont,Iguessthatwouldbe usefultoknowtoo.)Thanksmuch!Ihopetobeabletotrackdowntheproblemwithyourhelp.


Posted in meta | Tagged bug, firefox, GIF, images, latex | 22 Comments

Mandelbrot Maps
Posted on September 1, 2008

Run,dontwalk,toplaywiththeMandelbrotMapsapplet,whichletsyouplayaroundwiththeMandelbrotset whileseeingtheassociatedJuliasetsupdatedinrealtime.ThisfantasticappletwascreatedbyIainParris,anMSc studentofPhilipWadlerattheUniversityofEdinburgh.TheresevenashortscreencastofIainexplainingthe mathbehindtheMandelbrotandJuliafractalsandshowingsomeofthecoolthingsyoucandowiththeapplet.

Posted in fractals, links | Tagged applet, fractal, interactive, julia, mandelbrot, real-time

Rational and irrational numbers


Posted on August 31, 2008

Mostreadersofthisblogprobablyknowwhatarationalnumberis:itsanumberthatcanberepresentedasa ratio(hencerational)oftwointegers.Inotherwords,afraction.Examplesare3/4,99/2,and-20837/231,and even4(itcanberepresentedastheratio4/1).Theonlyrestrictionisthatyoucanthavezerointhe denominator(thebottomoftheratio)sincedivisionbyzero,ofcourse,isundefined. Youveprobablyalsoheardofirrationalnumbers,which,ofcourse,arenumbersthatarentrational.Butareyou surethatirrationalnumbersexist?Why?Justbecausesometeachertoldyou,oryouveheardofthembefore? TheancientGreeksthoughtirrationalnumbersdidntexistthateveryquantitycouldbedescribedbya ratioandtheywereinforarudesurprisewhentheythemselvesprovedthistobefalse!Iwontgointothat


Follow

155 of 224

FollowTheMathLess Traveled 10/16/12 16:55


Get everynewpost deliveredto your Inbox.

The Math Less Traveled | Explorations in mathema...

http://mathlesstraveled.com/

historyinmoredetailhere;itsactuallyquiteconfusedandunclearastowhodidwhatwhenandsoon.Maybe Illtrywritingaboutthatinafuturepost.Fornow,Illjustoffersomeincontrovertibleproofthatatleastone numberexistswhichisntrational.Probablymanyofyouhaveseenitalready,butitsagoodplacetostart,asI intendtowritemorepostsonthistopicinthenearfuture. Ready?OK,Imthinkingofanumberwhichwellcalls.Inparticular,sisthelengthofonesideofasquare whoseareais2.ThisishowtheGreekswouldhavethoughtofit,andcertainlynoonecanarguethatitisnta number!Thereobviouslymustbeasquarewithanareaof2,andthesidesofthatsquareobviouslymusthavea length,andthatlengthisanumber.(AstutereaderswillnotethatImplayingratherfastandloosewiththeterm number.Whichistrue.Sosueme.)Inmoremodernterms,wewouldsaythatsisthepositivesolutiontothe equation ,orjust . .Remember,wewantto

Now,supposethatsisarationalnumber,sothatscanberepresentedbyaratio

showthatsisntrational,soIllshowthatmakingthisassumptionleadstoanabsurdity,fromwhichwecan concludethatitwasanincorrectassumptioninthefirstplace.Thisiscalledaproofbycontradictionassumethe logicaloppositeofwhatyouwanttoprove,andshowthatthisleadstoanabsurdity,like beingbothtrueandnottrueatthesametime. Wellassumethatp/qisinlowestterms,thatis,thatpandqhavenocommonfactors.Itsoktoassumethis, sinceanyrationalnumbercanalwaysbeturnedintoanequivalentoneinlowestterms. Weknowthatsquaringsyields2,sosubstitutingp/qforsgivesustheequation that forsomeotherintegerr .Substituting,wefindthat ,andtherefore .Rearranging,wefind .Butthismeansthat ,orsomething

,so iseven.Butif iseven,soisp(youshouldconvinceyourselfofthis),sowecanwrite

iseven,andthereforesoisqbutthisisabsurd,becauseweassumedthatpandqhavenocommonfactor,so theycantbothbeeven!Therefore,ouroriginalassumptionthatsisrationalmustbefalse. InupcomingpostsIplantoexplorethisandotherrelatedtopics,possiblyincluding:theconnectionbetween rationalnumbersanddecimalexpansions,theperiodofrepeatingdecimals,repunits,setcardinalityandorders ofinfinity,somefamouslyirrationalnumbers(like ande),andmaybeevensomecontinuedfractions.Thereare manyfascinatingtopicsconnectedinonewayoranothertothisdistinctionbetweenrationalandirrational numbers,soitshouldbeinterestingIhopeyoullcomealongfortheride! Asafurtherchallenge,canyouextendtheaboveprooftoshowthatthesquarerootofanyprimenumberis irrational?Whataboutthesquarerootofanypositiveintegerwhichisnotaperfectsquare?(Thissecondoneis abittrickier,becauseifnisnotprime,youcannotassumethatifndivides thenitmustdividepaswellfor example,12divides ,butitdoesnotdivide6.)

Posted in algebra, challenges, number theory, proof | Tagged rational irrational proof | 11 Comments

Deriving simple divisibility rules


Posted on July 17, 2008

Ilearnedsomethingnewtoday!
Follow

156 of 224

FollowTheMathLess Traveled 10/16/12 16:55


Get everynewpost deliveredto your Inbox.

The Math Less Traveled | Explorations in mathema...

http://mathlesstraveled.com/

Manypeopleprobablyknowofthesesimpledivisibilityrules: Anintegerisdivisibleby10ifitendsinazero. Anintegerisdivisiblebyfiveifitendsinzeroorfive. Anintegerisdivisiblebytwoifitsfinaldigitis. Anintegerisdivisiblebythreeornineifthesumofitsdigitsis. Anintegerisdivisiblebysixifitisdivisiblebybothtwoandthree. Slightlylesswell-knownaretheserules: Anintegerisdivisiblebyfourifthetwo-digitnumberformedfromitslasttwodigitsis. Anintegerisdivisiblebyeightifthethree-digitnumberformedfromitslastthreedigitsis. Thatlastoneissimplerthanitsoundsyoudonthavetomemorizeallthethree-digitnumbersthataredivisible byeight(thatwouldberathersilly!),sinceitsprettyeasytotrydividingathree-digitnumberbytwoinyour headthreetimes.Ifyoucandivideitbytwothreetimes,itsdivisiblebyeight;ifyouhitanoddnumberbefore thethirddivision,itsnot.Forexample,29873458636->636->318->159,oops,notdivisiblebyeight. Youwillnoticethatthereisanicesimpledivisibilityrulelistedaboveforeverynumberuptotenexcept seven.TheonlydivisibilityruleIhadeverseenforsevenwasrathercomplicatedandhardtoperforminones head,makingitratheruseless.Atleast,Ithinkitwas,becauseIdontrememberit. Butjusttoday,IreadanexcellentpostoveratFoxmaths!explainingcompletewithderivationanice divisibilityruleforseven.Theruleisthis:takethenumber,andsubtracttwicethefinaldigitfromthenumber formedbytheremainderofthedigits.Theresultwillbedivisiblebysevenifandonlyiftheoriginalnumber was.Ofcourse,thisprocesscanberepeateduntilyoureleftwithsomethingsimple.Forexample,imagine startingwith3641:subtracttwice1from364,giving3642=362.Nowsubtracttwicetwofrom36,giving32, whichisnotdivisiblebyseven.Therefore,3641isnt,either.See,niceandsimple! ThereallyneatthingisthatthisisntpeculiartosevenFoxgoesontoshowhowthesametechniquecanbe used,inprinciple,toderiveasimilardivisibilitytestforanyprimenumber!Goreadit.
Posted in computation, links, number theory | 14 Comments

Change probabilities
Posted on June 9, 2008

Hello,dearreader!Ifyouareanewreader,welcome!(Iknowthereisatleastoneofyouthankstocommenter sg211forpromptingmetopostagain.=) ImustapologizefordroppingtheballonwritingupthesolutiontopartIIofthepowersoftwochallengefrom acouplemonthsago,andinabroadersensewrappingupmyongoingseriesonRecountingtheRationals.One readerevenwrotemeane-mailwithafriendlyremindertowriteupthesolution!Well,Ihaventforgottenabout it,andIpromisetogettoitsoon. Inthemeantime,heresaninterestingproblemforyou.Ihavealittleclearplasticpiano-shapedthingthatsorts Follow

157 of 224

FollowTheMathLess Traveled 10/16/12 16:55


Get everynewpost deliveredto your Inbox.

The Math Less Traveled | Explorations in mathema...

http://mathlesstraveled.com/

UScoinsyouputcoinsinthetop,anditsortsthemintostacksbytype.Iwenttoputsomecoinsintheother dayandnoticedthattherewereatonofquartersinit,butalmostnonickels,whichmademewonderwhether therewasareasonforthat,orwhetheritwasjustsomesortoffluke.So,thequestionforyouis:wasitafluke? Orwouldyouexpectmetohavefewernickelsthanquartersinmycoin-sortingpiano? [Foranynon-USreaderswhodon'tknow,thevaluesofUScoinsare25(quarters),10(dimes),5(nickels),and1 cents(pennies).Actually,therearealsocoinsworth50(half-dollar)and100cents(dollar),butthosearefarless common.] Tobealittlemoreprecise,youcanmakethefollowingassumptions,allofwhichare,ofcourse,completelyfalse intherealworld,buthey,thisisamathproblem.=) 1. Istartedwithacompletelyemptycoin-sortingpiano. 2. Ineveruseanycoinsinmakingpurchases,butalwayspaywithbillsonly. 3. ThecentsportionofthepricesofthingsthatIbuyarerandomanduniformlydistributedbetween0and99; inotherwords,foranyparticularnumbernbetween0and99,thereisa1/100chancethatthepriceof somethingIbuywillbesomenumberofdollarsplusexactlyncents. 4. Cashiersalwaysgivemechangeinquarters,dimes,nickels,andpennies(nodollarsorhalf-dollarcoins), usingthefewestnumberofcoinspossible.(ForexampleifIamowed16centsinchange,Iwouldgetone dime,onenickel,andonepenny,notthreenickelsandapennyoranythinglikethat.) Asusual,postcomments,questions,and/orsolutionsinthecomments(dontpeekifyouhaventtriedsolvingit first!=)
Posted in challenges, counting, probability | 17 Comments

Monday Math Madness!


Posted on April 28, 2008

Quan,Daniel,andSol,fromblinkdaggerandWildAboutMath!,haveteameduptocreateafunmathcontest, MondayMathMadness.Everyotherweek,theypostamathproblem;afteroneweek,awinnerisrandomly selectedfromamongthosesendinginacorrectanswer,andreceivesarealhonest-to-goodnessprize! Thecurrentcontestwasjustpostedtoday.Gogiveitatry,andsendinyoursolutionmaybeyoullwin!Even ifyoudont,ofcourse,itsstillafunproblem.Hereitis: Asidefrom1and9,arethereanyperfectsquareswhosedigitsareallodd?Justifyyouranswer. Dontpostanyanswersordiscussionhere!Ifyouthinkyouvesolvedit,sendyouranswerto mondaymathmadnessatgmaildotcom.


Posted in challenges, links, number theory | 2 Comments

Challenge #12 solution, part III


Posted on April 24, 2008

Follow

158 of 224

FollowTheMathLess Traveled 10/16/12 16:55


Get everynewpost deliveredto your Inbox.

The Math Less Traveled | Explorations in mathema...

http://mathlesstraveled.com/

Andnowforthesolutiontoproblem#3fromChallenge#12,whichasked:howmanywaysaretheretowritea positiveintegernasasumofpowersoftwo,withnorestrictionsonhowmanypowersoftwomaybeused? Continuereading

Posted in counting, pattern, recursion, sequences, solutions | 1 Comment

Challenge #12 solution, part I


Posted on April 21, 2008

IllbeginbyprovidingananswertothefirstofthethreequestionsIposedinapreviouspost. Continuereading

Posted in counting, pattern, proof, solutions | Tagged binary | 8 Comments

Challenge #12: sums of powers of two


Posted on April 18, 2008

Afewinterestingproblemsforyoutothinkabout: 1. Givenapositiveintegern,inhowmanywayscannbewrittenasasumofpowersoftwo,wheneachpower isallowedtooccuratmostonce?Forexample,11canbewrittenas allowedtowrite and dontcountasdifferentsums. denotesthenumberofdifferentwaystowritenasasumofpowersoftwo .Butwerenot ,since occurstwice.Theorderofthetermsinthesumsdontmatter,so

2. Nowwhatifweallowatmosttwocopiesofeachpoweroftwo?Bothoftheexamplesfor11shownabove wouldbeallowednow.Say withatmosttwocopiesofeachpowerallowed.Whatcanyousayaboutthefunctionh? 3. Whatifweallowanunlimitednumberofeachpoweroftwo?Nowhowmanywaysaretheretowritenas suchasum? Postyoursolutionsinthecomments(partialsolutionsarewelcomeandencouraged),butonlyifyoudidnt knowtheanswerassoonasyoureadthequestion.=)


Posted in challenges, counting, pattern | Tagged counting, partitions, powers of two, sums | 18 Comments

Hello again!
Posted on April 15, 2008

Helloagain,dearreader! IhaventwrittenanythinghereinquiteawhilenowsinceFebruary22,itwouldseem!Partly,thatsbecause Ivebeenbusyvisitinggraduateschoolsanddecidingwheretogo.AndIcannowhappilyannouncethatmywife andIwillbemovingtoPhiladelphiathissummer,whereIwillbeginacomputersciencePhDattheUniversityof Pennsylvaniainthefall.Imsuperexcited!


Follow

159 of 224

FollowTheMathLess Traveled 10/16/12 16:55


Get everynewpost deliveredto your Inbox.

The Math Less Traveled | Explorations in mathema...

http://mathlesstraveled.com/

So,whatdoesthatmeanforthisblog?Nottoworryasofrightnow,Iplantocontinuerightonwiththe sametastyaccessiblemathyouvecometo(hopefully)loveand(rightfully)expect!Planscanalwayschange,of course,butrightnowIintendandhopetocontinuethisblogwhileingradschool.WillIkeepwritingthisblog untilIdie?Probablynot!Illstopatsomepoint,butthatpointisnthereyet.=) Fornow,alink:ifyouhaventyetreadAMathematiciansLamentbyPaulLockhartatellingandinsightful lamentoverthecurrentstateofmathematicseducationintheUS,whichtranscendstheusualpettydebatesI highlyrecommendit,whetheryoureateacher,astudent,or(ideally)both!Itsbothdepressingandinspiringat thesametime.HeputseloquentandforcefulwordsonthingsIvethoughtonlydimly.


Posted in links, meta, people | Tagged education, grad school, Paul Lockhart, U Penn | 10 Comments

Carnival of Mathematics #1000!


Posted on February 22, 2008

#1000,youask?Surelythepreviousonewas#26? Well,yes,butthats1000inbase3,ofcourse.Duh.=) Anyway,yes,themostrecentCarnivalcanbefoundoveratJD2718.Someofmyfavoriteposts: Areallyneat,elegantcorrespondencebetweenrationalnumbersandPythagoreantriples MathonDisplaybeautifulmathematicalpictures! Afungeometrypuzzlethatseemsalotharderthanitreallyis=) Anicearticle(withmoreprettypictures)onvisualmethodsinmathematics Lotsofothergreatstufftheretoo,gotakealook!


Posted in links, meta | 2 Comments

Recounting the Rationals, part IVb: the Euclidean Algorithm


Posted on February 11, 2008

Supposewehavetwointegers,andwedliketofindtheirgreatestcommondivisor(GCD).Recallthatthe greatestcommondivisoroftwointegersmandnisexactlythat:thegreatestintegerwhichisadivisorofbothm andn.Theobviouswaytodothiswhichisprobablythewayyoulearnedinelementaryschool,whenreducing fractionsistofactoreachintegerintoprimes,andlookforoverlap.Forexample,letssaywewanttofindthe GCDof450and525.Webeginbyfactoring:

Nowwepulloutasmanyprimefactorsaswecanwhicharecontainedinthefactorizationsofbothnumbers.We cantuse2atall,sinceitisonlycontainedinthefactorizationof450;wecanuseafactorof3,butonlyone;and wecanusetwofactorsof5.Puttingthesetogether,thegreatestcommondivisorof450and525is .

Thismethodisintuitive,andworkswellforrelativelysmallnumbers,but(theresalwaysabut,isntthere?)it
Follow

160 of 224

FollowTheMathLess Traveled 10/16/12 16:55


Get everynewpost deliveredto your Inbox.

The Math Less Traveled | Explorations in mathema...

http://mathlesstraveled.com/

failshorriblyforlargernumbers.Forexample,whatifyouwantedtofindtheGCDof2257394839and 45466644967?(Goahead,tryit!=)Theproblemisthatfactoringishard,evenforcomputerswherebyhard Imeanhardtodoquickly.Itseasytowritecomputerprogramstodofactoring,itsjustthatnooneknows howtowriteaprogramwhichfactorslargenumbersquickly.(BylargeinthiscontextImeannumberswith hundredsofdigits;thetwonumbersIgaveasanexampleabovecouldbefactoredveryquicklybyacomputer, butIbetYOUcantfactorthemquickly,sothepointisthesame.) Well,asyouprobablyguessed,theresabetteralgorithmforcalculatingtheGCDoftwonumbers,sincetheGCD canactuallybefoundwithoutanyreferencetofactorizations.ThisbetteralgorithmiscalledtheEuclidean Algorithm,inhonorofEuclid,thefirstmathematiciantowriteitdown(around300BC,makingitoneofthe oldestalgorithmsknown!).ItsbasedonthepropertyIshowed(andproved)inmylastpost,namely,that . (Technically,inmylastpostIshowedthat versa.)Hereshowitworks: 1. Giventwointegersmandn,iftheyarethesame,thenclearlytheyareequaltotheirgreatestcommondivisor. 2. Otherwise,reducethelargerofthetwonumbersbysubtractingthesmallernumberfromit. 3. Lather,rinse,repeat. Thatsit.Wereyouexpectingsomethingmorecomplicated?Wealreadyknowthatdoingthesubtractionstep doesntchangetheGCD,andsincethenumbersarealwaysgettingsmaller,thealgorithmmusteventuallystop. Genius!ThatEuclidwasonesmartdude.Anyway,letstryitonasimpleexample:findingtheGCDof56and20. 1. 56isgreaterthan20,sosubtract20from56,leaving36and20. 2. 36isstillgreaterthan20,sosubtract20again,leaving16and20. 3. Subtract16from20,leaving4and16. 4. Subtract4from16threetimes,eventuallyleaving4and4. 5. HencetheGCDof56and20is4!(Thisiseasytoverifybyfactoring.) Neat!ButwhathasallthisgottodowiththeCalkin-Wilftree?Well,ifyouhaventalreadynoticed,the EuclideanalgorithmisexactlythesameasthemethodforfindingyourwayuptheCalkin-Wilftree!Inparticular, sincealltherationalsintheCalkin-Wilftreeareinlowestterms,findingyourwayuptheCalkin-Wilftreefrom m/nexactlycorrespondstousingtheEuclideanAlgorithmonmandninordertoshowthattheirGCDreallyis1. Andingeneral,if ,runningtheEuclideanAlgorithmonmandnislikefindingyourwayupa Calkin-Wilftreeinwhichallthenumbershavebeenmultipliedbyd(sothatitstartsatd/dinsteadof1/1).Inan importantsense,wecansaythattheCalkin-WilftreeistheEuclideanAlgorithm,intreeform! Inclosing,IshouldnotethatinpracticetheresaslightimprovementwecanmakeuponthebasicEuclidean Algorithm.Toseewhatitis,considerrunningtheEuclideanAlgorithmon20451and2.Well,20451isbigger than2,sosubtract2from20451,leaving20449and2.Subtract2again,leaving20447and2.20447isstill biggerthan2,sosubtractthisisthepointwherewestartgettingverybored.Everyonecanseeperfectlywell
Follow

,butsinceanythingthatdividesxalsodivides

-x,wecanfliparoundthem-nifwewant:anythingthatdividesm-nwillalsodivide-(m-n)=n-m,andvice

161 of 224

FollowTheMathLess Traveled 10/16/12 16:55


Get everynewpost deliveredto your Inbox.

The Math Less Traveled | Explorations in mathema...

http://mathlesstraveled.com/

thatsince20451isodd,aftersubtracting2enoughtimes,wewilleventuallybeleftwith1.Sothissuggestsour practicalimprovement:ateachstep,insteadofreplacingthelargernumberbytheremainderwhensubtracting thesmallernumber,replaceitbytheremainderwhendividingitbythesmallernumbersincedivisioncanbe viewedasrepeatedsubtraction.Now,usingthisimprovedmethod,canyoufindtheGCDof2257394839and 45466644967?


Posted in computation, iteration, number theory | 15 Comments

26th Carnival of Mathematics posted


Posted on February 8, 2008

The26thCarnivalofMathematicstheone-yearanniversaryedition!ispostedoverat360,soheadoverand giveitalook!OfparticularinteresttoreadersofTMLTareMusicFromMathematicalConstantsfromMikeat WalkingRandomly(piasmusic?sweet!),andThesecretofEgyptianFractionsfromDeniseatLetsplaymath!. Ofcourse,theresalsoapostbyyourstruly!Butthereslotsforeveryone,sogocheckitout.


Posted in famous numbers, links, meta, number theory

Recounting the Rationals, part IV


Posted on February 6, 2008

ContinuingaseriesabouttheCalkin-Wilftree(seethoselinksforsomebackground),todayIdliketoshowwhy alltherationalsinthetreemustbeinlowestterms.Letsstartoffwithalittlenumbertheory! Whatdowemeanwhenwesaythatafractionisinlowestterms?Wesimplymeanthatthenumeratorand denominatorhavenocommonfactorsotherthan1.So isinlowestterms,but isnt,sincethenumerator anddenominatorarebothdivisibleby7.Youwillsometimesalsohearthetermrelativelyprimetodescribetwo numberswhichhavenocommonfactors.Itdoesntreallyhaveanythingtodowithbeingprime;forexample,6 and49arerelativelyprime(andhence isinlowestterms),eventhoughneither6nor49isprime.

Amoreformalwaytotalkaboutallofthisisintermsofthegreatestcommondivisor,orGCD.TheGCDoftwo numbersmandn,oftenwritten thatmandnarerelativelyprimewhen ,isthelargestnumberwhichevenlydividesbothmandn.Forexample, . ,since6isthelargestnumberwhichevenlydividesboth12and30.So,inparticular,wecansay

YoushouldbeabletoverifyafewsimplepropertiesoftheGCDfunctionyourself.Forexample, ,and ourpurposes,isthat mandn,sowecanwrite and .Anotherslightlylessobviousproperty,butmoreimportantfor .Whyisthis?Well,suppose .Therefore .Thatmeansddividesboth ,soddividesm+ntoo!So,disa ,soitwouldbeacommondivisorofmandnbutwe .Theargumenttoshowthat isexactlythe

commondivisorofmandm+n.Notealsothatifanythingbiggerthandwasacommondivisorofmandm+n, thenthislargerdivisorwouldalsodivide ofmandm+n,and same.


Follow

alreadyknowthatdisthegreatestcommondivisorofmandn.Therefore,dreallyisthegreatestcommondivisor

162 of 224

FollowTheMathLess Traveled 10/16/12 16:55


Get everynewpost deliveredto your Inbox.

The Math Less Traveled | Explorations in mathema...

http://mathlesstraveled.com/

ApplyingthistotheCalkin-Wilftreeisapieceofcake.Wewanttoshowthatforevery itisthecasethat forsomenode .Well,itscertainlytruefortherootnode,1/1: ,itmustalsobetrueforthechildnodes,sinceweknowthat

intheCalkin-Wilftree, .

.Andsupposingitistrue

Andweredone!Thisisaniceexampleofaproofbyinduction:allwehavetodoisprovethatsomethingistrue forsomestartingvalue,andthatwheneveritistrueforonethingitmustbetrueforthenext,andvoil!We haveproveditforaninfiniteseriesofcases. So,wenowknowthattheCalkin-Wilftreecontainseverypositiverational,inlowestterms,exactlyonce!Next time,IlltalkaboutawaytocomputetheGCDoftwonumbers,calledtheEuclideanAlgorithmwhichis probablyalotdifferentthanthewayyoulearnedtocomputeGCDsinelementaryschoolanditscool connectiontotheCalkin-Wilftree.


Posted in induction, number theory, pattern, proof | 11 Comments

Recounting the Rationals, part III


Posted on January 24, 2008

First,aquickrecap:continuinganexpositionofthepaperRecountingtheRationals,wereinvestigatingthetree offractionsshownbelow(knownastheCalkin-Wilftree),whichisconstructedbyplacing1/1attherootnode, andgivingeachnode thetwochildren (ontheleft)and (ontheright).Herearethefirstfour levelsofthetree(ofcourse,ithasinfinitelymanylevels):

The Calkin-Wilf Tree

InmylastpostIclaimedthatthistreehasanumberofremarkableproperties;inthisandsubsequentpostsI plantoshowexactlywhythesepropertiesaretrue,sinceforthemostparttheyhaverelativelysimple explanationsalthoughthatinnowaymakesthemanylessremarkable! TodayletsbeginbytacklingthefirstpropertyIclaimed,namely,thateverysinglepositiverationalnumber occurssomewhereinthistree.Howcanweprovethis? First,noticethatgivenanyrationalinthetree,thereisanyeasywaytotellwhetheritistheleftorrightchildof itsparent.(Canyoufigureitoutbeforereadingon?) Sinceleftchildrenarealwaysoftheform numerator( ,theywillalwayshaveadenominatorgreaterthantheir and ,todecide
Follow

,sincejispositive);andviceversaforrightchildren,whichareoftheform

consequentlyhaveanumeratorgreaterthantheirdenominator.So,givensomefractioninthetree

163 of 224

FollowTheMathLess Traveled 10/16/12 16:55


Get everynewpost deliveredto your Inbox.

The Math Less Traveled | Explorations in mathema...

http://mathlesstraveled.com/

whetheritisaleftorrightchild,justcomparemandn.Ifnisgreater,itisaleftchild,anditsparentis ;otherwise,itistherightchildof .

Moving up the Calkin-Wilf tree

Forexample,ifwestartwith5/3,weseethat5>3,so5/3mustbetherightchildof(5-3)/3=2/3;2/3,in turn,hasagreaterdenominator,soitistheleftchildof2/(3-2)=2/1,and2/1istherightchildof1/1.

Moving up the Calkin-Wilf tree starting from 5/3

Infact,itiseasytoseethatIvejustdescribedasimpleprocedureforfindingyourwayupthetree,startingfrom anypositiverationalinlowestterms.Ateachstep,justtakethenumeratoranddenominator,andsubtractthe smallerfromthelarger.(Theycanneverbeequaluntilwegetto1/1,sincewestartoutwithsomethingin lowestterms.)Buthowdoweknowthatwewilleventuallyreach1/1thisway?Well,boththenumeratorand thedenominatorarealwayspositiveduringthisprocess(theystartoutpositive,andyoucantgetsomething negativebysubtractingsomethingsmallerfromsomethinglarger),andoneofthemgetssmallerateverystep. Thinkaboutthisforaminute.Theymusteventuallyreach1/1theresnowhereelseforthemtogo! Well,ifwecanfindapathupto1/1fromanyreducedpositiverational,everysuchrationalmustbeinthetree inthefirstplace!Youmayalsonotethatsincewehavenochoiceateachstep,thereisexactlyonepathfromany rationaluptotherootofthetreewhichmeansthateachreducedrationaloccursinonlyoneplaceinthetree; therearenoduplicates. Now,technically,onethingwehaventshownisthattherearentanyunreducedrationalswhichsneakilysneakin alongwiththereducedones;butthatsnottoohardtoshow,andIlltalkaboutthatnexttime. Thereareseveralotherinterestingthingsgoingonhereforexample,thepathfromanyreducedrationalupto therootofthetreecorrespondsdirectlytousingtheEuclideanAlgorithmtoprovethattherationalis,infact, reduced!AndifyouinterprettheleftsandrightsinthepathaszerosandonesinbinarybutImgettingahead ofmyself,thatsforalaterpost.=)
Posted in number theory, pattern, proof, recursion | 10 Comments

Fun "mathemagic" video


Follow

164 of 224

FollowTheMathLess Traveled 10/16/12 16:55


Get everynewpost deliveredto your Inbox.

The Math Less Traveled | Explorations in mathema...

http://mathlesstraveled.com/

Posted on January 12, 2008

Forsomethingonthelighterside,heresafunvideothatIveseenlinkedtofromanumberofplaces.Itsavideo ofArthurBenjamin,amathprofessoratHarveyMuddCollege,performingafifteen-minutemathemagicsshow. Hemultipliesnumbersinhishead,figuresoutthedayoftheweekonwhichpeoplewereborn,andsomeother things,allwhilekeepingupanentertainingbanter.Itsquitefun,watchitifyouvegotasparefifteenminutes!


Posted in computation, links, people, video | 1 Comment

Recounting the Rationals, part II (fractions grow on trees!)


Posted on January 7, 2008

TodayIdliketocontinuemyexpositionofthepaperRecountingtheRationals,whichIintroducedina previouspost.Recallthatourgoalistocomeupwithanicelistofthepositiverationalnumberswherebya nicelistwemeanonewhich: 1. iseasytodescribe, 2. includeseveryrationalnumber, 3. exactlyonce, 4. inlowestterms, 5. atafiniteindex. Soundslikeaprettytallorder!Butitspossible,asCalkinandWilfshow.So,withoutfurtheradodidyou knowthatfractionsgrowontrees?Suretheydo!Heresone:

The Calkin-Wilf Tree

Thisisatreeoffractionsdefinedbythefollowingsimplerules: 1. Thefractionatthetop(theroot)ofthetreeis1/1. 2. Eachfraction theleftis inthetreehastwochildren(thatis,fractionsunderneathitconnectedbybranches):to ;totherightis . ,sorule2saysthattheleftchildshouldbe .Likewise,thechildrenof are and

Forexample,startingatthetopofthetree,wehave ,andtherightchildshouldbe ,andsoon.

Prettysimple,right?Thepictureaboveonlyshowsthefirstfourlevelsofthetree,butitsclearhowtocontinue
Follow

165 of 224

FollowTheMathLess Traveled 10/16/12 16:55


Get everynewpost deliveredto your Inbox.

The Math Less Traveled | Explorations in mathema...

http://mathlesstraveled.com/

extendingthetreedownwardsasfarasyoudlike(forexample,thefirsttworationalsonthenextlevelare1/5 and5/4).Now,guesswhat? 1. Everypositiverationalnumberoccurssomewhereinthistree. 2. Therearenoduplicates:eachrationaloccursexactlyonce. 3. Everyrationalinthetreeisinlowestterms. 4. Ifyoulistouttherationalsinthistreeinorderbylevel(thatis,1/1,1/2,2/1,1/3,3/2,2/3,3/1,1/4), thedenominatorofeachrationalinthelistisequaltothenumeratorofthenext. 5. Thenthintegerinthelistofdenominators(1,2,1,3,2,3,1,4,3,5,)isthenumberofhyperbinary representationsofn,thatis,thenumberofwaystowritenasasumofpowersoftwo,whereatmosttwo copiesofeachpoweroftwoareallowed. Wow!Whatanamazingtree.Andallthatjustfromthosetwosimplerules!Itshouldbeclearthatthisisan answertoouroriginalgoal:justlistingtherationalsinthetreelevel-by-levelgivesusalistofthepositive rationalswithallthepropertieswewerehopingfor.Buttheresalotmoreinterestingstuffhereaswell.Next timeIllstarttalkingaboutwhyeachofthesepropertiesistruebutinthemeantime,youmaywanttotry figuringsomeofthemoutforyourself!Theyareallconsequencesofthetwosimplerulesweusedtodefinethe tree. (Hintfor#1,#2,and#3:rule2tellsushowtomovedownthetree,butwithalittlecleverness,youshouldbe abletofigureouthowtomoveupthetreeaswell.Forexample,assumingthat43/19occurssomewhereinthe tree,rule2tellsusthat43/62and62/19arebelowit;butcanyoufigureoutwhatfractionmustbeaboveit?)
Posted in infinity, number theory, pattern, recursion, sequences | 22 Comments

Next Carnival of Mathematics


Posted on January 3, 2008

ThenextCarnivalofMathematicswillbehostedatArsMathematicaonJanuary11.Itsnottooearlytosendin yoursubmissions!
Posted in links, meta

Carnival of Mathematics #23: Haiku Edition


Posted on December 28, 2007

Welcometothe23rdCarnivalofMathematics:HaikuEdition!First,Imustapologizeforthedelay:Iusuallyhave verylittletroublewithmyhostingprovider,butofcourseitwentdownjustwhentheCoMwassupposedtobe posted.Butitsfree,soIcantcomplain!Itsbackupnow,andwillhopefullystaythatway. ForthiseditionoftheCoM,Idecidedtowriteashortseventeen-syllablehaikuabouteachoftheexcellent seventeensubmissionsIreceived(alongwithadditionalcommentaryofthemoreprosaicvariety).Ivearranged thepostsmoreorlessinorderofrequiredmathematicalbackground,butdontstophalfwaythroughbecause thenyoullmisstheprettypicturesattheend.Enjoy! 1. Englishpolswantto


Follow

166 of 224

FollowTheMathLess Traveled 10/16/12 16:55


Get everynewpost deliveredto your Inbox.

The Math Less Traveled | Explorations in mathema...

http://mathlesstraveled.com/

makemathmoreinteresting. Itsnotalready? FromNaomiStevenssDiaryFromEngland:agovernmentbidtomakemathsmoreinteresting. 2. Neat,useperfectspheres todefinethekilogram! Offbyjustatoms HeatherLewis,of360,writesaboutAustralianscientistswhoaretryingtomakeaperfectsphere.Pretty incrediblestuff! 3. Freshmenworkingroups, andanswertheirownquestions. Effective?Discuss. JackieBofContinuitiesexplainsthepedagogicalapproachshetakeswithherfreshman.Besuretoread(or contributeto!)thefascinatingdiscussionthatensuesinthecommentssection. 4. Multiplechoice,now withbonuschoiceenhancement! Hardtests,nicetograde. MariaAndersen,attheTeachingCollegeMathTechnologyBlog,showsoffanewsortofmultiple-choicetest thatseasytograde,butavoidsmanyofthewell-knownproblemswithtraditionalmultiple-choicetests.I wishIdthoughtofthiswhenIwasteachinghighschool! 5. Areyoulearningtwo languagesmathANDEnglish? Greatsitesforyouhere. LarryFerlazzopresentsalistofthebestmathsitesforenglishlanguagelearners. 6. Mathematicsblogs aremany;whicharethebest? Heresoneopinion. DeniseofLetsplaymath!writesaboutherfavoritemathblogs. 7. Ihavenotyetread LettersToAYoungMathster. Imnotmissingmuch. Andrehaswrittena(not-too-favorable)reviewofIanStewartsbookLetterstoaYoungMathematician, overatherblogmeeyauw. 8. Albatrossesfly infractalpatterns!Ohwait experimentsucked. JulieRehmeyerdiscusseshowscientistsarerevisitingsomeresearchonfractalpatternsintheflightpatterns ofalbatrossatMathTrek.Apparently,justbecauseanalbatrosssfeetaredrydoesntnecessarilymeanits
Follow

167 of 224

FollowTheMathLess Traveled 10/16/12 16:55


Get everynewpost deliveredto your Inbox.

The Math Less Traveled | Explorations in mathema...

http://mathlesstraveled.com/

flying.Whoknew? 9. Eightninety-eight,eight ninety-nine,ninehundredsigh infinityyet? ThadGuyhasafunnycomicaboutinfinity.Checkoutsomeofhisothercomics,tooIma(new)fan! 10. Needsocksinthedark? Thepigeonholeprinciple comestoyourrescue! MaryPatCampbell(akameep)presentsacutevideoexplainingthepigeonholeprinciple.Didyouknowthat atleasttwopeopleintheUShavetheexactsamenumberofhairsontheirbody?Youcantarguewithmath! 11. Acountingproblem: howmanybraceletsarethere? Harderthanitlooks MathMomcameacrossaninterestingMathCountsprobleminvolvingbeadedbracelets,whichgeneratedsome greatdiscussion.Howwouldyousolveit? 12. Listofrationals, bothelegantandcomplete? Isitpossible? Yourstrulyhaspostedthefirstinaplannedmulti-partseriesexplainingaparticularlyelegantwayto enumeratethepositiverationalnumbers. 13. Kochsnowflakefractal: Area?Perimeter? Fractalsaresostrange OveratReasonableDeviations,rodusesgeometricseriestocalculatetheareaandperimeteroftheKoch snowflake.Theresultisrathersurprising! 14. TwelveDaysofChristmas? Howmanypresentsisthat? Letsfigureitout! OveratWildAboutMath!,SolLedermanpresentsaseasonally-appropriateexplorationincountingpresents. Fun! 15. Atrickypuzzle: rectanglesandanglesums. Isolvedit,canyou? JD2718sharesagemofapuzzleinvolvingthesumofsomeangles.Itstrickyareyouuptothechallenge?I wouldespeciallyencouragewould-besolverstocomeupwithanicegeometric solution(Icouldnt)! 16. PascalsTriangle:
Follow

168 of 224

FollowTheMathLess Traveled 10/16/12 16:55


Get everynewpost deliveredto your Inbox.

The Math Less Traveled | Explorations in mathema...

http://mathlesstraveled.com/

writingitoutisachore. Howfastdoesitgrow? Foxy,ofFoxMaths!fame,presentsaninterestingtwo-partanalysisoftheasymptoticgrowthoftherowsof Pascalstrianglenotthegrowthoftheactualvaluesintherows,butofthespaceneededtowritethem! makinguseofsomecleveralgebraicgymnasticsandasymptoticanalysis. 17. Inhowmanyways cantheNaurugraphbedrawn? Theanswer:alot! DavidEppsteinof0xDEpresentsThemanyfacesoftheNaurugraph:acollectionofdiversewaystovisualize aparticulargraphwhichhedubstheNaurugraph,duetothesimilarityofoneofitsdrawingstotheflagof Nauru.Planartesselation,hyperbolictesselation,embeddingonthesurfaceofatorusallthatandmuch more,with,yes,prettypicturesforeverything!Eventhosewhodontunderstandthearticleitselfshouldstill gotakealook,solelyforthesakeofthepictures.=) Thankstoeveryoneforthegreatsubmissions,Ihadafuntimereadingthemandputtingthistogether.Thenext CoMwillbehostedatArsMathematica.Asalways,emailAlonLevy(includingCarnivalofMathematicsinthe subjectline)ifyoudliketohostanedition. Wait!Beforeyougo,inhonorofthenewyear,heresonelastlinkfromMikeCroucheratWalkingRandomly, whowantstoknow:whatisinterestingaboutthenumber2008?
Posted in algebra, books, calculus, challenges, counting, fractals, geometry, infinity, links, meta, number theory, pascal's triangle, pattern, people, sequences, trig, video | 17 Comments

Recounting the Rationals, part I


Posted on December 27, 2007

ThisisthefirstinaseriesofpostsImplanningtowriteonthepaperRecountingtheRationals,byNeilCalkin andHerbertWilf,mathematiciansatClemsonUniversityandtheUniversityofPennsylvania,respectively.Im reallyexcitedaboutit,andIhopethatyoullsoonseewhy!Itsanincrediblyelegantandinterestingpaper,but itsalsoquiteaccessibletoanyonewithonlyamodestbackgroundinmathematicsthatis,itwouldbeifitwere longerthanthreeandahalfpages.CalkinandWilfsmainaudienceisothermathematicians,ofcourse,sotheir presentationisfairlyconcise,trustingthereadertofillinmanyofthedetails.So,Iplantotakemytimegoing throughtheirideas,fillinginmanyofthedetails,andhighlightingsomeinterestingtangentsalongtheway. However,IdonotplantodumbitdowninanywayImgoingtowriteabouteverysinglebitofmathematics intheirpaper. Today,bywayofintroduction,Idliketotalkaboutthemotivatingquestion:isitpossibletomakealistofall thepositiverationalnumbers?Ifitispossible,howcanwedoitinaniceway?

Well,letsthinkaboutit.Howwouldyoumakealistofallthepositiverationalnumbers?(Youmaywishtostop andactuallythinkaboutthisquestionforafewminutes.No,really!)Recallthatrationalnumbersaresimply
Follow

169 of 224

FollowTheMathLess Traveled 10/16/12 16:55


Get everynewpost deliveredto your Inbox.

The Math Less Traveled | Explorations in mathema...

http://mathlesstraveled.com/

fractionsoftheform example, ,and

,wherep(thenumerator )andq(thedenominator )areintegers,andqisntzero.For arepositiverationalnumbers.(Youmayhavelearnedaboutproperandimproper

fractionsinschool,whereproperfractionsarethoseinwhichthenumeratorislessthanthedenominator;I wouldadviseyoutoforgetaboutsuchamathematicallyuselessdistinctionasquicklyaspossible!) Heresoneidea:listalltherationalnumberswithadenominatorof1,followedbyalltherationalnumberswith adenominatorof2,andsoon.Likethis:

Unfortunately,whiletechnicallycorrect(forsuitablevaluesoftechnically),thisisnotaverygoodsolution. Theproblemisthatifyoutriedtowritedownthislist,youdneverevenmakeitthroughtherationalswith denominator1,sincethereareinfinitelymany!Youwouldnever,evergetto1/2,letalonesomethinglike 43/19.Wedlikeourlisttocontaineverypositiverationalatafiniteindex:thatis,ifsomeoneelse(say,your evil,plaid-pants-wearing,math-hatingarch-nemesis)picksapositiverationalnumber,nomatterwhatitis,you shouldbeabletostartwritingoutyourlistofrationalsfromthebeginningandeventuallyaslongasyoukeep writingforalongenough,yetfinite,amountoftimewritedownthechosenrational.Obviously,ourfirsttryis afailureinthisdepartment.Thatevilarch-nemesishasonlytopicksomethinglike1/3,andweresunk! Now,youmaythinkthisisimpossible:ourfirsttryfailed,soitwouldseem,becausetherearetoomany rationals.Wecouldntevenmakeitthroughalltherationalswithdenominator1;howcanwepossiblyhopeto rearrangethingssothateveryrationaloccursatafiniteplaceinthelist?Well,thefirstruleofdealingwith infinityis:donttrustyourintuitionwhendealingwithinfinity!Asitturnsout,therearenttoomanyrationals, wejusthappenedtoputtheminabadorder.Togetabettersenseofwhatsgoingonhere,thinkaboutmaking alistofallthepositiveintegers,likethis:first,writedownalltheoddpositiveintegers;then,writedownallthe evenones.

Thislistobviouslyincludesallthepositiveintegers,butitsabadlistinthesamewayasourinitialtryatalistof rationals:ifyoutriedtowriteitdown,youwouldkeeplistingoutoddintegersforever,withoutevergetting aroundtotheevenones.Butinthisexample,itseasytoseethatthiscanbefixed:everyoneknowsthatifyou writeoutthepositiveintegersinorder,youlleventuallygettoanypositiveintegeryouwant,aslongasyou keepwritinglongenough.

Ah,thatsbetter! Okay,soheresanotheridea:listthepositiverationalsinorderbythesumoftheirnumeratoranddenominator. Wecanagreetolistrationalswiththesamesuminorderbytheirnumerator.So,thefirstrationalinourlistis theonlyonewithasumof2,namely,1/1.Next,therationalswithasumof3:1/2,2/1.Then1/3,2/2,3/1, Follow


Follow

170 of 224

FollowTheMathLess Traveled 10/16/12 16:55


Get everynewpost deliveredto your Inbox.

The Math Less Traveled | Explorations in mathema...

http://mathlesstraveled.com/

followedby1/4,2/3,3/2,4/1,andsoon.Thisisthesamethingasputtingtherationalsinagrid,andlisting thembydiagonals,likethis:

Prettysimple,right?Now,itseasytoseethateverypositiverationalisincludedinthislist:thenumeratorand denominatorofeverypositiverationalsumtosomething(duh!),soanyrationalwecanthinkofwillgetincluded whenwegettothatsum.Moreexcitingisthefactthatthereareonlyafinitenumberofrationalswithanygiven sum(infact,youcanseethatthereareexactly rationalswithsum ),whichmeansthateveryrationalmust occuratafiniteplaceinthelist!Hooray!(Bytheway,anotherwaytosaythatwecanmakealistofthepositive rationalnumberslikethisisthatthepositiverationalsarecountable.Itisanastonishingfact(withaniftyproof) thatalthoughtherationalnumbersarecountable,therealnumbersarentbutthatsforanotherpost!) Theresstillsomethingunsatisfyingaboutthissolution,though:manynumbersoccurmorethanonce.Infact, everynumberoccursinfinitelymanytimes!Forexample,1/1,2/2,3/3,4/4,andsoonarereallyallthesame rationalnumber(namely,1)buttheyalloccurseparatelyinourlist.Thesameistrueof1/2,2/4,3/6andso on.Shouldweleavetherepeatsin?(Bleh.)Shouldwejustsaythatwewontwritedownanynumbersthathave alreadyoccurredearlier?(Yuck.)Eitherwayseemsinelegant. Isittoomuchtoaskforalistofthepositiverationalswhichincludeseachnumberexactlyonce,inlowest terms? Asyouhaveprobablyguessed,itisnttoomuchtoaskandthatswhereCalkinandWilfspapercomesin!They describeanextremelyelegantwaytolistallthepositiverationalnumbers,whichhasalltheniceproperties wevejusttalkedabout,andafewmorebesides.Inthenextpostinthisseries,Illrevealwhatthatlistactually is,andthenIllspendseveralpoststalkingabout(andproving)allofitsniceproperties.Alongthewaywelltalk aboutbinarytreesandbinarynumbers,theprincipleofinduction,recursion,andabunchofotherinteresting things. (PStheCarnivalofMathematicsiscomingsoon!Therearestillafewmorehoursleftforlast-minute submissionsIshouldhaveitupbymidnightattheearliestortomorrowmorningatthelatest.Getexcited!=)
Posted in counting, infinity, number theory, pattern, sequences | 19 Comments

Carnival of Mathematics first call for submissions!


Posted on December 19, 2007

TheCarnivalofMathematicsiscominghereonDecember28,anditsnottooearlytothinkaboutsubmitting! IvealreadygotafewbutImhopingforabunchmore.Imespeciallyhopingtohaveagoodmixofeducational, recreational,andresearch-levelsubmissions,sinceIenjoyandappreciateallofit,andintegratingthedifferent levelsispartofwhatImtryingtodowiththisblogtogetstudentsexcitedaboutmathematicsexploration


Follow

171 of 224

FollowTheMathLess Traveled 10/16/12 16:55


Get everynewpost deliveredto your Inbox.

The Math Less Traveled | Explorations in mathema...

http://mathlesstraveled.com/

andresearch. So,whetheryourewritingaboutcountingorcohomology,studygroupsorLiegroups,Ihopeyoullconsider submittingsomething!IalsohaveaparticularsoftspotfortheoreticalCS(ImcurrentlyapplyingtoPhD programs),soifyouvewrittenaboutanyCStheorywithamathematicalbent,Iencourageyoutosubmitthat too. Tosubmit,youcanusethesubmissionform,orjuste-mailme.


Posted in links, meta | 9 Comments

Google open-source programming contest


Posted on November 28, 2007

Areyouahighschoolstudent?Areyouinterestedincomputerprogramming?Youshouldparticipateinthe GoogleHighlyOpenParticipationContest!Theideaisthatstudentscompetetocontributefeaturestovarious opensourceprojects.Youreguaranteedtogetat-shirt,andyourealsoguaranteedtowinsomemoneyifyou completeatleastthreetasks.Plus,theresafunsenseofaccomplishmentincontributingtoopen-sourceprojects (Ispeakfromexperience).Grandprizewinnersgetanall-expensespaidtriptoGoogleHQinMountainview,CA. Theonlywayitcouldbecoolerisifitdidnthavesuchadumbname.


Posted in links

Perfect numbers, part III


Posted on November 27, 2007

Thisisthelastinaseriesofpostsaboutperfectnumbers.Aquickrecapoftheseriessofar:inpartI,Idefined perfectnumbersaspositiveintegersnforwhich revealedthatifnisfactoredintoprimepowersas ,where denotesthesumofthedivisorsofn.Ialso ,then canbecalculatedasfollows:

InpartII,wesawwherethisformulaactuallycomesfrom.Finally,inthechallengeinterlude,DBandSteve Gilberg(andyou?)foundthatthefirstfourperfectnumbersseemtoallbeoftheform additionallyclaimedthatthisonlyworkswhen Well,itstrue: isperfectwhenever isprime. isprime.Butyoudonthavetotakemywordforitlets if isprime. ;DB

proveit!Sincenisperfectif Applyingtheformulafor

,wewanttoshowthat

,thisisjustsomestraightforwardalgebra.(Note:bystraightforwardIdontmean

youredumbifyoudontseeitimmediately;Imeanifyourepatientandpersistent,youshouldbeableto workitoutforyourself.Mathematiciansarefondofsayingthatthingsareobviousorstraightforward,but thisiswhattheyactuallymean.)

Follow

172 of 224

FollowTheMathLess Traveled 10/16/12 16:55


Get everynewpost deliveredto your Inbox.

The Math Less Traveled | Explorations in mathema...

http://mathlesstraveled.com/

Voila!Justwhatwewantedtoshow.Notethattherestrictionthat formulafor canbefactoredfurther. So,thequestionbecomes,whenisanumberoftheform canbefactoredas formtobeprime? ,then canalsobefactored,as

mustbeprimeisveryimportant:the

assumesthatnisfactoredasaproductofprimepowers,sothecomputationaboveisinvalidif

prime?Well,firstofall,mmustbeprime;ifm .Butisitenough

Noticethatthefirstfourperfectnumberscorrespondtothefirstfourprimes,2,3,5,and7:

Thenextone,however,dashesourhopes: necessarilymeanthat

.Now,itisimportanttonotethatthisdoesnt isperfectwhen ,with

isntperfect:wehaveonlyprovenabovethat

isprime,whichdoesntnecessarilysayanythingaboutwhathappenswhenitisnt.However,itturnsout thatthisisindeedtrue.Infact,moreistrue:allevenperfectnumbersmustbeoftheform number,butitturnsoutthatthenextthreeprimes(13,17,and19)do: prime.Therearenootherkindsofevenperfectnumbers.So,wesawthat11doesntgiveusaperfect

areallperfect.Butthenafewmoreprimesgetskipped;thenextperfectnumbercorrespondsto31. Primesoftheform mightringabellforlong-timereadersofthisblog:thesearetheMersenneprimes!As

ofrightnow,weknowof44Mersenneprimes,andthereforeweknowabout44perfectnumbers.Thelargest knownMersenneprimehasalmosttenmilliondigits,sothelargestknownperfectnumberhasabouttwicethat many! Now,whataboutoddperfectnumbers?Arethereanyoddnumberswhichequalthesumoftheirproperdivisors? Nooneknows!!


Posted in algebra, number theory, open problems, primes, solutions | 11 Comments

Video: The Story of Pi


Posted on November 26, 2007

Afunvideoabout ,includinganicevisualexplanationoftheformulafortheareaofacircle,andsome interestingplacesthat showsupwhereyoumightnotexpectitto.ThevideowasmadebyTomM.Apostoland


Follow

173 of 224

FollowTheMathLess Traveled 10/16/12 16:55


Get everynewpost deliveredto your Inbox.

The Math Less Traveled | Explorations in mathema...

http://mathlesstraveled.com/

JimBlinnaspartofProjectMATHEMATICS!.

Also,dontforgettopostyourdiscoveriesaboutperfectnumberfactorizationsascommentstotheprevious post!
Posted in famous numbers, geometry, links, video | 5 Comments

Perfect numbers, interlude (Challenge #11)


Posted on November 20, 2007

Recallthatthefirstthreeperfectnumbersare6,28,and496.Andifyoutriedcomputing here?(Hint:tryfactoring)

asIsuggested

neartheendofpartI,youmighthavenoticedthat8128isperfectaswell!Doyouseeanyinterestingpatterns

Postyourdiscoveriesascommentshere(unlessyoualreadyknowtheanswer,inwhichcaseyoushouldrefrain fromspoilingotherpeoplesfun=).
Posted in challenges, famous numbers, number theory | 4 Comments

Video: Mbius transformations revealed


Posted on November 18, 2007

Foryourviewingpleasure,afantasticallybeautifulvideoaboutMbiustransformations,whicharefunctionsof theform

wherez,a,b,c,anddarecomplexnumbers,and withb=2,c =1,anda=d=0. isnt,because number

.Forexample,

isaMbiustransformation

isalsoaMbiustransformation.However,

.SinceanyMbiustransformationsendsacomplexnumberztoanothercomplex

,itcanbethoughtofasatransformationonthecomplexplane.Thequestionis,whatsortsof

transformationsarepossible?Thatswhatthevideoisabout.

Follow

174 of 224

FollowTheMathLess Traveled 10/16/12 16:55


Get everynewpost deliveredto your Inbox.

The Math Less Traveled | Explorations in mathema...

http://mathlesstraveled.com/

ThevideowasmadebytwomathematiciansattheUniversityofMinnesota,DouglasArnoldandJonathan Rogness.Theresmoreinformationaboutthevideohere.
Posted in complex numbers, geometry, video

Perfect numbers, part II


Posted on November 15, 2007

InmylastpostIintroducedtheconceptofperfectnumbers,whicharepositiveintegersnforwhich where denotesthesumofallthedivisorsofn.Incidentally,wecouldwritethedefinitionof , thatis,thesumrangingoveralldforwhichddividesn( as

).Ofcourse,thisisslightlynonstandard -notation;

usuallythereissomesortofindexvariablewhichcountsbyonesfromastartingtoanendingvalue,butherethe indexvaraible,d,doesntcountbyonesbutinsteadonlytakesonvaluesthatevenlydividen.Thisnotationis actuallyfairlycommon,though,andtheresnotreallyanychanceofconfusion. WhileImonatangent,Ishouldalsomentionthatingeneral, ofthedivisorsofn,thatis, isusedtodenotethesumofthekthpowers

Forexample,

.So,thefunctionthatwehavebeencalling

isreally

,and

tellshowmanydivisorsnhas,insteadofgivingtheirsum(doyouseewhy?). Now,asyoumayrecall,inmylastpostIclaimedthefollowingformulaforcalculating befactoredas : ,assumingthatncan

Follow

175 of 224

FollowTheMathLess Traveled 10/16/12 16:55


Get everynewpost deliveredto your Inbox.

The Math Less Traveled | Explorations in mathema...

http://mathlesstraveled.com/

ButIdidntexplainwherethisformulacomesfrom!Well,todayIdliketorectifythat.Letsstartwithan example:letsaddupallthedivisorsof18.Simpleenough:1+3+9+2+6+18=39.Butwecanwritethis inaslightlydifferentformwhichshowsmoreclearlywhatisgoingon:

Doyouseethepattern?18canbefactoredas

,andeverydivisorof18isoftheform

,where

and

.Thismakessenseclearly,anydivisorof18canonlyusethesameprimefactorsthat18has,anda givenprimefactormustoccurbetweenzerotimesandthenumberoftimesitoccursin18(sinceotherwisewe wouldntgetadivisorforexample,nothingwithafactorof canbeadivisorof18).Infact,wecanmakea strongerstatementthanthis:wecanobtaineverysingledivisorof18exactlyoncebytaking possiblecombinationofvaluesfor and . ,theneverydivisorofn forevery

Theresnothingspecialabout18here,ofcourse.Ingeneral,ifwehave willbeoftheform ,where

(thatis,each fallsbetween0andthecorresponding ,

inclusive).Wegeteverydivisorexactlyonceifwetakeallpossiblecombinationsofvaluesforthe s. Now,goingbacktoourexampleofaddingupthedivisorsof18foramoment:noticethatwecanfactoritnicely, likeso:

Inotherwords,togetadivisorof18,firstchooseapowerof2 ofalldivisorsoftheform

andthenchooseapowerof3

;multiplyingtheseexpressionsgivesusthesumofallpossiblechoices.Inthegeneralcase,thesum ,forallpossiblecombinationsofvaluesforthe s,canbefactoredas:

Thedotsinthemiddleindicatethatthereisoneterm

foreverydifferentprime .Again, ;multiplyingallof :weobtain

thisrepresentsfirstchoosingapowerof ,thenchoosingapowerof ,andsoonupto thesetermsgivesusthesumofallpossiblechoices.Asanexample,consider

Youcanverifyforyourselfthatthisis,infact,thesumofallthedivisorsof720!OK,werealmostthere.The onlystepthatremainsistosimplifythesumsoftheform .Youmayrecognizethisasa geometricseries,asumoftermswhereeachtermisaconstantmultipleoftheprevious.Notethat ,soifwesubtractS fromthis,allthetermscanceloutexceptforthefirstandlast, giving

andtherefore
Follow

176 of 224

FollowTheMathLess Traveled 10/16/12 16:55


Get everynewpost deliveredto your Inbox.

The Math Less Traveled | Explorations in mathema...

http://mathlesstraveled.com/

Looksfamiliar,doesntit?Sureitdoes!Nowdoyouseewheretheformulafor Nextup:findingmoreperfectnumbers!
Posted in algebra, famous numbers, number theory | 8 Comments

comesfrom?

Perfect numbers, part I


Posted on November 12, 2007

TodayIdliketotalkaboutperfectnumbers,whichtouchonsomecleveralgebraandsomeneattopicsinnumber theory.Atleast,Illstarttalkingaboutthem,sinceitwillprobablytakemethreeorfourpoststosayeverythingI wanttosay.=) Withoutfurtherado,thedefinitionofperfectnumbersisverysimple:anumbernisperfectifitsproperdivisors addupexactlyton.Adivisorofn,ofcourse,isanumberwhichevenlydividesn;talkingabouttheproper divisorsofnsimplyexcludesnitself.Forexample,theproperdivisorsof6are1,2,and3,whichhavethesum1 +2+3=6so6isaperfectnumber.Anequivalentwayofexpressingthedefinitionisthatnisperfectifall itsdivisors(includingnitself)adduptotwicen.Thesumofallthedivisorsofnisoftenwritten formally,wecanwritethatnisperfectif . ,so

Thenextperfectnumberafter6is28(28=1+2+4+7+14),andthenextafterthatis496=1+2+4+ 8+16+31+62+124+248.Arethereanymore?Well,thatswhatIdliketotalkabout.TodayIllstartby showinghowwecaneasilycompute Howwouldyoucompute,say, .

?Onewaywouldbetosimplycheckeverynumberfrom1to252tosee

whichonesdivide504,andaddthemup.(252ishalfof504,sothereclearlycantbeanylargerproperdivisors of504.)However,thatstediousanderror-prone.Itseemslikethereshouldbeabetterway. Asyouknow,wecanwriteanyintegerintermsofitsprimefactorization:forexample, canalsosimplifythisfurtherbycollectinglikeprimes: theform .We

.Ingeneral,wecanwriteanynumbernin

where

aremdifferentprimenumbers,andthe sareallpositiveintegerpowers.Inthecaseof504, and .

forexample,wewouldhave

Now,Iclaimthatifnisfactoredintheformabove,then

[Incidentally,areyouoneofthosepeoplewhofreaksoutwhenyouseeascary-lookingequationliketheone above?Well,stopitthisinstant!Ifyouthinksomethingistooscaryorcomplicatedforyoutounderstand,then
Follow

177 of 224

FollowTheMathLess Traveled 10/16/12 16:55


Get everynewpost deliveredto your Inbox.

The Math Less Traveled | Explorations in mathema...

http://mathlesstraveled.com/

you'reabsolutelyright--becauseyouhaven'tevengivenyourselfachance.Butifyouforceyourselftogetover yourinitialfear,takeagoodhardlook,andconsideritslowlyinsmallpieces,youmightbesurprisedathow muchyoucanunderstandafterall.] BigPi( )isjustlikeBigSigma( ),exceptthatitdenotesaproductinsteadofasum.Youcanreadmoreabout sigmaandbigpinotationhere.Inthiscase,theformulafor ofn,wecompute justmeansthatforeach inthefactorization : ,andthenmultiplythemalltogether.Asanexample,wecancompute

Letsalsotryiton perfectnumbers.

and

.Weknowthisshouldyield12and56respectively,since

for

Itworks!OK,nowitsyourturn:canyouusethismethodtocompute

Thequestionyoushouldbeaskingatthispointis:whydoesthiswork?Wheredidthiscrazy-lookingformulafor comefrom?ThatswhatImgoingtotalkaboutnexttime.Andafterthat,Illshowhowthisformulacan helpusdiscovermoreperfectnumbers.


Posted in famous numbers, number theory, sequences | 3 Comments

Menger sponge video


Posted on November 10, 2007

CheckoutthefollowingtotallysweetvideoofzoomingintoaMengersponge!

ThisvideowasmadebyDavidMakin,whohaslotsofothercoolimagesandvideosathiswebsite.Youcan probablyfigureoutwhataMengerspongeisjustfromwatchingthevideo,butitsafractalobjectwhichisvery Follow

178 of 224

FollowTheMathLess Traveled 10/16/12 16:55


Get everynewpost deliveredto your Inbox.

The Math Less Traveled | Explorations in mathema...

http://mathlesstraveled.com/

easytomake.Hereswhatyoudo: 1. Startwithasolidcube. 2. Slicethecubeinto27equallittlecubes,bymakingtwoparallelslicesineachdimension(justlikeaRubiks cube). 3. Removethecubeintheverycenter,andthesixcubesinthecenterofeachfaceofthebigcube.Youllbeleft withacube-shapedobjectwithsquareholesgoingstraightthroughthemiddleoneachside. 4. Repeatthisprocedureoneachofthe20remaininglittlecubes,andsoonrecursivelyforever. TheWikipediapagehassomenicepicturesthatshouldmakethisprettyclearifitsnotalready.Incasethevideo didntblowyourmindenough,youshouldnotethatMengerspongeshavezerovolumebutinfinitesurfacearea! (Howisthatpossible?!Ihearyoucryindismay.Well,infinityplaysveryweirdgameswithyourintuition!) FoundviaGodPlaysDice.
Posted in fractals, video | 6 Comments

Geometric multiplication: an explanation


Posted on November 10, 2007

Nowforanexplanation/proofofthatweirdmethodofmultiplicationthatItalkedaboutinapreviouspost. Youllrecallthatitsreallyquitesimple:tomultiplyaandb,drawalinefrom to andseewhereit crossesthey-axis.Butwhyonearthshouldthatcrossthey-axisatheight ?Well,ImafraidIstilldontknow howtoanswerthewhyinawaythatmakesintuitivesense,butwecancertainlyshowthatitistrue. Thefirststepistofigureouttheequationofthelinepassingthroughthepoints is and .Todothat,we

firstneedtheslope.Slopeisriseoverrun,thatis,thechangeinydividedbythechangeinx,whichinthiscase

Wecansimplifythis,however,bynotingthat .Inthatcase,wedhave wecanalwayscompute

factorsas

,sothe

scancelaslongas ,butthatsOKsince

andwedbetryingtodrawalinefromapointtoitself,whichdoesnt

makesense.Thisjustmeansthatwecantusethismethodtocomputeaproductlike andnegatetheresult.So,wenowhave

Now,howcanwefindtheequationoftheline?Sincetheslopeofalineisthesameeverywhere,theslope between andanypoint onthelineshouldalwaysbeequaltotheslopemthatwecomputed.Wecan writethisrequirementas

Thisisalreadyaperfectlygoodequationfortheline,butletssimplifyitabit:
Follow

179 of 224

FollowTheMathLess Traveled 10/16/12 16:55


Get everynewpost deliveredto your Inbox.

The Math Less Traveled | Explorations in mathema...

http://mathlesstraveled.com/

Nowthatwehaveanequationfortheline,wewanttoknowwhereitcrossesthey-axis.Butthisissimple:it crossesthey-axiswhenxiszero;plugging0intotheaboveequationforxyields
Posted in algebra, geometry, proof

Geometric multiplication
Posted on November 8, 2007

Ivejustlearnedaboutafairlyuseless,yetutterlybeguilingmethodforperformingmultiplication,andIdliketo shareit! Supposeyouhavetwonumbersyouwishtomultiply,callthemaandb.Yourfirstinstinctisprobablytowrite oneundertheotherandstartwritingdownpartialproducts,ortoreachforacalculator.Butwaittheres anotherway!Allyouneedisapencil,aruler,andaVeryLargesheetofgraphpaper.Thefirststepistodrawthe parabolawhichisthegraphof correspondingto and (actually,thisstepisoptional).Now,findthepointsontheparabola .Inotherwords,plotthepoints and and .So,forexample,ifwe .Now,usetherulertoconnectthetwo

wantedtomultiply4by9,wewouldplotthepoints

pointswithastraightline,andlooktoseewhereitcrossesthey-axisandvoila!Youhaveyourproduct. Prettyamazing,huh?Ihavesomequestionsforyou: Canyouexplainwhythisworks?Hint:youllneedtodustoffyourknowledgeonfindingslopesand equationsoflines(unlessitisntdusty). Doesitstillworkevenifaandbhavedifferentsigns? Whataboutifaand/orbareequaltozero? TheancientGreeks(whodidalltheirarithmeticandalgebrausinggeometry)wouldbeproud. ThismultiplicationmethodcomescourtesyofSolLedermanatWildAboutMath.FansofTheMathLessTraveled mightwanttocheckoutSolsblog;hesgotsomeinterestingpostsonvariousmathtopicsandmathteaching.


Posted in algebra, geometry, links, people | 4 Comments

Hello, reader!
Posted on November 3, 2007

Follow

180 of 224

FollowTheMathLess Traveled 10/16/12 16:55


Get everynewpost deliveredto your Inbox.

The Math Less Traveled | Explorations in mathema...

http://mathlesstraveled.com/

TherearetimeswhenIgetdiscouragedwritingthisblog:itshardtokeepwritingsomethinglikethiswhen theresnotawholelotoffeedbackandyoureallyhavenoideawhetheranyoneisreadingitatall.Thenthe otherday,Iinstalledapluginthatletsmetrackvariousstats,andIfoundoutthatthisbloggetsaround100 visitsperday!So,hellothere,mysteriousanonymousreaders.Ihopeyouenjoy! (AndIdolikegettingcomments,sodontbeshy.=)


Posted in meta | 6 Comments

Nuclear Pennies Game: Analysis


Posted on November 2, 2007

Andnow,forthepromisedanalysisoftheNuclearPenniesGame! First,recalltherulesofthegame:thereisasemi-infinite(i.e.withabeginningbutnoend)stripofsquares,each ofwhichcancontainastackofanynumberofpennies(ornopenniesatall).Youareallowedtosplitapenny byreplacingitwithtwopennies,oneinthesquareoneitherside.Thisrulecanalsoberuninreverse;two penniesseparatedbyexactlyonespacecanbefusedintoasinglepennyonthemiddlesquare. Wesawhowtogetfromasinglepennyonsquare#7toasinglepennyonsquare#1.However,Iclaimedthatit isimpossibletomoveasinglepennyfromsquare#7tosquare#2.Letmeshowyouwhy. Weregoingtostartbydoingsomethingthatmightseemabitodd,butyoullseewhysoonenough.Weregoing tolabelthenthsquarewith .Likethis:

(Remember,

.)Now,wewillthinkofthenumberofpenniesonagivensquareasbeingmultipliedbythe

appropriatepowerofx.Thatway,everyconfigurationofpenniesisrepresentedbyapolynomial.Forexample,if therearethreepenniesinsquare#0,twopenniesinsquare#2,andonepennyinsquare#5,wewouldrepresent thatbythepolynomial .

Now,heresthekeyidea:wewanttopickavalueforxsothattwopolynomialshaveequalvaluesifthey representpennyconfigurationsthatyoucanconvertbetweenwhileonlymakinglegalsplittingandfusingmoves. Ifwecandothat,allyouneedtodotoseewhetheritispossibletomovefromoneconfigurationtoanotheristo writedowntheirpolynomials,pluginthemagicalvalueofx,andseeiftheyarethesame! So,howcanwefindthismagicalvalueofx?Well,letssupposewestartwithasinglepennyonsquare#1.That wouldberepresentedbythepolynomial .Bythesplittingrule,wecanreplacethiswithapennyonsquare#0 andapennyonsquare#2,givingusthepolynomial .Wewantthesepolynomialstohavethesamevalue, .Now,whatifwehadstarted but sincewecanconvertbetweenthemwithalegalmove,sowemusthave thatsreallythesamethingas

withapennyon,say,square#4andappliedthesplittingrule?Inthatcasewewouldhave Thesamegoesforanystartinglocation,sotheequation

,sincewecanjustdividebothsidesby (assumingthatxisnotzero). coversallpossibleapplicationsofthesplitting


Follow

rule.Whataboutthefusionrule?Well,remember,thefusionruleisreallyjustthesplittingrulebackwards,soit Follow

181 of 224

FollowTheMathLess Traveled 10/16/12 16:55


Get everynewpost deliveredto your Inbox.

The Math Less Traveled | Explorations in mathema...

http://mathlesstraveled.com/

givesustheexactsameequation. So,bothrulescanbesummedupbythissingleequation: number,justasIpromised! Now,ifitwerepossibletomoveasinglepennyfromsquare#7tosquare#2,thenwewouldhave isthesameas singlepennyfromsquare#7tosquare#2isntpossible!Infact,thesmallestpowerofxforwhich else,youcanonlymoveitbysixsquaresatatime! TheNuclearPenniesgameisoriginallyfromDanPiponiatANeighborhoodofInfinity(fromwhomIalsostolea lotofthenicepictures),whobaseditontheideasinthepaperSevenTreesinOne,byAndreasBlass.The mathematicallyintrepidmightwanttotryreadingsomeoftheoriginalpaper,whichisreallyquiteexcellent butthecontentofthepaperiswaydeeperthanwhatIvewrittenabouthere,sodontbediscouragedifyou dontunderstandalotofit(Idontunderstandtheentiresecondhalfofthepaper!).Justtryingtogivecredit wherecreditisdue.
Posted in algebra, complex numbers, games, proof | 3 Comments

.Allwehavetodoissolveittofindour .Aha,acomplex

magicalvalueofx.Well,usingthequadraticformula,thatsnothard:weget

,which is6.x,

.Well,isthattrue?Itturnsoutthatitisnt,asyoucanverifyforyourselfso,movinga

infact,isoneofthecomplexsixthrootsof1.Soifyouwanttostartwithasinglepennyandmoveitsomewhere

Nuclear Pennies Game: Solution


Posted on October 30, 2007

Itseemsthateveryonewithablogisalwaysapologizingfornotpostinginawhile,asifthishassomehow inconveniencedtheirreaders.Ofcourse,withthemagicoffeedreaders,emailnotifications,andthelike,thisis nottrueatall.You,gentlereader,inalllikelihoodhavenoteventhoughtaboutmyblogoncesincethelasttime Iposted.(Yes,Iforgiveyou.)SoIwontapologizefornotpostinginawhile.IfIdid,Imightaskyoutoplease forgivemeduetothefactthatIamhardatworkongraduateschoolapplications.ButIwont,remember? Onwards! YoumayrecallthatmylastpostconcernedthegameofNuclearPennies.Ifyoudontrememberitorifyou haventhadachancetotrysolvingityourselfnowisagoodtime! FromGeorgeBell(whoalsohasagreatsiteaboutpegsolitaireifyouenjoythesesortsofmathematicalgames, youshouldcheckitout)comesthefollowingexcellentsolution: Thispuzzleseemsratherdifficulttosolvebytryingrandommovesthatappeartogetoneclosertothe solution.However,considertheboardasequenceofintegers,eachbeingthenumberofpenniesatthat location,andthemovesasaddingorsubtracting(+1,-1,+1)tothreeconsecutivevalues. Thenbyinspectionwecancomeupwithasolution,notworryingaboutwhetherthenumberofpennies isalwayspositive,byaddingthecolumns:

Follow

182 of 224

FollowTheMathLess Traveled 10/16/12 16:55


Get everynewpost deliveredto your Inbox.

The Math Less Traveled | Explorations in mathema...

http://mathlesstraveled.com/

000000+1(start) 0000-1+1-1 000-1+1-10 0+1-1+1000 +1-1+10000 --------------------------+1000000(end)

Thisgivesyouthefourcriticalmovesthatmustbeinanysolution.However,applyingthemdirectlyis illegal.[Butyoucan]just[keep]splittingtheleftpenny(7times),then[dothe]critical4movesand youcomeupwiththemirrorimageposition,soyounowcombinetothefinalposition.Ergo,

000000010 000000101 000001011 000010111 000101111 001011111 010111111 101111111 110211111<--+1-1+1 111121111<--+1-1+1 111112011 111111101<---1+1-1 111111010<---1+1-1 111110100 111101000 111010000 110100000 101000000 010000000

IaddedlabelstoshowwherethefourcriticalmovesarehappeninginthemidstofGeorgessolutionabove. Andheresavideoofaslightlydifferentyetessentiallysimilarsolution(canyouseehowtheyaredifferent,and whythisdifferenceisunimportant?):

Follow

183 of 224

FollowTheMathLess Traveled 10/16/12 16:55


Get everynewpost deliveredto your Inbox.

The Math Less Traveled | Explorations in mathema...

http://mathlesstraveled.com/

Ifyoutriedthesecondchallenge,thatofmovingapennyonlyfivespacestotheleftinsteadofsix,youprobably didntsucceed:itturnsoutthatthisisimpossible!Asinglepennycanonlybemovedbymultiplesofsix. TomorrowIllproveittoyouusing,ofallthings,complexnumbers!Complexnumbers!?Ihearyouexclaim. Whatonearthcancomplexnumberspossiblyhavetodowithpenny-shufflinggames?Justwaitandsee,dear reader!


Posted in games, links, solutions | Tagged game, nuclear, pennies, solution | 1 Comment

The Nuclear Pennies Game


Posted on October 6, 2007

HaveIgotaninterestinggameforyou! Imaginethatyouhaveaninfiniterowofsquares,numberedbynon-negativeintegers.Square#0isthefirst square.Toitsrightissquare#1,thensquare#2,andsoonforever.Eachsquarecanholdasmany(orasfew) penniesasyoulike.Thereareonlytworules. First,thefissionrulestatesthatyoumayreplaceapennybyapairofpennies,placingoneineachofthe immediatelyadjacentsquares.Forexample:

Thefusionrule,ontheotherhand,statesthatiftherearetwopenniesseparatedbyexactlyoneintervening square,youmayreplacethembyasinglepennyinthatmiddlesquare.Forexample:

Follow

184 of 224

FollowTheMathLess Traveled 10/16/12 16:55


Get everynewpost deliveredto your Inbox.

The Math Less Traveled | Explorations in mathema...

http://mathlesstraveled.com/

Inasense,theresonlyonerule,butyoureallowedtoapplyiteitherforwardsorbackwards.Notealsothat theresnorequirementforanyofthesquaresinvolvedtobeemptyoranythinglikethat. Heresthechallenge:startingwithjustasinglepennyinsquare#7,

canyouendupwithjustasinglepennyinsquare#1,

makingonlymovesallowedbytherulesabove?Whatsthefewestnumberofmovesthatyouneed? Onceyouvesolvedthatone,trystartingwithapennyinsquare#7andendingupwithapennyinsquare#2. TuneinnexttimefortheanswerandanamazingmathematicalanalysisoftheNuclearPenniesGame!(Note:I didntcomeupwiththeNuclearPenniesGame;Illgiveproperattributionnexttime,butfornowIdontwantto giveawaytheanswerbylinkingtoit.)


Posted in challenges, games, pattern | 8 Comments

Explicit Fibonacci numbers


Posted on September 13, 2007

Dontworry,thispostisntgoingtobeX-rated!ByexplicitImeannotrecursive.RememberthattheFibonacci numbersaredefinedrecursively,thatis,eachFibonaccinumberisgivenintermsofpreviousones: .Doesntitmakeyouwonderwhethertheresaformulawecouldusetocalculate directlyin termsofn,withouthavingtocalculatepreviousFibonaccinumbers? Itsagreatquestion.Suchaformulawouldbecalledanexplicitformula,andingeneral,youcoulddevotea wholegraduate-levelclasstothetopicofturningrecursiveformulasintoexplicitones!However,findingan explicitFibonacciformulaisntasdifficultasallthat.Infact,thekeytounderstandingitisthegoldenpowers identityfrommylastpost:

Firstofall,wenotethateverysinglepieceoftheaboveidentitysproofworksfor justaswellasfor ,soitis alsotruethat

Now,considersubtractingonefromtheother:

Butnowitisasimplemattertosolvefor :

Follow

185 of 224

FollowTheMathLess Traveled 10/16/12 16:55


Get everynewpost deliveredto your Inbox.

The Math Less Traveled | Explorations in mathema...

http://mathlesstraveled.com/

Amazing!Itseemsweirdthataformulatogenerateintegersinvolvesallthesestrangenon-integerslike and butthereitis.Wecanevensimplifythisfurtherbynotingthatsince than1,ingeneral isverysmall.So,whatshappeninghereisthat than ,andsubtractingoffthe(verysmall) round(upordown)tothenearestinteger: hasanabsolutevaluesmaller isjustatinybitbiggerorsmaller and

gives exactly.Wecouldjustaswellcompute

Excellent!NowwecancomputeFibonaccinumbersdirectly,withoutcomputingallthepreviousones.
Posted in famous numbers, fibonacci, golden ratio, proof | 4 Comments

Golden powers
Posted on September 10, 2007

So,weknowfromapreviouschallengethat

.Thatsaprettyinterestingproperty,whichissharedonly

byitscousin, .Iwonderwhetherotherpowersof havespecialpropertiestoo?Letssee:

Interesting!Whatabout ?

And ?

Curiouserandcuriouser!Areyounoticingapattern?(Tryworkingout toseeifitfitsthepatternyoususpect.) Hopefully,younoticedthatFibonaccinumbersareinvolvedhere.Ifthispatternholds,wewouldexpecttofind that , ,when . ,andsoon.Tostateitmoregenerally,wesuspectthat

Isthistrue?Itturnsout,amazingly,thatitis,andwecanproveitusingatechniquecalledmathematical induction.Theideaofmathematicalinductionissortoflikeknockingoverachainofdominoes.First,wellshow thattheabovestatementistrueforsomeparticularvalueofn(thebasecase;forthisstatement,wellusen=2). Then,wellshowthatifthestatementistrueforanygivenvalueofn,thenitmustbetrueforn+1aswell(the inductivestep).Doyouseehowitslikeknockingoverdominoes?Ifthestatementistruewhennis2,thenit mustbetruefor3aswell,whichmeansitmustbetruefor4,whichmeansandsoon,forallpositiveintegers! Ready?First,recallthattheFibonaccinumbers aredefinedasfollows:


Follow

186 of 224

FollowTheMathLess Traveled 10/16/12 16:55


Get everynewpost deliveredto your Inbox.

The Math Less Traveled | Explorations in mathema...

http://mathlesstraveled.com/

Nowforthebasecase:whennis2,thestatementsaysthat equalto1. Finally,theinductivestep.Wellbeginbysupposingthat toconcludethatthestatementistruefor suggestsamethodofattack:

;thisistruesince and areboth

forsome

,andseeifthisallowsus

aswell.Themethodweusedabovetoevaluate , andsoon

So,sinceweveprovedthatthestatementistruewhennis2,andwevealsoprovedthatitmustbetrueforn+ 1wheneveritstrueforn,itmustbetrueforall !

Nextup:wellusethisneatrelationshipbetween andtheFibonaccinumberstodiscoveranotherone!
Posted in famous numbers, fibonacci, golden ratio, induction, proof | 6 Comments

Challenge #10 Solution


Posted on September 8, 2007

HaveyoutriedsolvingChallenge#10yet?Gotryitfirstifyouhavent.Itsnottoohard,Ipromise! Continuereading

Posted in famous numbers, golden ratio, proof, solutions | 3 Comments

Golden ratio properties (Challenge #10)


Posted on September 6, 2007

Rememberthegoldenratio, (phi)?Itsthepositivesolutiontotheequation , whichcanbefoundusingthequadraticformula:

Closelyrelatedisitscousin, (phi-hat) ofthefollowingistrue? 1.

.Aswellsee,thesefamousconstantsactuallyrelatetoFibonacci

numbersinsomeamazingways.Butfirst,wellneedafewpropertiesofthesenumbers.Canyoushowwhyeach

Follow

187 of 224

FollowTheMathLess Traveled 10/16/12 16:55


Get everynewpost deliveredto your Inbox.

The Math Less Traveled | Explorations in mathema...

http://mathlesstraveled.com/

2. 3. 4. Extrapointsforespeciallyslickproofs.=)
Posted in challenges, famous numbers, fibonacci, golden ratio, number theory, proof | 6 Comments

USAMTS problems
Posted on August 30, 2007

Lookingforaninterestingmathematicalchallenge?Tryyourhand(brain)atthecurrentchallengeproblemsfrom theUSAMathematicalTalentSearch(USAMTS).Thesearesomegreatproblemstheyareinterestingwhile stillbeingaccessibletomosthighschoolmathematicsstudents.TheideabehindtheUSAMTScompetitionisto encouragecreativeproblem-solving.Youcansignupforfreeandsubmityoursolutions(youhavetowritean explanationofyoursolution,notjustanumber)andagraderwillactuallyscoreyoursolutionsandoffer comments.Funmathproblems,withfeedback,forfreewhatsnottolike?Solutionsforthecurrentsetof problemsaredueinearlyOctober,whenthenextsetofquestionscomesout.


Posted in challenges, links

Prime Shooter!
Posted on August 15, 2007

TodayIcameacrossthisniftySpaceInvaders-likegameexceptinsteadofshootingbulletsatUFOs,youshoot primefactorsatintegers.Forexample,ifthenumber66isfallingtowardsyou,youneedtoshootitwith2,3, and11tomakeitgoaway.Sillybutfun! (foundviaGodPlaysDice)


Posted in games, primes | 2 Comments

A few related problems


Posted on August 8, 2007

Hereisacollectionofinterestingmathproblems.Despiteappearances,theyallhavesomethingincommon.Can youfigureoutwhatitis? Asinglepairofbabyrabbitsisplacedonanisland.Theytakeonemonthtogrowup;afterthat,theyhave onenewpairofbabyrabbitseverymonth.Eachnewpairofbabyrabbitstakesamonthtogrowup,then starthavingbabyrabbitsoftheirown,andsoon.Howmanypairsofrabbitsareontheislandaftern months? Susiecanclimbeitheroneortwostepsatatime.HowmanydifferentwayscanSusieclimbastaircasewithn stairs?(Forexample,shecouldclimbastaircaseof3stairsin3differentways:1-2,2-1,or1-1-1.) Howmanydifferentwaysaretheretotilea2xnrectanglewith21dominoes? Ifmalebees(drones)areproducedasexuallyfromafemale(queen),butfemalebeeshavetwoparents,one
Follow

188 of 224

FollowTheMathLess Traveled 10/16/12 16:55


Get everynewpost deliveredto your Inbox.

The Math Less Traveled | Explorations in mathema...

http://mathlesstraveled.com/

maleandonefemale,howmanyancestorsdoesamalebeehaveinthenthgenerationbeforeit? Well,ok,Isortofgaveitawaywiththetagsbelieveitornot,thesolutiontoeachproblemistheFibonacci numbers!Foreachproblem,canyoufigureoutwhy?Canyoucomeupwithyourownproblemwhichhasthe Fibonaccinumbersasthesolution?


Posted in famous numbers, fibonacci, pattern, sequences | 3 Comments

The Fibonacci sequence: an introduction


Posted on July 22, 2007

TheFibonaccisequenceisprobablyoneofthemostfamousandmostwidelywritten-aboutnumber sequencesinallofmathematics.Sowhywriteaboutit,ifitsbeenwrittenaboutsomuchalready?Well,because Ican,andbecauseitsfuntherearegoodreasonsitssopopular!Iplantospendthenextfiveposts(atleast) talkingabouttheFibonaccisequenceanditsconnectionstootherareasofmathematics. TheFibonaccisequenceisdefinedbytherecurrencerelation

InEnglish,thissaysthatthefirsttwoFibonaccinumbersarebothequalto1,andanyFibonaccinumberfrom thethirdonwardscanbeobtainedbyaddingtheprevioustwoFibonaccinumbers.SothethirdFibonacci numberis1+1=2;thefourthis1+2=3;then2+3=5;andsoon.ThefirstfifteenFibonaccinumbersare

Asyoucansee,theygetlargefairlyquickly.The31stFibonaccinumberisthefirsttoexceedonemillion,andthe 45thisalreadybiggerthanonebillion. ThedefinitionoftheFibonaccisequenceisquitesimple,buttheresaLOTmoreherethanmeetstheeye,whichI hopetoexpandoninsubsequentposts.Fornow,wellstartwiththissimpleobservation:whathappensifyou sumalltheFibonaccinumbersuptoacertainpoint?Forexample,letsaddupalltheFibonaccinumbersupto 5:1+1+2+3+5=12.Okay.Howaboutgoingonefurther:1+1+2+3+5+8=20.Ifweinclude13 aswell,weget33.Doyouseeapattern?(Hint:comparethesumswegot(12,20,33)withtheFibonacci sequenceitself.)Doesthisalwayswork,andifso,canyoushowwhy(andifnot,canyougiveanexamplewhere itdoesnt)?Feelfreetopostcomments!


Posted in famous numbers, fibonacci, iteration, pattern, sequences | 11 Comments

More drafts of my book


Posted on June 26, 2007

Asyoumayrecall,Iamwritingamathbook,andIfinallyhavemoredraftsreadyfortest-reading(justclick here)!TherearerevisedversionsofthePrefaceandChapter0,aswellasfirstdraftsofChapter1(ProblemSolvingandProof)andChapter2(Numbers).Asusual,anyandallfeedbackiswelcome!
Follow

189 of 224

FollowTheMathLess Traveled 10/16/12 16:55


Get everynewpost deliveredto your Inbox.

The Math Less Traveled | Explorations in mathema...

http://mathlesstraveled.com/

Bytheway,thedraftsarehostedonanewblogwhichImusingformoreacademic/professional/mostlycomputer-relatedthings,incaseanyonereadingthisisinterestedinthatsortofthingtoo.
Posted in books, meta | 6 Comments

Carnival of Mathematics #9
Posted on June 4, 2007

TheninthCarnivalofMathematicsisupoveratJD2718,includingacoupleofpostsbyyourstruly.PostswhichI thinkmaybeofparticularinteresttoreadersofTMLTinclude: InvestigatinganAlgebraII/SATprobleminabitofdepth SomediscussionofthePrisonersDilemma andTheWasonSelectiontask. Butthereslotsofotherinterestingstufftobefoundaswell.


Posted in links, meta | 2 Comments

Trig sum identities, the easy way


Posted on June 1, 2007

What?!DidIjustuseallofthewordstrig,identitiesandeasyinthesamesentence? Actually,Ididnt,sinceTrigsumidentities,theeasywayisntasentence.Thatsbecauseitnoverb,justlike thissentence. Anyway,rememberthoseangleadditionidentitiesforsineandcosine?

Ifyouveseenthesebefore,youllknowwhatImtalkingabout;ifyouhavent,trustme:theyreveryuseful,but abigpaininthebutttomemorize.Hmm,nowwait,wasthatcosineAsineum,cosineBcosineA somethinglikethatand,uh,cosineistheonewhereyouswitchthesignorwait,wasthatsineItsway tooeasytogetconfused.Andwheredothoseequationsevencomefrom,anyway?Ifyouvelearnedthemin school,chancesaretheyweresimplypresentedtoyouasfact,withoutanyderivation. Totherescuecomesthisreallyniftyderivation:allyouneedtoknowisEulersFormula(which,ifyourecall,is alsowherewegotTheMostBeautifulEquationintheWorldfrom)!Justtoremindyou,Eulersformulasaysthat

Nowletsjustreplace with(A+B)andseewhathappens!

Follow

190 of 224

FollowTheMathLess Traveled 10/16/12 16:55


Get everynewpost deliveredto your Inbox.

The Math Less Traveled | Explorations in mathema...

http://mathlesstraveled.com/

Lookatthat!Wecanjustequatetherealpartsandtheimaginarypartstogetbothidentitiesfromthissingle equation.Nifty,eh? (IgotthisfromTrigWithoutTears,foundviaLetsPlayMath!.)


Posted in proof, trig | 3 Comments

Open problems: Collatz conjecture


Posted on May 8, 2007

Pickanyintegeryoulike.Gotaninteger?OK,nowapplythissimplerule:iftheintegeriseven,dividebytwo; otherwise,multiplybythreeandaddone.Forexample,ifyoustartedwith10,youwouldget5;ifyoustarted with13,youwouldget40.Prettysimple,right?Youcanprettymuchdoitinyourheadforsmallnumbers. (Actually,ifyourewillingtoconcentrateyoucanprobablydoitinyourheadforbignumberstoo!Quick,what wouldyougetifyoustartwith217?) Youmightnotthinktheresanythingveryinterestingtobesaidaboutthisrulebutyouwouldbewrong. Thefunstartswhenweconsideriteratingthisrule.Ivetalkedaboutiterationbefore:itjustmeansrepeatingthe rule.So,forexample,ifwestartwith10,weget5;applyingtheruleagainto5,weget16;applyingitto16 gives8,andsoon.Whathappensifyoukeepgoing?Tryitbeforereadingon! Continuereading

Posted in iteration, open problems, sequences | 33 Comments

Happy birthday, Euler!


Posted on April 15, 2007

Today,April15,2007,marksthe300thbirthdayofLeonhardEuler.Notthathesstillaliveoranything.Euler (rhymeswithspoiler,NOTBueller!)wasthemostprolificmathematicianofalltime,publishingmorethan800 papers(!)inhislifetime.Haveyoueverusedthenumbere?Howaboutthenumberi?Haveyoueverusedthe notationf(x)toindicateafunction?RememberTheMostBeautifulEquationintheWorld? Yeah,thosewereallhim.Andthereareaboutabajillionotherthingsinmaththathediscoveredand/orare namedforhim:Eulercharacteristic,Eulerstheorem(thereareactuallyseveralofthese),Euleridentity,Eulerian path,Euler-Mascheroniconstant,Eulerstotientfunction,andsoon.Hewasonesmartdude. MarkDominusoveratTheUniverseofDiscoursehasamoredetaileddiscussionofafewofthesethings,instead ofrepeatingitIlljustsendyouovertheretoreaditifyoureinterested.=)


Follow

191 of 224

FollowTheMathLess Traveled 10/16/12 16:55


Get everynewpost deliveredto your Inbox.

The Math Less Traveled | Explorations in mathema...

http://mathlesstraveled.com/

Posted in links, people

An even better explanation


Posted on April 14, 2007

AfterIhadalreadypublishedmypreviouspostaboutTheMostBeautifulEquationintheWorld,Icameupwith anevenmoreelegantwaytoexplainthecentralproof.Ifyougotitinyoure-mailorthroughRSS,youmaywant togobackandrereadtheeditedversionespeciallyifitconfusedyouthefirsttimearound!


Posted in meta, proof | 1 Comment

The Most Beautiful Equation in the World


Posted on April 13, 2007

Thispostisaspecialshout-outtomyformerstudentswhoarenowtakingcalculus(ifyoudontknowany calculus,justhangtighttherewillbemorecalculus-lessmathgoodnesscomingyourwaysoon).Thispostis 100%money-backguaranteedtonotreallyhelpyouatallontheAPexam!Butdontworry,itsstillawesome. First,considerthisequation:

Perhapsyouveseenitbeforeamongotherthings,itgivesusaveryneatconnectionbetweencomplex numbersexpressedinpolarcoordinates( isthecomplexnumberwithpolarcoordinates )andexpressed inrectangularcoordinates.Buttoseewherethisequationcomesfrom,youneedsomecalculus! Letsset (remember, ): .Thekeyobservationisthatdifferentiatingzisthesameasmultiplyingitbyi

Sowewritethisobservationasasimpledifferentialequation;afterswapping andzwecanintegrate:

Andsince differentiating gives

isequalto1when iszero,Cmustbeequalto1.Thereforewehave ,aspromised!Notethatthismakessensewithourkeyobservationfrombefore: ,thesameasmultiplyingbyi. .Notingthat ,weendupwiththeMostBeautifulEquation

Nowforareallygoodtime,set intheWorld:

.
Follow

192 of 224

FollowTheMathLess Traveled 10/16/12 16:55


Get everynewpost deliveredto your Inbox.

The Math Less Traveled | Explorations in mathema...

http://mathlesstraveled.com/

Almostmakesyoucry,doesntit?ItrelatesfiveoftheMostImportantNumbersintheWorld(0,1,e,i,and ) usingthreeoftheMostImportantOperationsintheWorld(addition,multiplication,andexponentiation)and nothingelse.Beauty,simplicity,eleganceitsallrighthere. Andforashorttimeonly,thisequationcanbeyoursforthelow,lowpriceofthenumberofgrainsofriceonthe lastsquareofachessboardifyouputjustonegrainonthefirstsquare,twograinsonthesecondsquare,four grainsonthethird,andmerelydoublethenumberofgrainseachtime.Ordernow,suppliesarelimited!


Posted in calculus, famous numbers, proof | 46 Comments

Open problems: Twin prime conjecture


Posted on April 10, 2007

Oops,somuchforpostingonceaweek!MyexcuseisthatIvebeenhardatworkonmybook.Well,nothingto dobutgetrightbackatit.Ipromise*Iwillbebetter**aboutpostingregularly***fromnowon. *IhadmyfingerscrossedwhenItypedthat.Seriously. **Forlowvaluesofbetter. ***Somerestrictionsmayapply. Anyway,IthoughtIwouldbeginanintermittentseriesdescribingsomecurrentlyopen(unsolved)problemsin mathematics.Itsprettyfascinatingthatinmathematics(unlikeinmanyotheracademicdisciplines)itsnottoo hardtocomeupwithquestionsthatareveryeasytounderstand,butincrediblydifficulttoanswer!Itjustgoes toshowthatsometimes,thesimplestquestionsendupbeingthedeepest. Today,Imgoingtotalkaboutthetwinprimeconjecture.Aprimenumber ,asyoumayrecall,isapositiveinteger greaterthan1whichhasnodivisorsotherthan1anditself.Forexample,17isprime,sincethereisnonumber lessthan17thatevenlydividesit,but18isnotprime(itiscomposite)sinceitisdivisibleby,forexample,3.The firstfewprimenumbersarethus2,3,5,7,11,13,17,19,23,29,31,37Primenumbersserveasbuilding blocksforalltheotherintegers,sinceanyintegercanbewrittenuniquelyasaproductofprimes(thisiscalled theintegersfactorization). Letslookatthesequenceofdistancesfromeachprimetothenext:1,2,2,4,2,4,2,4,6,2,6,Seea pattern?Isuredont.Infact,thisisoneofthemostenigmaticthingsabouttheprimenumbers:althoughthere aretheoremsdescribingapproximatelyhowoftenprimesoccur,thereisnoneatpattern(thatanyoneknowsof) whichdescribesexactlywhenprimeswilloccur.Asyougothroughbiggerandbiggerintegers,primestendtoget rareryetthereisstillnopattern.Onaverage,thehigheryougo,thefartheraparttheprimeswillbeyet everynowandthen,primesstilloccurrightnexttoeachother,like29and31.Pairsofprimeswhichareonly twoawayfromeachotherarecalledtwinprimes.Largerexamplesoftwinprimesinclude101and103,281and 283,andthewhopping

agargantuanpairoftwinprimeswith58711digitseach!Thisisactuallythelargestknownpairoftwinprimes, discoveredjustthisyear,onJanuary15.So,heresthequestion:arethereinfinitelymanypairsoftwinprimes,
Follow

193 of 224

FollowTheMathLess Traveled 10/16/12 16:55


Get everynewpost deliveredto your Inbox.

The Math Less Traveled | Explorations in mathema...

http://mathlesstraveled.com/

justlikethereareinfinitelymanyprimenumbers?Ordotheprimeseventuallygetsofewandfarbetweenthat thetwinprimesstop?Nooneknows!Thetwinprimeconjecture(aconjectureislikeamathematicalguess) statesthatthereareaninfinitenumberoftwinprimesmostmathematiciansbelievethatitstrue,butnoone hasbeenabletoproveit. Achallengetoconclude:wecoulddefinetripletprimesasasetofthreeconsecutiveoddnumbersp,p+2,and p+4whichareallprime.Thiswouldntbeaveryusefuldefinition,however,since3,5,7istheonlysetoftriplet primes.Why? YoucanlearnmoreabouttwinprimesandthetwinprimeconjectureatMathWorld.


Posted in challenges, famous numbers, infinity, primes | 3 Comments

Happy pi day!
Posted on March 14, 2007

Happy day!Today,March14,iscelebratedas day,sincewhentodaysdateiswritteninAmericanformat (3/14)itcorrespondswithacommonapproximationof (3.14).(IdontknowwhenEuropeanscelebrate day. [Edit:AccordingtoJester(seebelow),EuopeanscelebrateitonJuly22--22/7isanothercommon approximationof .]) ,ofcourse,istheratiobetweenanycirclescircumferenceanddiameter,andshowsupall overtheplaceinmathematics(notonlyingeometry).Asyouprobablyknow,itisntequalto3.14,or22/7;its actuallyirrational,whichmeansthatitcantbeexpressedasafraction,anditsdecimalrepresentationcontinues onforeverwithoutrepeating. 3.14159265358979323846264338327950288419716939937510582097494459230781640628620899 86280348253421170679821480865132823066470938446095505822317253594081284811174502 84102701938521105559644622948954930381964428810975665933446128475648233786783165 27120190914564856692 Incelebrationofthisamazingnumber,herearesomefun factsandlinksforyou!

Thefirst1milliondigitsofpi. Searchthedigitsofpi. Findyournameinpi!


Follow

194 of 224

FollowTheMathLess Traveled 10/16/12 16:55


Get everynewpost deliveredto your Inbox.

The Math Less Traveled | Explorations in mathema...

http://mathlesstraveled.com/

Makepiintomusic. CadaeicCadenza:anincrediblyamazingpieceofconstrainedwriting. ReadaboutpionMathWorld. ReadaboutpionWikipedia. Edit:DeniseofLetsplaymath!hasafewmoreinterestingpi-relatedlinksandquotes,includingafewexplaining howtocalculatepibythrowinghotdogsonyourkitchenfloor.Imnotkidding,gocheckitout.


Posted in famous numbers | 6 Comments

The Binomial Theorem


Posted on March 9, 2007

TheBinomialTheoremisanextremelyimportantandgeneral(andtotallysweet)resultinthefieldof combinatorics(whichisthebranchofmathematicsaboutcountingthings).Withoutfurtherado,hereitis:

Wait!Dontletallthefancysymbolsandvariablesscareyou;itsactuallynottoocomplicated.Gobackandread itagain,carefully,andthenletsbreakitdownabit.Itsaysthatthenthpowerofthebinomial theoremgetsitsname)canbefoundbyasum( )whichstartswith sumiscomposedofthenextbinomialcoefficient,onefewerx,andonemorey.Forexample: (whencethe ,andeachsubsequenttermofthe

(Recallthatanythingtothezeropoweris1.)Makesense?Anotherwaytothinkaboutitisthat expanded,consistsofanumberoftermsoftheform withoutcollectingliketerms;forexample: thetermsaredeterminedbytheentriesinarowofPascalsTriangle.Whyisthistrue?Expandout

,when

,whereaandbalwaysaddupton;thecoefficientsof

Ingeneralyougetatotalof terms,onecorrespondingtoeachpossiblewaytochooseastringofnxsandys. Everytermwithexactlykxs(andtherefore(n-k)ys)canbecombined,since,forexample, thenumberofsuchtermsisthenumberofwaystochoosekxsoutofnletters.Thus,theBinomialTheorem! Now,whyistheBTsosweet?Hereareafewreasons,intheformofproblemsforyoutotry: 1. Provethebinomialcoefficientsumequationfromlasttimeinyetanotherway(hint:expand BinomialTheorem). 2. Proveanotherinterestingbinomialcoefficientidentitybyexpanding 3. Compute , , , , happenswhenyoucompute ?Canyouthinkofawaytofixit? . .Noticeanythinginteresting?Whydoesthathappen?(Hint:11=10+1.)What
Follow

usingthe

195 of 224

FollowTheMathLess Traveled 10/16/12 16:55


Get everynewpost deliveredto your Inbox.

The Math Less Traveled | Explorations in mathema...

http://mathlesstraveled.com/

4. AccordingtotheBT,whatis

?Doesthatlookfamiliar?

5. Ifyouknowsomecalculus,usetheBTandthedefinitionofthederivative

toprovethepowerruleforderivatives:

Posted in counting, pascal's triangle, pattern | 2 Comments

Challenge #9 Solution
Posted on March 4, 2007

InWhichOurHero(You)DiscoversSeveralMethodsofProvingaCombinatorialIdentityInvolvingPascals Triangle (ReadChallenge#9firstifyouhaventalready) Continuereading

Posted in counting, pascal's triangle, proof, solutions | 4 Comments

More fun with Pascal's triangle (Challenge #9)


Posted on February 27, 2007

RememberPascalstriangle?

1 11 121 1331 14641 15101051 1615201561 172135352171 18285670562881

Itschock-fullofamazingpatterns(bigsurprise).IthinkIllspendapostortwo(orthree?)lookingatafewof them. Firstup:asnotedbySteveGilberg,addingupthenumbersineachrowrevealsaninterestingpattern:1=1; 1+1=2;1+2+1=4;1+3+3+1=8;1+4+6+4+1=16amazing!Theentriesineachrowaddupto successivepowersoftwo! Asasidenote,letmetellyouastory.Considerthepolynomialfunction

Follow

196 of 224

FollowTheMathLess Traveled 10/16/12 16:55


Get everynewpost deliveredto your Inbox.

The Math Less Traveled | Explorations in mathema...

http://mathlesstraveled.com/

Gosubstituteallthevaluesfrom1to6intoitforx,andtellmewhatyoufind.Thingsarentalwayswhatthey seem. OK,sorry,thatwasntreallyastory,sosueme.GettingbacktoPascalstriangle,it*ahem*appearsthatthe entriesineachrowadduptosuccessivepowersoftwo.Atleast,theydoforthefirsteightorninerows.Butdo theyalways?Simplyobservingthisapparentpatternisagreatstartbutitsnotenough. Anotherwayofexpressingthepatternwevenoticed,intermsofbinomialcoefficients:itseemsthat

forallintegers

.(Heresanexplanationofsigmanotationifyoudontknowwhatthe means.)

Iwontkeepyouinsuspense;itturnsoutthatthisistrue.Butcanyoufigureoutwhy?Postyourexplanations here!Iknowofatleasttwo,onehavingtodowiththewayPascalstriangleisdefined(intermsoftheadditionrule),andonehavingtodowiththewaybinomialcoefficientsaredefined.Butthereareprobablymore.
Posted in challenges, pascal's triangle, pattern, proof | 25 Comments

Carnival of Mathematics
Posted on February 24, 2007

MymostrecentpostabouttetrahedralnumbersisincludedinthisweeksissueoftheCarnivalofMathematics, whichcanbefoundhere,inMarkChu-Carrollsblog,GoodMath,BadMath.(Justsoyouarentmisledinto thinkingthisissomesortofhonor,itsincludedbecauseIsubmittedit.=)TheCarnivalofMathematicsisa biweeklycompendiumofblogpostsrelatedtomathematics,witheachissuehostedonadifferentblog.This weeksissueincludespostsonawiderangeoftopics,withawiderangeofdifficultylevels,sotheressomething foreveryone.Gocheckitout!


Posted in links

Tetrahedral numbers, exposed!


Posted on February 20, 2007

Andnow,thetriumphantterminationtothetantalizingtaleoftetrahedraltotals! Tosumupsofar:webeganbyobservingthatthenumberofgiftsinthepopularsongTheTwelveDaysof Christmascanbedescribedbytetrahedralnumbers.Unsatisfiedwithaddinguptetrahedralnumbersbyhand,we firstexperimentedwithusingacomputertocalculatethemforus,butfoundeventhatmethodhaditslimits.In searchofthewayofcunningratherthanforce,wefirstdevelopedaformulaforcalculatingthesimplertriangular numbers,thenmadeaseeminglyunrelateddetourtodiscussbinomialcoefficients,andsawanamazing connectionbetweenbinomialcoefficientsandPascalstriangle.Itmightnotbeobvious,butwereincredibly closetoourgoalthepiecesareinplace;allthatremainsistoputthemtogether. ThefirstthingtoclearupiswhyonearthbinomialcoefficientshaveanythingtodowithPascalstriangle. Remember,thebinomialcoefficient
Follow

197 of 224

FollowTheMathLess Traveled 10/16/12 16:55


Get everynewpost deliveredto your Inbox.

The Math Less Traveled | Explorations in mathema...

http://mathlesstraveled.com/

tellsushowmanywaystherearetochoosekthingsoutofn.Aswesawlasttime,ifwemakeatableof binomialcoefficients(withndownthesideandkalongthetop),itisexactlythesameasPascalstriangle,which isformedbyasimpleruleofaddition:togeteachnewentryinthetable,addtheentryaboveitandtheone aboveandtotheleft.

1 11 121 1331 14641 15101051 1615201561 172135352171 18285670562881

ItsclearwhytheleftandrightsidesofPascalstrianglearemadeofall1 s,anditsclearwhythecorresponding binomialcoefficientsarealsoequalto1(theyareoftheform additionrulethatcreatesPascalstriangle? LetstranslatetheadditionruleofPascalstriangleintothelanguageofbinomialcoefficients.Theadditionrule saysthattogetanyentryinPascalstriangle,youaddtheentryaboveitandtheoneaboveandtotheleft;ifa particularentryisgivenby theadditionrulesaysthat ,thentheonesaboveitandaboveandtotheleftare and respectively.So or theresonlyonewaytochoose nothing,oreverything).Sotherealquestionis,whydobinomialcoefficientsobeythesamekindofsimple

Whyisthistrue?Theresactuallyasimpleexplanation.Supposeyouwanttochoosekthingsoutofntotal things.TakethelastofthenthingsandcallitThingZ.(ZforZucchini.Yes,zucchiniisafruit.)Clearly,youhave exactlytwochoiceswhenitcomestoThingZ:youcanincludeThingZinthekChosenThings,ornot.Ifyou dontincludeit,thenyoustillhavetochoosekthings,butnowyouonlyhaven-1totalthingstochoosefrom,so youhave waystodoit.Ontheotherhand,ifyoudoincludeThingZ,thenyouonlyhavetochoosek-1 waystodothat.Sincethesearetheonly thingsfromtheremainingn-1tocompleteyourselection;thereare

twopossibilities,andtheyobviouslydontoverlap,thebinomialadditionidentityisborn!Neato![Youcanalso provethisidentitybyresortingtothedefinitionofbinomialcoefficientsintermsoffactorials;alittlemassaging ofdenominators,alittlefactoring,andheypresto!I'llleavethatasanexercisefortheinterestedreader.] Cool,nowweknowwhyPascalstriangleisjustbinomialcoefficientsindisguise(orviceversa).Butwhatdoes allofthishavetodowithtetrahedralnumbers?Well,doyourememberwhatthefirstfewtetrahedralnumbers are?Letsseethefirstfewtriangularnumbersare1,3,6,10,15,21whichwouldmakethefirstfew tetrahedralnumbers1,4,10,20,35,56right?NowgobackandstareatPascalstriangle. Doyouseeit?!WecanjustreadoffthetetrahedralnumbersfromthethirdcolumnofPascalstriangle!


Follow

198 of 224

FollowTheMathLess Traveled 10/16/12 16:55


Get everynewpost deliveredto your Inbox.

The Math Less Traveled | Explorations in mathema...

http://mathlesstraveled.com/

(Remember,theleftmostcolumnisthezerothcolumn.)Forthatmatter,thetriangularnumbersareinthe secondcolumn.(Andyes,thefourthcolumnisthepentatopenumbers,apentatopebeingthefour-dimensional analogofatetrahedron;pentatopenumbersarecreatedfromsumsoftetrahedralnumbers.Andthefifthcolumn is)Itsnottoohardtoseewhythisisso:thinkabouttheadditionrule,andtheprogressionfromacolumnof all1 s,toacolumnofthecountingnumbers,toacolumnoftriangularnumbers,toacolumnoftetrahedral numbers(ifyouhavetroublefiguringoutwhythisworks,leaveacommentandIllexplaininmoredetail). SothetetrahedralnumbersarejustthenumbersinthethirdcolumnofPascalstriangle.Butheresthe punchline:wealreadyknowhowtocomputetheentriesofPascalstriangle,usingbinomialcoefficients!A momentsthoughtrevealsthatthenthtetrahedralnumbercaninfactbegivenby (the+2 isbecausethe firsttetrahedralnumberisactuallyfoundinthethirdrow).Wecanexpandthisabitifwewant:

Woohoo!Letstryit:

Itworks!Ifyoutrycalculating calculating toyou!

youllget

justlikewefoundbefore.AndIllleavethefunof

Posted in counting, famous numbers | 8 Comments

Binomial coefficients and Pascal's triangle


Posted on February 17, 2007

Inapreviouspost,Iintroducedbinomialcoefficients,andwesawthattheycanbegivenbytheformula

Letsmakeatableofbinomialcoefficientvaluesthatis,wellmakeatablewhereyoucanlookuparow correspondington,acolumncorrespondingtok,andfindthevalueof before. attheintersection.Itsnottoohard (andprobablyagoodexercise)todobyhandforsmallvalues,butfornowIlluseJ,whichIvewrittenabout

Follow

199 of 224

FollowTheMathLess Traveled 10/16/12 16:55


Get everynewpost deliveredto your Inbox.

The Math Less Traveled | Explorations in mathema...

http://mathlesstraveled.com/

|:!/~i.12 100000000000 110000000000 121000000000 133100000000 146410000000 15101051000000 161520156100000 1721353521710000 18285670562881000 19368412612684369100 11045120210252210120451010 1115516533046246233016555111

Notethatthefirstrowandcolumnofthetablecorrespondton=0andk=0,respectively.Soifwewantedto knowthevalueof,say, ,wewouldlookinthefourthrowandthesecondcolumntofind3,asexpected.Ifwe wantedtoknowthevalueof ,wewouldlookinthefifthrow,thirdcolumnsureenough,its6. Noticethatthefirstcolumn(correspondingtok=0)containsall1 s.Doesthatmakesense?Suretheres alwaysonlyonewaytochoosenothing!Noticealsothatalltheentriesintheupper-rightarezero.Theseare placeswherek>n.Thismakessensetoo,since,forexample,theresnowaytochoose7thingsifyouonlyhave 3options.Onemoreinterestingthingtonoteisthateachrowofthetableissymmetric.Forexample, and .Thismakessenseifyouthinkaboutitchoosingfourthingsoutofsix(forexample)isthesame aschoosingthetwothingsoutofsixthatyourenotgoingtochoose! Nowletsdosomethingcompletelyunrelated.Getoutapieceofpaper,oropenNotepad,orwhatever.Were goingtowritedownabunchoflinescontainingnumbers.Startbywritingdownasingle1byitselfonthefirst line.Nowforeachnewlineafterthat,startbywritinga1inthefirstcolumn,andthenforeachsubsequent columnwriteanumberwhichisthesumofthenumberaboveitandthenumberaboveandtotheleft(ifthere isnonumberabove,treatitaszero).Remember,thisiscompletelyunrelatedtobinomialcoefficients.Sothe firsttworowswouldlooklikethis:

1 11

Prettyexciting,Iknow.Thenextrowwouldbe121writedownaninitial1,then1+1is2,then1+0is1. Makesense?Keepgoingforafewrows.Oh,anddidImentionthatthisiscompletelyunrelatedtobinomial coefficients? Soifyoukeepgoingforawhile,yougetsomethingthatlookslikethis:

Follow

200 of 224

FollowTheMathLess Traveled 10/16/12 16:55


Get everynewpost deliveredto your Inbox.

The Math Less Traveled | Explorations in mathema...

http://mathlesstraveled.com/

1 11 121 1331 14641 15101051 1615201561 172135352171

ThisisknownasPascalsTriangle,namedfortheFrenchmathematicianandphilosopherBlaisePascal.Keepin mindthatPascalsTrianglehasabsolutelynothingwhatevertodowithbinomialcoefficients. Exceptforthefact,whichyouhaveprobablyguessedbynow,thatIamahorribleliar.Egads! Atthispointyouprobablyhaveanumberofburningquestions.Whyisthetableofbinomialcoefficientsthe sameasPascalstriangle?Whatonearthdoesthishavetodowithtetrahedralnumbers?WhydoItoywithyour mindso?Tuneinnexttimefortheexcitingconclusion!


Posted in counting, famous numbers | 8 Comments

Challenge #8 solution
Posted on February 16, 2007

inwhichyouwereaskedtofindaformulafor Continuereading

,thenthtriangularnumber.

Posted in famous numbers, solutions | 6 Comments

Binomial coefficients
Posted on February 14, 2007

TodayImgoingtotalkaboutsomethingprettyniftycalledbinomialcoefficients.Itmaynotbeimmediately apparenttoyouwhytheyresonifty,buttakemywordforit:theyshowupallovertheplaceinmathematics! [Andyes,ifyou'rewondering,we'restillonourwaytowardsaformulafortetrahedralnumbers.Itwillbecome apparentwhatbinomialcoefficientshavetodowithitlater.] Ingeneral,abinomialcoefficientlookslikethis: .Youpronouncethatasnchoosek,sincethesimplestway

tounderstandthisbinomialcoefficientisthatittellsyouhowmanywaystherearetochoosekthingsoutofn possiblechoices.Thisisknownasthenumberofcombinations.(Onmanycalculators,youenterbinomial coefficientsusingakeyorfunctionlikenCrCforCombinations.)Thisdefinitionisenoughtoletusfigureout whatsomebinomialcoefficientsareforsmallvaluesofnandk.Forexample,ifIhavethreethings(say,aguava, astarfruit,andamango)andtellyoutochooseone,howmanydifferentchoicesdoyouhave?Obviously,three. So .HowaboutifIhavefourthings(say,anApple,aBanana,aCherry,andaDate)andaskyoutochoose anytwohowmanywayscanyoumakeyourchoice?Ifweusethefirstletterofeachfruitasanabbreviation, youhavesixchoices:AB,AC,AD,BC,BD,orCD.(Noticethattheorder inwhichyoupickthetwofruitsdoesnt


Follow

201 of 224

FollowTheMathLess Traveled 10/16/12 16:55


Get everynewpost deliveredto your Inbox.

The Math Less Traveled | Explorations in mathema...

http://mathlesstraveled.com/

matter:ABisthesameasBA.)So,

Whenwestartgettingintobiggernumbers,however,thisgetscumbersome:howcanwebesurethatwehavent missedapossiblechoice?Whatifthenumberofchoicesissolargethatitwouldtakeustenyearstocount them?Whatweneedisaformulawhichcantellusthevalueof directly.

Togetthere,letsstartwithsomethingabitsimpler.Sayyouhavendifferentfruits(anApple,aBanana,a Cherry,aDate,aumanEgguh).Howmanywayscanyoulinethemupinarow,puttingoneparticular fruitfirst,onesecond,andsoon?Imagineplacingthemonebyone.Forthefirstfruit,youcanchooseanyone ofthem,soyouhavenchoices.Forthesecondfruitintheline,youcannolongerchoosetheonethatsfirst,so youonlyhave(n1)choices.Thenforthethirdfruityouhave(n2)choices,andsoon,untilforthelastfruit youonlyhaveonechoice,sinceitstheonlyoneleft.Intotal,then,youhave haveonlythreefruits,youhave Forfourfruits,thereare aswell.=) Nowforanother,relatedquestion:supposeyoutakeyournfruits,andyouwanttolinethemupasbefore,but youonlywanttomakealineofkfruitsoutofthetotaln?Again,youhavenchoicesforthefirstfruit,(n1) choicesforthesecond,andsoon,exceptthistimeyoustopafterthekthfruit,forwhichtherewillbe(nk+ 1)choicessothetotalnumberofwaystodothisis : .Noticewecanwritethisas waysof puttingyourfruitsinorder.(Theexclamationpointrepresentsthefactorialfunction.)So,forexample,ifyou waysofputtingtheminorder:ABC,ACB,BAC,BCA,CAB,CBA. waysofputtingtheminorder.Iwontbotherwritingthemallout

here,butfeelfreetoverifythisforyourself.Itshouldbeobviousthatthisargumentextendstonon-fruitobjects

Forexample, seethisfunctioncallednPr.

.Whatweredoinghereiscountingpermutations;oftenoncalculatorsyouwill

Werealmostthere.Wenowhaveawayofcountingthenumberofwaystochoosekthingsoutofatotalofn, whenweconsidereachpossibleorderofthekthingstobedifferent.Butreallywhatwewantisawaytocountthe numberofwaystochoosekthingsoutofnwhentheorderdoesntmatter.Startingwithwhatwehave (countingeachorderseparately),itsclearthatwerecountingeachpossiblegroupofkthingsmultipletimes, oneforeachpossibleorder.Well,howmanywaysaretheretoputkthingsinorder?Yougotit,k!.Sowere countingeachgroupofkthingsk!timeswhenwereallyonlywanttocountitonce.Well,thatseasytofix:just divideeverythingbyk!.Here,finally,isourformulaforfindingabinomialcoefficient:

Letschecktomakesurethisformulasquareswiththesmallexampleswedidbefore. ,check! HmmIwonderhowmanywaystherearetochoose4thingsoutof10possibilities?Letssee:

,check!

.Nowyouknow.Isurewouldnthavewantedtocountallthosepossibilitiesby

Follow

202 of 224

FollowTheMathLess Traveled 10/16/12 16:55


Get everynewpost deliveredto your Inbox.

The Math Less Traveled | Explorations in mathema...

http://mathlesstraveled.com/

hand! Nowholdontoyourhatsnowthatweknowthebasics,nexttimeIllshowyousomethingtrulyamazing aboutbinomialcoefficients!


Posted in computation, counting, famous numbers | 6 Comments

Triangular number formula (Challenge #8)


Posted on February 12, 2007

Morehostingissuesrecentlysigh.ImseriouslyconsideringmovingTMLTtoadifferenthostingproviderthe onlyproblembeingthatrightnow,myhostingisfree,anditmightbehardtofindanotherhostwhichprovides allthetoolsIneed(LaTeX,imagemagick,PHP,andmySQL,forastart). Inothernews,giventhedismalfrequencyofmypostsrecently,Ivedecidedtotrystickingtoapost-a-week schedule.Wellseehowitgoes! Asyoumayrecall(youprobablydont),Iminthemiddleofaseriesofpostsbuildinguptoawayofcalculating tetrahedralnumbersbyhand.Thatsstillwherewereheaded,butitwillprobablytakeafewmorepoststoget there!Today,welltakeasmallstepintherightdirectionbyderivingasimplerformulaforcalculatingtriangular numbers. Recallthatthenthtriangularnumber(whichwellcall )isthesumofthefirstnpositiveintegers:

[IfthebigE-lookingthingscaresyou,don'tworry.It'sjustacompactwayofrepresentingsums.Ifyou'venever seenitbefore,here'sashortexplanation.] Whatifyouwereaskedtoaddupallthenumbersfrom,say,1to100?(Putdownthatcalculator!)Youcould probablydoitbybruteforce,butitwouldtakeawhile,andyoudprobablymakeamistake.(Oh,youwouldnt, eh?Well,excuseme,Mr.perfectpants.)LegendhasitthattheyoungGausswasonceaskedtodojustthatbya schoolteacherwhopresumablywantedtogettheirstudentstobequietforanhourortwo.Gauss,however, stunnedhisteacherbyimmediatelywritingdownthecorrectanswer,havingcomputeditinhishead. So,howdidhedoit?Nooneknowsforsurewhatwentthroughhishead,ofcourse,butitmighthavegone somethinglikethis.FirstletswritedownthesuminquestionandcallitS :

Andnowforsomethingstrange:letswriteS again,butbackwardsthistime.Ofcourse,sinceanysumisthe samebackwardsasforwards,thisisobviouslypointlessorisit?Theinspirationhitswhenweaddthesumto itself:

Follow

203 of 224

FollowTheMathLess Traveled 10/16/12 16:55


Get everynewpost deliveredto your Inbox.

The Math Less Traveled | Explorations in mathema...

http://mathlesstraveled.com/

Lookatthat!MatchingupthefirsttermofS withthelastterm,thesecondtermwiththesecond-to-lastterm, andsoon,formspairsoftermswhicheachaddto101.Howmanyofthesepairsarethere?Thatseasy;thereare 100.Werealmostthere:

Voila!Thesumofthenumbersfrom1to100is5050. Now,asachallengeforyou,Illletyouusethistechniquetofindageneralformulaintermsofnwhichcantell usthesumoftheintegersfrom1tonthatis,thenthtriangularnumber, Ontothenextpostintheseries!


Posted in challenges, computation, famous numbers, sequences | 12 Comments

Book excerpt
Posted on January 23, 2007

Lately,IvebeenhardatworkonthebookImwritingandIfinallyhavesomethingtoshowforit!Youcan downloadthefirstchapterhere(318KPDF).Ifyoudliketotakethetimetoworkthroughitandgiveme feedback,Iwouldbemostgratefulbutyoushouldalsofeelfreetojustcheckitoutforfun.


Posted in books | 9 Comments

Computing tetrahedral numbers


Posted on December 22, 2006

Ileftoffthediscussionoftetrahedralnumbersinmylastpostbyposingthequestionofwhatthe144th tetrahedralnumberis,andpromisingtoexplainaquickwaytofigureitoutwithouthavingtoactuallyaddup thefirst144triangularnumbers.Istillintendtodothatbutfirst,Illdemonstratehowwecanuseacomputer tocalculateitforus.Althoughitwouldbeincrediblytediousanderror-proneforahumantocalculateandadd upthefirst144triangularnumbers,anymoderncomputercandoitinafractionofasecond(minecandoitin aboutthreemilliseconds). IllusetheprogramminglanguageJ,whichiscompletelyfreeandisgreatforplayingaroundwithmath.Ifyouve neverprogrammedacomputerbefore,dontworryJisntmuchlikeotherprogramminglanguages;althoughit doeshavealotofadvancedfeatures,itsprettyeasyandintuitivetogetstarted.Itmostlyjustactslikeareally sophisticatedcalculator.Ifyouwanttotrydownloadingittoplayaroundwithyourself(goahead,itsfun!),the websiteisjsoftware.com;heresalinktothedownloadpage.Somebooksandreferencematerialscanbefound here(Ihaventreadmostofthem,soIcantvouchforthedifficultylevel/quality.) Atitscore,Jactsjustlikeacalculator:youtypeinexpressions,anditevaluatesthem.Inallofthefollowing examples,anythingindentedthreespacesiswhatyoutype,andanythingnotindentedistheanswerprintedby J.
Follow

204 of 224

FollowTheMathLess Traveled 10/16/12 16:55


Get everynewpost deliveredto your Inbox.

The Math Less Traveled | Explorations in mathema...

http://mathlesstraveled.com/

3+5 8 2-9 _7 9*8*5 360 8%5 1.6 !5 120 !20 2.43290200818e18 !20x 2432902008176640000

Obviously,Jcandoaddition,subtraction,multiplication,anddivision.Itcanalsodosuchthingsasfactorial(!). (Ofcourse,itcandotonsofotherthingstoolikeexponentiation,sineandcosine,andderivatives,toname justafew.Heresacompletelist,justtoboggleyourbrain.)Afewthingstonoteabouttheaboveexamples: first,Jusesanunderscoretorepresentnegativenumbers(e.g._7),andapercentsigntorepresentdivisiona bitdifferentthanyourcalculator,buteasytogetusedto.Noticealsothatusinganxafterthe20intheabove examplemadeJcalculate20factorialexactly,insteadofusingscientificnotation. Okay,sohowarewegoingtouseJtocalculatethe144thtetrahedralnumber?Illstartbyillustratingthe processofcalculatingthe12thtetrahedralnumber,andwecaneasilyextendit.Wellstarthere:

i.12 01234567891011

Thei.functioncreatesalistofintegersfromzerotoonelessthanthenumberyougiveit.Sotypingi.

12prints

allthenumbersfromzeroto11.Okay,butwhatwereallywantisallthenumbersfrom1to12,not0to11.No problem:wellusethe>:function,whichaddsonetowhatevernumberyougiveit.Aswithmostfunctionsthat operateonnumbers,ifyougiveitalistofnumbers,itwillsimplyoperateoneachnumberinthelistseparately.

>:i.12 123456789101112

Nowwehavethefirsttwelvewholenumbers.Butwhatwereallywantisthefirsttwelvetriangularnumbers,so wecanaddthemup.Recallthattriangularnumbersareformedbyaddingupconsecutivewholenumbers(1, 1+2,1+2+3,andsoon). Heresastart:

+/>:i.12 78
Follow

205 of 224

FollowTheMathLess Traveled 10/16/12 16:55


Get everynewpost deliveredto your Inbox.

The Math Less Traveled | Explorations in mathema...

http://mathlesstraveled.com/

Whathappened?Youmayrecallthat78isthetwelfthtriangularnumber.The+/addedupourlistofnumbers from1to12!Ingeneral,puttingafrontslashafterafunctioncausesthatfunctiontobeappliedthroughoutan entirelist.Forexample,+/ likethis:


1 2 3 4 5isthesameas1 + 2 + 3 + 4 + 5.Asanotherexample,wecoulddosomething

*/>:i.12 479001600 !12 479001600

Apparently,479001600istheproductofallthenumbersfrom1through12aswecanconfirmbyfindingthe factorialof12directly.Anyway,backtotriangularnumbers.Weusedthelistofnumbersfrom1to12tofind the12thtriangularnumber;butwhatwereallywantisallthetriangularnumbersfromthefirsttothe12th! Well,theresaneasywaytodothattoo.Ifyouputabackslashafterafunction,itcausesthefunctiontobe appliedtoalltheprefixesofalist.Thatis,itwillbeappliedtothefirstthinginthelist;thentothefirsttwo thingsinthelist;thentothefirstthreethingsandsoon.Forexample:

+/\>:i.12 136101521283645556678

Gotthat?The>:

i. 12part,ofcourse,generatesthelistofnumbersfrom1to12.Weknowthat+/meanstoadd

upallthenumbersinalist.Sowriting+/\meansaddupthefirstnumberinthelist;thenaddupthefirsttwo numbersinthelist;thenaddupthefirstthreenumbersinthelist;andsoon.Soweget1,1+2,1+2+3,the firsttwelvetriangularnumbers! Werealmosttherethenthtetrahedralnumber,ofcourse,isjustthesumofthefirstntriangularnumbers.So nowthatweknowhowtocreatealistofthefirstntriangularnumbers,allwehavetodoisaddthemup(using


+/again,ofcourse):

+/+/\>:i.12 364 +/+/\>:i.144 508080

Obviouslywecanjustreplacethe12with144tofindthe144thtriangularnumber508,080! Fun,huh?Eventhismethodhasitslimits,thoughusingJtocomputethenthtetrahedralnumberactually causesyourcomputertostorealistofthefirstnwholenumbersinitsmemory,whichisfineforvaluesofnlike 12or144.Butforvalueslikeonemillion,ittakesawhile,andforsomethinglikeonetrillion,itwouldberather hopeless.Butneverfearwithabitofslickmathematics(isntitall?),youllsoonknowhowtocomputethe onemillionthtetrahedralnumberforyourselfusingpencilandpaper*! *(Whyyoudeverneedtodothis,Ihavenoidea.Butshush,thatsnotthepoint.)


Follow

206 of 224

FollowTheMathLess Traveled 10/16/12 16:55


Get everynewpost deliveredto your Inbox.

The Math Less Traveled | Explorations in mathema...

http://mathlesstraveled.com/

Posted in computation, famous numbers, geometry, sequences | 3 Comments

The Twelve Days of Christmas and Tetrahedral Numbers


Posted on December 20, 2006

(Edit,12/5/07:Iguessitsthattimeofyearagain,isntit?=)Ivestartedtoseealotmoretraffictothispostinthe lastfewdays.ForanyonefindingmethroughaGooglesearch,Iwantedtonotethatthiswasactuallythefirstpostin aseven-partseries,andyoucanreadtherestoftheserieshere:computingtetrahedralnumbers,triangularnumbers, binomialcoefficients, asolutiontothetriangularnumberchallenge,pascalstriangle,andtheconclusiontyingeverythingtogether.) OnthefirstdayofChristmas,mytruelovegavetome ThustheopeningwordstothepopularChristmassong.Incaseyoudontknowit(hey,itspossible),itconcerns anindividualwhorecounts,inobsessivedetail,acatalogueofgiftsreceivedfromtheirtruelove.Itisamatter ofsomespeculationwhetherthetruelovebitisintendedassarcasm,forreasonsweshallsoonsee.Atany rate,oneachofthetwelvedaysofChristmasthetruelovegivesthenarratorallthesamegiftsasonthe previousday,plusanewgiftiteminaquantitydeterminedbythenumberofdayssincethestartofChristmas. Forexample,onthefifthday,thenarratorreceivesfivegoldenrings,fourcallingbirds,threefrenchhens,two turtledoves,andapartridgeinapeartree.(Orfiveonionrings,fourcallingcards,threefrenchfries,twoturtle necks,andanMCHammerCD,dependingonwhichversionofthesongyouknow.)Thissongraisesseveral interestingquestions,includingWhattheheckisaturtledove?andWhodoesthat?Moretothepointofthis post,however,italsoraisesthequestion,So,howmanygiftsISthat!? Goodquestion,Imgladyouasked!Letsstartbythinkingabouthowmanygiftsaregivenoneachday.Onthe firstday,thenarratorreceivesonegift:apartridgeinapeartree.Onthesecondday,thenarratorreceivestwo turtledovesandapartridgeinapeartree:2+1=3giftsintotal.Onthethirdday,thereare3+2+1=6 gifts,onthefourthday,4+3+2+1=10gifts,andsoon.Ingeneral,itsnothardtoseethatonthenthday, thenarratorreceivesanumberofgiftsequaltothesumofalltheintegersfrom1ton.Sothenumberofgifts thenarratorgetsoneachdayare1,3,6,10,15,21,28,36,45,55,66,78. Thesenumbersareknownastriangularnumbers,duetothefactthattheycanberepresentedpictoriallybydots arrangedintriangles.Likethis:

Itshouldntbetoohardtoseethatthenumberofdotsinthenthtriangleabovecorrespondstothesumofthe integersfrom1throughn(justcountthenumberofdotsineachrow).Canyoufindapatternandcomeupwith awaytoquicklyfigureoutthenthtriangularnumber,withouthavingtoaddupallthenumbersfrom1ton? (Moreonthisinthenextpost.)


Follow

207 of 224

FollowTheMathLess Traveled 10/16/12 16:55


Get everynewpost deliveredto your Inbox.

The Math Less Traveled | Explorations in mathema...

http://mathlesstraveled.com/

Ofcourse,werenotultimatelyinterestedinhowmanygiftsthenarratorreceivedoneachdaybyitself;wewant toknowhowmanygiftsthenarratorgotintotal.Well,afterthefirstdaythenarratorhadonegift;afterthe secondday,1+3=4gifts;afterthethirdday,1+3+6=10gifts;andsoon.Ingeneral,thetotalnumberof giftsafterthenthdayisjustthesumofthefirstntriangularnumbers:1,4,10,20,35,56,84,120,165,220, 286,364. Theseareknownastetrahedralnumbers,sincetheycanberepresentedpictoriallybydotsarrangedintetrahedra (i.e.,triangularpyramids).Likethis:

Doyouseewhy?Eachlayerofatetrahedronisatriangleofdotsrepresentingatriangularnumber:henceeach tetrahedronisasumoftriangularnumbers.Thetwelfthtetrahedralnumber,364,tellsushowmanygiftsour narratorreceivedintotal.364giftsisalot.Andwithallthosegeeserunningaroundlayingeggseverywhere,the continualracketfromthedrummers,andsoon,ImnotsureIdreallywantthem. So,wevelearnedabouttriangularnumbersandtetrahedralnumbers,andthatthetotalnumberofgiftsreceived bythenarratorofthesongcorrespondstoatetrahedralnumber.Butwhatiftherewere144daysinChristmas insteadof12?Howmanygiftswouldourintrepidnarratorreceivethen?Easy,yousay,itsthe144thtetrahedral number!Well,true,butwhatisthatnumber?Wedontyetknowaquickwaytofigurethatoutotherthanjust addingeverythingupbutforanumberasbigas144thatwouldbeincrediblytedious(nottomentionthat wedprobablymakeamistake).Itturnsoutthereisaquickwaytofigureitoutbutthatwillhavetowaitfor thenextpost!
Posted in famous numbers, games, geometry, sequences | 17 Comments

Go resources
Posted on October 28, 2006

SomedaysoonIwillgetbacktowritingaboutsomecoolmath,Ipromise!Inthemeantimeasyoumayknow ifyouvereadmymini-bio,IloveplayingtheancientAsiangameofGo.Itsareallyamazinglybeautifulgame, andifyoulikegamesbutyoudontknowhowtoplayGo,youarereallymissingout!TodayIhadthegreat pleasureofteachingacoupleofintroductoryworkshopsonGo,forwhichIcompiledalistoflinkstosomeof myfavoriteGoresourcesontheweb.IfyoureinterestedinlearningmoreaboutGo,thislistoflinksisan excellentplacetostart.So,withoutfurtherado:


Goresourcesontheweb!

IfyouenduplearningGo,letmeknow!MaybeIcouldfindtimetogiveyoualessonortwoonKGS.
Posted in games, links, meta

Hosting issues
208 of 224

Follow

FollowTheMathLess Traveled 10/16/12 16:55


Get everynewpost deliveredto your Inbox.

The Math Less Traveled | Explorations in mathema...

http://mathlesstraveled.com/

Posted on October 6, 2006

TheMathLessTraveledwasdownforawhileyesterdayyoumayhavereceivedstrangeerrormessagesinyour newsreaderifyouresubscribedviaRSS,butdontworry,allisbacktonormalnow.ThesitethathostsTMLT washavingDNSissues,butsinceitsrunbyvolunteersandIdonthavetopayanythingtohaveTMLThosted there,Icertainlycantcomplain!Lookformoremathcomingyourwaysoon.


Posted in meta | 1 Comment

Circles are awesome


Posted on September 26, 2006

CirclesAreAwesome.Enoughsaid.
Posted in geometry, links | 2 Comments

New bookshelf entry: The Book of Numbers


Posted on September 20, 2006

AfterseeingJohnH.ConwayandRichardGuys TheBookofNumberscitedinyetanotherinterestingarticle/book/whatever,Ifinally decidedthatIclearlyhadtoreadit.(Itseemstogetcitedalotincertaincircles.)I wasntdisappointeditsafun,well-written,andfar-rangingtourofmathematical ideasallstemmingfromtheconceptofnumber .Youmightthinksuchatopicwould besomewhatlimiting,butyouwouldbeverywrong!AlongthewayConwayand Guymanagetotouchonsuchtopicsasnumbertheory,geometry,algebra,pi, fractions,partitions,Babylonians,infinity,irrationals,primes,pineapples,Fibonacci numbers,Pascalstriangle,complexnumbers,quaternions,surrealnumbers,harmonicnumberstonamejusta few. Inordertopacksomuchinterestingstuffintothebook,thepresentationisbynecessityfairlyconcise;forthis reasonandbecauseofthetopicscovered,thelevelofthebookisdefinitelymoreadvancedthanmanyofthe otherbooksonmybookshelf.Theauthorsdontshyawayfromadvancedtopics,butthewritingstyleisstill friendlyandcompelling.Evenifyoudontunderstandeverythinginthebook(evenIdidntfollowafewthings thefirsttime),youllundoubtedlylearnsomecoolthings,anditcouldbeabooktogrowwithasyoucontinue tolearnmoremathematics. AtsomepointIhopetocreateapagewithmoredetaileddescriptionsofallthebooksonmybookshelf(which canbefoundintherightmargin).Inthemeantime,ifyoureadabookthatyouthinkIshouldinclude,orread oneofthebooksalreadyonthebookshelfandwanttotalkaboutitorofferyouropinionorcomments,of coursefeelfreetoleaveacommentore-mailme. [Note:Amazon.comisn'tpayingmetolinktothemoranything,it'sjustaconvenientwayformetolinktomore infoaboutthebook.Ifyouwanttobuyacopy,ofcoursefeelfreetobuyitfromanywhereyouwant!]
Posted in books, famous numbers, geometry, golden ratio, infinity, links, primes | 3 Comments

Follow

209 of 224

FollowTheMathLess Traveled 10/16/12 16:55


Get everynewpost deliveredto your Inbox.

The Math Less Traveled | Explorations in mathema...

http://mathlesstraveled.com/

Proof by animation
Posted on September 20, 2006

AproofofthePythagoreanTheorem: [java_applet] Cool,huh?IfyoudontimmediatelyseehowitsaproofofthePythagoreanTheoremjuststareatitfora while.=)Youcanpausetheanimation(andthenrestartitagain)byclickingonitwiththemouse.Imadethis usingtheprogramminglanguageProcessing,whichisalotlikeJavaexceptithasallkindsofgraphicsstuffbuilt in. (Pleasebearwithmeifitdoesntwork,IvenevertriedembeddingaJavaappletinapostbeforeifitdoesnt workforyou,letmeknowinacomment.Andbytheway,theresaverygoodchancetheappletwontworkif youarereadingthisinanewsreaderorinane-mail;togetthefullapplet-ygoodness,visittheoriginalpost.)


Posted in geometry, proof | 2 Comments

New Mersenne prime, for real this time!


Posted on September 11, 2006

GIMPShasjustannouncedtodaythattheyhaveindeedfoundanewMersenneprime!

ObviouslyIcantprintthewholethinghere,butIcantellyouthatthefirst50digitsare 12457502601536945540085550157479950312279598515115,andthelast50digitsare 212445737104635692000092659011752880154053967871.Thisisnowthelargestknownprimenumber, breakingthepreviousrecordsetinDecember2005.(Ofcoursetherearelargerprimenumbersthereare infinitelymanybutthisisthelargestnumberthatwecanactuallypointtoandsay,thisnumberisprime.) Youcandownloadithere.Youcanevenorderaposterofit!(Itsprettyexpensivethough,sinceitsverydifficult toprintdigitstinyenoughtomakethewholenumberfitonaposter.) Unfortunately,at9,808,358digits,itsjustshyofthetenmilliondigitsneededtoclaimthe$100,000prize offeredbytheEFF.Butitsstillprettyexciting. Moreinformationcanbefoundinmypreviouspost.
Posted in links, primes

Challenge #5 Solution
Posted on September 7, 2006

Okay,timeforasolutiontoChallenge#5 Continuereading

Follow

Posted in famous numbers, golden ratio, solutions

210 of 224

FollowTheMathLess Traveled 10/16/12 16:55


Get everynewpost deliveredto your Inbox.

The Math Less Traveled | Explorations in mathema...

http://mathlesstraveled.com/

New Mersenne prime (probably)!


Posted on September 5, 2006

TheGreatInternetMersennePrimeSearchhasjustannouncedthattheyhave(probably)discoveredanew Mersenneprime.Theyhaventreleasedanydetailsyetsospeculationabounds.Willthisbetheprimetowinthe $100,000prizeawardedbytheElectronicFrontierFoundationtothefirstdiscovererofatenmillion-digit prime?Wellseeinaboutaweek,onceanindependentcomputercheckconfirmstheresult. So,whatsthebigdeal?AMersennenumberisanumberoftheform ;forexample, , ,and

areallMersennenumbers.Mersennenumberswhicharealsoprime(thatis,whichhavenodivisors otherthanthemselvesandone)arecalled(notsurprisingly)Mersenneprimes.Forexample,3and7are Mersenneprimes,but1023isnot( ).ThethingaboutMersennenumbersisthattherearewaysto writecomputerprogramstocheckwhethertheyareprimewhichareaLOTfasterthansimplytryingallpossible divisors.Forawhilenow,thelargestknownprimenumbershavebeenMersenneprimes. TheGreatInternetMersennePrimeSearchisadistributedcomputingeffortsortoflikeSETI@Homeifyouve everheardofthatwhichtriestofindnewMersenneprimes.Anyoneatallcansignuptohelp;allyouhaveto doisgototheirwebpage(linkedabove),downloadsomesoftware,andthenwheneveryouarenotusingyour computeritwillworkoncheckingreallybigMersennenumberstoseeiftheyareprime.Giveitatry!Itwont runwhileyoureactuallyusingyourcomputer,soyouwontnoticeanyslowdown.Andyouhaveasmallchance ofbeingfamous(well,sortof)orevenreceivingacashprizeifyourcomputerhappenstobetheonetodiscover anewprime. So,howbigarethesenumbersweretalkingabout?Well,thelargestcurrentlyknownprimenumber(not countingtheonethatwasjustannounced,whichwedontknowyet)is is9MBinsize! YoucanreadmoreaboutMersenneprimesonMathWorld.
Posted in links, primes | 1 Comment

,whichhasawhopping

9,152,052digits!Youcandownloadithere,butkeepinmindthatsinceeachdigittakes1bytetostore,thefile

Challenge #7
Posted on July 28, 2006

ThisdelightfulproblemiscourtesyofDianaDavis(dontfollowthelinkunlessyouwanttoseethesolution, though!),althoughIdidtakethelibertyofformulatingitslightlydifferently.=) Onebrightandsunnyday,yourewalkingdownthestreetthinkingaboutfractals,andyouareaccostedbya strange-lookingwomanwithclothescoveredinprintedtriangles,protractors,andcompasses(alaMs.Frizzle fromtheMagicSchoolBus).Ihaveamathematicalpropositiontomake!shesays.Yourinitialreactionisto walkrightbymumblingsomethingaboutthanksbutIalreadyhaveaprotractorbutsomethingmakesyou pausetolisteninstead.Wewillpickarandomtriangle,shecontinues,andifitsobtuse,youpayme$1,butif itsacute,Illpayyou$2!Hmmyousay,whatdoyoumeanbyarandomtriangle?Oh,wewilljustroll thesetwospecialdicehere,shereplies.Theyhaveinfinitysides,andwhenyourollthem,theygiveyoua completelyrandomnumberbetween0and180like28.3,94.11106749,andsoon.Wellusethemtopick Follow

211 of 224

FollowTheMathLess Traveled 10/16/12 16:55


Get everynewpost deliveredto your Inbox.

The Math Less Traveled | Explorations in mathema...

http://mathlesstraveled.com/

twooftheangles,andofcourse,wecanfigureouttheotheranglesincealltheanglesofatrianglehavetoadd upto180degrees.Sherollsherspecialdiceafewtimestoshowyou,andalthoughyoucantquiteunderstand howdicecouldhaveinfinitysides,theydoseemtogiverandomnumbersbetween0and180assheclaims. Shouldyoutakethebet?


Posted in challenges, geometry, probability | 7 Comments

Fractal Art
Posted on July 15, 2006

Follow

212 of 224

FollowTheMathLess Traveled 10/16/12 16:55


Get everynewpost deliveredto your Inbox.

The Math Less Traveled | Explorations in mathema...

http://mathlesstraveled.com/

TakealookatthefractalartofJockCoopertherearesomeamazingimagesthere!Andtonsofthem.Hereare acouplemoreorlessrandomlychosenexamples:

Andtherearehundredsmore!Hehasalsomadeseveralfractalanimationswhichareverycool(andkindof trippy). Ofcourse,Ivewrittenaboutfractalsbefore.MostofJocksimageswereprobablymadeusingacombinationof iteratedfunctionsofvarioustypes(someusingcomplexnumbers)andvariouscomputer-assistedmanipulations. ThankstoSamLawrenceforthelink.Ifyouevercomeacrosscoolmath-relatedthingsontheweb,orhavean ideaofsomethingyoudliketoseemewriteabout,justdropmeane-mail!


Posted in fractals, links | 6 Comments

Challenge #6
Posted on July 12, 2006

Follow

213 of 224

FollowTheMathLess Traveled 10/16/12 16:55


Get everynewpost deliveredto your Inbox.

The Math Less Traveled | Explorations in mathema...

http://mathlesstraveled.com/

IknowImsortofgettingaheadofmyselfwithsolution-lesschallenges,butIcouldntresist.Hereisafun(I hope)littlecode-breakingchallengethathasalottodowithmath.Thefirstfivepeopletosendmethesecret word(e-mailittome,dontpostithere)willget,um,aspecialprize!(prizeTBD) Continuereading


Posted in challenges

Challenge #4 Solution, Part II


Posted on July 7, 2006

Butwait!WhathappenedtoPartI,youask!?Well,letsplaytheMultipleChoiceGame! (a)ThereisnoPartI. (b)ThereUSEDtobeaPartI,butitdisappeared. (c)PartIwillcomelater.HaventyouseentheStarWarsmovies? (d)Quitwhining. (e)Alloftheabove. Andtheanswerisactually,beforeItellyoutheanswer,letmejustmentionthattheonlyreasonanyoneever givesmultiplechoicetestsisbecausetheyreeasytograde,notbecausetheymakeforagoodtest.ButIdigress. Continuereading

Posted in sequences, solutions

MegaPenny Project
Posted on June 14, 2006

Howisyourintuitionforlargenumbers?Ifyoureadanysortofnewspaperormagazine,orwatchnewson television,youcanthelpbutencounterreallybignumberslike40million,3.2billion,or2trillion.Doyou REALLYknowhowbigthosenumbersare?Ifyourelikemostpeople(meincluded),yourbrainprobablyjust seesillionandtranslatesitintosomefuzzynotionlikeaLOT.ThatswheretheMegaPennyProjectcomesto therescue!Ithelpsdevelopintuitionforthesizeofbignumbersbyusingpicturesofpennies.Itsquite charming,nottomentionthatitsalotoffuntryingtowrapyourmindaroundthingslikeapileofpenniesthe sizeandshapeoftheEmpireStateBuilding! DontforgettocheckouttheMegaMoosectionwhileyourethere,too.


Posted in links

Challenge #5
Posted on June 11, 2006

Andnowforsomethingcompletelydifferentorisit?
Follow

214 of 224

FollowTheMathLess Traveled 10/16/12 16:55


Get everynewpost deliveredto your Inbox.

The Math Less Traveled | Explorations in mathema...

http://mathlesstraveled.com/

ThisproblemcomesfromtheancientGreeks(Euclid,tobeexact).Supposeyouhavearectanglewhichisone unittallandhasthisspecialproperty:ifyoucutoffasquarepiecefromtheendoftherectangle,youreleftwith asmallerrectanglethathasthesameproportionsastheoriginalrectangle.Howlongistheoriginalrectangle? Maybethispicturewillhelpyouseewhatisgoingon.Startingwiththebigbluerectangleatthetop,thewhite squareiscutofffromtheleftside,leavingthesmallerbluerectanglewhichisjustasmallercopyofthebig bluerectangle.

E-mailmewithquestions,comments,orsolutions,orpostthemhere.Also,ifanyonehassolvedeitherorboth partsofChallenge#4,feelfreetopostyoursolutionnow!
Posted in challenges, geometry | 2 Comments

Pi Music
Posted on May 23, 2006

Whilewereonthesubjectof ,takealookatthislink.Theresnotmuchactualmathbehindit,butitsjustkind offun.Theideaisthatyouchoosetennotes,oneforeachofthedigits0-9,andthenitplaysthefirst10,000 digitsofpiusingthenotesyouchose.Inotherwords,itmakesthedigitsof intoasong!Itsnotparticularly useful.Butwhocares?


Posted in famous numbers | 1 Comment

Searching $latex \pi$


Posted on May 19, 2006

TakealookatthePiSearchPage.Youcantypeinabunchofdigitsandittriestofindthemsomewhereinthe first3.2billiondigitsofthedecimalexpansionof . ,ofcourse,istheratioofanycirclescircumferencetoitsdiameter,andisapproximatelyequalto


Follow

215 of 224

FollowTheMathLess Traveled 10/16/12 16:55


Get everynewpost deliveredto your Inbox.

The Math Less Traveled | Explorations in mathema...

http://mathlesstraveled.com/

Thedigitsneverrepeatsince isirrational.Trysearchingforyour7-digitphonenumber.Surprisedthatitwas found?EvenmoresurprisedthatIwassosureyourphonenumberwouldbefound? Well,letsthinkaboutitforaminute.Assumingthedigitsof areessentiallyrandom,thechanceofaparticular digithavingaparticularvalueis1in10;inotherwords,theprobabilityofaparticulardigithavingaparticular valueis .Sotheprobabilitythat7digitsinarowwillallhavecertainvalues(suchasyourphonenumber)is ,oronechanceintenmillion.Thismightsoundlikeaverysmallchance,butremember,wearetalking about3.2billiondigitsinwhichtosearch.Soinfact,wewouldexpectthatyourphonenumberoccursabout timesinthefirst3.2billiondigitsof !Thisalsomeansthatyouhaveagoodchanceoffinding anyeight-digitsequenceofnumbers,andafairlygoodchanceoffindinganynine-digitsequenceofnumbers,but yourchancesoffindinganyparticularsequenceoftenormoredigitsarenotsogood.Tryitandsee!
Posted in famous numbers

Summer Math Tutoring


Posted on May 16, 2006

ThissummerIllbeofferingmyservicesasamathtutor.Whetheryouareinpre-algebraorcalculus,whetheryou needhelpreviewingoryoujustlovemathandwanttolearnmore,whetheryouwanttobetutoredone-on-one orwithagroupoffriends,Idlovetheopportunitytotutoryou.Imalsoofferingtotutoranyonewhowantsto learnhowtoprogramacomputer.Ifyoureinterested,youcanreadallaboutithere (mathlesstraveled.com/tutoring.html). Ofcourse,thisonlyappliestoanyoneintheDCarea.AlthoughImightbewillingtoconsideronlinetutoring (tele-tutoring,asitwere)ifanyonewereinterested. Backatyousoonwithmoredeliciousmath


Posted in meta

Get TMLT in your inbox!


Posted on May 10, 2006

Sincenoteveryoneknowshow(orcares)tousenewsreadersandRSSandallthatjazz,Iveaddeda handy-dandye-mailsubscriptionservice(providedbyFeedBurner).Justtypeyoure-mailaddressintotheboxat right,hitSubscribe,andyoucangetnewpoststoTheMathLessTravelede-maileddirectlytoyou,soyou donthavetoremembertocheckthissitefornewposts.Anddontworry,itstotallyfreeandthereareno advertisementsinvolved. AsalwaysyoucanstillsubscribeviaRSStoo.


Posted in meta | 1 Comment

The Mandelbrot Set

Follow

216 of 224

FollowTheMathLess Traveled 10/16/12 16:55


Get everynewpost deliveredto your Inbox.

The Math Less Traveled | Explorations in mathema...

http://mathlesstraveled.com/

Posted on May 8, 2006

ForthoseofyoualreadyfamiliarwiththeMandelbrotSet,Isupposethiswillbelikevisitingwithanoldfriend. Forthoseofyouwhoarentyoureinforatreat!

Okay,yousay,thatlooksprettycoolIguess,buthuh?Well,toanswerthefundamentalquestionofhuh?we needtodustoffourComplexNumberSkills.(Ifyoudontknowwhatacomplexnumberisorneeda refresherreadmyexplanationofcomplexnumbers.) Hereswhatyoudo.Picksomecomplexnumberc anddefinethefunction

(Notethatzoftendenotesacomplexnumber.)Nowstartwith

anditeratethefunctionf,bytakingeach andfollowthisprocessforafew

valueoutputfromthefunctionandputtingitbackintothefunction.Inotherwords,find ,andsoon.Forexample,letspick steps:

andsoon.Thisisaverysimpleprocess,andcanbeworkedoutbyhandfairlyquickly.Itcanbeworkedoutbya computerintheblinkofaneye. Forsomevaluesofc ,iteratingthefunctionfwilltendtoproducecomplexnumbersthatjustgetbiggerand bigger.Forothervaluesofc ,iteratingthefunctionwillproducecomplexnumbersthatstayrelativelysmall,no matterhowlongyoukeepiteratingthefunction.(Ofcoursetherearetechnicaldefinitionscorrespondingtothe Follow

217 of 224

FollowTheMathLess Traveled 10/16/12 16:55


Get everynewpost deliveredto your Inbox.

The Math Less Traveled | Explorations in mathema...

http://mathlesstraveled.com/

phrasesbiggerandbiggerandstayrelativelysmall,butfornowwewontworryaboutwhattheyare.)Try startingwiththevalue toseeanexampleofthelatter.

Well,nowwerereadytodefinetheMandelbrotset:theMandelbrotsetisthesetofallcomplexnumbersc for whichiteratingthefunction producescomplexnumbersthatstayrelativelysmall. correspondtothepointwith

WecanmakeapictureoftheMandelbrotsetbylettingeachcomplexnumber coordinates

.Forexample,acomputercaneasilymakeapictureoftheMandelbrotsetbylookingateach

pointonthescreenonebyone,decidingwhichcomplexnumberc thatpointcorrespondsto,then(say)coloring thepointblackifc isintheMandelbrotset,andwhiteotherwise.(Often,insteadofjustwhite,programswill choosedifferentcolorsforpointswhicharenotintheMandelbrotset,basedonhowmanyiterationsthe programhadtodobeforeitcoulddecidewhetherthepointwasinthesetornot.) Youmightthinkthatwithsuchasimplefunction,thepicturewouldbesimpleaswelllikeacircle,ora parabola,orsomethinglikethat.Butinfactyougetthatcrazythingshownabove.Iterationcanmakeeventhe simplestfunctionsbehaveinverycomplexways! Infact,theMandelbrotsetiswhatisknownasafractal,anobjectwhichisinfinitelydetailedandcontains copies(ornear-copies)ofitselfonalldifferentscales.Thismeansthat(theoretically)youcankeepzoominginto theMandelbrotforever,andyouwillalwaysseedetailsjustasfineandcomplexasyoudoatthetoplevel. Moreover,asyouzoomin,youwillfindstructuresthatappeartobetinycopiesoftheentirepicture. Butdonttakemywordforitherearesomenicezoomed-inpicturesoftheMandelbrotset,andyoucanfind lotsmorewithGoogleimagesearch.Youmightalsowanttodownloadsomesoftwareforviewingfractalstobe abletoplayaroundwithityourself. Themostamazingthingisthatnoonemadeupthesepicturestheyhaveexistedforever,builtintothe mathematicalstructureoftheuniverse,justwaitingforsomeonetocomealonganditerateacertainfunction andmakeapictureoutofit.Andinfact,itsonlybeensincetheinventionofcomputersthatwevebeenableto dosuchthings(althoughitseasytocarryouttheiterationdescribedaboveforaparticular valueofc byhand,to doitforenoughdifferentvaluesofc tomakeadecentpicturewouldtakesolong,andbesomind-numbingly tedious,astomakeitpracticallyimpossible.) MoreabouttheMandelbrotset,ifyoureinterested: onDr.Math onMathWorld
Posted in convergence, fractals, infinity, iteration | 1 Comment

Challenge #4
Posted on May 3, 2006

Anothersequence-relatedchallengeforyoursequencingpleasure.ForthoseofyouinmyElementaryFunctions class,thisisextracredit.(Hence,ifyoupostacommentwithyoursolutions,pleaserefrainfrompostingan
Follow

218 of 224

FollowTheMathLess Traveled 10/16/12 16:55


Get everynewpost deliveredto your Inbox.

The Math Less Traveled | Explorations in mathema...

http://mathlesstraveled.com/

explanationalongwithit.) 1. Findthesumofthefirst500termsofthisseries:

2. Findthesumofthefirst100terms:

Posted in challenges, sequences | 1 Comment

Challenge #3 Solution
Posted on April 28, 2006

SolutionstoChallenge#3follow.Ifyouabsolutelymustreadthesolutionswithoutattemptingthechallenge,I certainlycantstopyou.ButSantaClausknows.Continuereading
Posted in sequences, solutions | 2 Comments

Challenge #3
Posted on April 22, 2006

ItsbeenalittlewhilesincemylastpostIpleadspringbreakvacation.Atanyrate,hopefullyyoufiguredout thatthelastpostwasintendedasanAprilFoolsjoke.Ihopeyoudidntembarassyourselftoobadlytryingto guessrandomnumbers.(= Herearesomenumbersequences.Canyoufigureoutthepatternandgivethenextthreetermsforeach?(Each sequencefollowsadifferentpattern.) 1. 2. 3. 4.


Posted in challenges, sequences | 2 Comments

"Psychic" Math Powers


Posted on April 1, 2006

Iknowthisisgoingtosoundcrazy,butitactuallyworks.Thereissomuchmathallaroundus,builtintothe veryfabricoftheuniverse,fillingeverycornerofourtechnologicalsocietybutmostpeoplejustignoreitor dontevenrealizethatitsthere. Herestheideawithalittlepracticeandtuningintothenumbersallaroundus,youcanlearntoguess numbersthatotherpeoplearethinkingofnotthatyouwillberightallthetime,butyouwillbeabletodo muchbetterthanyouwouldjustbyrandomguessing.Thefactisthatwhenyouasksomeonetochoosea numberbetweenoneandten,theirchoiceisnotrandombutisactuallyinfluencedinsubtlewaysbythe


Follow

219 of 224

FollowTheMathLess Traveled 10/16/12 16:55


Get everynewpost deliveredto your Inbox.

The Math Less Traveled | Explorations in mathema...

http://mathlesstraveled.com/

numbersallaroundthemtheirage,phonenumberstheyknow,numberstheyhavejustrecentlybeenthinking of,ornumbersthatareprintedonthingsthattheycansee.(Forexample,stoprightnowandtakealookaround you,whereveryouarehowmanynumberscanyousee?Probablyhundreds.)Bybeingawareofthisyoucan learntoguesswhatnumberspeoplehaverandomlychosen. Thefirststepistospendsometimebecomingawareofthenumbersaroundyoutuningin,sotospeak.Pay attentiontonumbersyouseearoundyou,andnumbersthatyouhearpeoplesay.Althoughitdoesnthelp directlywithguessingnumberspeoplearethinkingof,Ivefoundthatdoingthingslikereadingthedigitsofpi outloudcanhelpjustbygettingmybraininamorenumber-mindfulstate. Afterthat,itjusttakespractice.Askyourfamilyandfriendstochooseanumber(startwithanumberfrom1-10, butwithmorepracticeyoumightbeabletodo1-100).Thentrytoguesswhatnumbertheyrethinkingof.But dontjustguessrandomly:trytobeawareofthesurroundingnumberenvironmentnumbersthathavebeen spokenrecently,numbersprintedonthingsnearby,andsoon.(Ifyourereallycleveryoucantrytodothings likeworkacertainnumberintoconversationseveraltimesbeforehavingtheotherpersonchooseanumber,to trytosubliminallyinfluencetheirchoice;butbecarefulnottobetooobvious.)Aboveall,trytosensewhat numberitfeelsliketheyarethinkingof.Iknowthatsoundssilly;ofcourse,mindreadingisntactually possible,butwhatImeanisthatifyouaretunedintothenumberenvironment,yoursubconsciousmight suggestanumberwithoutyourconsciousmindreallyknowingwhywhichcanfeelalotlikemind-readingeven ifthereareactuallymorecomplexandrealisticthingsgoingon. Ofcourse,dontbediscouragedifyoudontdoverywellatfirst!Itwilltakesomepracticebeforeyoustartto getbetteratit.Andeventhebestnumber-guesserscantguesscorrectlyallthetime.Ifyoucanguesscorrectly oneoutofeveryfivetimes,youredoingextremelywell!(Remember,bypurechanceyouwouldonlyexpectto guesscorrectlyoneoutofeverytentimes,iftheotherpersonchoosesanumberfrom1to10.) Happyguessing!Feelfreetocommentandletmeknowifitworksordoesntwork,interestingexperiencesyou havetryingthistechnique,oranythingelse.
Posted in meta | 4 Comments

Convergence
Posted on March 20, 2006

LetsdigalittledeeperbehindthesolutionstoChallenges#1and#2.Whatonearthdoesitmeanforaninfinite expressiontohaveavalue?Well,asnotedinthesolutiontoChallenge#1,whatwerereallytalkingaboutis thevalueoftheexpressionatvariousstoppingpointsalongtheway:whathappenstothesevaluesasthe stoppingpointsgetfurtherandfurtheralong? LetslookagainattheinfinitecontinuedfractionfromChallenge#1.Tomakethingseasiertoread(andsothey wonttakeupasmuchspace),welluseacommonnotationforcontinuedfractions:

Follow

220 of 224

FollowTheMathLess Traveled 10/16/12 16:55


Get everynewpost deliveredto your Inbox.

The Math Less Traveled | Explorations in mathema...

http://mathlesstraveled.com/

So,forexample,theexpressionfromChallenge#1canbewrittenas Challenge#2canbewrittenas whichisequalto11/4. .Asanotherexample,

,andthesecondexpressionfrom (notetherearenoellipses)means

Now,letsanalyzevariousstoppingpointsalongthewaytotheinfiniteexpression then ,then ,andsoon:

,startingwith1,

(Tryconfirmingthesevaluesforyourself.)Asyoucansee,itseemslikethesenumbersaregettingcloserand closertosomethingheresagraphicalviewofwhatsgoingon:

Theredsquaresindicatethestopping-pointvaluesthatwecalculated:theleftmostsquare(inthebottomleft)is thevalueatthefirststoppingpoint,andthevaluesprogresstotheright.Itseasytoseethattheyseemtobe quicklyzoominginonaparticularvalue(indicatedbythehorizontalblueline),somewherearound1.62. Themathwordforthisisconvergencetheredsquares(thevaluesofsuccessivestopping-pointsalongtheway to )convergetoaparticularvalue.Technically,thismeansthatwecangetascloseaswewanttothat particularvalue,aslongaswearewillingtopickastoppingpointthatisfarenoughalong.Thestoppingpoint valueswillkeepgettingcloserandcloser(converging)tothisparticularvalueforever,eventhoughtheywill neveractuallyreachitexactly. Notealsothatthisiswhythevalueof gettingbiggerandbigger: isinfinity.Ifwelookatthevalueatsuccessive

stoppingpoints(i.e.1,1+1,1+1+1,etc.),theyarenotgettingcloserandclosertoanything;theyaresimply

Follow

221 of 224

FollowTheMathLess Traveled 10/16/12 16:55


Get everynewpost deliveredto your Inbox.

The Math Less Traveled | Explorations in mathema...

http://mathlesstraveled.com/

Sayingthatthevalueofsomethingisinfinityisreallyjustashorthandwayofsayingthatitdoesnotconverge toanything(itdiverges). Sowhenweaskforthevalueoftheinfiniteexpression pointvaluesconvergeto? Well,whatdotheyconvergeto?Youalreadyknowtheanswertothis:theyconvergeto knownas ! Thereismuch,muchmoregoingonhereaswellbutthatwillhavetowaitforanotherpost.Asalways,feel freetocommentwithquestions,ideas,comments,oranythingatall(exceptspam).


Posted in convergence, infinity | 3 Comments

,wearereallyasking:whatdothestopping

,otherwise

Challenge #2 Solution
Posted on March 19, 2006

AndherearethesolutionstoChallenge#2 Continuereading

Posted in infinity, solutions | 4 Comments

Challenge #2
Posted on March 10, 2006

CanyouapplythetechniqueslearnedfromthesolutiontoChallenge#1inordertofindavalueforthese expressions?

1.

2.

3.
Follow

222 of 224

FollowTheMathLess Traveled 10/16/12 16:55


Get everynewpost deliveredto your Inbox.

The Math Less Traveled | Explorations in mathema...

http://mathlesstraveled.com/

Posted in challenges, infinity | 9 Comments

Challenge #1 Solution
Posted on March 10, 2006

ReadonforthesolutiontoChallenge#1butgiveitatryfirstifyouhaventalready! Continuereading

Posted in famous numbers, golden ratio, infinity, solutions | 7 Comments

Challenge #1
Posted on March 8, 2006

Whatbetterwaytostartthingsoffthanachallenge?Challengingproblemsarethegoldenroadtodiscovering mathematicalbeauty:theysparkinterest,inviteexploration,requirethoughtandcreativity,andusuallybegtobe tweaked,generalized,andotherwiseturnedintoavenuesforfurtherexploration.Andbeautydiscoveredfor oneselfisthebestsort. So,enoughrambling,ontothechallengealready!Itisthis:canyoufindavalueforthefollowingexpression?

Thedots( )meanthatthepatterncontinuesforever.Ifyouhaveseenthissortofproblembeforethenitwill notbetoohard;butifnot,thesolutionisnotobviousandrequiresacertaincreativeleap.Itdoesnot,however, requireanymathmoreadvancedthanbasicalgebra. (Andno,theanswerisnotinfinity,eventhoughthatisthevalueoftheverysimilar-lookingexpression ) Feelfreetocommentwithguesses,questions,solutions,orwhatever.


Posted in challenges, infinity | 8 Comments

Welcome!
Posted on March 8, 2006

Welcome,dearreader,toTheMathLessTraveled!Doanyofthefollowingapplytoyou? 1. Youareahighschoolstudent. 2. Youarenotahighschoolstudent. 3. Youareintriguedbytheideathatmathematicscanbefunandbeautiful. 4. YoutookGeometralgebralculunometryIIIwithMrmrs.Soandsowhatserisfaceandhadsuchaterrible, horrible,no-goodverybadexperiencethatyouhavenowdecidedthatyouHATEmath. 5. Youenjoytryingtosolvechallengingmathproblems.


Follow

223 of 224

FollowTheMathLess Traveled 10/16/12 16:55


Get everynewpost deliveredto your Inbox.

The Math Less Traveled | Explorations in mathema...

http://mathlesstraveled.com/

6. Youhatesolvingchallengingmathproblemsbutdontmindreadingabouttheir(interestingandbeautiful) solutions. Ifso,youvecometotherightplace!Whetheryouareamathnerd,aclosetmathlover,amathhaterwillingto giveitasecondchance,orjustcurious,Ihopeyouwillbeabletofindthingsheretointrigueandinspireyou: fractionsandfractals,infinitiesandintuitions,numbersandnuances,andmostimportantly,beauty. Enjoy! (IfyouareintriguedbutwanttoknowmorewhoIam,whatthisblogisreallyallabout(worlddomination, ofcourse),whatsortsofmathyoumightfindherecheckouttheaboutpageslinkedtofromtheright margin.)


Posted in meta

The Math Less Traveled


Theme:TwentyTen BlogatWordPress.com.

Follow

224 of 224

FollowTheMathLess Traveled 10/16/12 16:55


Get everynewpost deliveredto your Inbox.

You might also like